You are on page 1of 378

版权信息

书名:余翔GRE330+的Verbal全科解题思维

作者:余翔

出版社:中国人民大学出版社

出版日期:2021-05-01

ISBN:9787300292663
目录
CONTENTS
前言
第一部分 GRE逻辑
第一章 GRE逻辑总论
第二章 归纳题
第三章 假设题
第四章 解释矛盾题
第五章 评价题
第六章 句子作用题(黑体题)
第七章 混合逻辑题练习
第二部分 GRE阅读
第八章 阅读的整体解题思维
第九章 阅读中的题型介绍与详细解法
第十章 短文章和长文章的实战练习&易错题和难题汇总练习
第十一章 补充文章练习与讲解
第三部分 GRE填空
第十二章 填空解题思维
第十三章 考前必看机经词汇
第十四章 考前必练20道经典机经题
前言

2005年我第一次接触到GRE是刚考完托福(取得了637分)。当时
考GRE的考生大多是名校中的精英,GRE课堂上学习气氛非常浓厚。时
间一晃,从接触GRE到一线教授GRE,我和GRE的“缘分”已经超过了16
年,有14年的一线GRE教学经验(真正在一线专攻GRE教学且教龄12年
以上的GRE老师,全国目前不超过8人)。在本书中,我详细讲解了
GRE Verbal(语文部分)中的逻辑题、阅读题、填空题的核心考法和解
题思维与步骤,希望这些内容能帮助中国考生获取足以申请美国排名前
30高校的GRE分数。

从我第一次教授GRE到现在,解题切入点和系统性方法已迭代了很
多次。拿阅读举例,逐句读并翻译成中文再讲题不适合冲刺GRE高分的
学生。因为在实际考试中,学生面临时间压力和心理压力,不可能把文
章和题目读得很透彻。所以在教学后期,我模拟考生在真实考场中的状
态,思考如何通过一些思考方式和解题切入点来找到答案。学无止境,
教研更无止境。在教学的第11年我才渐渐摸索到了GRE的解题精髓,并
将其传授学生,之后,又思考如何以托福90分考生的能力水平为出发
点,帮助他们在GRE考试中考到高分,比如330分。此书讲解的解题思
维,正是我的教研成果,望各位考生好好体会这些被无数考生验证过切
实有效的解题技巧。

最后,由衷希望各位考生通过学习此书能够获取GRE高分,入读名
校,实现人生理想。

余翔
第一部分 GRE逻辑

第一章 GRE逻辑总论

一、对GRE逻辑题考查本质的理解

为什么美国研究生三个主流考试GMAT(管理类入学考试)、
GRE(通用研究生入学考试)和LSAT(法学院入学考试)都有测试学
生逻辑能力的模块呢?

在备考过程中,逻辑能力的提升对于学生以后进入商学院或毕业后
工作会有哪些具体的好处呢?

批判性逻辑(critical reasoning)的“批判性(critical)”如何理解?

逻辑测试就是在测智力吗?

…………

上述问题是初入GRE备考期考生常遇的问题。为更好地解答这些问
题,我们必须明确GRE逻辑题的考查核心。概括而言逻辑题考查的核心
有:考查学生是否具有“客观性思维”、“批判性思维”以及“决策能力”。

1. 何为客观性思维?客观性思维就是在获得某项信息时,除了常识
推理外,不可以加入考生本身主观的联想,过往经历所带来的过度推
理,或任何有感情色彩的价值观倾向等。我们所知道的事情只可以是信
息本身的内容,以及构成信息内容的前提条件,其他任何事情只要不是
常识可联想到的,我们只能当作不知。不可以话里有话地理解信息。例
如,我们学习修辞的时候读过“窗帘是蓝色的,体现出了作者的忧郁心
情”。这句话当作修辞来学习没有问题,但这句话本身不符合客观性逻
辑。“窗帘是蓝色的”这条信息只能让我们知道信息内容本身(窗帘是蓝
色的这件事情),以及信息内容的前提(如“有窗帘”)。从窗帘的颜色
推理出作者的心情,就是主观性的判断,不符合GRE考试中客观性思维
的要求。我们必须客观地接受信息,才能不被主观或感情因素影响,从
而作出正确的决策。

2. 何为批判性思维?简单讲,批判性思维肯定不是吵架或抬杠。批
判性思维在考试中的原则是“我们从某选项是错误的角度来对待这项,
但如果我们找不到这项错误的理由这项就不算错,可以放着待比较”。
在商业中,某个方案被排除,一定要有明确的理由。因此,大家在GRE
逻辑题或阅读题中会看到答案非常虚,因为虚就表示不会错。比如“针
对这个问题,我们会采取相应的方案”就是根本不会错的表达,毕竟不
采取方案也叫“相应的方案”。

3. 决策思维。某些逻辑题的答案在另外一道题中就是诱惑选项。这
是为什么呢?因为在整个GRE语文部分考试中,根本没有最好的答案,
只有相对于其他选项而言更好的答案。在商业中资源有限,我们必须要
学会如何在几项都有缺陷的选择中找一个相对较好的,或在几项都正确
的选择中寻找出相对更好的那一项。相信读到这,考生会理解“为什么
某些题的答案在另外一些题中就是诱惑选项”的说法了。

以上这三项能力是笔者从事GRE教学14年以来的感悟。中国考生逻
辑部分得分不高的原因之一在于读不懂题干和选项。建议考生在托福至
少有90分的前提下,再准备GRE考试。

为应对GRE考试中思维的考查,考生不能仅靠题海战术,而是要多
思考,多研究错题,在此基础上再来刷题。

二、GRE逻辑原文元素罗列和题型分类

逻辑题可按结构、内容、逻辑实质三个角度大致分类为四种题型:
推理链题,归纳题,句子作用题(黑体题)和其他类。
(一)推理链题

逻辑链(argument)可以理解为前提推出结论的过程。GRE逻辑题
中围绕推理链出的考题就称为推理链题,其又可以细分为以下六种考
题:

(1)增加前提能够推导出结论可能性的考题——支持题。

(2)减少前提能够推出结论成立可能性的考题——削弱题。

(3)认为前提和结论间缺少某些补充性的必要因素——假设题。

(4)认为前提不能推出结论,结论由其他原因(他因)导致——
解释题。

(5)测试通过前提推出结论的过程是否成立——评价题。

(6)用填空的方式考查之前五种题型——填空题。

考试中,最常见的题型就是推理链题,归纳题、句子作用题(黑体
题)、其他类题基本在一次考试中各自出现1~2道。

何为前提,何为结论呢?

逻辑原文可以由六个元素组成:前提(premise)、结论
(conclusion)、中间性结论(intermediate conclusion)、背景
(background)、与结论方向相反的前提(counter-premise)、与结论
方向相反的结论(counter-conclusion)。在做题时,不是所有元素都需
要被概括进推理链,但前提和结论两个元素必须被概括进推理链。

1.前提(premise)的特征

(1)必须概括进推理链。

(2)用于支持结论(conclusion)。

(3)每一份逻辑原文都存在至少一个前提。

(4)可以被理解为得出结论(conclusion)的理由,故常接在表示
原因的接续词(例如because, due to , for, the reason is that...等)之后。

(5)呈现方式可以是历史事件、统计数据、观点、事实等。

2.结论(conclusion)的特征

(1)不是所有逻辑文章都有结论。

(2)必须被概括进推理链。

(3)主要呈现方式是描述作者的主要观点,或预测,或是能解释
文中现象的说法等。

(4)至少被一个前提(premise)所支持。

上述两个元素必须概括进推理链,需要注意的是前提可以不止一
个,结论可以被多个理由(就是“前提”)所支持,而答案可以针对与理
由相关的任何一个事件进行设置。

下面是四个不需要被概括进推理链的元素:

3.背景(background)

(1)不需要被概括进推理链。

(2)出现的目的是帮助读者更好地理解前提如何推出结论。

(3)呈现的方式可以是以客观事实、统计数据、某个术语的解释
或定义等。

(4)并不是所有逻辑原文都存在背景。

4.中间性结论(intermediate conclusion)

(1)不需要被概括进推理链。

(2)是比较长的文章中的一个过渡性(先期性)结论,其逻辑方
向和结论(conclusion)一致;结论就是中间性结论的一个进阶推理。
(3)中间性结论被某个前提所支持。

(4)中间性结论其本身又作为前提,支持最终的结论。

5.与结论方向相反的前提(counter-premise)

(1)不需要被概括进推理链。

(2)支持和结论态度相反的结论(counter-conclusion)。

(3)起到让考生混淆,增加找到正确推理链困难的作用。

6.与结论方向相反的结论(counter-conclusion)

(1)不需要被概括进推理链。

(2)态度方向和结论相反。

(3)被与结论方向相反的前提(counter-premise)所支持。

(4)和与结论方向相反的前提(counter-premise)一样,增加考生
找到正确推理链的困难。

GRE考试中最常见的逻辑题题型如下:

In the United States, of the people who moved from one state to
another when they retired, the percentage who retired to Florida has
decreased by three percentage points over the past ten years. Since many
local businesses in Florida cater to retirees, this decline is likely to have a
noticeably negative economic effect on these businesses.

Which of the following, if true, most seriously weakens the


argument?

A. Florida attracts more people who move from one state to another
when they retire than does any other state.

B. The number of people who move out of Florida to accept


employment in other states has increased over the past ten years.

C. There are far more local businesses in Florida that cater to tourists
than there are local businesses that cater to retirees.

D. The total number of people who retired and moved to another state
for their retirement has increased significantly over the past ten years.

E. The number of people who left Florida when they retired to live in
another state was greater last year than it was ten years ago.

为原文, 为题目,再加上五个选项构成一道逻辑单选题。

注: 做逻辑题时,先读问题,知道问题问的方向和精准的考点之
后再读原文,然后带着预测的答案方向再找选项,得出答案。

逻辑文章元素举例(1):

An arrest made by a Midville police officer is provisional until the


officer has taken the suspect to the police station and the watch commander
has officially approved the arrest.Such approval is denied if the commander
judges that the evidence on which the provisional arrest is based is
insufficient.(background) A government efficiency expert has found that
almost all provisional arrests meet standards for adequacy of evidence that
watch commanders enforce. (counter-premise) The expert therefore
recommends that the watch commander's approval should no longer be
required since the officers' time spent obtaining approval is largely wasted.
(counter-conclusion) This recommendation should be rejected as
dangerous, (conclusion) however, since there is no assurance that the
watch commanders' standards will continue to be observed once approval is
no longer required. (premise)

解读: 句1、句2(An arrest made by a Midville ..... arre


st is based is insufficient.)描述一个现象。现象本身没有直接
支持某个结论,所以概括为“背景”。
句3(A government efficiency expert ... that watch comma
nders enforce)支持了一个态度和作者最终结论相反的结论,故概
括为与结论方向相反的前提。

句4(The expert therefore recommends that .... obtaining


approval is largely wasted.)中的“therefore recommends that
..”说明本句是一个结论,但这个结论和作者最终的结论态度相反,
因为之后有一个however,因此本句是与结论方向相反的结论。

句5的主句(This recommendation should be rejected as dan


gerous)有明确的态度评价,和句4呈相反关系,是作者最终的观点
,故被认为是结论。

句5的原因状语从句(since there is no assurance that ...


is no longer required)支持主句,作为结论的原因存在,被认为
是前提。

逻辑文章元素的举例(2):

Hunters alone are blamed for the decline in Greenrock National Forest's
deer population over the past ten years. (background) Yet clearly, black
bears have also played an important role in this decline. (conclusion) In
the past ten years, the forest's protected black bear population has risen
sharply, and examination of black bears found dead in the forest during the
deer hunting season showed that a number of them had recently fed on deer.
(premise)

解读: 句1(Hunters alone are blamed for ... past ten ye


ars.)描述了一个观点或现象,因此该句可概括为“背景”。

句2(black bears have also played an important role in t


his decline)中Yet提示本句和之前句子观点态度相反,句2之后是
具体时间,表明后文是支持这句话的细节,所以句2被后文所支持,
为结论。
句3(In the past ten years, the forest's protected ... h
ad recently fed on deer.)属于句2的原
因,即为前提。

7.考生答题的障碍:推理链的提取

解GRE逻辑题时,切记读文章的目的就是把推理链总结出来,读出
推理链之前,不建议读选项。在总结的过程中,有以下几点需要特别强
调:

* 推理链的总结步骤:先思考结论是哪句话,再想共有哪些事情推
出了结论,这些事情全部是前提。

(1)多个前提的辨识。

一个结论可以被多个前提推出,那么如果前提有n个句子,就表示
有n个前提吗?肯定不是这样。如果几个句子在说同一件事情,那就只
有一个前提。多个句子在说不同的事情,这些事情都作为结论的理由或
依据,这才是多个前提。(所有前提都需要被概括进推理链中)

(2)指代性词汇不进推理链。

Which of the following most logically completes the argument below?

Although the number of large artificial satellites orbiting the Earth is


small compared to the number of small pieces of debris in orbit, the large
satellites interfere more seriously with telescope observations because of the
strong reflections they produce. Because many of those large satellites have
ceased to function, the proposal recently has been made to eliminate
interference from nonfunctioning satellites by exploding them in space. This
proposal, however, is ill conceived, since ____.

此文的结论是“this proposal is ill conceived”,需要为这个结论找一


个理由,此题相当于对一个计划进行削弱。如果考生把短文概括为“这
个策略不行”,那么选项如果出现了与这个策略具体内容相关的信息,
考生不会在第一时间反应过来其和原文推理链是相关的。因此如果把指
代性词汇总结进推理链中,必须要把具体内容归纳进去。不能记成“这
个策略不行”,而要记成“把坏的了卫星爆破的方案不行”。

(3)结论和中间性结论的判断。

中间性结论不需要被概括进推理链,而结论需要被概括进推理链。
如果在原文中发现多个结论,哪个结论才是最终的,并且需要被总结进
推理链呢?答案是“时间靠后”的那一个结论才是最终的结论。

例 The owner of a small publishing company plans to lease a new office


space that has floor-to-ceiling windows and no internal walls, arguing that the
new space will enhance worker productivity. The owner cites a recent study
showing that workers exposed to natural light throughout the day tended to
report, on average, a higher level of job satisfaction than did those who
worked in office spaces that used fluorescent lighting. Thus, the owner
concluded, exposure to natural light has a positive effect on workers' job
satisfaction.

解读: 原文中出现两个结论,其一是“the new space will en


hance worker productivity”,其二是最后一句话“the owner con
cluded, exposure to natural light has a positive effect on w
orkers' job satisfaction”。从外在形式来看,第二个更像结论。
但是在GRE考试中,如果考生仅凭形式而不思考意思来作判断的话,
基本上总分不会超过680分。“增加创造力”和
“提升满意程度”哪个“时间靠后”?“提升了满意程度,所以才会
最终增加创造力”,所以第一个结论才是最终的结论。

(二)归纳题

从逻辑原文范围中可以概括总结出哪些信息——归纳题。

(三)句子评价题
某个或某两个句子在逻辑原文中扮演的角色是什么,又称黑体题。

(四)其他类

上述题型之外的题均称为其他类题型,出现概率很小。

面对如此多的题型,我们只需要记住一点,即“看清问题,回答问
题”。在灵活的出题方式下,不能思维固化,在实际考试中碰到新变体
题型,考生需做到随机应变,即不管出题形式如何改变,看准题目在问
我们什么,答其所问即可。

例1 Yeasts capable of leavening bread are widespread, and in the many


centuries during which the ancient Egyptians made only unleavened bread,
such yeasts must frequently have been mixed into bread doughs accidentally.
The Egyptians, however, did not discover leavened bread until about 3000
B.C. That discovery roughly coincided with the introduction of a wheat
variety that was preferable to previous varieties because its edible kernel
could be removed from the husk without first toasting the grain.

Which of the following, if true, provide the strongest evidence that the
two developments were causally related?

解读: 题目问哪个选项提供证据表明文章中两个事件的发展是
相关的。此题略像支持题,但支持题是增加前提和结论成立的概率,
而此题所问的对象是文中的两个事件(two developments),和一般
支持题有差异。

例2 Sonya: The government of Copeland is raising the cigarette tax.


Copeland's cigarette prices will still be reasonably low, so cigarette
consumption will probably not be affected much. Consequently, government
revenue from the tax will increase.

Raoul: True, smoking is unlikely to decrease, because Copeland's


cigarette prices will still not be high. They will, however, no longer be the
lowest in the region, so we might begin to see substantial illegal sales of
smuggled cigarettes in Copeland.

Raoul responds to Sonya's argument by doing which of the following?

解读: 题目问Raoul回复Sonya观点的方式(即Raoul如何评价So
nya的观点),不属于直接的推理链考题,考生需要读出Raoul削弱或
支持Sonya的角度。

例3 In Patton City, days are categorized as having heavy rainfall(more


than two inches), moderate rainfall(more than one inch, but no more than
two inches), light rainfall(at least a trace, but no more than one inch), or
no rainfall. In 1990, there were fewer days with light rainfall than in 1910
and fewer with moderate rainfall, yet total rainfall for the year was 20 percent
higher in 1990 than in 1910.

If the statements above are true, then it is also possible that in Patton
City

解读: 此题并非归纳题,因为归纳题中答案描述的事件一定是
逻辑原文中所描述事件的一个子集,或原文能推理总结出的一个事件
。而此题问的是哪个选项可能对。何为“可能对”?只要不和原文内
容冲突,任何事件都“可能对”。此题的答案范围完全可以脱离原文
描述的事件范围,因此此题不能被认为是归纳题。

三、各种题型的举例和分析

1.填空题

Garnet and RenCo each provide health care for their employees. Garnet
pays for both testing of its employees' cholesterol levels and treatment of
high cholesterol. This policy saves Garnet money, since high cholesterol left
untreated for many years leads to conditions that require very expensive
treatment. However, RenCo does not have the same financial incentive to
adopt such a policy, because ____.

A. early treatment of high cholesterol does not entirely eliminate the


possibility of a stroke later in life

B. the mass media regularly feature stories encouraging people to


maintain diets that are low in cholesterol

C. RenCo has significantly more employees than Garnet has

D. RenCo's employees are unlikely to have higher cholesterol levels


than Garnet's employees

E. the average length of time an employee stays with RenCo is less than
it is with Garnet

解析: 此题相当于解释题,即寻找Renco没有经济动机采取省钱
措施的原因。

此题论证过程是:Renco和Garnet有共同点。(句1:两个公司都
提供医疗保健,表示两个公司都会为员工报销医药费)Garnet公司会
为员工检测胆固醇水平并提供相应治疗,因为长期来看会为公司省钱
(high cholesterol left untreated for many years leads to co
nditions that require very expensive treatment),所以RenCo
公司应该也采取同样的措施。RenCo公司没有采纳的理由是什么?找
一个表达不采取策略理由的选项。

推理链题答案基本就在推理链的范围中,如果选项所描述的事件
和文章论证过程中某一个事情有关联,则这个选项就和正确答案沾边
,可留下备选。

A选项描述的早期治疗不可完全消除中风,但只要能省钱RenCo公
司依然可以采纳文中所说的措施,为什么没有采纳?A项没说,因此
此项无关。

B项讲媒体鼓励人们采纳低胆固醇的饮食。为什么有一个可省钱
的措施,RenCo公司不去采纳?此项依然没有描述,此项无关。

C项表示RenCo公司人更多。如果人更多,RenCo会承担更多的医
疗费用。此项不能选。

D项描述RenCo公司员工的胆固醇指标不可能比Garnet公司员工的
更高。分析D项之前,大家先想一个问题:GRE既然考查逻辑,那GRE
逻辑题会不会有一些违反常识的原文或答案?GRE逻辑需要进行由前
提至结论的推理,这个推理如何去理解?用背景推理还是用经验推理
?这里先不予解答,在后文详细说到推理链题型解法的时候,再深入
描述。暂且先放着D选项,因为肯定有学生这么认为:RenCo公司员工
胆固醇不会更高,所以RenCo没有采纳这措施的动机。

原文中说“高的胆固醇如果不治疗,数年的话会导致公司昂贵的
医疗费用”,E项的“average length of time”(平均工作时间)
与原文中的“数年”属于有话题沾边,故E项和原文有联系,选E。

2.支持题

Studies in restaurants show that the tips left by customers who pay their
bill in cash tend to be larger when the bill is presented on a tray that bears a
credit-card logo. Consumer psychologists hypothesize that simply seeing a
credit-card logo makes many credit-card holders willing to spend more
because it reminds them that their spending power exceeds the cash they have
immediately available.

Which of the following, if true, most strongly supports the


psychologists' interpretation of the studies?

A. The effect noted in the studies is not limited to patrons who have
credit cards.

B. Patrons who are under financial pressure from their credit-card


obligations tend to tip less when presented with a restaurant bill on a tray
with credit-card logo than when the tray has no logo.

C. In virtually all of the cases in the studies, the patrons who paid bills in
cash did not possess credit cards.

D. In general, restaurant patrons who pay their bills in cash leave larger
tips than do those who pay by credit card.

E. The percentage of restaurant bills paid with given brand of credit card
increases when that credit card's logo is displayed on the tray with which the
bill is prepared.

解析: 题目问支持心理学家解释(psychologists' interpreta


tion)的选项是哪个,所以看原文的时候解释本身和针对的问题要读
清楚。(答题技巧:题目问什么,就回答什么)

原文先说了一个现象:如果盘子上有一个信用卡标志,则小费会
给得更多,后文对这个现象进行解释:因为看到信用卡标志,人们会
觉得自己有更大的购买力。要选一个能支持这种解释的选项。

此题是典型的果因推理,要找出支持原文所描述现象的原因,一
种方式是排除其他可以推出这一现象的原因,即排除他因;另外一种
方式是“原文的理由”可推出“原文的现象”,这样相当于把原文的
推理链再描述一次,达到支持的作用。类似的角度可以是给出一个其
他场合的例子或理论来解释文中现象。出题机构往往会以“给出他因
”的方式进行出题,因为如果是第二种方式,那选项必须把文章中所
说的现象和解释在一个选项中都提到,这样的出题方式太容易让学生
觉得该选项是沾边选项。因此,此题的答案方向是:如果选项说了一
个新事件,而这新事件不能推出文章中所说的现象,则无关,因为不
属于排除他因;要么一个选项把原文中的现象和原因再说一次。

A项:说了新事件,此新事件又不能推出“给小费更多”这个现
象,则A项无关。

B项:提了信用卡和给小费多少之间的关系,属于原文推理链的
范围,先放着此项。

C项:是否拥有信用卡和信用卡标志会刺激购买力是两件事情,
所以有没有信用卡和是否被刺激是两回事。

D项:本题要找的选项应能支持看到信用卡标志和给更多小费这
一说法,此项即使沾边也逻辑方向相反,不选。

E项:特例可以允许任何情况发生,但是特例没有代表性(not r
epresentative),只有大多数的、可以代表某个集体的样本才能说
明问题,此项的特定品牌(given brand)一样不能说明问题,完全
可以把特定品牌当成一个特例来对待。

答案为B,属于第二种答案设计的模式。

3.归纳题

Shipping Clerk: The five specially ordered shipments sent out last week
were sent out on Thursday. Last week, all of the shipments that were sent out
on Friday consisted entirely of building supplies, and the shipping department
then closed for the weekend. Four shipments were sent to Truax Construction
last week, only three of which consisted of building supplies.

If the shipping clerk's statements are true, which of the following must
also be true?

A. At least one of the shipments sent to Truax Construction last week


was specially ordered.

B. At least one of last week's specially ordered shipments did not consist
of building supplies.

C. At least one of the shipments sent to Truax Construction was not sent
out on Thursday of last week.

D. At least one of the shipments that were sent out on Friday of last
week was sent to Truax Construction.

E. At least one of the shipments sent to Truax Construction last week


was sent out before Friday.

解析: 题目问如果船员的话正确,下面哪个选项也对。相当于
哪个选项可以确定地从上文概括出来。解题过程类似阅读题,因为阅
读题中正确选项也是从文章信息中总结出来的,只是归纳题的文章篇
幅短于阅读题的文章。因此,正确选项不可以出现新元素、新概念,
或原文中没有提到的绝对化程度等,毕竟不可能从文章中总结出文章
没有提到过的信息。

短文告诉我们四件事情:(1)周四有特殊订单货物(special o
rder shipment)发出。(2)周
五发出的货物(shipment)中都是建筑材料(building supplies)
。(3)周五后就不再发货物(shipment)。(4)Truax Constructi
on收到的四批货物中有三批是建筑材料。

阅读GRE逻辑文章时,必须以事件为单位边阅读边总结文章中一
共说了几件事。

在上述四件事情中,根据事件一,我们可否得知周四只发了特殊
订单货物?有没有可能在周四也发出建筑材料?以及,根据事件二,
我们可否得知只有周五这一天发出了建筑材料,而其他任何一天都没
有发出建筑材料?当然不能。因为我们并没有看到类似only的限制词
。所以,选项ABCD可以都去掉,比如B项,因为我们不知道在周四前
是否有非特殊订单的货物发出,以及我们不知道特殊订单货物和建筑
材料之间是否存在关联,所以没法判断B项是否正确。事件四表示有
一批货物不是周五发出,而事件二告诉我们周五发出的都是建筑材料
,则不是建筑材料的货品要么在周五前发出,要么在周五后发出,然
后事件三告诉我们非建筑材料的货物只能在周五前发出,故只有E项
正确。

4.解释题
When a new restaurant, Martin's Cafe, opened in Riverville last year,
many people predicted that business at the Wildflower Inn, Riverville's only
other restaurant, would suffer from the competition. Surprisingly, however, in
the year since Martin's Cafe opened, the average number of meals per night
served at the Wildflower Inn has increased significantly.

Which of the following, if true, most helps to explain the increase?

A. Unlike the Wildflower Inn, Martin's Cafe serves considerably more


meals on weekends than it does on weekdays.

B. Most of the customers of Martin's Cafe had never dined in Riverville


before this restaurant opened, and on most days Martin's Cafe attracts more
customers than it can seat.

C. The profit per meal is higher, on average, for meals served at Martin's
Cafe than for those served at the Wildflower Inn.

D. The Wildflower Inn is not open on Sundays, and therefore Riverville


residents who choose to dine out on that day must either eat at Martin's Cafe
or go to neighboring towns to eat.

E. A significant proportion of the staff at Martin's Cafe are people who


formerly worked at the Wildflower Inn and were hired away by the owner of
Martin's Cafe.

解析: 题干中explain一词提示此题为解释题。文章中有一个前
后矛盾的事件(discrepancy),需要考生在选项中找到使结论成立
的理由。因而,在解答解释题的过程中要特别关注结论是什么。

原文的事件包括:(1)Martin's Cafe开业影响Wildflower Inn


的生意。(2)Wildflower Inn在Martin's Cafe开业后反而卖了更多
的饭。

根据时间先后顺序,可知结论是第二个事件,需要找一个能推出
“竞争对手Martin's Cafe存在的情况下,Wildflower Inn生意更好
”的选项。
A项:Martin's Cafe自己与自己进行对比,属于纵比,而原文是
自己和别人比,属于横比。此项直接可判定为无关项。即使考生不清
楚这个技巧,此项也没有描述Martin's Cafe的存在会使Wildflower
Inn生意更好的信息,故可排除此项。

B项:此项描述Martin's Cafe容纳不了所有光顾的客人,那些未
能在Martin's Cafe 就餐的客人会不会去Riverville的其他餐馆即Wi
ldflower Inn呢?基于常识来推理,当然有可能。因此,此项增加了
“Wildflower Inn可以卖更多饭”的可能性,先放着可作为答案的
备选。

C项:逻辑题不难,但如果考生修改了题目所问的事件或原文中
描述的事件的意思,并按错误方向去找选项,尤其是读完了某个选项
之后受其干扰,就容易选错。此题问的方向是“Wildflower Inn卖更
多的饭”,不是问赚不赚钱,即使不赚钱但如果某项表达的是Wildfl
ower Inn的饭卖得多不多的话题,也可以放着这个选项待选。排除C
项。

D项:此项没有联系到Wildflower Inn卖饭更多的事件,只谈到
了Wildflower Inn不营业的时候,人们要么去Martin's Cafe要么去
附近的城镇,这项不能告诉我们为什么Wildflower Inn会卖更多的饭

E项:员工会不会影响到营业额?答案是不确定的。选项没有联
系Wildflower Inn会卖更多饭的事情,排除E项。

只有B项有点逻辑关联,得出答案为B项。

5.评价题

Community activist: If Morganville wants to keep its central shopping


district healthy, it should prevent the opening of a huge SaveAll discount
department store on the outskirts of Morganville. Records from other small
towns show that whenever SaveAll has opened a store outside the central
shopping district of a small town, within five years the town has experienced
the bankruptcies of more than a quarter of the stores in the shopping district.

The answer to which of the following would be most useful for


evaluating the community activist's reasoning?

A. Have community activists in other towns successfully campaigned


against the opening of a SaveAll store on the outskirts of their towns?

B. Do a large percentage of the residents of Morganville currently do


almost all of their shopping at stores in Morganville?

C. In towns with healthy central shopping districts, what proportion of


the stores in those districts suffer bankruptcy during a typical five-year
period?

D. What proportion of the employees at the SaveAll store on the


outskirts of Morganville will be drawn from Morganville?

E. Do newly opened SaveAll stores ever lose money during their first
five years of operation?

解析: evaluating这个词提示此题为评价题。评价题是从其他
角度来论述原文中的前提可以推导出结论。

原文讨论的事件是“SaveAll开店与其他店倒闭”,选项中只有C
和E两项沾边。E项中lose money和“倒闭”是一个方向,因此也可以
先待选。对比选项后发现C项和原文核心事件描述得更近,故选C。

何为选项与原文事件“关系更近”?选项描述事件与原文的密切
程度有以下的排列顺序:原文再现>改写>方向对>无关选项。

选项描述内容与原文的事件一模一样即为“原文再现”,密切度
最高。

选项描述内容是原文事件改写,或通过常识可以从选项推出原文
事件的,密切度次之。
选项描述事件和原文事件是一个方向,但是推不出原文事件,密
切度优于无关选项。

因此,只要选项和原文事件是一个方向,即可以先放着,没有更
好的选项的时候,只能退而求其次,而不需要追求最佳选项。(GRE
逻辑考查决策思维——没有最好的,只有相对更好的)

6.削弱题

Five years ago, as part of a plan to encourage citizens of Levaska to


increase the amount of money they put into savings, Levaska's government
introduced special savings accounts in which up to $3,000 a year can be
saved with no tax due on the interest unless money is withdrawn before the
account holder reaches the age of sixty-five. Millions of dollars have
accumulated in the special accounts, so the government's plan is obviously
working.

Which of the following, if true, most seriously weakens the argument?

A. A substantial number of Levaskans have withdrawn at least some of


the money they had invested in the special accounts.

B. Workers in Levaska who already save money in long-term tax-free


accounts that are offered through their workplace cannot take advantage of
the special savings accounts introduced by the government.

C. The rate at which interest earned on money deposited in regular


savings accounts is taxed depends on the income bracket of the account
holder.

D. Many Levaskans who already had long-term savings have steadily


been transferring those savings into the special accounts.

E. Many of the economists who now claim that the government's plan
has been successful criticized it when it was introduced.
解析: weaken一词表明本题是削弱题,即降低前提推出结论的
可能性。此题论证过程属于“通过现象得到解释”的果因推理题。此
题除了果因推理的解题模式外,还可以用“通过计划达到目标”的方
式解题。我们假设有这样的场景:“余翔想要变得更美,所以他决定
去泰国做整容手术。”如果是这样的计划和目的,我们在生活中会如
何进行反驳(削弱)呢?估计有人会讲“泰国手术技术太落后,整形
后的鼻子风一吹就歪了”,这叫计划本身不能达到目标。假设计划可
达到目标,可能又有人会讲“泰国做手术前后期成本和机会成本太大
,余翔承受不了”,这叫计划本身不能被执行。我们再假设计划可执
行,并且计划可达到目标,可能又有人会说“做完手术之后是美了,
但是可能就不长胡子了,这是余翔不能承受的痛”,这叫计划有副作
用。基本上,“计划达不成目标,计划不可被执行,计划有副作用”
是“从计划到目标”出题套路的大多数答案模式。

此题的D选项讲大多数居民把长期存款存入特别账户(special a
ccount),因此达不成文中所描述的目标(plan to encourage citi
zens of Levaska to increase the amount of money they put int
o savings),因为转移了存款并没有增加存款。

7.句子作用题(黑体题)

Astronomer: Observations of the Shoemaker-Levi comet on its collision


course with Jupiter showed that the comet broke into fragments before
entering Jupiter's atmosphere in 1994, but they did not show how big those
fragments were. Nevertheless, some indication of their size can be inferred
from spectrographic analyses of Jupiter's outer atmosphere. After the
fragments' entry, these analyses revealed unprecedented traces of sulfur.
The fragments themselves almost certainly contained no sulfur, but
astronomers believe that the cloud layer below Jupiter's outer atmosphere
does contain sulfur. Since sulfur would have seeped into the outer
atmosphere if comet fragments had penetrated this cloud layer, it is
likely that some of the fragments were at least large enough to have passed
through Jupiter's outer atmosphere without being burned up.

In the astronomer's argument, the two portions in boldface play which


of the following roles?

A. The first is a claim that the astronomer seeks to show is true; the
second acknowledges a consideration that weighs against the truth of that
claim.

B. The first is a claim that the astronomer seeks to show is true; the
second provides evidence in support of the truth of that claim.

C. The first and the second are each considerations advanced in support
of the conclusion of the argument.

D. The first provides evidence in support of the conclusion of the


argument; the second is that conclusion.

E. The first is a circumstance for which the astronomer seeks to provide


an explanation; the second acknowledges a consideration that weighs against
the explanation provided by the astronomer.

解析: 此题考查黑体句的作用。如前所述,逻辑文章中句子的
作用分为:背景、前提、结论、中间性结论、与结论方向相反的前提
和与结论方向相反的结论。因此,梳理清楚六个逻辑元素之间的区别
后,做此类考题对学生而言不在话下。

句子作用题的基本解题步骤为:(1)判断两个黑体句之间的逻
辑关系是顺承还是转折。(2)判断黑体句所起作用是六大逻辑元素
中的哪一类。

将上述步骤应用于此题:

解题步骤1:两个黑体句分别描述了两个现象,合在一起推出结
论,因此两句话之间不存在转折关系。

解题步骤2:通过意思来判断,得出文中最后一句话的主句(it
is likely that some... being burned up)是结论(conclusion)
,第一个黑体句描述了一个现象(现象本身可以是背景或前提),第
二个黑体句开头的Since提示该句是前提。

A项:后半句weight against 提示“转折逻辑”,据此可排除本


项。

B项:后半句意思为第二个黑体句支持第一个黑体句。即使没有
学习过逻辑六大元素,也知道从句的作用是服务于主句的,所以此项
肯定不正确。

C项:没有发现错误点,两个现象被认为是用于支持结论的因素
,至少不能说错,留着此项待比较。

D项:认为第一句话是支持结论的论据(属于前提),据此排除
此项。

E项:后半句weight against 表明此项可排除。

此题答案为C项。

8.其他类

Yeasts capable of leavening bread are widespread, and in the many


centuries during which the ancient Egyptians made only unleavened bread,
such yeasts must frequently have been mixed into bread doughs accidentally.
The Egyptians, however, did not discover leavened bread until about 3000
B.C. That discovery roughly coincided with the introduction of a wheat
variety that was preferable to previous varieties because its edible kernel
could be removed from the husk without first toasting the grain.

Which of the following, if true, provide the strongest evidence that the
two developments were causally related?

A. Even after the ancient Egyptians discovered leavened bread and the
techniques for reliably producing it were well known, unleavened bread
continued to be widely consumed.
B. Only when the Egyptians stopped the practice of toasting grain were
their stone-lined grain-toasting pits available for baking bread.

C. Heating a wheat kernel destroys its gluten, a protein that must be


present in order for yeast to leaven bread dough.

D. The new variety of wheat, which had a more delicate flavor because
it was not toasted, was reserved for the consumption of high officials when it
first began to be grown.

E. Because the husk of the new variety of wheat was more easily
removed, flour made from it required less effort to produce

解析: 此题让考生找一个选项,这个选项可以支持文中两个事
件的因果联系逻辑。我们必须先找到具体事件。

原文前两句话(Yeasts capable of leavening bread ....Egyp


tians, however, did not discover leavened bread until about
3000 B.C)描述“发酵面包”的事件;之后的句子描述大麦(wheat
)。所以,我们需要找“描述‘发酵面包与大麦’之间的关系的选项
”。

A项提到发酵面包。

B项说了烤面包相关。

C项提到大麦之后又提到发酵面包。

D项提到大麦被消耗。

E项讲了大麦的壳。

只有C项两个事件都提到,最符合题目所问,因此得出答案为C项

本章小结

本章核心测试题

1. 逻辑六大元素是哪些?

2. 何为推理链(argument)?

如答案不能脱口而出,请回读本章。
第二章 归纳题

一、归纳题的解题要点

位于题目之“上”的为原文,题目之“下”为选项,“从下推上”为推理
链题,“从上推下”为归纳题。

首先讲解归纳题,因为此题型可被认为考试中占比最大的推理链题
的基础练习。归纳题在考试中往往会出现1道题左右,并非属于难题题
型。归纳题在前文粗略地介绍过,解题过程类似于阅读题,即从原文段
落中概括总结出选项,可简单理解为“缩小版”阅读题。

归纳题考查学生的客观性思维,即考生是否会在处理信息的过程中
加入自己主观的、带有某些情感色彩或偏向性(biased)的错误理解。
从客观性角度出发,对一件事,我们只能知道这件事本身,以及事件的
前提假设。

例如“小明在GRE考试中考了335分”这件事,我们只能知道:事件
本身是小明考了335分;事件的前提是小明去参加了GRE考试,世界上
有小明这个人,世界上有GRE考试。以前提为起点,我们可否“往后
推”,推出“小明很聪明”,“小明很懂GRE”,“用小明的方法别人也可以
考出GRE高分”,“小明在复习GRE时很努力”,“小明考试结束之后很高
兴”等?显然是不可以的。

* “小明很聪明”不可被推出,因为小明可能是考场中蒙对了很多
题,考335分纯粹是运气好。

* “小明很懂GRE”不可被推出,因为小明可能是因为考试当天碰到
了很多之前正好看到过的题,考高分不是因为懂考试本身。

* “用小明的方法别人也可以考出GRE高分”不可被推出,因为小明
可能智商很高加上英语很好,他得到GRE高分可以几乎没有方法,或他
的方法不适合其他人。
* “小明考试结束之后很高兴”不可被推出,因为可能小明目标是338
分,这次考试没有达到他的目标,他反而很郁闷。

相信各位考生读到这里,会对客观性思维有一定的理解。面对一件
事情,只能以事件本身的信息加上事件的前提为基础解题,不可借助自
己的经验、专业背景、偏见喜好或其他主观性因素为事件添加新元素、
新话题。

综上所述,大多数归纳题的解题步骤和解阅读题类似。步骤:
(1)读懂原文,先概括出原文中一共说了哪些不同的事件。(2)思考
之前总结的不同事件,看是否可总结出一个说法。(3)看选项是否出
现了新话题或新元素,如果有,则不选。为什么上述解法只针对大多数
归纳题?因为归纳题里面也会有题型细分,具体在后文详述。总之,记
住解逻辑题的根本原则——“题目问什么,就回答什么”。

二、归纳题的解题能力(逻辑题的基础能力)——如何读懂句

对于所有GRE逻辑题,中国考生可能会碰到的困难点,通常有以下
五点:

(1)根本读不懂句子。

(2)读懂了内容记不住事件。

(3)特别容易往后推理。

(4)没有全部读完选项的情况下,以为选项所述内容和自己所想
内容一致。

(5)读选项的过程中,毫无意识的情况下改变了原文事件内容。

逻辑题和阅读题需要读懂句子的能力,所以下面特意针对如何读懂
句子,来给大家分享一些技巧和经验。

“读句子”的先期思维铺垫:
1. “读句子”的目标是做到逻辑通顺地一遍把句子读懂。

2. 灵活地翻译英语单词。

3. 边阅读边总结边预测后文走势。

4. 逻辑通顺优于翻译精准。

5. 以“主体+行为”为单位,把长句子切割成不同的事件来记忆。

6. 读到“行为”时,把其描述的主体词补全。

7. 读到指代性词汇时一定要回找其指代的内容。

8. 注意句子与句子间的“等价词汇”。

9. 用逻辑推测读不懂的局部词汇。

10. 读完长句子后,只需要记住此句主要描述的“主体+行为”。

上述“读句原则”1-4的核心词:灵活、逻辑预测、尽量不回读。

解读: 我们读中文基本就是从左至右地扫读,碰到稍难理解的
内容时,才会局部地回读一下。美国人读英语句子时,不会刻意进行
语法分析,也不会先理解后半句,再回头理解前半句,不会进行特别
复杂的阅读过程处理。因此,如果考生在读句子时刻意地分析语法结
构,读到定语从句的时候,先读定语从句内容,再带定语内容返回主
句,这个过程显然就不正确了。就像模仿美国人讲话来练好口语,模
仿美国人的阅读过程,我们才能读懂英语句子。

要灵活地翻译英语单词是指灵活理解英语词汇意思。灵活理解英语
词汇意思可理解为“只要别把所要理解的英语词汇解读成这个词的反义
词,只要逻辑通畅,怎么容易理解就怎么翻译”。

一个英语单词在词典中有多个中文意思,因为在不同语境中可表达
不同的含义。但是,大家见过一个英语单词同时有两个相反意思的吗?
有,但很少。在整个GRE题库中只有2个左右的词汇同时有相反的意
思,比如condescending(两个相反的意思是:自大,即showing superior
attitude towards others;谦虚,即modest, unassuming)。基本上GRE考
试的词汇只有一个方向,要么是贬义词,要么是褒义词。

所以只要不往相反的方向去翻译,结合语境怎么能方便地让我们从
逻辑上理解整句话的意思,就怎么翻译。

例1 Leo Tolstoy wrote many works of nonfiction and professed a


preference for these explorations of ethics and religion compared with his
novels and short stories. The fiction writer in him, however, was hard to
suppress. Handi Murdd is a short with the breadth and power of an epic, with
vivid characterization and intense storytelling that sweep the reader away.

句1:Leo Tolstoy wrote many works of ... with his novels and short
stories.

解读: 我们使用逻辑线索来理解这句话,and前后肯定是说类似
事情,因此之前说Leo Tolstoy写了很多非小说作品,看到and之后就
带着“写非小说”的预测内容再读下去。一个动词如果没有dis-、un
-、a-、de等含有负面意思的词根或词缀,我们可把这个动词全部替
换成do,即语境或逻辑线索决定这个词的意思。所以我们可以把prof
essed a preference for 理解为“表达了一个喜欢”,而不是较难
理解的“公开声称了一个倾向”。假设不认识ethics,也可以通过an
d 后的内容进行推测。因此,首句可读成“Leo Tolstoy写了非小说
,喜欢宗教相关的非小说”。

句2:The fiction writer in him, however, was hard to suppress.

解读: 什么叫“fiction writer in him”?单个词可能都认识


,但多个词组合在一起就不一定懂了。不懂的原因还是和没有进行逻
辑推测相关。在此句中however一词表明本句和之前一句话取反,之
前说写非小说,现在就应带着“会写小说”的方向去理解这句话。带
着“会写小说”这层含义再看此句,“the fiction writer in him
”就可灵活地理解为“写小说的灵魂”、“写小说的特征”,或“写
小说的才能”等。“写小说的才华很难被克制住”可灵活简单地记成
“会写小说”。

句1、句2可读成:Leo Tolstoy写了非小说,喜欢宗教相关的话
题,他也写小说。

句3:Handi Murdd is a short with the breadth and power of an epic,


with vivid characterization and intense storytelling that sweep the reader
away.

解读: 我们带着逻辑去阅读,前句说会写小说,句子3和前句之
间没有转折词,则也应围绕会写小说的方向继续描述。所以可读成“
HM是一个短的小说,这个小说和英雄诗有点沾边”。vivid characte
rization可理解为“内容挺具体”;假设读不懂“intense storytel
ling that sweep the reader away”这半句中的“sweep the reade
r away”,但整个语境是在说这本书挺好,所以可理解“讲故事方式
让人印象深刻”。

我们阅读句子的目标是把话题读出来,并不是在做严谨的翻译。因
此,请大家务必牢记读句子的重要原则——灵活地、带着逻辑预测地去
理解句子,而不要生搬硬套词典中的中文释文。

“读句原则”5、6的关键词:“主体+行为”;句子切割;逻辑补

解读: 无论是逻辑题还是阅读题,其正确答案必须是文章中谈
论的事情,所以要确定原文中共谈论了多少不同的话题。所谓一个话
题,就是一个“事件”。一个事件由主体和行为组成。这件事围绕的
中心即主体,这个主体具体做的动作或拥有的属性统称为“行为”。
学生读不懂英语长难句的很大一个原因是想一下把这个句子在描述什
么都读出来,这势必增加理解难度。正确的方法是在一个英语句子中
“切割事件”式地一部分一部分去理解句子。一旦读出了句子局部的
“主体+行为”,就表示已经读出一个事件了,这时可以停顿一下,
然后继续找下个事件(主体+行为)。有同学可能会问:“一句话中
如果有多个事件,那么这句话真正想要表达的是什么?”一句话的核
心是什么?是主谓宾,如果谓语是不及物动词,则就是“主谓”。一
句话可能表达很多事件,但通常核心事件就是“主谓(宾)”所描述
的那件事。为什么是“通常”?因为如果碰到“it is”做主谓,那
当然不可能是核心。主谓如果有明确的意思,则句子的重心就是“主
谓(宾)”。

“逻辑补全”的意思是指,如果主体离行为特别远,为了逻辑通
畅,在读行为前,可以加一个主体进去,使其构成“主体+行为”完
整的事件。

例2Smithtown University's fund-raisers were successful in their


contacts with potential donors who had never given before about as
frequently as were fund-raisers for other universities in their contacts with
such people.

解读: 以“主体+行为”为目标地切割这句话,可分为下面三个
成分:

(1)Smithtown University's fund-raisers were successful

(2)in their contacts with potential donors who had never given bef
ore

(3)about as frequently as were fund-raisers for other


universities in their contacts with such people.
“Smithtown的募捐人是成功的”是这句话告诉我们的第一个事件。
第一个事件逻辑不完整,因为需要结合后续内容才能知道哪方面成功
了,前面两个成分应结合在一起去理解,即“Smithtown的募捐人在让那
些过去从来不捐款钱的人捐钱方面是成功的”。

读到as frequently as 时,逻辑意思上有一个断点,即“什么东西”的


频率是一样,为了读出“主体+行为”要进行逻辑上的主语补全,所以第
三个成分会读成“(成功接触比率)是一样的”。和谁一样呢?再读下去
发现是“和其他学校的募捐人一样”。于是这句话连起来就是:
Smithtown的募捐人在让那些过去不捐钱的人捐钱这件事上的成功比率
和其他大学的募捐人差不多。

例3 A minor criticism of the book, which is tempered by an


understanding of the difficulty of doing direct research in Hong Kong's, is
that So relied on secondary sources to tell the story of Hong Kong's political
development, with previous histories of the period playing a large role in his
research.

解读: 此句可切割为以下四个成分:

1. A... criticism...

2. which is tempered by an understanding of the difficul


ty of doing direct research in Hong Kong's

3. So relied on secondary sources to tell the story of H


ong Kong's political development

4. with previous histories of the period playing a large


role in his research

成分一:“读到一个批评”之后,没有立刻出现“这个批评怎样(行
为)”,就先搁置。

成分二:读到which,需要明确地理解指代性词汇的对象,所以会
读成“这个批评的严重性被一种理解缓和了,这个理解是直接做调查在
香港挺难的”。“这个理解”是为了形成“主体+行为”,构成逻辑完整的事
件而补全的词。

成分三:直接理解内容显示逻辑不完整,补一个主体,所以成分三
读成:(这个批评的内容是)So依靠二手的来源讲香港政治发展的事
情。

成分四:假设学生读不懂这里,但是整句话的核心主谓我们已经读
出了,成分四即使读不出也不重要。

此句可理解:有一个批评,这个批评被一个理解缓和了,这个批评
的内容是So这个人依靠间接来源去描述香港政界的故事,并且之前也是
类似方式(而导致的批评)。

请考生注意上述句子中的“逻辑补全”“灵活翻译”“逻辑推测”技巧的
应用。

读句原则7~9的关键点:指代性词汇需要回找、等价词。

解读: 读句子的时候,考生最担心读着读着就懵圈了。因此,
指代性词汇不能读到后让其一闪而过,而是要思考其具体指代的对象
是什么。指代性词汇往往是代词和定冠词。

文章的作者往往会在一句话中用不同的词汇来表达相同的对象。
在阅读句子的过程中,要察觉出表达相同意思的不同词汇,不要把其
当成不同的对象。

例4 (摘自GRE阅读文章)

This position represents the new mainstream called social


constructivism. The constructivists gain acceptance by misrepresenting
technological determinism: technological determinists are supposed to
believe, for example, that machinery imposes appropriate forms of order on
society.
解读: 读到the constructivists时,考生要判断出它与social
constructivism的指代关系,不要认为又出现了新名词。

“technological determinists are supposed to believe, fo


r example, that machinery imposes appropriate forms of order
on society”这句话中machinery 指的就是“科技”,不要认为又出
现了一个新的描述对象。

读句原则10的关键点:读完一句话后只记一件事情。

解读: 一个长句子会包括很多事,但只有这句话的“主谓(宾
)”才是核心事件(假设这句话不是“it is”这种形式)。因此,
读完长句之后,可以把这句话浓缩成很短的“主体+行为”来降低记
忆难度。

例5 For similar cars and drivers, automobile insurance for collision


damage has always cost more in Greatport than in Fairmont. Police studies,
however, show that cars owned by Greatport residents are, on average,
slightly less likely to be involved in a collision than cars in Fairmont. Clearly,
therefore, insurance companies are making a greater profit on collision-
damage insurance in Greatport than in Fairmont.

解读: 上文三句话,可简单记成如下三件事情:

(1)G保险金更贵。

(2)G碰撞少。

(3)G更赚钱。
所以实际阅读文章的节奏是“一边预测下文,一边总结正在读的句
子”。

三、归纳题的题型差异

逻辑原文在题目的上方,称为“上”,选项在题目下方,称
为“下”,“上推下”为归纳题。归纳题的普遍解法是不可从文章中总结归
纳得出新元素、新事件,或新程度。但同属“上推下”出题形式,归纳题
之间也会有部分差异,应看清题目所问,从提问来分析正确答案的方
向。

“上推下”题型可细分为:

1. 必定是真类考题(must be true)

2. 最能支持类考题(most strongly support)【推理题(infer)】

3. 其他形式“上推下”题

4. 归纳题中的“数学题”

解读: 如果出题方式是“If the information given as facts


above is true, which of the following must also be true”,
那么选项一定可以从文章中推测出来,“must be”提示不允许有反
例出现。

如果出题方式是“The information above best supports whic


h of the following hypotheses”(文章最能支持下列哪种假说)
,需不需要这个假说里的词汇全部由文章内容所组成?当然不需要。
类似题型的答案一定是选项描述事件和原文的事件一致,但是允许出
现新词汇,只要选项的整体意思能从原文推出即可。

“下面哪个选项可以被推理出来”类问题的答案允许有反例存在
,但是不能选择无法判断的事件,答案可以是原文中描述事件的一个
子集。
其他类的“上推下”考题较少,如果出现,考生们也要注意和一
般归纳题的解题差异。“If the statements above are true, then
it is also possible that...”问哪个选项可能对,只要和原文不
冲突的选项都可能对,所以类似这样的题型就不能再使用发现新词汇
就排除特定选项的方法了。

出题方会如何灵活地出题无法预测,总之考生们应注意从题目的问
法上去分析,千万不要固化自己的思维。上述三种题型中,只有第一种
需要找到一个没反例的选项,其他题型找一个相对最能推出,但不需要
百分之百没有反例的选项。

例1 The theory of military deterrence was based on a simple


psychological truth, that fear of retaliation makes a would-be aggressor
nation hesitate before attacking and is often sufficient to deter it altogether
from attacking. Clearly, then, to maintain military deterrence, a nation would
have to be believed to have retaliatory power so great that a potential
aggressor nation would have reason to think that it could not defend itself
against such retaliation.

If the statements above are true, which one of the following can be
properly inferred?

A. A would-be aggressor nation can be deterred from attacking only if it


has certain knowledge that it would be destroyed in retaliation by the country
it attacks

B. A nation will not attack another nation if it believes that its own
retaliatory power surpasses that of the other nation.

C. One nation's failing to attack another establishes that the nation that
fails to attack believes that it could not withstand a retaliatory attack from the
other nation.

D. It is in the interests of a nation that seeks deterrence and has


unsurpassed military power to let potential aggressors against it become
aware of its power of retaliatory attack.
E. Maintaining maximum deterrence from aggression by other nations
requires that a nation maintain a retaliatory force greater than that of any
other nation.

解析: 文章内容:(1)军事威慑与怕报复之间的关系;(2)
国家要有足够报复能力让潜在的入侵国相信打不过它。

A项:出现了only,表示此项不能选,因为原文中没有绝对化词
汇出现。文章是说有报复能力会让潜在入侵国不敢动手,但也可能有
其他原因让潜在入侵国不敢动手,比如某国受第三国保护,虽然某国
没有足够报复能力,但潜在入侵国也不敢动手。

B项:报复力更强。文章中只说会避免被其他国家侵犯,没有说
报复力更强的国家是否会入侵别国,不可选此项。

C项:此项和A项描述的事件类似。但是别国不入侵,完全可以是
因为其他理由,并不只是因为此国的报复能力,比如这两个国家自古
以来关系甚佳。

D项:“符合国家利益的是国家要有强的军事力量让潜在入侵者
意识到它的报复能力。”有同学可能会说“符合国家利益”这些词原
文中找不到。这种想法完全不正确。逻辑题不要求选项词汇和文章词
汇一一对应。比如选项用一个词能概括原文中的两句话,这个词虽然
没有明确说出来,但通过逻辑是可以接受的,则被认为是正确的。因
此,我们做题时,不能追求任何的形式,比如只看选项的词是否和原
文一样,而是要通过逻辑去思考。“让国家不被入侵”这件事情当然
可以被认为是“符合国家利益”的,故此项任何一处都不能说是错误
的,放着此项待选。

E项:出现两个最高级,肯定不正确,排除此项。

答案为D项。

例2 People with a certain eye disorder are virtually unable to see in


moderately bright light, which seems to them unbearably intense, since the
cells of their retinas are overwhelmed by moderately bright light. These
people do, however, show normal sensitivity to most components of dim
light. Their retinal cells are also not excessively sensitive to red components
of moderately bright light.

The information above best supports which of the following hypotheses


about people with the disorder described, if they have no other serious visual
problems?

A. In all moderately dim light in which people without the disorder can
read large print, people with the disorder cannot read such print.

B. In an otherwise darkened concert hall, these people will see a dimly


illuminated red exit sign more clearly than small dim white lights that mark
the aisles.

C. These people typically see more acutely at night and in dim light than
do most people who do not have the disorder.

D. Eyeglasses that are transparent to red components of light but filter


out other components of light help these people see in moderately bright
light.

E. These people perceive colors other than red in the same way as do
most people who do not have the disorder.

解析: “hypotheses about people with disorder”表明选项


和原文是一个方向即可成为备选答案,不要因为某选项出现新词汇而
排除这个选项。

原文描述三件事:(1)有眼病的人正常亮度下看不清;(2)暗
的光线里面可看见;(3)正常亮度里看得见红色光线。

在做所有的GRE逻辑题过程中,事件描述的范围一定要多加关注
,比如这道题中的范围有正常亮度和暗的光线。
A项:原文中讲有眼病的人在光线暗的环境中看得见,所以此项
和原文内容冲突。

B项:在暗光线里面可看得见,在正常亮度下有眼病的人看得见
红色光线,但在暗光线下有眼病的人对红色光线会有什么样的反应,
文章没有告诉我们,所以排除此项。

C项:暗光线中,有眼病的人看得见。从原文得不出这些人比正
常人能够看得更清楚。

D项:“过滤掉红色光线之外的其他光线的眼镜可让有眼病的人
在正常亮度中看得见。”这项和原文描述的事件三是一个方向,可放
着待选。不可因为eyeglass在文章中没有出现过而直接排除此项。

E项:从原文中我们只知道看得见红色光线,接收红色光线的方
式我们不清楚,所以E项不能选。

D为答案。

例3 In Patton City, days are categorized as having heavy rainfall(more


than two inches), moderate rainfall(more than one inch, but no more than
two inches), light rainfall(at least a trace, but no more than one inch), or
no rainfall. In 1990, there were fewer days with light rainfall than in 1910
and fewer with moderate rainfall, yet total rainfall for the year was 20 percent
higher in 1990 than in 1910.

If the statements above are true, then it is also possible that in Patton
City

A. the number of days with heavy rainfall was lower in 1990 than in
1910.

B. the number of days with some rainfall, but no more than two inches,
was the same in 1990 as in 1910.

C. the number of days with some rainfall, but no more than two inches,
was higher in 1990 than in 1910.

D. the total number of inches of rain that fell on days with moderate
rainfall in 1990 was more than twice what it had been in 1910.

E. the average amount of rainfall per month was lower in 1990 than in
1910.

解析: 原文里面说了三件事情:(1)天气可以被归类为四类(
没有雨;降雨量小于1英寸;降雨量1~2英寸;降雨量2英寸以上)。
(2)1990年下小雨和中雨的天数少。(3)1990年的降雨总量更多。

请问考生“通过1990年下小雨和中雨的天数少,以及1990年的降
雨量多,可否推出1990年降大雨的天数更多”?答案是不一定。导致
1990年总降雨量更多有其他理由,比如1990年下大雨的天数和1910年
一样多,但是因为大雨是降雨量2英寸以上,文章中并没有说明最高
值是多少,所以1990年的大雨可以降10英寸,而1910年只有3英寸,
这样也会导致1990年的总降雨量更多。即1990年降大雨天数可以少于
1910年,只要1990年的单次降雨量更大即可。

此题问哪个选项可能对,表示只要和文章不冲突的选项即可成为
答案。因此,五个选项一定是由四个和文章冲突的选项和一个推出或
推不出但和原文不矛盾的选项组成。

A项:正如上文所分析的,1990年可能大雨的降雨天数更少,但
只要降雨量大到一定值,即可导致1990年总降雨量大于1910年的。

B和C选项:1990年下雨但不超过2英寸,那只有下小雨和中雨。
文中倒数第二行明确地说明是“fewer days”,所以B和C两项全错。

D项:假设1990年和1910年的下中雨天数一致,中雨降雨量是开
区间1~2英寸,那么即使两年的下中雨天数一样,也不会得出某年降
雨量是另一年降雨量的两倍这一结论,更何况文章告诉我们1990年下
中雨的天数更少,所以D项也和原文冲突。

E项:平均月降雨量是年降雨量除以12后得到的,所以年降雨量
更高的1990年不可能月平均降雨量更低,E项也和原文冲突。

此题答案选A。

归纳题中的“数学题”

从归纳题的内容来看,有些归纳题带有数学概念。比如原文中告诉
我们某辆车的时速以及行驶时间,则行驶距离是可计算的,这时如果在
选项中出现“距离”不能认为是原文没有提及而被当作新概念。可以通过
常识(包括数学知识)推测出来的内容都不算新概念。

例4 Fact: Asthma, a bronchial condition, is much less common ailment


than hay fever, an allergic inflammation of the nasal passages.

Fact: Over 95 percent of people who have asthma also suffer from hay
fever.

If the information given as facts above is true, which of the following


must also be true?

A. Hay fever is a prerequisite for the development of asthma.

B. Asthma is a prerequisite for the development of hay fever.

C. Those who have neither hay fever nor asthma comprise less than 5
percent of the total population.

D. The number of people who have both of these ailments is greater than
the number of people who have only one of them.

E. The percentage of people suffering from hay fever who also have
asthma is lower than 95 percent.

解析: 原文可用表示集合的韦恩图表示:大圈是hay fever,小


圈是ashma。95% ashma是两者交集。
从原文中并不能得知两种疾病谁先谁后发生,因此A和B项中含有
prerequisite,都是错误的。

C项:“没有得病的人数是5%”,此项在问上述两个圈之外的空
间是多大,外框多大,我们无法得知,故此项不正确。

D项:“两项之间的交集比任何一项都大”,如上图所画,交集
不可能比“hay fever”这一元素更大,此项不正确。

E项:我们得知hay fever的总量比asthma大,二者的交集占了as
thma 的95%,那么相同大小的交集占hay fever的比例肯定低于95%,
所以E项正确。

例5 From 1980 to 1989, total consumption of fish in the country of


Jurania increased by 4.5 percent, and total consumption of poultry products
there increased by 9.0 percent. During the same period, the population of
Jurania increased by 6 percent, in part due to immigration to Jurania from
other countries in the region.

If the statements above are true, which of the following must also be
true on the basis of them?

A. During the 1980's in Jurania, profits of wholesale distributors of


poultry products increased at a greater rate than did profits of wholesale
distributors of fish.

B. For people who immigrated to Jurania during the 1980's, fish was less
likely to be a major part of their diet than was poultry.

C. In 1989 Juranians consumed twice as much poultry as fish.

D. For a significant proportion of Jurania's population, both fish and


poultry products were a regular part of their diet during the 1980's.

E. Per capita consumption of fish in Jurania was lower in 1989 than in


1980.

解析: 文章描述四件事情:(1)鱼的消耗增加4.5%。(2)家
禽的消耗增加9%。(3)人口增加6%。(4)人口多部分是因为移民的
到来。

A选项:从文中四件事情中,不可能推出利润多少,所以排除本
项。

B选项:“鱼可能成为主食的可能性较小”,在原文中我们只知
道鱼和家禽的消耗增加了。人们是把鱼当主食吃还是当零食吃?是偶
尔吃还是经常吃?偶尔吃的东西会成为主食,还是经常吃的东西会成
为主食?皆有可能。所以B项的“major part of the diet”不可被
确定地推出来,排除此项。

C选项:文章中描述的是百分比,属于相对值。此项描述的是绝
对值,如果此项正确那只得假设鱼和家禽的基值一样,但文章中没有
告诉我们基值的信息,所以排除此项。

D选项:排除此项的理由和B项类似。

E选项:“人均鱼肉消耗量在1989年更低”。1989年鱼的人均消
耗量= [1980年鱼的消耗量×(1+4.5%)]÷[1980年人口数(1+×6%
)]
分母比分子变化更大,所以E项为答案。

例6 The total market value of real estate in Altonville has steadily


declined over the past four years. This decline has meant that the overall
figure on which the city's property tax is based—the assessed value of that
real estate—has also declined. Moreover, the percentage of assessed value
that was paid as property taxes to the city did not change during this period.

The information above most strongly supports which of the following?

A. Money collected from property taxes provided most of Altonville's


revenue during the past four years.

B. The percentage of Altonville's overall revenue that was collected


from property taxes did not change over the past four years.

C. Altonville officials had proposed raising property tax rates during the
past four years but never did so because of strong opposition from property
owners.

D. The total amount of revenue that Altonville has collected from


property taxes has declined over the past four years.

E. During the past four years, Altonville officials also did not increase
tax rates on other sources of revenue such as retail sales or business profits.

解析: 文章讲两件事情:(1)房地产估值小了。(2)房产税
率不变。

通过这两件事情,自然可推理出交的房产税额(房地产估值×房
产税率)变小了。

A项:“房产税贡献了A城市大部分收入”,在原文中我们只能得
知房产税会给A城市提供收入,但所提供的收入是否占大多数无法得
知,此项排除。如果此项把most改为part,则A项没有出错的地方。
B项:“房产税占总收入的比例没变”,此比例是“房产税÷A城
市总的收入”,但A城市的总收入为多少我们无法得知。虽然房产税
变少了,但其他的税收可能变多、变少或持平。所以排除此项。

C项:从原文中根本不可能得知官方做了什么事情,排除此项。

D项:“房产税税额少了”,正确。解归纳题时,在总结完所有
事件后,可先想想这些事件可以让我们推理出什么,答案并不一定和
自己推出的内容一致,但建议考生做下推理,因为这样会增加对正确
答案的敏感度。

E项:排除该项的理由和C项一致,官方会做什么事情我们无法得
知。

答案为D。

补充知识:对程度词的理解

在GRE逻辑考试中,对程度词掌握不佳会大大影响逻辑推理的正确
性。most、majority、some、certain、many、a few、nearly all等程度词
内容上有何不同?

1. some=a few = certain = many,都表示超过零个,但不表示超过


50%的量。

2. majority =most = nearly all,都表示超过50%的量。

例如:

句1:Some students get GRE 338. Some students go to Harvard.

句2:A few students get GRE 338. Many of them go to Harvard.

句3:Most students get GRE 338. A majority of them go to Harvard.

这三件事情表达的意思有何区别?哪件事情表达了GRE得338分即
可进入哈佛大学?
首先,三件事情都不能表达GRE考338分即可进哈佛大学,因为
most表示“最多”,超过50%。

其次,some表示“一部分,一些”,句1的两件事之间不一定产生交
集。句1和句2并不能推出有些学生GRE考338分,且去了哈佛大学,因
为a few和many都表示“一些”的意思。

most和majority表示超过50%,所以句3可表示有些学生GRE考338
分,其中有些学生考入哈佛大学。

四、重点归纳题练习

例1 That the application of new technology can increase the


productivity of existing coal mines is demonstrated by the case of Tribnia's
coal industry. Coal output per miner in Tribnia is double what it was five
years ago, even though no new mines have opened.

Which of the following can be properly concluded from the statement


about coal output per miner in the passage?

A. If the number of miners working in Tribnian coal mines has remained


constant in the past five years, Tribnia's total coal production has doubled in
that period of time.

B. Any individual Tribnian coal mine that achieved an increase in


overall output in the past five years has also experienced an increase in output
per miner.

C. If any new coal mines had opened in Tribnia in the past five years,
then the increase in output per miner would have been even greater than it
actually was.

D. If any individual Tribnian coal mine has not increased its output per
miner in the past five years, then that mine's overall output has declined or
remained constant.

E. In Tribnia the cost of producing a given quantity of coal has declined


over the past five years.

解析: 原文两件事:(1)科技可提高产量。(2)在没有新矿
开采的情况下,人均产量翻倍。

归纳题中,总结出原文的事件之后,先思考下通过这些事件可否
推测出结论的方向。

A项:人均产量×总工作时间×矿的数量=总产量,因此A项正确

B项:科技因素可提高产量,但并不是只有科技因素会让产量上
升,也可以有其他因素,比如矿工人数增加,因此B项不正确。

C项:新矿出现会让总产量上升,但并不影响人均产量。“人均
产量×总工作时间×矿的数量=总产量”,第三个因素上升或下降只
能影响总产量,并不能影响第一个因素。因此,C项不正确。

D项:其他因素也会导致总产量变化,比如矿工人数,故D选项错
误。

E项:“生产煤的成本”在原文中未提及,所以排除此项。

例2 Although most smoking-related illnesses are caused by inhaling the


tar in tobacco smoke, it is addiction to nicotine that prevents most smokers
from quitting. In an effort to decrease the incidence of smoking-related
illnesses, lawmakers in Sandonia plan to reduce the average quantity of
nicotine per cigarette by half over the next five years. Unfortunately, smokers
who are already addicted to nicotine tend to react to such reductions by
smoking correspondingly more cigarettes.

The information above most strongly supports which of the following


predictions about the effects of implementing the Sandonian government's
plan?
A. The average quantity of tar inhaled by Sandonian smokers who are
currently addicted to nicotine will probably not decrease during the next five
years.

B. Sandonian smokers who are not already addicted to nicotine will


probably also begin to smoke more cigarettes during the next five years than
they had previously.

C. The annual number of Sandonian smokers developing smoking-


related illnesses will probably decrease during the next five years.

D. The proportion of Sandonians attempting to quit smoking who


succeed in that attempt will probably decrease during the next five years.

E. The number of Sandonians who quit smoking during the next five
years will probably exceed the number who quit during the last five years.

解析: 题目问上文会支持哪个预测,既然是“预测”,就不追
求答案百分之百地可被推出,找一个最可能正确的选项即可。

原文描述的事件:(1)尼古丁让人戒不了烟。(2)计划把烟的
尼古丁含量减半。(3)人们抽更多的烟。

A项:通过“人们抽更多的烟”是否一定能推出尼古丁吸入量和
过去一样呢?显然不能,因为我们不知道抽的烟是否增加了一倍。只
有抽烟频率增加一倍,结合单支烟尼古丁含量减半,才能推出吸入的
尼古丁量和过去一样。但是此题不追求一定推出,因此A项待选。

B项:文章中讲述的是“已经对尼古丁上瘾的人群”,此项描述
的不属于此范围,肯定不正确。

C项:第三件事情告诉我们抽烟的人依然会抽非常多的烟,这和
患病人数会下降是反方向,故排除此项。

D项:从“吸烟的人会抽更多的烟”,无法推出人们会戒烟。同
理,E项也不可推出。
得出答案为A。

例3 A medieval manuscript called L contains all eighteen extant


tragedies by the Greek playwright Euripides. Of these, ten called the “select
plays,”are accompanied in L by ancient commentaries and also appear in
other medieval manuscripts; this group includes some of Euripides' best-
known works, including the Medea . The other eight, which appear only in L,
appear in alphabetical order, without commentary. The Electra is one of the
alphabeticals.

Which of the following can be reliably concluded on the basis of the


statements given?

A. Only Euripides' best-known works are accompanied by ancient


commentaries in extant medieval manuscripts.

B. The select plays are accompanied by ancient commentaries because


they were the best known of Euripides' works.

C. No commentaries were written about Euripides' Electra in ancient


times.

D. Euripides' Medea never appears in medieval manuscripts


unaccompanied by ancient commentary.

E. Euripides' Electra does not appear accompanied by a commentary in


any extant medieval manuscript.

解析: 此题考查的是考生对事件范围的归纳能力。归纳题的答
案一般不包括绝对化内容,除非原文中出现了绝对化概念。我们不可
能概括总结出一个原文中没有提到的极端程度词。原文中如果出现最
高级等描述程度词汇,必须记住这些绝对化内容。因此,我们在概括
事件之后,可再总结下事件的范围。

原文可记成:(1)L包括18个文件。(2)10个带评论,在其他
手卷中也出现,Medea是其一。(3)另外8个只有在L里面出现,没有
评论,Electra是其一。

A项:第二件事情描述10个带评论的里面包括一些很有名的作品
(some of the best known works),表明还有其他带评论的作品不
广为人知,因此A项很明显错误。

B项:写评论的理由无法得知,排除此项。

C项:我们只知道在手卷(manuscript)里面没有对Electra的评
论,没有看到评论不代表评论一定没有写过。因此C项的逻辑不严谨

D项:第二件事情讲Medea在其他手卷里面出现,那么在其他手卷
里面出现时是否有评论,无法得知,故D错误。

E项:因为Electra只在L里面出现,所以原文中描述的第三件事
情表明Electra没有评论就是表示在任何手卷中都没有评论。此项正
确。

例4 The quality of unrefined olive oil is not actually defined in terms of


acidity, yet extensive tests have shown that the less free oleic acid an
unrefined olive oil contains per liter, the higher its quality. The proportion of
free oleic acid that an olive oil contains is an accurate measure of the oil's
acidity.

If the statements above are all true, which of the following conclusions
is best supported by them?

A. When an olive oil is refined, the concentration of oleic acid in the oil
is reduced.

B. The quality of an unrefined olive oil can be determined only by


accurately measuring its acidity.

C. If an unrefined olive oil is intermediate in acidity between two other


unrefined olive oils, it will also be intermediate between them in quality.

D. Free oleic acid is the only acid that unrefined olive oil contains.

E. People who judge the quality of unrefined olive oils actually judge
those oils by their acidity, which the judges can taste.

解析: 文章主要在描述“酸度与油质之间的关系”。

A项:上文只告诉我们未精炼(unrefined)的油的情况,精炼后
会有什么变化是无法通过常识推理的,所以此项不正确。

B项:原文中没有出现过的动词、形容词、副词、名词和绝对化
词汇出现在选项中,则该选项不正确。绝对化词汇包含形容词最高级
、only、solely、merely等。此项出现了原文没有的绝对化词汇only
,不正确。

C项:描述了“酸度与质量的关系”,此项暂没发现错误。

D项:出现绝对化词汇only,排除。

E项:前半句没问题,逗号之后表示这样的“酸度可以尝出来”
,酸度的测量方式我们无法得知,非常可能是仪器测试得出,并不能
尝出来。不选E项。

得出C项为正确答案。

整个GRE考试都是挑错的过程,如果我们发现某选项挑不出错误,
就先放着此项,再与其他也挑不出错误的选项进行比较。这就是“批判
性思维”在考试中的运用。

五、补充归纳题练习

例1 Each December 31 in Country Q, a tally is made of the country's


total available coal supplies that is, the total amount of coal that has been
mined throughout the country but not consumed. In 1991 that amount was
considerably lower than it had been in 1990. Furthermore, Country Q has not
imported or exported coal since 1970.

If the statements above are true, which one of the following must also be
true on the basis of them?

A. In Country Q, more coal was mined in 1990 than was mined in 1991.

B. In Country Q, the amount of coal consumed in 1991 was greater than


the amount of coal mined in 1991.

C. In Country Q, the amount of coal consumed in 1990 was greater than


the amount of coal consumed in 1991.

D. In Country Q, the amount of coal consumed in 1991 was greater than


the amount of coal consumed in 1990.

E. In Country Q, more coal was consumed during the first half of 1991
than was consumed during the first half of 1990.

解析: 事件1:所有挖的煤减去使用的称为“T”(可简单理解
为煤的存货)。

事件2:1991年的存货少于1990年的。

事件3:自从1970年没有进出口。

从这三件事情中,我们可得知“1990年的存货+1991年煤的开采
量-1991年煤的消耗量=1991年的存货量”。由事件2得知1991年存货
更少,于是可推出1991的开采量没有当年消耗量大。

A项:无法得知1990年挖煤的量,此项没法判断。

B项:正如之前的分析,1991年煤的消耗量更多,所以才会动用1
990年存货,故而导致1991年的存货量变小,正确。
C项:无法得知1990年煤的消耗量,错误。

D项:原文中导致1991年的存货数量低于1990年的原因只可能是1
991年的生产量不如消耗量,导致会用到之前的“存货”,所以才会
有1991年存货量少于1990年存货量这一结果。D项仅描述消耗量的对
比,并不一定正确。在“必对(must be)考题中”,只要找到一个
反例能削弱选项,此项就不可选。在原文中只知道1991年的净存货量
是负的,所以才会用到之前的存货,但并不知道1991年的消耗量与19
90年的消耗量谁更大,因为原文并没有告知“多少煤被开采”。不知
道煤的生产量,所以无法判断煤的消耗量,此项错误。

E项:只知道1991年整年存货量的情况,不知道前半年的情况,
此项必错。

得出B项正确。

例2 Book Review: When I read a novel set in a city I know well, I must
see that the writer knows the city at least as well as I do if I am to take that
writer seriously. If the writer is faking, I know immediately and do not trust
that writer. When a novelist demonstrates the required knowledge, I trust the
storyteller, so I trust the tale. This trust increases my enjoyment of a good
novel. Peter Lee's second novel is set in San Francisco. In this novel, as in his
first, Lee passes my test with flying colors.

Which one of the following can be properly inferred from the passage?

A. The book reviewer enjoys virtually any novel written by a novelist


whom she trusts.

B. If the book reviewer trusts the novelist as a storyteller, the novel in


question must be set in a city the book reviewer knows well.

C. Peter Lee's first novel was set in San Francisco.

D. The book reviewer does not trust any novel set in a city that she does
not know well.
E. The book reviewer does not believe that she knows San Francisco
better than Peter Lee does.

解析: 事件1:作者要至少和评论员一样熟悉城市。

事件2:描述的和我熟悉的一样我才相信。

事件3:Lee在书里面描述的旧金山和我熟悉的一样。

A项:出现了原文中没有出现的绝对化词汇(any),不可选。此
项把原文的范围扩大,归纳题的答案可以是原文的一个子集,但不可
能推出范围更大的事件。

B项:B项错在“knows well”这个程度上。书的作者不需要对城
市十分熟悉(knows well),按照原文的程度,只需要达到评论员对
于城市的熟悉程度即可。故此项错误。

C项:first一词是无中生有,肯定错误。

D项:针对评论员不知道的城市,原文中评论员会采取什么态度
,我们无法判断,故D项错误。

E项:原文中“know at least as well as I do”表示书的作者


对于城市的了解必须多于或等于评价员的知识,E项内容相当于是多
于或等于的关系中取多于的内容,故此项可被推出来。

得出答案为E。

归纳题允许有反例出来,但答案必须与原文事件相关,或是原文事
件的一个子集。

例3 The only way that bookstores can profitably sell books at below-
market prices is to get the books at a discount from publishers. Unless
bookstores generate a high sales volume, however, they cannot get discounts
from publishers. To generate such volume, bookstores must either cater to
mass tastes or have exclusive access to a large specialized market, such as
medical textbooks, or both.

Which one of the following can be properly inferred from the passage?

A. If a bookstore receives discounts from publishers, it will profitably


sell books at below-market prices.

B. A bookstore that caters to mass tastes or has exclusive access to a


large specialized market will have a high sales volume.

C. A bookstore that profitably sells books at below-market prices gets


discounts from publishers.

D. A bookstore that does not sell books at below-market prices does not
get discounts from publishers.

E. A bookstore that not only caters to mass tastes but also has exclusive
access to a large specialized market cannot profitably sell books at below-
market prices.

解析: 事件1:如果想低于市场价卖书(below-market price)


,唯一的方法只能是从出版商那里低价进货。

事件2:不大量采购就不能低价进货。

事件3:大量进货必须是要么书符合大众口味,要么在专门市场
有独家渠道。

总结出三件事情后,先分析下三者之间的关系。“想要低价卖,
唯一方法是低价进货”,那如果先有了低价进货的条件,书商一定会
低价出售吗?不一定,书商完全可以不打折,从而使自己的利润更高
。大量进货要么符合大众口味,要么有独家渠道进入特别市场。如果
有后两个条件,就一定大量进货吗?显然是不一定的。

根据上述分析,A和B项都可排除,C项合理,符合事件1的描述。
D项属于否命题,不正确。E项也无法推出。答案为C。
例4 An advertisement states: Like Danaxil, all headache pills can stop
your headache. But when you are in pain, you want relief right away. Danaxil
is for you—no headache pill stops pain more quickly. Evelyn and Jane are
each suffering from a headache.

Suppose Evelyn takes Danaxil and Jane takes its leading competitor.
Which one of the following can be properly concluded from the claims in the
advertisement?

A. Evelyn's headache pain will be relieved, but Jane's will not.

B. Evelyn's headache pain will be relieved more quickly than Jane's.

C. Evelyn's headache will be relieved at least as quickly as Jane's.

D. Jane's headache pain will be relieved at the same time as is Evelyn's.

E. Jane will be taking Danaxil for relief from headache pain.

解析: 事件1:所有头疼药能治疗头疼(注意绝对化词汇)。

事件2:没有别的药比Danaxil治疗更快(可理解为Danaxil和市
面上治疗最快的头疼药药效一样快)。

事件3:Evelyn吃Danaxil,Jane吃其他药。

总结出事件之后,先归纳三者之间的关系。Evelyn和Jane都吃药
了,那么根据事件1,两位都能药到病除。Evelyn吃了Danaxil,可否
表明Evelyn比Jane更快地被治愈呢?不能。因为Danaxil并不是市面
上唯一治疗最快的头疼药,事件2表明这种药和市面上治疗最快的头
疼药一样快。

A项:Jane不能治愈,和事件1矛盾。

B项:无法推出Evelyn更快地被治愈,只能推出不比Jane治愈的
速度慢。

C项:“两人被治愈至少一样快”,正确,符合之前的事件分析

D项:不一定是同时被治愈,除非二人同时间开始头疼,同时间
吃药,并且Jane吃市面上治疗最快的药,这才可能两人同时被治愈。

E项:“Jane会吃Danaxil”,在原文中没有这个信息,排除。

选项C正确。
第三章 假设题

一、对假设条件的理解

能让结论成立,但是论证过程没有提到的事件可被认为是“假设条
件”。假设条件在前提和结论之间架一座桥,让前提可过渡到结论,并
且假设条件没有固定的内容。

GRE逻辑题中考查的assumption在英语词典中的定义是“a statement
that is assumed to be true and from which a conclusion can be drawn”。从字
面意思上说是与前提一起,帮助结论成立。

假设条件可以有以下属性:

(1)一定是文章中没有说过的事件。

用英语来理解是“assumption is something one takes for granted and is


unstated”。注意unstated一词,如果考生发现某选项可以帮助推出结论,
但这项的内容在原文中明确看到过,则这项一定不是假设题的答案。

(2)和结论是一个方向。

因为要支持结论成立,假设条件在逻辑上和结论必须是一个方向。

(3)可简单地认为考题中的假设条件都是必要条件。

其实假设条件可分为必要性假设条件和充分性假设条件,然而GRE
逻辑考试中只考必要性假设题,因此我们也不需要了解这两种假设条件
之间的区别。既然是必要条件,则如果没有这个条件,结论必须无法得
出。

(4)答案倾向于和前提更相关的选项。
可以这么看假设条件:出题人认为前提推出结论有缺陷,逻辑不严
谨,需要新增其他条件才能得出结论,所以假设条件可理解为前提和结
论之间的过渡,解题过程中倾向于找一个和前提更相关的选项,因为要
帮助前提架桥,以便于推出结论。

(5)假设条件是支持题的一种答案形式。

支持题要选出可增强前提推出结论可能性的选项,所以假设条件可
能成为支持题答案的一种,但并不是所有支持题的答案都是假设条件。
假设条件必须是原文中没有提过的事情,而支持题答案可以是文章中已
经被描述过的事。假设题答案取非(negation)之后,会让原文结论得
不出来,然而支持题不需要选项取非导致结论取非。

(6)假设题的整体解法和推理链题的其他题型类似。

(7)实在做不出时,猜选主语前或谓语后带有否定词的选项。

例1 For similar cars and drivers, automobile insurance for collision


damage has always cost more in Greatport than in Fairmont. Police studies,
however, show that cars owned by Greatport residents are, on average,
slightly less likely to be involved in a collision than cars in Fairmont. Clearly,
therefore, insurance companies are making a greater profit on collision-
damage insurance in Greatport than in Fairmont.

Which of the following is an assumption on which the argument


depends?

A. Repairing typical collision damage does not cost more in Greatport


than in Fairmont.

B. There are no more motorists in Greatport than in Fairmont.

C. Greatport residents who have been in a collision are more likely to


report it to their insurance company than Fairmont residents are.

D. Fairmont and Greatport are the cities with the highest collision-
damage insurance rates.

E. The insurance companies were already aware of the difference in the


likelihood of collisions before the publication of the police reports.

解析: 原文描述三件事情:在Greatport保险更贵;在Greatpor
t碰撞少;保险公司在Greatport更赚钱。

假设要支持结论成立,因此我们要带着结论(保险公司在Greatp
ort更赚钱)看选项,同时不能忘记前提,如果只是记结论,万一选
项说到了前提相关的事件会来不及反应。

A项:“Greatport的车辆修理成本不会更贵”。车辆的修理成本
和“赔偿金”一样,都是由保险公司支付的。支付得越多,则保险公
司的利润越少,故此项可留下。

B项:“摩托车”是文章推理链之外的话题,不能选。

C项:“Greatport的居民更可能向保险公司报告事故”,报告越
多,则公司利润越小,此项和结论的方向相反,不可选。

D项:“两个城市有一个共同点”,结论是一个比较级,两个城
市的共性不能推出一个比较级,因此此项不正确。

E项:“保险公司意识到某件事”,如果只是意识到而没有采取
行动,则客观世界不会受影响。排除E项。

正确答案为A。

例2 The milk of many mammals contains cannabinoids, substances that


are known to stimulate certain receptors in the brain. To investigate the
function of cannabinoids, researchers injected newborn mice with a chemical
that is known to block cannabinoides from reaching their receptors in the
brain. The injected mice showed far less interest in feeding than normal
newborn mice do. Therefore, cannabinoids probably function to stimulate the
appetite.

Which of the following is an assumption on which the argument


depends?

A. Newborn mice do not normally ingest any substance other than their
mothers' milk.

B. Cannabinoids are the only substances in mammals' milk that


stimulate the appetite.

C. The mothers of newborn mice do not normally make any effort to


encourage their babies to feed.

D. The milk of mammals would be less nutritious if it did not contain


cannabinoids.

E. The chemical that blocks cannabinoids from stimulating their brain


receptors does not independently inhibit the appetite.

解析: 题目问推理链的假设,所谓推理链就是要找“前提与结
论”。

原文的结论为“cannabinoids probably function to stimulat


e the appetite”,可简单记为 “C和胃口”有关,导致结论成立的
原因即为前提。因此原文的前提是“给老鼠注射一种化学物质,以阻
止C影响大脑,导致老鼠没啥胃口”。

我们带着“C会影响胃口”来看选项,只要选项和这个事件有部
分交集,或者和文章之前所述某个话题沾边,此项即可待选。

A项可读成“老鼠不消化除了乳汁之外的其他东西”。答案的方
向是“C会影响胃口”,此项和所要的答案方向完全没有交集,属于
“不沾边”的选项。

B项读成“C是唯一会刺激胃口的物质”。我们在做逻辑题和阅读
题的时候,要有这样的思维:(1)要找一个出错概率小的选项。(2
)答案是比较后的结果。因为我们要找“C和刺激胃口”之间的关系
,此项和A比起来,肯定更好一些,因此B项优于A项。
C项读成“老鼠母亲不会鼓励小老鼠吃东西”,老鼠母亲和小老
鼠之间的关系和答案方向“C与胃口”无关,肯定不对。

D项读成“乳汁有没有营养”。此项和答案方向完全无关。

读完前四个选项,得出B项更好一些。如果E项也无关,则答案即
为B。但是同学们切记:(1)在没有看完五个选项之前,不要得出答
案,因为答案是比较后的结果,不把五个选项都比较完,无法得出最
好的选项。(2)排除一定不对的选项,不要对答案事先有明确的预
判。只要选项和答案方向有交集,就可以推理了,所以先放着和答案
的方向或原文提过的事件有交集的选项,然后再来对比哪个选项更好

E项读成“阻止C的化学物质本身不会导致没有胃口”,此项和答
案的方向肯定有交集。放着此项和B项进行对比。

在实际考试中时间压力特别大,很多情况下不允许考生进行深入
的思考和推理,因此在逻辑题和阅读题中如果要对比几个选项,考生
切记“尽量不选有最高级或比较级”的选项。(此招有风险,不是10
0%正确)所以此项答案选E。通过推理也可排除B选项。假设条件是必
要条件,必要条件意味着没有这个条件结论不能成立。如果把B项取
非,结论依然可以成立,则B项不是假设条件。不管cannabinoids是
不是唯一导致没有胃口的物质,这个唯一性不是结论要追求的必要条
件。是不是唯一导致没有胃口的物质结论一样可以成立,所以B项不
可选。

例3 Most household appliances use electricity only when in use. Many


microwave ovens, however, have built-in clocks and so use some electricity
even when they are not in use. The clocks each consume about 45 kilowatt-
hours per year. Therefore, households whose microwave oven has no built-in
clock use 45 kilowatt-hours per year less, on average, than do comparable
households whose microwave oven is otherwise similar but has a built-in
clock.

Which of the following is an assumption on which the argument


depends?
A. Households that do not have a microwave oven use less energy per
year, on average, than do households that have a microwave oven.

B. Microwave ovens with a built-in clock do not generally cost more to


buy than microwave ovens without a built-in clock.

C. All households that have a microwave oven also have either a gas
oven or a conventional electric oven.

D. Households whose microwave oven does not have a built-in clock are
no more likely to have a separate electric clock plugged in than households
whose microwave oven has one.

E. There are more households that have a microwave oven with a built-
in clock than there are households that have a microwave oven without a
built-in clock.

解析: 题目问“argument”,依然找“前提和结论”

读完此文之后把话题串记为“有无内置钟和消耗45度电”之间的
关系。(“话题”是理解和记忆全文和选项的最小单位。如何把读到
的句子浓缩为“话题”?推荐两招:第一,把句子总结为“什么东西
+怎么着”;第二,把句子总结为“XX与XX”之间的关系,XX优先为
名词)。

总结完原文推理链之后,带着“钟和45度电”的关系来读选项,
只要选项和话题部分沾边,就可以先放着进行对比。

A项有比较级,出现比较级就应思考比较对象和比较内容是否在
文章中出现。此项是有无微波炉之间的对比,不是微波炉里面的钟和
45度电的关系,因为比较内容没有出现,可以果断排除此项。

B项读成“内置钟和卖价”的关系,是无关话题,排除此项。

C项和“钟与45度电”这一话题完全没有联系,排除此项。

D项:一个选项中出现文章未提及的内容,如果借助常识推理几
步后所得内容和预测的答案话题处于一个方向,也属于沾边选项。并
不是不能推理,是不允许主观地自以为是地推理,但可以借用常识推
理。如果有一个单独的钟,则也会使用电,所以此项提到了“内置钟
与使用电”的关系,与话题有交集。

E项谈论户数相关的话题,和话题完全无关。

答案为D选项。

考生切记: 不可选项读到一半即对选项进行对与错的判断。正确
的步骤是读完选项,然后把选项以话题方式总结出来,看总结出来的选
项话题是否和原文提过的话题有联系,再进行判断。

例4 In response to mounting public concern, an airplane manufacturers


implement a program with well-publicized goal of reducing by half the total
yearly amount of hazardous waste generated by its passenger-jet division.
When the program began in 1994, the division's hazardous waste was 90
pounds per production worker, last year it was 40 pounds per production
worker. Clearly, therefore, charges that the manufacturer's program has not
met its goal are false.

Which of the following is an assumption on which the argument


depends?

A. The amount of nonhazardous waste generated each year by the


passenger-jet division has not increased significantly since 1984.

B. At least as many passenger jets were produced by the division last


year as had been produced in 1994.

C. Since 1994, other divisions in the company have achieved reductions


in hazardous waste output that are at least equal to that achieved in the
passenger-jet division.

D. The average number of weekly hours per production worker in the


passenger-jet division was not significantly greater last year than it was in
1994.
E. The number of production workers assigned to the passenger-jet
division was not significantly less in 1994 than it was last year.

解析: 在逻辑考题中,平均值和相对数是两个高频考点。此题
考查平均值问题。如果题目中有平均值,则和计算平均值过程相关的
事件就出题方向。(切记:不要对答案的方向有预判,这里只是展示
了统计数据,如何选择还是应具体问题具体分析)

原文从“有害废物数值由人均90磅降至人均40磅”推出“计划成
功了”。此结论是虚的内容,答案只会和“有害废物人均值”相关。
出现平均值则意味着可能会考查其计算过程,所以在看选项之前想下
“总有害废物数量除以总人数”得到人均有害废物值,如果不提前思
考此计算过程,万一选项提到计算过程相关的事件,很多考生会反应
不过来。

A项描述“非有害废物”和原文中“有害废物”没有关联,或可
理解为没有话题的交集。排除此项。

B项中“生产一样多的飞机”不能联系到“人均有害废物减少”
,但比A项好一些。

C项“其他部门”和原文中提到的部门没有交集,属于可以直接
排除的选项。

D项的话题是“工作时长没有增加”。

E项的话题是“人数没有增加”。

根据原文中的“平均人数”的话题,肯定E项的“人数”与原文
“平均人数”更沾边,故E项为答案。

技巧延伸: 如果原文说子集,选项说子集,两者不一定有关。就
像C项和原文的关系一样。但如果原文说子集,选项说全集,子集和全
集是肯定有交集,有类似逻辑关系的选项待比较。
例5 In January of last year the Moviemania chain of movie theaters
started propping its popcorn in canola oil, instead of the less healthful
coconut oil that it had been using until then. Now Moviemania is planning to
switch back, saying that the change has hurt popcorn sales. That claim is
false, however, since according to Moviemania's own sales figures,
Moviemania sold 5 percent more popcorn last year than in the previous year.

Which of the following, if true, most strongly supports the argument


against Moviemania's claim?

A. Total sales of all refreshments at Moviemania's movie theaters


increased by less than 5 percent last year.

D. Total attendance at Moviemania's movie theaters was more than 20


percent higher last year than the year before.

E. The year before last, Moviemania experienced a 10 percent increase


in popcorn sales over the previous year.

分析: A项中的食物和饮料(refreshments)与原文的爆米花是
全集和子集的关系,此项为正确答案。

D项:原文并未说明人数增加与业绩额提升之间的转换率到达哪
种程度才算是“好”,D项仅告知人数增加20%,我们无法判断原文中
销售额增加5%是算好还是不好,故此项无关。

E项:前年的销量如何影响不了现在。除非文章告知不同时空会
相互影响,比如“过去会影响到现在”,否则“过去”“现在”“将
来”不同的时空相互独立。例如一个女生去年被同一个男生追求了十
次没有成功,那么今年这个女生再被这个男生追求的话,结果是成功
还是不成功?我们没法判断。因为结果可能是男生追求成功,也可能
追求不成功。故E项不正确。

补充:“相对数”的考题
例6 In the United States, of the people who moved from one state to
another when they retired, the percentage who retired to Florida has
decreased by three percentage points over the past ten years. Since many
local businesses in Florida cater to retirees, this decline is likely to have a
noticeably negative economic effect on these businesses.

Which of the following, if true, most seriously weakens the argument?

A. Florida attracts more people who move from one state to another
when they retire than does any other state.

B. The number of people who move out of Florida to accept


employment in other states has increased over the past ten years.

C. There are far more local businesses in Florida that cater to tourists
than there are local businesses that cater to retirees.

D. The total number of people who retired and moved to another state
for their retirement has increased significantly over the past ten years.

E. The number of people who left Florida when they retired to live in
another state was greater last year than it was ten years ago.

解析: 文章是通过“3%”这一数字进行逻辑推理,为“相对数
”相关的考题。原文用“相对数”出题,答案往往用一个“绝对数”
来削弱。(切记,不可仅记“相对数对应绝对数”的答案模式,还是
要具体考题具体分析)

A项中有比较级,那么就应思考比较对象和比较内容是否在原文
出现过,如果有一个没有出现,则属于无关对比,可以直接排除。

B项“离开佛罗里去其他州就业”和“退休后去佛罗里达的人数
下降3%”无关。

C项:有部分学生觉得此项与原文有关,认为满足游客需求的生
意好,则表示佛罗里达还是能赚到钱的。请问,这些学生犯了什么错
误?他们把原文结论的内容修改了。原文结论并没有说佛罗里达是否
能赚钱,而是“佛罗里达针对退休的人的生意不行了”。不可修改原
文中的事件,这点在解阅读题时也尤其重要。把定位句的内容改了,
答案就会选错。

D项说“退休后迁至其他州的总人数增加了”。该项没有针对佛
罗里达来说,是一个全集性的说法,可涉及佛罗里达,属于沾边选项
。并且,“相对数和绝对数”的出题套路让D项更加有理由留着待选

E项与“佛罗里达针对退休人员的生意”无关,排除此项。

答案为D选项。有学生可能问,D选项为什么会成为正确答案呢?
首先,如果五个选项中只有一个选项沾边,不需要再考虑这唯一的选
项是如何推理出来的,只需要判断沾边即可。其次,逻辑题的支持或
削弱不是一定要推出答案所涉事件,能增加或降低推理链所涉事件成
立的概率即为支持或削弱。D选项通过逻辑可推理出:虽然相对数3%
下降了,但是绝对总量多,即使退休人数少了3%,也会有充足的绝对
数量来保障与退休人士相关的生意不受影响。但在实际考试中,各种
压力会导致学生失去理性判断能力,所以纯逻辑推理的思路在考试中
往往不适用。因此“沾边思维”对于解逻辑题和阅读题都有很大的好
处。

通过补充的试题,学生可发现削弱题、支持题和假设题三者的做
题思路类似,都是先判断选项是否和原文有话题交集,即是否沾边,
然后再判断哪个选项与原文内容更相近。其实,不仅是这三种题型,
评价题、解释矛盾题的解题方式也和沾边思维密切联系。逻辑推理能
力薄弱的学生尤其要学习此类思维。

例7 Press Secretary: Our critics claim that the President's recent


highway project cancellations demonstrate a vindictive desire to punish
legislative districts controlled by opposition parties. They offer as evidence
the fact that 90 percent of the projects canceled were in such districts. But all
of the canceled projects had been identified as wasteful in a report written by
respected nonpartisan auditors. So the President's choice was clearly
motivated by sound budgetary policy, not partisan politics.
Which of the following is an assumption on which the press secretary's
argument depends?

A. Canceling highway projects was not the only way for the President to
punish legislative districts controlled by opposition parties.

B. The scheduled highway projects identified as wasteful in the report


were not mostly projects in districts controlled by the President's party.

C. The number of projects canceled was a significant proportion of all


the highway projects that were to be undertaken by the government in the
near future.

D. The highway projects canceled in districts controlled by the


President's party were not generally more expensive than the projects
canceled in districts controlled by opposition parties.

E. Reports by nonpartisan auditors are not generally regarded by the


opposition parties as a source of objective assessments of government
projects.

解析: 第一步,找出原文的推理链。由“无党派审计人员说取
消的项目是浪费的”推出“项目取消是因为预算,而不是党派偏见”

第二步:看哪个选项和推理链或原文话题沾边。

A项:“唯一性”不是原文要研究的必要条件,类似的错误选项
在之前的试题中出现过。排除此项。

B项:此项读成“浪费的项目和执政党”的话题,这个话题明显
和文章内容有交集,先放着待选。

C项:“被取消的项目是执政党要承担的”。此项有党派倾向的
意思,感觉是执政党故意要取消这些项目,和文章的结论相反,不选
此项。
D项:“执政党控制的区域内取消的项目不会比反对党控制的区
域内取消的项目更贵”。此项发现了比较级,要立刻思考比较对象和
比较点是否都在原文中出现过。明显地,原文只提到过在“反对党控
制的区域内项目取消”而不存在“执政党控制的区域内项目取消”的
事件,所以此项属于无关对比,可排除此项。

E项:如果非党派人士的评估不被认为是客观的信息评价来源,
那么此条件对于原文结论不是一个支持的方向,违反了假设条件要支
持结论成立的原则,排除E选项。

只有B项和原文有关,无须推理,直接得出答案为B项。

例8In Brindon County, virtually all of the fasteners—such as nuts,


bolts, and screws—used by workshops and manufacturing firms have for
several years been supplied by the Brindon Bolt Barn, a specialist wholesaler.
In recent months many of Brindon County's workshops and manufacturing
firms have closed down, and no new ones have opened. Therefore, the
Brindon Bolt Barn will undoubtedly show a sharp decline in sales volume
and revenue for this year as compared to last year.

The argument depends on assuming which of the following?

A. Last year the Brindon Bolt Barn's sales volume and revenue were
significantly higher than they had been the previous year.

B. The workshops and manufacturing firms that have remained open


have a smaller volume of work to do this year than they did last year.

C. Soon the Brindon Bolt Barn will no longer be the only significant
supplier of fasteners to Brindon County's workshops.

D. The Brindon Bolt Barn's operating expenses have not increased this
year.

E. The Brindon Bolt Barn is not a company that gets the great majority
of its business from customers outside Brindon County.
解析: 原文的推理链可记成“Brindon的许多生产公司倒闭了”
,推出“紧固件(fastener)供应商Barn收入比去年下降”,可简单
记成“其他公司倒闭,Barn收入下降”。

A项:比较对象“前年”在文中没有提到,属于无关对比,可排
除A项。

B项:此项的比较内容在原文中没有见过。假设条件是必要条件
,如果取非必要条件,结论不能成立。即使幸存的公司业务量比去年
大,因为生产公司的数量下降,依然可以导致Barn公司的收入下降,
没有B项这个条件原文结论依然可以得出,所以排除此项。

C项:原文没有涉及唯一性,排除该选项。

D项:“运营成本没有增加”,此项和原推理链没有交集。单独
看运营成本是否增加,无法得知Barn收入是否增加。

E项:“Barn不会从Brindon 之外获得生意”。学生不需要推理
,肯定知道此项在描述“Barn和订单”的话题,则肯定与原文的结论
“Barn收入”意思上有关联,所以E项为答案。从推理的角度也很容
易理解,原文的前提是Brindon Country的生产公司数量减少,下游
的公司倒闭,上游的供应商收入下降也符合生活中的经历。但如果Ba
rn的大多数生意是来自Brindon这一城市之外呢?则即使城内收入下
降,城外其他地方是Barn主要的收入来源,Barn的主要收入依然不受
影响。故E为正确答案。

二、假设题错误选项的形式

假设题属于推理链题,又称“演绎题”。所有推理链考题都会涉及的
错误选项和诱惑选项类型:样本不足、诉诸大众、主观内容、循环论证
(重复现象)。

这四个错误是所有推理链题中常见的,不仅限于假设题。
1. 样本不足

选项中提示样本不足的词汇有:some、few、certain、sample、
small percentage等,也会有其他表示“小范围”“小样本”的内容。之前讲
过“some”本身只是告诉我们数字大于零,至于会不会达到有代表性的程
度我们并不清楚,完全可理解为含有some的事件只是一个特例。样本不
足则没有代表性(not representative)。但是样本不足不意味着选项必
错,只是属于高概率错误的选项。如果其他四个选项描述的事情和原文
都无逻辑交集,只有一个选项与原文话题部分沾边,即使这个选项
有“some”或类似词汇,我们也只能选择这个选项。

例1 Which of the following most logically completes the argument?

The irradiation of food kills bacteria and thus retards spoilage. However,
it also lowers the nutritional value of many foods. For example, irradiation
destroys a significant percentage of whatever vitamin B1 a food may contain.
Proponents of irradiation point out that irradiation is no worse in this respect
than cooking. However, this fact is either beside the point, since much
irradiated food is eaten raw, or else misleading, since

A. many of the proponents of irradiation are food distributors who gain


from food's having a longer shelf life.

B. it is clear that killing bacteria that may be present on food is not the
only effect that irradiation has.

C. cooking is usually the final step in preparing food for consumption,


whereas irradiation serves to ensure a longer shelf life for perishable foods.

D. certain kinds of cooking are, in fact, even more destructive of vitamin


B1 than carefully controlled irradiation is.

E. for food that is both irradiated and cooked, the reduction of vitamin
B1 associated with either process individually is compounded.

解析: 填空题其实就是演绎题的间接考法,实质是考查支持题
、假设题、削弱题等。原文讲“辐照和营养减少”之间的关系,带着
这个话题看选项。

A项描述“支持者和食物保质期”的话题,并不是在讨论“辐照
与营养价值”。排除此项。

B项讲“杀菌不是辐照唯一的作用”,此项和“营养价值”无关
,排除此项。

C项讲“辐照确保长的保质期”,和“营养价值”无关,排除此
项。

D项发现比较级,要立刻思考比较关系和比较对象是否在原文中
被提到过,有一项没有提到过即为无关对比。“certain cooking”
既有诱惑词汇“certain”,又有无关对比内容。并且从逻辑意思角
度分析,如果某种烹饪比辐照更有破坏性,那么能否说明辐照本身对
营养的负面影响?我们并不能得知。所以排除此项。

E项包含了“辐照和营养降低(deduction of Vitamin B1)”,


是唯一的沾边选项,选E项。

切记: some指代的词汇没有代表性,但most、much指代的词汇有
代表性。

例2 The chemical adenosine is released by brain cells when those cells


are active. Adenosine then binds to more and more sites on cells in certain
areas of the brain, as the total amount released gradually increases during
wakefulness. During sleep, the number of sites to which adenosine is bound
decreases. Some researchers have hypothesized that it is the cumulative
binding of adenosine to a large number of sites that causes the onset of sleep.

Which of the following, if true, provides the most support for the
researchers' hypothesis?

A. Even after long periods of sleep when adenosine is at its lowest


concentration in the brain, the number of brain cells bound with adenosine
remains very large.

B. Caffeine, which has the effect of making people remain wakeful, is


known to interfere with the binding of adenosine to sites on brain cells.

C. Besides binding to sites in the brain, adenosine is known to be


involved in biochemical reactions throughout the body.

D. Some areas of the brain that are relatively inactive nonetheless


release some adenosine.

E. Stress resulting from a dangerous situation can preserve wakefulness


even when brain levels of bound adenosine are high.

解析: 题目要求支持假说(hypothesis),则读文章时重点找
假说的内容。如有选项谈到与假说相关的内容,则优选。

因为在实际考试中不可能记得很详细,所以考生可简单把原文读
成“Adenosine结合多少”与“睡觉”之间的关系。

A项:“睡觉之后腺苷(adenosine)最低,结合的腺苷也非常多
”,此项谈论了“睡觉与腺苷多少”相关的话题,属于沾边选项。

B项:“清醒与腺苷结合”的关系,清醒就是不睡觉,此项相当
于也谈到了“睡觉和腺苷结合”的关系,此项待选。

C项:此项没有谈到“睡觉与腺苷”相关的话题,排除此项。

D项:出现了两个some,即使此项沾边也肯定不如A项、B项更好
,因为这两项没有诱惑选项的词汇,成为答案的概率会高于D项。

E项:本项讲“清醒和腺苷”相关的话题,此项也沾边。

五个选项中A项、B项、E项三者都在谈论“清醒与腺苷”的关系
,所以都为待选项。

比较选项前,可以先思考一下,如果你是出题人,你会如何让几
个沾边选项中的一个成为答案?只有这几种可能性:情况一:假设有
些不沾边,而是学生的误读,把一些不沾边的选项误认为是“有关选
项”。情况二:假设所有待选项都沾边,则又分为两种情况——沾边
选项的方向和题目问的一致、沾边选项的方向和题目所问的不一致。
沾边选项和题目所问的都一致,则可以思考哪个选项出错概率小,或
哪个选项和原文的距离最近。

在此题的A项、B项、E项对比中,可清楚地发现B项和E项是逻辑
意思完全相反的关系,B项是“清醒与‘interfere with the bindin
g’”,E项是“清醒和大量的‘bound’(bound意思等同于binding
)”。此时,再核实一下原文的内容,原文的假说内容是“the cumu
lative binding of adenosine to a large number of sites that
causes the onset of sleep”。“结合和睡觉”是正相关关系,试
题问的是支持方向,所以B项优于E项。同时A项的意思是“睡觉之后
腺苷的结合也会非常多”,和原文方向相反,排除A,即B项为正确答
案。

2. 诉诸大众

解读: 此项逻辑错误是我们在日常生活中也会犯的类型,大众
认为对的事情不一定是正确的。

例 Before the discovery of pellagra's link with niacin, it was widely


believed that the disease was an infection that could be transmitted from
person to person.

一直以来这个疾病被相信是人传人的感染,实际是如此吗?答案是
无法判断。因为大众相信的东西不一定是事实,不可从这句话中得出这
个病就是通过人际传染的。

3. 主观内容
解读: 逻辑论证具有客观性。人的思考具有主观性,只思考而
不做出某些行为是不会对客观的世界有所影响的,从而也不会形成“
逻辑演绎”。因此,如果选项只表达主观思考(知道、想要、激励、
目标)等,而并没有采取行动,则基本可以排除此项。但如果原文问
的就是一个人的主观想法,另当别论。

例 Newspaper editorial: In an attempt to reduce the crime rate, the


governor is getting tough on criminals and making prison conditions harsher.
Part of this effort has been to deny inmates the access they formerly had to
college-level courses. However, this action is clearly counter to the
governor's ultimate goal, since after being released form prison, inmates who
had taken such courses committed far fewer crimes overall than other
inmates.

Which of the following is an assumption on which the argument


depends?

A. Not being able to take college-level courses while in prison is


unlikely to deter anyone from a crime that he or she might otherwise have
committed.

B. Former inmates are no more likely to commit crimes than are


members of the general population.

C. The group of inmates who chose to take college-level courses were


not already less likely than other inmates to commit crimes after being
released.

D. Taking high school level courses in prison has less effect on an


inmate's subsequent behavior than taking college-level courses does.

E. The governor's ultimate goal actually is to gain popularity by


convincing people that something effective is being done about crime.

解析: (1)试题问“假设”,则要找一个和结论事件相同方向
的选项。(2)原文读成:“减少犯罪率,让监狱变得更糟一些,不
让罪犯接触到大学课程。但是这个有悖于政府最终目的,释放之后参
加大学课程的人犯罪更少。”简单记为“参加课程与犯罪少”的关系

A项可读成“不让罪犯参加课程不会阻止他们犯罪”,粗看起来
此项与“参加课程与犯罪”沾边,但我们要找的方向是“参加课程与
犯罪少”的关系,而此项是“不参加课程”与“犯罪少”的关系,所
以A项排除。

B项并没有提到“课程与犯罪少”的话题,排除此项。并且此项
犯了“无关对比”的错误。

C项肯定能读出“大学课程”与“犯罪少”的关系,保留此项。

D项谈到了“大学课程”但是没有讲“犯罪少”,排除此项。

E项,政府的目标与“大学课程和犯罪少”没有事实上的关联。

只有C沾边,正确答案为C。C项用了“排除他因”的方式进行增
强。C项有一个“not”,意思是排除了参加大学课的人本身会更少犯
罪的可能性,从而增加了是因为“大学课程”而不是“他们本身就会
犯罪更少”导致的“犯罪少”。

4. 循环论证

解读: 在支持题中,如果选项重复了结论,可以是正确答案。
但在假设题中,必须找一个原文没有说过的事件。此外,所有的逻辑
题都不允许答案重复原文中“已经提到的事实”,或“已经告诉我们
会成立的条件”。逻辑演绎过程就是生活中的场景。例如:A说“假
如明天不下雨,你能陪我去看电影吗?”B如果回答“明天会下雨”
算不算反驳了A?在生活中,算。GRE、GMAT考试中追求客观性逻辑,
文章之外的情况我们都不能主观推理。生活充满了博弈,需要在和他
人互动的过程中了解到其真实想法,要听出“话里有话”。但是这样
的思想不可带进备考的状态,之前A说“如果明天下雨”,说明问的
是在这个条件下,B会如何行动,而不是削弱条件本身。只驳斥条件
,并没有回答A的提问。再比如,假设原文内容是“如果你中了1000
万元彩票,你会给慈善机构捐款1000元”,问支持方向。选项A:你
真的会中1000万元。选项B:你平时就有乐于助人的习惯。这两个选
项哪个是支持?A项只是在重复条件,并没有说明有了这个条件之后
你会采取什么样的行动。B项讲,你平时就乐于助人,所以会增加你
做慈善的概率,因此答案是B项。

例 Smithtown University's fund-raisers succeeded in getting donations


from 80 percent of the potential donors they contacted. This success rate,
exceptionally high for university fund-raisers, does not indicate that they
were doing a good job. On the contrary, since the people most likely to
donate are those who have donated in the past, good fund-raisers constantly
try less-likely prospects in an effort to expand the donor base. The high
success rate shows insufficient canvassing effort.

Which of the following, if true, provides more support for the argument?

A. Smithtown University's fund-raisers were successful in their contacts


with potential donors who had never given before about as frequently as were
fund-raisers for other universities in their contacts with such people.

B. This year most of the donations that came to Smithtown University


from people who had previously donated to it were made without the
university's fund-raisers having made any contact with the donors.

解析: B项只是在重复原文中的事实——“since the people m


ost likely to donate are those who have donated in the past
”,因此B项不是答案。

A项的意思是Smithtown大学的募捐人和其他大学的募捐人在劝说
那些不大可能捐款的人捐款的成功可能性上是一样的,说明别的大学
未能劝说这些人捐款,Smithtown大学的人也未能成功。这样就不能
说Smithtown大学的募捐人做得很好(doing a good job),属于支
持的方向。此项的内容相当于在解释原文倒数第2句话“good fund-r
aisers constantly try less-likely prospects in an effort to
expand the donor base”,好的募捐人应该让不太可能捐款的人群
捐钱,而Smithtown大学的人并没有做得比别的学校的人更好,故A项
正确。

假设题的错误选项类型如下:

(1)重复文章中描述的事情。

(2)与结论事件方向相反。

(3)选项支持结论,但与论证无关。

(4)逻辑顺序颠倒。

(5)其他无关选项。

类型1:重复文章中描述的事情

错误选项或重复前提,或重复结论,或重复原文已经描述的某个现
象。

例 Radio stations with radio data system(RDS)technology broadcast


special program information that only radios with an RDS feature can
receive. Between 1994 and 1996, the number of RDS radio stations in
Verdand increased from 250 to 600. However, since the number of RDS-
equipped radios in Verdland was about the same in 1996 as in 1994, the
number of Verdanders receiving the special program information probably
did not increase significantly.

Which of the following is an assumption on which the argument


depends?

A. Few if any of the RDS radio stations that began broadcasting in


Verdland after 1994 broadcast to people with RDS-equipped radios living in
areas not previously reached by RDS stations.

B. In 1996 Verdlanders who did not own radios equipped to receive


RDS could not receive any programming from the RDS radio stations that
began broadcasting in Verdand after 1994.

解析: 此题为假设题,表明选项和原文必须是“求同”的方向

原文说了这些事:

(1)只有RDS收音机才能收听到特别电台节目。

(2)RDS发射站多了,但是RDS收音机数量一样。

(3)收到特别电台节目信号的人数并没有增加。

推理链就是前两件事情推出第三个件事情,来比较A项和B项。

A项可读成“很少有基站给这些人广播,他们在1994年之前是收
不到信号的”。如果1994年之前收不到信号的人很多,而1994年之后
因为基站多了,使这些人们能收到信号了,则原文的结论“收到信号
的人数不多”就推不出来了。此项削弱后能使原文结论无法得出,属
于必要条件的特征。此项可作为答案备选。

B项读成“没有RDS收音机就不能接收RDS的信号”。读到此时,
学生会想如果一般的收音机也能收到RDS信号,那么接收信号的人数
就增加了,同A项一样会导致结论得不出。但是,如果这样想的话,
就犯了假设答案的错误。假设条件必须是原文未提及的条件,它能支
撑原文结论成立,但在文章中没有说过类似事件。原文明确说了,只
有RDS收音机才能接收到RDS信号,所以此项只是在针对“已有的事实
”再说一次,B项不能成为答案。

A项为答案。
类型2:与结论所涉事件方向相反

假设条件是结合前提一起推出结论,必须和结论是一个方向性的内
容。此条很容易理解。

类型3:选项支持结论,但与论证无关

假设相当于在前提与结论之间架一座桥,铺一条路,让前提能够顺
利推导出结论。所以如果仅是增加结论成立的概率,而忽略了前提,不
属于正确的假设条件。

例 A notched wooden stick from South Africa's Border Cave dating to


24,000 years ago contains the earliest evidence of humans using poison.
Moreover, the poison applicator is just one of several artifacts, some dating to
as early as 44,000 years ago, that resemble objects used by the San. Others
include a digging stick, ostrich eggshell beads, carved pig tusks, bone
arrowheads, and a lump of beeswax. Archaeologists have hypothesized that
the finds indicate that San culture emerged about 44,000 years ago, making
these the earliest link to a culture of modern humans.

Which of the following, if true, most strongly supports the


archaeologists' hypothesis?

A. Carbon-dating of human skeleton indicates that human were not able


to make artifacts until 44,000 years ago.

B. Successor cultures at given cave do not often adopt the style of


agricultural implements used by earlier inhabitants of the same site.

分析: 此题为支持题,支持题和假设题一样,都承认结论正确
,要找一个选项进一步增加结论成立的概率。题目问支持假说的选项
,于是要从原文找到假说的内容。

文章说了以下这些事:

(1)一根木棒显示最早的用毒的证据。
(2)发现一些文物。

(3)从这些文物中推出了San文化出现的时间。

文章的结论是“San文化出现在4.4万年前”,前提是“一些物品
追溯至4.4万年前”,所以推理过程是通过文物的时间推出San文化出
现的时间。

A项读成“人类不能够在4.4万年前制作物品”,此项支持了结论
,但此项和前提无关。我们要分析是否可以通过前提来得出结论,并
不是通过其他角度单纯来支持结论,因为此项没有涉及“文物”相关
内容,所以和推理链无关,不正确。

B项读成“之后的文化不会模仿之前的农具”。如果可以模仿,
那么就不能从文物时间来推测出文化的时间,因此此项说到了“文物
与文化”的关系,和原文的推理链沾边,得出答案为B项。

类型4:逻辑顺序颠倒

逻辑顺序颠倒类似“因果倒置”,这条不仅在假设题中常见,在支持
题中也是高频考点。

例1 Researchers took a group of teenagers who had never smoked and


for one year tracked whether they took up smoking and how their mental
health changed. Those who began smoking within a month of the study's start
were four times as likely to be depressed at the study's end than those who
did not begin smoking. Since nicotine in cigarettes changes brain chemistry,
perhaps thereby affecting mood, it is likely that smoking contributes to
depression in teenagers.

Which of the following is an assumption on which the argument


depends?

A. Participants who were depressed at the study's start were no more


likely to be smokers at the study's end than those who were not depressed.
解析: 原文的推理链为“抽烟导致抑郁”。

A项:“抑郁的人不可能更容易成为抽烟者”。A项如果没有“no
more likely to be”就会导致“因果倒置”,即不是先有抽烟再有
的抑郁,而是先有抑郁再成为抽烟者。加上否定词就排除了因果倒置
的可能。此项即为答案。

例2 In the Middle Ages, bloodletting was considered beneficial to health


and the procedure was used to treat many illnesses, as well as to improve
general health. Modern medicine has shown bloodletting to be harmful in
treating many of the illnesses for which it was prescribed in the Middle
Ages.Yet people who donate blood regularly to blood banks live longer, on
average, than those who never donate blood, so bloodletting probably has
some beneficial effect.

Which of the following, if true, points to an error of reasoning in the


argument?

A. Medieval medical theories assigned a different role to the blood than


modern medicine does.

B. Not all of the conditions for which medieval medicine prescribed


bloodletting are worsened by the procedure.

C. Bloodletting diminishes a patient's ability to fight off certain kinds of


infection.

D. Many people who donate blood regularly to blood banks do so less


frequently in old age than they did when they were younger.

E. People who are not generally healthy are not permitted to donate
blood to blood banks.

解析: 此题为削弱题,题目问在推理中有何错误。问法表明正
确答案描述的事件一定是推理链内部说过的内容,不会通过其他事件
来削弱推理链,因为问法是“推理过程中有哪个错误”,答案只能和
推理过程相关联。

文章推理链是从“献血的人活得更久”推出“放血有好处”。

A项出现比较级,比较内容并没有在文章中出现,排除此项。

B项,not all 相当于some,放在名词前面,说明所指情况没有


代表性,也是诱惑选项的信号词。此项不太好。

C项针对结论反着说,属于削弱的方向。前三个选项对比,C项胜
出,可放着待选。

D项又发现一个比较级。两个比较对象(old age,younger)的
对比在文章中并没有见到,所以此项和A项一样,犯了无关对比的错
误,排除此项。

E项意思是“不健康的人不让献血”。原文中提到“献血会有好
处(健康)”,此项和因果倒置相关。如果健康的人才允许被献血,
就推理不出献血会有益健康这个说法了。E项正确。
第四章 解释矛盾题

一、出题原理

解释矛盾题是一种在GRE考试中出现频率极小的题型。其出题结构
是逻辑文章中含有一个矛盾点,即按大家的常规性理解事件一理应推出
事件二,但结果却发生了与事件二方向相反的事件,要找一个选项解
释“事件二的反面”发生的原因。

解释矛盾题、支持题、假设题有一个共同的特性:都承认文章中逻
辑链的结论(conclusion)成立。

解释矛盾题也可以转换成削弱题,例如:

Nitrogen dioxide is a pollutant emitted by automobiles. Catalytic


converters, devices designed to reduce nitrogen dioxide emissions, have been
required in all new cars in Donia since 1993, and as a result, nitrogen dioxide
emissions have been significantly reduced throughout most of the country.
Yet although the proportion of new cars in Donia's capital city has always
been comparatively high, nitrogen dioxide emissions there have showed only
an insignificant decline since 1993.

Which of the following, if true, most helps to explain the insignificant


decline in nitrogen dioxide emissions in Donia's capital city?

解读: 题干部分有explain、account for、discrepancy等词汇


的即为解释矛盾题。此题的原文主要内容是:“别的地方二氧化氮下
降很多,首都下降得不多”,要求从选项中找一个话题能解释为什么
首都下降不多。

此题可以转换为“别的地方下降二氧化氮多→首都二氧化氮下降
也多”,要削弱结论,即“首都下降二氧化氮不多”。

二、解题思维

第一步:确定需要解释的“矛盾事件”是文章中哪句话。

解读: 结论一般在段落末句,当然也有结论在第一句的情况。
题目要求找到一个选项解释为什么结论所涉事件会发生,所以首要任
务就是去找矛盾关系在哪些话题中产生。

第二步:记住结论。

解读: 需要找一个选项能推出结论,所以要记住结论。

第三步:留下本身或推几步能解释文中结论的选项。

解读: 答案不是一定能推出结论的,只要能增加结论成立的概
率即可放着选项对比。

例 When a new restaurant, Martin's Cafe, opened in Riverville last year,


many people predicted that business at the Wildflower Inn, Riverville's only
other restaurant, would suffer from the competition. Surprisingly, however, in
the year since Martin's Cafe opened, the average number of meals per night
served at the Wildflower Inn has increased significantly.

Which of the following, if true, most helps to explain the increase?


A. Unlike the Wildflower Inn, Martin's Cafe serves considerably more
meals on weekends than it does on weekdays.

B. Most of the customers of Martin's Cafe had never dined in Riverville


before this restaurant opened, and on most days Martin's Cafe attracts more
customers than it can seat.

C. The profit per meal is higher, on average, for meals served at Martin's
Cafe than for those served at the Wildflower Inn.

D. The Wildflower Inn is not open on Sundays, and therefore Riverville


residents who choose to dine out on that day must either eat at Martin's Cafe
or go to neighboring towns to eat.

E. A significant proportion of the staff at Martin's Cafe are people who


formerly worked at the Wildflower Inn and were hired away by the owner of
Martin's Cafe.

解析: 原文的矛盾事件是“竞争对手Martin's Cafe开业→Wild


flower Inn平均用餐人数更多了”。此题相于说“因为有了竞争对手
Martin's Cafe开业,所以Wildflower Inn卖餐量少了,问哪个选项
能削弱结论”。

带着结论“Wildflower卖餐量更多”看选项。

A项讲“Martin's Cafe什么时候卖餐卖得更多”。短文的结论是
Wildflower Inn平均卖餐量增加,A项说的是Martin's Cafe某些日子
卖得多,所以是无关选项。

B项讲 “Martin's Cafe吸引顾客数超过可容纳的量”。Martin'


s Cafe顾客多到坐不下,增加了顾客从Martin's Cafe去Wildflower
Inn吃饭的可能性,符合“Wildflower Inn卖餐量增加”的方向,待
选。

C项讲Martin's Cafe平均每餐利润高。结论只讲到了用餐数字,
没有讲利润情况。此项无关。
D项讲Wildflower Inn不营业时居民要么去Martin's Cafe要么去
其他地方吃饭。此项和Wildflower Inn卖餐量没有关系,或方向相反
。排除此项。

E项讲Wildflower Inn的员工去了Martin's Cafe。员工的转移推


不出顾客去Wildflower Inn用餐的数字,排除此项。

答案为B。虽然B选项没有直接告诉我们Wildflower Inn卖餐量增
加,但至少此项描述的话题和结论是同方向,而其他选项无关,故只
能选此项。

三、试题练习

例1 Nitrogen dioxide is a pollutant emitted by automobiles. Catalytic


converters, devices designed to reduce nitrogen dioxide emissions, have been
required in all new cars in Donia since 1993, and as a result, nitrogen dioxide
emissions have been significantly reduced throughout most of the country.
Yet although the proportion of new cars in Donia's capital city has always
been comparatively high, nitrogen dioxide emissions there have showed only
an insignificant decline since 1993.

Which of the following, if true, most helps to explain the insignificant


decline in nitrogen dioxide emissions in Donia's capital city?

A. More of the cars in Donia's capital city were made before 1993 than
after 1993.

B. The number of new cars sold per year in Donia has declined slightly
since 1993.

C. Pollutants other than nitrogen dioxide that are emitted by automobiles


have also been significantly reduced in Donia since 1993.

D. Many Donians who own cars made before 1993 have had catalytic
converters installed in their cars.
E. Most car trips in Donia's capital city are too short for the catalytic
converter to reach its effective working temperature.

解析: 第一步:找到矛盾事件。原文的矛盾性事件是“别的城
市的汽车安装了设备后二氧化氮排放下降很多,但是首都只下降一点
”。

第二步:带着结论(首都二氧化氮排放只下降一点)找答案。

第三步:选项分析。

A项读成“首都旧车多”。此项方向对,但是和文章中已有的事
实冲突。原文最后一句话表明“新车比例很多”,所以此项不可选。

如果在结尾有含因果关系或让步关系的句子,考生要警惕,往往
会针对这些句子设置诱惑选项。

B项读成“新车销售在Donia轻微下降”。要找的方向是“首都”
,而此项谈论的对象是Donia这个国家,不可选此项。

C项读成“二氧化氮之外其他废气排放量下降了”。此项是无关
选项,既说到了原文之外的其他对象,又针对Donia这个国家来谈,
没有针对性地说到Donia的首都为何二氧化氮下降不多。

D项读成“Donia的人在1993年之前已经安装转换器了”。此项和
前两项犯的错误一样,要解释的是“首都”的情况,而不是整个国家
的情况。为何这个国家的其他城市汽车废气排放减少,而首都却没有
?此项未给出原因,排除。

E项读成“首都短途旅行让转换装置不能工作”。首都汽车的设
备不能正常工作,正好解释了为什么首都二氧化氮下降不多。此项即
为答案。

知识点展开:
此题可变化为削弱题:“因为别的城市废气下降多,所以首都废气
也下降多”,找一个削弱结论的选项。如果要找削弱或支持结论的选
项,则此类题可统称为演绎题,即前提通过演绎推导得出结论。此题运
用了类比推理的方式由前提推出了结论。

类比推理的特征:前提是对象1,结论是对象2。当考生发现结论的
对象和前提的对象无交集时,这样的论证过程就是类比推理。此题通过
其他城市类比推理到首都。类比推理题的答案有固定的模式。在GRE考
试中,目前只出现如下答案模式,即两个类比对象有共性(题目问支持
方向),以及两个类比对象有差异性或某个对象有独特之处(题目问削
弱)。此题的E项即符合“某个类比对象有独特之处”的思路。

除了类比推理之外,还有“样本推总量”“原因推结果”“现象推说
法”“计划推目标”的论证过程,这些论证过程将在其他的试题中展开论
述。

例2 Escalating worldwide demand for corn has led to a sharp increase in


the market price of corn, and corn prices are likely to remain high. Corn is
extensive used as feed for livestock, and because profit margins are tight in
the livestock business, many farmers are expected to leave the business. With
fewer suppliers, meat prices will surely rise. Nonetheless, observers expect an
immediate short-term decrease in meat prices.

Which of the following, if true, most helps to justify the observers'


expectation?

A. The increase in corn prices is due more to a decline in the supply of


corn than to a growth in demand for it.

B. Generally, farmers who are squeezed out of the livestock business


send their livestock to market much earlier than they otherwise would.

C. Some people who ate meat regularly in the past are converting to
diets that include little or no meat.

D. As meat prices rise, the number of livestock producers is likely to rise


again.
E. Livestock producers who stay in the business will start using feed
other than corn more extensively than they did in the past.

解析: 第一步:找到矛盾事件。矛盾事件:“供应少,肉价高
,但短期内肉价反而会下降。”

第二步:带着结论找选项。结论:“短期肉价会下降。”

第三步:选项分析,留下部分沾边或和结论方向一致的选项。

A项读成“玉米价格上升的原因”。“什么原因导致玉米价格上
升”和“肉价短期内会下降”是两个完全不相关的事情,直接排除此
项。

B项读成“农民提前送动物进市场”。“农民提前送动物进市场
”会推出“家禽供应量多了,从而肉价下降”,此项和结论方向一致
,待选。

C项读成“过去吃肉的人开始吃素了”。some这个词不好。C项告
知我们一些人过去吃肉现在不吃肉了。这件事与文中预测“短期内肉
价下降”没有关联。首先我们不知道有多少比例的人现在不吃肉了,
some的出现表明这个事件可能只是一个特例,特例缺少代表性。如果
把some变成most,此项能推出“短期内肉价下降吗”?还是推不出。
因为消费者少了,供应方会降价,但不可推出“短期内肉价下降”中
的“短期”。综上,C项不可选。

D项读成“肉价上升后,养猪的人变多了”。原文要解释的事件
是“肉价会上升”,而不是“上升之后出现什么情况”。此项相当于
在论述假如肉价上升,会出现什么情况。要找的答案方向是“肉价会
上升”,这与“假设肉价已经上升了,然后会发生什么情况”不是在
谈论一件事,排除此项。

E项读成“留在行业的人用玉米之外的饲料去喂动物”。“用玉
米之外的饲料喂动物”会导致肉价是上升还是下降?从常识来看并不
能推出“肉价会上升”的情况,排除此项。
正确答案为B项。

例3 A diet high in saturated fats increases a person's risk of developing


heart disease. Regular consumption of red wine reduces that risk. Per-capita
consumption of saturated fats is currently about the same in France as in the
United States, but there is less heart disease there than in the United States
because consumption of red wine is higher in France. The difference in
regular red-wine consumption has been narrowing, but no similar
convergence in heart-disease rates has occurred.

Which of the following, if true, most helps to account for the lack of
convergence noted above?

A. Consumption of saturated fats is related more strongly to the growth


of fatty deposits on artery walls, which reduce blood flow to the heart, than it
is to heart disease directly.

B. Over the past 30 years, per-capita consumption of saturated fats has


remained essentially unchanged in the United States but has increased
somewhat in France.

C. Reports of the health benefits of red wine have led many people in
the United States to drink red wine regularly.

D. Cigarette smoking, which can also contribute to heart disease, is only


slightly more common in France than in the United States.

E. Regular consumption of red wine is declining dramatically among


young adults in France, and heart disease typically does not manifest itself
until middle age.

解析: 第一步:发现矛盾事件。矛盾事件:“喝酒量差不多,
法国人得心脏病的比例比美国人低”。

第二步:记住结论。结论:“法国人得心脏病的比例比美国人低
”。

第三步:分析选项。

A项读成“饱和脂肪的摄入会导致的身体情况”。此项和“法国
人得心脏病”的话题完全无关,排除。

B项读成“饱和脂肪的摄入在美国不变,在法国增加”。原文中
描述的是“法国人得病的比例低”,此项可推出法国人得病多,属于
和结论方向相反的选项,排除此项。

C项读成“美国人常喝红酒”。原文中已经提到了美国人喝红酒
,所以此项等于把原文中的一个现象再重复一次,显然不能解释“法
国人得心脏病的比例低”相关的话题。

D项读成“法国人抽烟更多”。此项的slightly more common不


能解释原文中的两国公民患心脏病比例的差异,排除此项。

E项读成“红酒消耗在法国年轻人中下降,心脏病在中年前没有
显示出来”。此项提到了“法国”,又提到了“心脏病”,即使猜也
要猜E选项为答案,因为之前几个选项根本没提到结论中的关键词。
从逻辑角度来分析,此项正确是因为心脏病只是没有显示出来,并不
是没有得病,从而解释了原文结论。

例4 Denoma, a major consumer-electronics maker, had a sizeable


decline in sales revenue for its most recent fiscal year. This result appears
surprising, because electronics retailers report that although their overall sales
were considerably lower than in the previous year, their sales revenue from
Denoma models actually grew, largely thanks to some innovative and popular
models that Denoma introduced.

Which of the following, if true, does most to explain the apparently


surprising result?

A. Because of the need to educate the public about its new models'
capabilities, Denoma's advertising spending was higher than normal over the
period.

B. For the period at issue, Denoma's major competitors reported declines


in revenue that were, in percentage terms, greater than Denoma's.

C. A significant proportion of Denoma's revenue comes from making


components for other consumer-electronics manufacturers.

D. Unlike some of its major competitors, Denoma has no lines of


business outside consumer electronics to provide revenue when retail sales of
consumer electronics are weak.

E. During the period, consumer-electronics retailers sold remaining units


of Denoma's superseded models at prices that were deeply discounted from
those models' original prices.

解析: 第一步:挑出矛盾事件。

矛盾点:“零售商说销售收入增加,但制造商说收入下降。”

第二步:记住结论话题。结论:“制造商报告收入下降。”

第三步:找出沾边选项。

A项读成“Denoma的广告费用多”。广告费用多,推不出Denoma
收入下降。广告费用多还可能推出Denoma收入多,因而不能达到解释
原文的作用。

B项读成“竞争对手利润下降更多”。Denoma的竞争对手和Denom
a有何关联?可以立刻排除此项。

C项读成“Denoma的很多的收入来自其他方面”。如果significa
nt变成certain此项就不能解释原文,但“收入的大部分来自其他地
方”则表明“零售商的收入好,并不代表Denoma的收入好,因为Deno
ma很多收入来自零售商之外的地方”,此项可起到解释作用,可保留

D项读成“Denoma没有其他的收入渠道”。此项并没解释文中矛
盾。为何零售商收入高,而Denoma收入低呢?此项最多说明Denoma收
入低的事件方向,但没有解释零售商与Denoma收入的“矛盾点”,排
除此项。

E项读成“对部分产品零售商打折卖”。零售商如何卖是过程,
结果是零售商收入高。此项最多只是在描述“零售商收入高的可能原
因”,相当于把原文事件再提一次,没有联系到Denoma,所以无关。

正确答案为D项。

例5 The number of applications for teaching positions in Newtown's


public schools was 5.7 percent lower in 1993 than in 1985 and 5.9 percent
lower in 1994 than in 1985. Despite a steadily growing student population
and an increasing number of teacher resignations, however, Newtown does
not face a shortage in the late 1990's.

Which of the following, if true, would contribute most to an explanation


of the apparent discrepancy above?

A. Many of Newtown's public school students do not graduate from high


school.

B. New housing developments planned for Newtown are(shared)for


occupancy in 1987 and are expected to increase the number of elementary
school students in Newtown's public.

C. The Newtown school board does not contemplate increasing the ratio
of students to teachers in the 1990's.

D. Teachers' colleges in and near Newtown produced lower graduates in


1994 than in 1993

E. In 1993 Newtown's public schools received 40 percent more


applications for teaching positions than there were positions available.
解析: 此题较容易,学生可自行分析,答案为E项。
第五章 评价题

一、评价题和其他演绎题型的区别

评价题又称评估题。识别评价题:如果一道题要考生评估文中的前
提是否可推出文中结论,此题即为评价题。常见的是题目本身包括
evaluate或evaluation。

常见的评价题问法示例:

Which of the following would be most useful to know in evaluating the


argument above?

Which of the following would it be most useful to know in order to


evaluate the argument?

Which of the following, if known, would be most relevant to evaluating


the claim above about how new-coal burning plants could save money?

Which of the following would it be most important to ascertain in


determining whether implementing the shipping manager's proposal would
have the argued-for effect on costs?

从上面具体示例可以看出问法都是“提问哪个选项能评估、确定文
章中的推理链或某个事件”。评价题的选项基本带有疑问词,
如“Whether physicians are likely to succumb to pressure from patients to
prescribe inappropriate medications”。

完整的评价题示例:

Because visual inspection cannot reliably distinguish certain skin


discolorations from skin cancers, dermatologists at clinics have needed to
perform tests of skin tissue taken from patients. At Westville Hospital,
dermatological diagnostic costs were reduced by the purchase of a new
imaging machine that diagnoses skin cancer in such cases as reliably as the
tissue tests do. Consequently, even though the machine is expensive, a
dermatological clinic in Westville is considering buying one to reduce
diagnostic costs.

Which of the following would it be most useful for the clinic to establish
in order to make its decision?

A. Whether the visits of patients who require diagnosis of skin


discolorations tend to be shorter in duration at the clinic than at the hospital.

B. Whether the principles on which the machine operates have been


known to science for a long time.

C. Whether the machine at the clinic would get significantly less heavy
use than the machine at the hospital does.

D. Whether in certain cases of skin discoloration, visual inspection is


sufficient to make a diagnosis of skin cancer.

E. Whether hospitals in other parts of the country have purchased such


imaging machines.

间接方式考查的评价题示例:

例1 Yeasts capable of leavening bread are widespread, and in the many


centuries during which the ancient Egyptians made only unleavened bread,
such yeasts must frequently have been mixed into bread doughs accidentally.
The Egyptians, however, did not discover leavened bread until about 3000
B.C. That discovery roughly coincided with the introduction of a wheat
variety that was preferable to previous varieties because its edible kernel
could be removed from the husk without first toasting the grain.

Which of the following, if true, provide the strongest evidence that the
two developments were causally related?

A. Even after the ancient Egyptians discovered leavened bread and the
techniques for reliably producing it were well known, unleavened bread
continued to be widely consumed.

B. Only when the Egyptians stopped the practice of toasting grain were
their stone-lined grain-toasting pits available for baking bread.

C. Heating a wheat kernel destroys its gluten, a protein that must be


present in order for yeast to leaven bread dough.

D. The new variety of wheat, which had a more delicate flavor because
it was not toasted, was reserved for the consumption of high officials when it
first began to be grown.

E. Because the husk of the new variety of wheat was more easily
removed, flour made from it required less effort to produce.

此题哪个选项说明了文章中所提到的两个事件有因果关系,哪个选
项就是正确答案。把此题看做支持题也可以,反正演绎题的做法基本雷
同,只有选项对比时有细微差异而已。

评价题和支持题、假设题、削弱题之间的关系是什么?解题过程有
何差异?评价题是考试中常出的一种演绎题型。就做题步骤而言,考生
们可思考一下是支持题的做题步骤多,还是评价题的做题步骤多。对于
常见的支持、削弱等逻辑演绎题,我们在筛选选项时先把沾边的选项留
下,然后再进行对比,比如题目要找削弱推理链的选项,一个选项沾边
但是支持结论,自然不会是此题的答案。评价题本身不存在“求
同”或“求异”的方向,评价题的正确答案就是那个和原文沾边的选项,
不需要再思考选项是否与题目所问处于同一个方向。故而评价题比支持
题、削弱题、假设题要少一个解题步骤,只需要判断选项与原文是否沾
边。

因此,解评价题在看选项之前的解题过程和做其他的演绎题无差
别,对比选项的过程中只需要找到与题目所问或文章事件最有关联度的
选项即可。

例2 Because visual inspection cannot reliably distinguish certain skin


discolorations from skin cancers, dermatologists at clinics have needed to
perform tests of skin tissue taken from patients. At Westville Hospital,
dermatological diagnostic costs were reduced by the purchase of a new
imaging machine that diagnoses skin cancer in such cases as reliably as the
tissue tests do. Consequently, even though the machine is expensive, a
dermatological clinic in Westville is considering buying one to reduce
diagnostic costs.

Which of the following would it be most useful for the clinic to establish
in order to make its decision?

A. Whether the visits of patients who require diagnosis of skin


discolorations tend to be shorter in duration at the clinic than at the hospital.

B. Whether the principles on which the machine operates have been


known to science for a long time.

C. Whether the machine at the clinic would get significantly less heavy
use than the machine at the hospital does.

D. Whether in certain cases of skin discoloration, visual inspection is


sufficient to make a diagnosis of skin cancer.

E. Whether hospitals in other parts of the country have purchased such


imaging machines.

解析: 题目是针对诊所作决定(clinic to make its decision


)这件事情,所以在看文章的时候要注意“决定(decision)”的内
容。

原文描述的事件包含:Westville医院购买了一台设备能并能降
低成本,诊所为了降低成本也考虑购买一台类似设备。

此论证过程非常明显是一个“类比推理”,如果大家发现通过前
提中的对象推出了结论中的另外一个对象,即前提和结论分别在谈论
两个不同的对象,则这样的推理一定是类比推理。类比推理的正确答
案模式只有一种,即两个类比对象有共同性,可以进行类比,即“求
同”方向;两个类比对象差异性较大,不可进行类比,即“求异”方
向。【求同:答案要与文章说描述的事件方向相同。比如假设题,支
持题的答案均为“求同”。求异:答案要与文章中描述的事件方向相
反。比如削弱题的答案要和原文描述事件的方向相反。】比如小时候
妈妈问你为什么隔壁小明考了90分,而你只考了60分,你会不服气地
说“他是他,我是我”。这就是通过描述“他和我”是不同的,因而
两者不可对比的角度来进行削弱。另外一种答案模式从来没有出现过
,这里不作论述。

如果学生没有看出是类比推理,则文章描述的事件是“医院和小
诊所与购买设备并降低成本”之间的关系。带着这事件来看哪个选项
与其最有关联度。

A项讲“诊断时间”的差异,这和“购买设备与降低成本”无关
,排除此项。

B项讲“机器的运作原则”,和“购买设备与降低成本”完全无
关,排除此项。

C项讲“机器使用频率”,和成本肯定有交集,如果机器买来却
不使用的话那肯定亏本。所以此项与原文沾边,可放着待选。

D项,“certain cases”这两个词提示该选项犯了“没有代表性
”的错误。而且此项并没有告诉我们是否可以通过“医院如何做”推
出“诊所也如何做”,属于无关选项。演绎题就是讨论是否可以通过
前提推出结论。

E项,“别的地方购买”和小诊所并没有关系,排除此项。

只有C项沾边,答案为C选项。

如果从类比推理的逻辑推理角度来分析此题,C项在论述两者的
差异,属于之前描述过的类比答案模型。A项也涉及两者的差异,但
并不是所有描述两个类比对象差异的选项都可以选择,这个差异必须
和原文的话题沾边。A项的诊断时间和“成本”并没有联系,除非文
章告诉我们诊断时间与成本有关,否则我们并不知情,不知情就属于
“没法判断”,不可选。
与“类比推理”类似的推理套路在GRE的逻辑考试中还有两类:“因
果推理”和“果因推理”。在本书后续章节的例题分析中,会提到相关知
识点。

例3 Though sucking zinc lozenges has been promoted as a treatment for


the common cold, research has revealed no consistent effect. Recently,
however, a zinc gel applied nasally has been shown to greatly reduce the
duration of colds. Since the gel contains zinc in the same form and
concentration as the lozenges, the greater effectiveness of the gel must be due
to the fact that cold viruses tend to concentrate in the nose, not the mouth.

In order to evaluate the argument, it would be most helpful to determine


which of the following?

A. Whether zinc is effective only against colds, or also has an effect on


other virally caused diseases.

B. Whether there are remedies that do not contain zinc but that, when
taken orally, can reduce the duration of colds.

C. Whether people who frequently catch colds have a zinc deficiency.

D. Whether either the zinc gel or the lozenges contain ingredients that
have an impact on the activity of the zinc.

E. Whether the zinc gel has an effect on the severity of cold symptoms,
as well as on their duration.

解析: 评估推理链,找文章的前提与结论。原文的事件一共包
含这些:“通过吃锌锭(zinc lozenges)治疗感冒没有持续效果。
一个锌膏(zinc gel)应用在鼻子上可以缩短感冒周期。因为种类和
浓度一致,锌膏效果好一定是因为感冒病毒集中在鼻子而不是嘴巴上
。”归纳所读到的内容:“锌锭与锌膏治疗感冒时间与病毒在鼻子还
是嘴巴有关系”。

A项中“other disease”表明此项为无关选项。
B项中“不包含锌的治疗”(do not contain zinc)表明此项和
文章中说的锌锭与锌膏都没关系,谈论对象不是文章中所描述的内容
,当然不可能成为答案。

C项与“患感冒的频率”与“病毒的位置”的话题完全无关,排
除该项。

D项所谈论的对象与内容至少是我们在原文中都看到过的原词,
则表明此项肯定沾边,放着此项,待比较。

E项的duration一词在文章中出现过,但是“严重程度”(sever
ity of cold symptoms)在原文中没出现,所以排除。

只有D选项沾边,为正确答案。

从逻辑推理角度来分析,D项是在论述有没有其他原因(简称“
他因”)会导致疗效不同。如果是因为两个产品其中一种含有一些成
分会让锌失去部分效果,那么最终导致治疗感冒效果不同的就不是文
章中所述的“病毒的位置”,而是这些让锌失效的成分导致的。因此
D项在研究是否有他因导致结论。

若通过逻辑套路来分析,此题属于果因推理论证。果因推理是指文
章先论述一个结果,然后再说明导致这个结果的原因。文章先有果再有
因,所以叫果因推理。在实际做题过程中,如果发现文章的结论在时间
上是在原因之前的,则一定是果因推理。比如你通过一个人长得很黑,
推出他是做户外工作的。此时前提是“现象”(一个人长得很黑),推
出“结论”(他做户外工作)。但从时间先后顺序来看,先有“在户外工
作”,才会有“长得黑”这个现象。所以如果一个推理链的结论在前提之
前,则可归为“果因推理”,即通过“现象”,得到一个“推出现象的原
因”。此题就是结论先于前提的“果因推理”。果因推理类试题的答案类
型有两个,其中最可能的就是“排除他因”或“给出他因”。如果题目问的
是“求同”方向,答案即为“排除他因”——排除其他推导出结论的原因,
从而增加对文章中所述原因的肯定。同理,在“求异”的试题中,答案即
为“给出他因”,如同此题。还有一个更有效的答题技巧,即如果发现文
章是通过一个现象得到一个说法,那么基本就是果因推理。此题即为先
说“现象”——疗效更好,再说“原因”——病毒在人体的位置不同。另外
一种果因推理的这里先不谈论,后文会再提到。

例4 Trancorp currently transports all its goods to Burland Island by


truck. The only bridge over the channel separating Burland from the
mainland is congested, and trucks typically spend hours in traffic. Trains can
reach the channel more quickly than trucks, and freight cars can be
transported to Burland by barges that typically cross the channel in an hour.
Therefore, to reduce shipping time, Trancorp plans to switch to trains and
barges to transport goods to Burland.

Which of the following would be most important to know in


determining whether Trancorp's plan, if implemented, is likely to achieve its
goal?

A. Whether transportation by train and barge would be substantially less


expensive than transportation by truck.

B. Whether there are boats that can make the trip between the mainland
and Burland faster than barges can.

C. Whether loading the freight cars onto barges is very time consuming.

D. Whether the average number of vehicles traveling over the bridge


into Burland has been relatively constant in recent years.

E. Whether most trucks transporting goods into Burland return to the


mainland empty.

解析: 此题评价计划是否能达到目标。要找到文章中的“计划
与目标”。

原文读成:“T公司用卡车运输货物。唯一的桥很拥挤,会花费
很多时间。火车和船的运输方式更快。因此,T公司计划用火车与船
去运输。”

原文话题为“用火车与船运输”和“节省时间”的关系。这两个
话题正好就是“计划”与“目标”。
A项:“贵不贵”不是文章讨论的核心,原文只讨论“节省时间
”的话题,贵不贵文章没提到,我们不知道T公司是否在乎价格。排
除此项。

B项:发现此项有一个对比,然后立刻思考比较对象和比较内容
是否在原文出现。很明显,“小船(boat)”和“驳船(barge)”
的对比不存在,所以排除此项。

C项:货运车厢(freight car)和时间的关系这个话题文章中讨
论过,留下此项。

D项:关注车辆多少,没有直接说到时间的话题,前四个选项对
比起来,C项可胜出。

E项:“是不是空车返回”完全和时间无关,排除此项。

答案为C项。

如果留下的沾边选项不止一个,那么需要借助逻辑推理或从其他角
度判断哪个选项更好一些。若只有一个选项沾边,无须推理,答案只能
是这个选项。

从论证类型来分析,此题又可被归纳为“计划与目标”类考题。论证
方法在GRE考试中,共有四种:类比论证,因果推理,果因推理,计划
目标类。如果题目问“计划”或和“计划”相关的词汇(比如“proposal”)即
为“计划目标类”。一般的试题考查推理链,我们需要在原文中总结的事
件是“前提和结论”。在“计划目标类”考题中,需要总结的事件就是“计
划与目标”,并且答案要针对“计划”的内容来谈,因为如果选项不是针
对“计划”在谈,何来支持或削弱某个计划呢。

“计划目标类”试题的答案形式可通过一个假想的例子来阐明:

原文: “余翔想要变得更美,所以余翔计划去韩国做整容手术。”

原文简单记成“目标→变美;计划→去韩国做手术”。

“计划目标类”试题从削弱角度出题方式有三种:
1. 假设计划达不成目标。比如某选项:“韩国的审美方式与中国的
大不一样。”这说明即便执行了计划,也达不到目标。

2. 假设计划本身可达成目标,出题方式又可以分为:

(1)计划不可执行。比如某选项:“去韩国整容手术费用太高,余
翔承受不了。”虽然计划本身有效,但不可行。

(2)计划可执行,但会产生不可接受的副作用。比如某选项:“去
韩国整容之后余翔长不出胡子了,这是余翔无论如何不能接受的。”在
这里可能会出一个陷阱,比如说了一个副作用,但是原文并没有说这个
副作用是做计划的那个对象不可接受的,则这个副作用不能成为答案。
比如“服用一种药物目的是治疗心脏病”,题目问哪个选项可削弱这个计
划(通过这种药物治疗心脏病),一个选项的内容是“这种药很贵”,这
个选项不可以成为答案,因为文章并没有说药品贵就承受不了。又比
如“一个营销计划可让公司增加50%收益”,题目问哪个选项可削弱这个
营销计划成功的可能性,某选项说“这个营销计划不道德”。如果原文中
没有涉及公司在乎道德的话题,那么“不道德”并不构成削弱因素,因为
我们的任务是推理“计划”是否可达到“目标”,目标外的考虑因素如果文
章没有提过,我们就都不知道且不用去管。

二、特色评价题练习与解析

解答评价题的过程和解答一般演绎题的过程类似,均是找沾边的选
项,只是评价题不需要再思考选项描述事件方向与题目所问是否一致。
下面将探讨一些有特色的评价题,并从中继续把一些细微的解题技巧列
举出来。

例1 Community activist: If Morganville wants to keep its central


shopping district healthy, it should prevent the opening of a huge SaveAll
discount department store on the outskirts of Morganville. Records from
other small towns show that whenever SaveAll has opened a store outside the
central shopping district of a small town, within five years the town has
experienced the bankruptcies of more than a quarter of the stores in the
shopping district.
The answer to which of the following would be most useful for
evaluating the community activist's reasoning?

A. Have community activists in other towns successfully campaigned


against the opening of a SaveAll store on the outskirts of their towns?

B. Do a large percentage of the residents of Morganville currently do


almost all of their shopping at stores in Morganville?

C. In towns with healthy central shopping districts, what proportion of


the stores in those districts suffer bankruptcy during a typical five-year
period?

D. What proportion of the employees at the SaveAll store on the


outskirts of Morganville will be drawn from Morganville?

E. Do newly opened SaveAll stores ever lose money during their first
five years of operation?

解析: reasoning(推理过程)和argument(推理链)没有区别
,依然要找文章的“前题与结论”。

原文描述的事件可读成:“为了让购物区商业生态健康,需要阻
止SaveAll开店。别的地方发生的事表明SaveAll开店之后,四分之一
的商店倒闭。”简单概括文章事件并且以话题为单位,就得出“Save
All开店”与“别的商店倒闭”的关系,带着这个的话题来看选项。
选项说到了“SaveAll开店”或选项说到了“SaveAll不开店”,都属
于一个话题,都需要放着待选。同理,选项说到了“别的商店倒闭或
不倒闭”也都需要放着待选。

A项可读成“有没有活动人士(activists)反对开店”。首先,
原文中并没有提到活动人士这个概念,其次活动人士在别的地方成功
地反对了SaveAll开店,那么SaveAll就不开店了吗?尽管活动人士反
对,SaveAll不顾反对依然开店,这种可能性也有。再者,在别的地
方活动人士即使成功地阻止了SaveAll开店,那么和文章中所说的Mor
ganville有何关联?在别的地方成功了,在Morganville这个地方也
会成功吗?我们不得而知。最后,此项说的话题是“活动人士和Save
All”的关系,并不是我们要找的“SaveAll开店”与“其他商店倒闭
”的话题,所以此项不可能正确。

B项可读成“居民在Morganville购物”,此项没有提到SaveAll
到来之后对其他商店有何影响,排除此项。

C项可读成“健康的商业区中公司倒闭的比例”,此项肯定与要
找的答案话题有部分交集,保留此项。

D项可读成“员工与Morganville”的关系,这个话题与要找的答
案话题没有太多交集,所以前四个选项对比来看,C项优选。

E项可读成“SaveAll自己损失钱”,我们要找的答案方向是Save
All开店之后,其他商店是否倒闭。此项并没有谈到对其他商店的影
响,因此不正确。SaveAll即使自己损失钱,那么损失钱的SaveAll会
不会对其他商店有影响?我们无法判断。

大家可以思考下,如果E项改成“SaveAll商店开业之后,其他商
店会损失钱”,这样E项可以成为答案吗?

例2 Journalist: Well-known businessman Arnold Bergeron has long


been popular in the state, and he has often talked about running for governor,
but he has never run. However, we have just learned that Bergeron has
fulfilled the financial disclosure requirement for candidacy by submitting a
detailed list of his current financial holdings to the election commission. So,
it is very likely that Bergeron will be a candidate for governor this year.

The answer to which of the following questions would be most useful in


evaluating the journalist's argument?

A. Has anybody else who has fulfilled the financial disclosure


requirement for the upcoming election reported greater financial holdings
than Bergeron?

B. Is submitting a list of holdings the only way to fulfill the election


commission's financial disclosure requirements?

C. Did the information recently obtained by the journalists come directly


from the election commission?

D. Have Bergeron's financial holdings increased in value in recent


years?

E. Had Bergeron also fulfilled the financial disclosure requirements for


candidacy before any previous gubernatorial elections?

解析: 评估推理链(argument)。

原文的事件:(1)Arnold一直说要竞选,但从来没有参加过选
举。(2)要披露财产,就要提交财产清单。(3)他非常可能会竞选

思考文章的推理链是什么。根据之前提到的方法,先思考结论是
什么事件,再思考最能直接导致结论的原因。这些原因就是前提。【
实战解题技巧:前提最多两条,如果超过两个原因把其概括为最多两
个前提】

此题结论为最后一句话,在文章最后一句话的“So”之前只谈论
了“披露财产”的事情,所以原文可记成为“披露财产”与“参加竞
选”之间的关系。

A项:别人和Arnold Bergeron没有关系,排除此项。

B项:可读成“提交财产清单是不是披露财产的唯一方式”,此
项的话题是“如何披露财产”,我们要找的答案方向是“披露财产”
与Arnold Bergeron之间的关系,但此项没有提到披露财产之后会如
何,所以不正确。用之前提过的例子做下补充说明:“如果你发财了
,你会捐钱给慈善机构”,要选择一个支持这样论证的选项。某选项
:“买彩票会让你发财。”这项只是说明了发财的方式,我们无法判
断你发财之后会不会捐钱。故不选此项。
C项:可读成“信息是否来自选举委员会”。通过此项可以引出
一个错误选项类型:如果选项只是在研究信息的来源,而不是针对信
息的内容来谈,此类选项必错,简称信息来源类错误。我们在概括文
章事件的时候,会把事件的内容总结出来,然后思考事件一是否会推
出事件二,不会把信息的来源概括进推理链。所以如果选项在研究信
息来源,可以直接排除。换句话说,即使信息来源不可靠,那么所谈
论事件内部的逻辑关系一定是不成立的吗?我们无法判断。可能我们
会觉得某个事件是假的,但是我们不需要关注事件的真假,要做的只
是关注事件之间是否有“逻辑关系”。

D项:“是否增值”和“竞选”的话题完全无关,排除此项。

E项:可读成“Arnold Bergeron之前是否披露过财产”,此项肯
定和“披露财产与竞选”的话题有联系,因此E选项为正确答案。

有同学可能会有疑问:E项是描述“过去如何做”,而原文是研
究将要如何做,这“过去”和“将来”难道不是两个事件吗?

答: (1)所有的GRE逻辑题都是找相对正确的选项,在五个选
项中只有E项和原文有话题交集,E项是相对最好的,所以无须纠结是
否为完美的答案。(2)原文其实有提到时间概念,“but he has ne
ver run”表明他过去没有竞选过。如果过去披露了财产,而过去也
没有竞选,那当然不能通过披露财产就推出现在要参加竞选,通过逻
辑意思来分析,答案依然为E项。

例3 Scientists have made genetic modifications to cotton to increase its


resistance to insect pests. According to farmers' report, the amount of
insecticide needed per acre to control insect pests was only slightly lower for
those who tried the modified seed than for those who did not. Therefore,
since the modified seed costs more than ordinary seed without producing
yields of higher market value, switching to the modified seed would be
unlikely to benefit most cotton farmers economically.

Which of the following would it be most useful to know in order to


evaluate the argument?
A. Whether farmers who tried the modified cotton seed had ever tried
growing other crops from genetically modified seed.

B. Whether the insecticides typically used on ordinary cotton tend to be


more expensive than insecticides typically used on other crops.

C. Whether for most farms who grow cotton it is their primary crop.

D. Whether the farmers who have tried the modified seed planted as
many acres of cotton, on average, as farmers using the ordinary seed did.

E. Whether most of the farmers who tried the modified seed did so
because they had previously had to use exceptionally large quantities of
insecticide.

解析: 文章读成:(1)修改基因的棉花能抵抗害虫。(2)杀
虫剂的使用量只能节省一点点。(3)修改过基因的种子更贵,所以
没有经济上的好处。

假设同学读完原文后,仍不知道如何总结推理链的话题,则哪个
选项在文章中曾读到过,就可以把这个选项先放着,因为沾边的选项
就可能正确。

A项:“种其他作物”和文章的话题无太多交集,此项可排除。

B项:有一个对比,应立刻思考有没有犯无关对比的错误。在原
文中没有与其他作物的对比,所以排除此项。

C项:“是不是主要作物”和文章中说的“节省杀虫剂、更贵的
种子、没有经济上的好处”无关联度,排除此项。

D项:“使用基因被修改的种子的农民和使用一般种子的农民种
植一样多的棉花”可读成“种植棉花多不多”,棉花多和经济好处处
于相同的方向,可留下待比较。在解答逻辑题和阅读题的过程中,只
要选项与原文的话题部分沾边,或无须推理,只要方向一致,即可先
放着待比较。
E项:可读成“种植修改过基因种子的农民”与“大量杀虫剂”
的关系。文章中有与“杀虫剂的量”类似的概念,留下E项。

D项和E项两个选项再进行对比。

对比两项前,大家可思考一下:逻辑题的答案只选那些不需要推
理,意思直接和原文话题相关的选项吗?肯定不是。如果真是这样的
话,逻辑题就太简单了,我们无须推理,所有题目找沾边的选项即可
。有些逻辑题必须借助推理,直接靠沾边思维会留下几个选项。只能
说大多数试题只有一个选项和原文事件沾边时,不需要推理。一个选
项如果通过常识可推理出与预测的答案事件方向相同,依然需要放着
待选。通过常识推理出的能够和原文产生关联度的选项也属于沾边选
项。只是在几个沾边选项对比时,优选不需要推理或推理步骤少的选
项。

此题的D和E项,哪个不用推理,直接说出了原文的事件?显然是
E项。如果没有E项,D项也可以成为答案,因为我们追求的是相对正
确的答案。大家在做题的时候可以推理,但推理只能靠每个人都会有
一样反应的常识性内容,如果对于一个话题内容不同人会有不同的反
应,则不属于常识性推理。例如“产量越大”能推出“价格越便宜”
吗?显然不是。有些产品的产量越大,需要的投资或者交的税会导致
价格上涨。在实际做题中,可这么理解深常识性推理:一个事件全世
界不同的人会有相同的反应,那么这件事情就是和常识性推理有关,
比如“冬天我们大概率会感觉到冷”。

其次,靠常识推理或联想的选项如果沾边,大家要知道这个选项
经过推理了,要优先找一个不推理的选项。如果不存在不推理即与原
文沾边的选项,则之前经过推理的选项依然可以成为答案。

例4 Fish currently costs about the same at seafood stores throughout


Eastville and its surrounding suburbs. Seafood stores buy fish from the same
wholesalers and at the same prices, and other business expenses have also
been about the same. But new tax breaks will substantially lower the cost of
doing business within the city. Therefore, in the future, profit margins will be
higher at seafood stores within the city than at suburban seafood stores.
For the purposes of evaluating the argument, it would be most useful to
know whether

A. more fish wholesalers are located within the city than in the
surrounding suburbs.

B. any people who currently own seafood stores in the suburbs


surrounding Eastville will relocate their businesses nearer to the city.

C. the wholesale price of fish is likely to fall in the future.

D. fish has always cost about the same at seafood stores throughout
Eastville and its surrounding suburbs.

E. seafood stores within the city will in the future set prices that are
lower than those at suburban seafood stores.

解析: 原文的事件包含:(1)在城市和郊区鱼的成本一样。(
2)进价和运营成本差不多。(3)城市里税更低。(4)城市的利润
比郊区的利润更高。

读完这四件事之后,大家可能会找出前三件事推出第四件事的一
些理由。如果想到了一个理由,那么直接看哪个选项是针对这个理由
的,这样做题能更快。请问,这样思考的方式正确吗?显然不正确。
可能有些试题这样做对了,但这样的方式不可被当作逻辑题的标准解
法。如何让选项与原文产生联系是出题机构的安排,不是考生的主观
想象。因此我们只能记住原文的话题,然后判断哪个选项与原文有逻
辑意思交集,不能自己“预定答案”。

A项读成“批发商人数多”。文章中已经提到了“价格不变”,
所以人数多不会对价格产生影响,此项与文章事件无联系,排除此项

B项读成“人会如何做”。此项与“城市与郊区”哪个地方赚钱
多无关,排除此项。
C项可读成“将来价格可能会下降”。“可能会下降”表示或下
降或不下降。此项并不能支持或削弱原文结论,属于无关选项。

D项可读成“鱼的成本是一样的”。此项和原文说的事一样,无
法评估原文。此项不正确。

E项可读成“城市海鲜店会不会设置比郊区店更低的售价”。此
项的话题是“售价低不低”,售价一定会和利润有联系,只有E项沾
边。

得出答案为E选项。
第六章 句子作用题(黑体题)

一、句子作用题的解法

1. 题型介绍

首先来看两道完整的句子作用题。

例1 Historian: Newton developed mathematical concepts and techniques


that are fundamental to modern calculus. Leibniz developed closely
analogous concepts and techniques. It has traditionally been thought that
these discoveries were independent. Researchers have, however, recently
discovered notes of Leibniz' that discuss one of Newton's books on
mathematics. Several scholars have argued that since the book includes a
presentation of Newton's calculus concepts and techniques, and since the
notes were written before Leibniz’ own development of calculus concepts
and techniques, it is virtually certain that the traditional view is false. A
more cautious conclusion than this is called for, however. Leibniz’ notes are
limited to early sections of Newton's book, sections that precede the ones
in which Newton's calculus concepts and techniques are presented.

In the historian's reasoning, the two boldfaced portions play which of the
following roles?

A. The first provides evidence in support of the overall position that the
historian defends; the second is evidence that has been used to support an
opposing position.

B. The first provides evidence in support of the overall position that the
historian defends; the second is that position.

C. The first provides evidence in support of an intermediate conclusion


that is drawn to provide support for the overall position that the historian
defends; the second provides evidence against that intermediate conclusion.

D. The first is evidence that has been used to support a conclusion that
the historian criticizes; the second is evidence offered in support of the
historian's own position.

E. The first is evidence that has been used to support a conclusion that
the historian criticizes; the second is further information that substantiates
that evidence.

题目问文章中两处黑体的部分(boldfaced portions)扮演什么角
色。句子作用题不需要读题目,看到原文有黑体句,题目的问法全部都
是要找出黑体句在文章中的功能。

句子作用题又称为“黑体题”或“黑脸题”,往往是文章中有两个或一
个黑体句。从来没有过两个以上黑体句的“句子作用题”。

一个黑体句的句子作用题的举例如下。

例2 Hunter: Hunters alone are blamed for the decline in Greenrock


National Forest's deer population over the past ten years. Yet clearly, black
bears have also played an important role in this decline. In the past ten
years, the forest's protected black bear population has risen sharply, and
examination of black bears found dead in the forest during the deer hunting
season showed that a number of them had recently fed on deer.

In the hunter's argument, the boldface portion plays which of the


following roles?

A. It is the main conclusion of the argument.

B. It is an objection that has been raised against the main conclusion of


the argument.

C. It is a judgment that the argument opposes.

D. It is a finding that the argument seeks to explain.

E. It provides evidence in support of the main conclusion of the


argument.

2. 句子作用题的具体解题步骤

题目问的是两句话的功能,因此如果句间关系简单的话,不读整篇
文章就可以得知句子之间的关系。如果文章较难,或文章当中转折比较
多,建议先读完文章,然后再判断句子的功能。如果只有一个黑体句,
则做题时无须判断两个句子之间的逻辑关系。

句子在文章中的具体功能其实与前文提到的文章六大元素密切相
关。换个角度讲,句子作用题考查的就是考生对于句子信息的属性是否
清楚,即所读的句子是背景、前提、中间性结论还是最终的结论等。在
解题过程中,可以结合文章六大元素,以及下述三大步骤得出答案。

步骤1:找出“大方向”。

这里的“大方向”是指画线句之间的逻辑关系,以及文章的作者对于
句子内容的态度评价。

任意两个句子之间的逻辑关系只有三种:顺接、逆接、描述新话
题。比如“做人要学习孔子,做事要学习老子”,听到这句话之后的回应
只有三种可能:要么同意,要么不同意,要么说一个新话题。在句子作
用题中,文章共三四句话,所以句子与句子间的关系不可能是开启新话
题,因此句间关系不是顺接(同意上句话),就是逆接(反驳上句
话)。只有一个黑体句的考题无须判断句间逻辑关系。

若能判断作者最终结论是哪句话,则与结论态度相反的句子全部为
作者持有否定态度的句子。有一个很好的技巧来判断句间关系,即“没
有直接驳斥意思的词(比如however、but)或逻辑意思,即为‘顺接’的
逻辑关系”。

例 Scientists typically do their most creative work before the age of


forty. It is commonly thought that this happens because aging by itself
brings about a loss of creative capacity. However, a study has found that
almost all scientists who produce highly creative work beyond the age of
forty entered their fields late and less than a dozen years before their creative
breakthroughs. Since creative breakthroughs by scientists under forty also
generally occur within a dozen years of the scientists' entry into the field,
the study's finding strongly suggests that the real reason why scientists over
forty rarely produce highly creative work is not due to age but rather because
most have spent too long in their fields.

In the argument given, the two portions in boldface play which of the
following roles?

A. The first is the position that the argument as a whole depends; the
second is evidence that is advanced as part of that evidence.

B. The first and second are both claims that have been advanced in
support of a position that the argument as a whole opposes.

C. The first is an explanation that the argument challenges; the second


provides evidence in support of a competing explanation that the argument
defends.

D. The first is an explanation that the argument challenges; the second is


evidence that has been used against an alternative explanation that the
argument defends.

E. The first is an explanation that the argument defends; the second is


evidence that has been used to challenge that explanation.

解析: 本文较简单。假设在实际考试中遇到一篇类似上文逻辑
关系但语言难懂的文章,靠逻辑意思来判断两句话的方向及作者态度
就困难了。此时,发现两句话之间有一个转折词(However),所以
两句话是一个负相关(逆接)的逻辑关系。从全文最后一句话的“th
e study's finding strongly suggest that”可推测出本句表达作
者最终的观点。第二个黑体句作为最后一句话的原因呈现(因为有si
nce)。所以文章的“大方向”是:(1)两句话之间“负相关”。(
2)作者支持第二句话,反对第一句话所支持的观点。

有了“大方向”之后,带着“大方向”来看选项:

A项:前半句“the argument as a whole depends”导致此项不


正确。因为作者反对第一句话的观点,排除此项。

B项:两句话不是正相关逻辑,排除此项。

C项:前半句和作者态度相反,没法挑错。后半句和作者态度一
致,也无法挑错。可留下此项。

D项:前半句的内容没法挑错。后半句的内容是“反驳作者支持
一个的观点”就错了,第二句话和作者的最终态度一致。排除此项。

E项:前半句就错了,原因同A选项。排除此项。

只有C挑不出错,即为此题答案。

步骤2:结合逻辑文章六大元素,思考句子功能。

绝大多数句子作用题通过“步骤1”已经可以解决了,有些难度大的
试题才会用到“步骤2”。在现有GRE、GMAT逻辑题库中只有不多于2道
句子作用题才会用到“步骤3”。通过下面的例题来详细了解。

例 Editorial: An arrest made by a Midville police officer is provisional


until the officer has taken the suspect to the police station and the watch
commander has officially approved the arrest. Such approval is denied if the
commander judges that the evidence on which the provisional arrest is based
is insufficient. A government efficiency expert has found that almost all
provisional arrests meet standards for adequacy of evidence that watch
commanders enforce. The expert therefore recommends that the watch
commander's approval should no longer be required since the officers'
time spent obtaining approval is largely wasted. This recommendation
should be rejected as dangerous, however, since there is no assurance that
the watch commanders' standards will continue to be observed once approval
is no longer required.

In the editorial, the two portions in boldface play which of the following
roles?
A. The first is a recommendation made by the editorial; the second
acknowledges a potential objection against that recommendation.

B. The first is a proposal against which the editorial is directed; the


second is a judgment reached by the editorial concerning that proposal.

C. The first provides evidence in support of a recommendation that the


editorial supports; the second is the conclusion reached by the editorial.

D. The first is a position that the editorial challenges; the second is a


judgment that was made in support of that challenged position.

E. The first is a recommendation that the editorial questions; the second


provides evidence against that recommendation.

解析: 通过意思来判断,第二句话否定第一句提及的建议,第
二句话之后文章结束,所以第二句话为作者最终的观点。故而“大方
向”是作者反驳第一句,支持第二句;两句话之间是“逆接”的逻辑
关系。

带着“大方向”来看选项:

A项:第一个黑体句和作者观点是相反的逻辑关系,所以此项的
前半句就不正确。排除此项。

B项:前半句表示第一句话所提建议和作者观点相反,不能说错
;后半句和作者态度方向一致,该项没有挑出错误,可以放着此项待
比较。

C项:前半句和A选项犯的错误一样,排除此项。

D项:前半句认为第一句话和作者的态度相反(challenges),
不能说错。后半句讲“第二个黑体句支持第一个黑体句”,完全错误
,排除此项。

E项:前半句讲作者反驳一个建议(recommendation);后半句
讲第二个黑体句和第一个是相反(against)的逻辑关系,“大方向
”来看,E项也符合,需要对比B项和E项之间的差异。

比较B项和E项时,利用“步骤2”,借助六个功能思考黑体句的
角色是什么。第二个黑体句是结论(conclusion)。B项的后半句使
用“judgement”一词,E项的后半句是使用“evidence”一词,显然
答案为B选项。

步骤3:找出错概率最小的选项。

一般情况下,在GRE考试中句子作用题靠前两个步骤已经可以解
题,没有试题需要用到第3个步骤。在GMAT考试中会碰到需要运用步
骤3的情况,所以简单地提一下步骤3的内容。

所谓出错概率小就是一个选项没有说明好坏具体评价,说得越空洞
越抽象就越不会被挑出错误。比如第一句话被读出作者对其有负面评
价,选项A讲“作者反驳第一句话”,肯定放着A项待比较。选项B讲“作
者对第一句话有一个评价”。不管是好评价还是坏评价,都属于评价,
所以B的出错概率会比A小一些,得出B项为答案。这一方法在其他的逻
辑题和阅读题中一样适用,后文会有具体说明。

二、练习与解答

例1 A product that represents a clear technological advance over


competing products can generally command a high price. Because
technological advances tend to be quickly surpassed and companies want
to make large profits while they still can, many companies charge the greatest
price the market will bear when they have such a product. But large profits
on the new product will give competitors a strong incentive to quickly
match the new product's capabilities. Consequently, the strategy to
maximize overall profit from a new product is to charge less than the greatest
possible price.

In the argument above, the two portions in boldface play which of the
following roles?
A. The first is an assumption that forms the basis for a course of action
that the argument criticizes; the second presents the course of action endorsed
by the argument.

B. The first is a consideration raised to explain the appeal of a certain


strategy; the second is a consideration raised to call into question the wisdom
of adopting that strategy.

C. The first is an assumption that has been used to justify a certain


strategy; the second is a consideration that is used to cast doubt on that
assumption.

D. The first is a consideration raised in support of a strategy the


argument endorses; the second presents grounds in support of that
consideration.

E. The first is a consideration raised to show that adopting a certain


strategy is unlikely to achieve the intended effect; the second is presented to
explain the appeal of that strategy.

解析: 文章的内容可读成:“体现科技进步的产品可以卖高价
。科技优势会被超越,公司希望赚取大量利润,拥有这种产品的公司
收取最高的费用。但是高额利润会刺激别的公司也获得这样的新功能
,因此,最好的策略是……。”

读文章的目标是了解“大方向”以及黑体句有哪些功能,所以原
文最后一句话不需要读完就能判定最后一句话是结论。句子作用题不
考查段落意思只考查逻辑关系和作者的态度方向,读出之前的句子意
思也是为了了解句子逻辑关系。

读出意思之后,思考“大方向”与“六大功能”。

第一个黑体句和第二个黑体句之间有一个转折词(But),通过
意思也能得知第二个黑体句是前提,因为它支持最后一句结论句。第
一个黑体句是与结论方相反的前提(counter-premise),但第一个
黑体句的内容不能被作者否定,因为这是一个客观事实。学生们切记
:“作者不会对一个现象作出支持或削弱的评价。作者只能对基于一
个现象得出的主观说法进行评价。”因为客观现象是既存事实,不需
要被作者支持或削弱。从意思来看 “ technological advances ten
d to be quickly surpassed”是一个客观事实,故不可被削弱。

选项分析:

A项:前半句不能挑出错;第二个黑体句“别的公司也会开发一
样的新功能”很明显是一个说法,并不是一个行动(course of acti
on),所以A项的后半句有些问题,待选。

B项:前半句没挑出错;后半句讲“第二句削弱第一句”,大方
向来看貌似可行,先放着B选项,总之B项优先A选。

C项:前半句和B项的前半句一样,不能挑出错;后半句讲第二句
削弱第一句,无法判定。和B项比较,发现和B项的区别在于:C项是
直接削弱第一句的内容。因为第一个黑体句话是一个事实,客观事实
不能被削弱,第二个黑体句是削弱第一个句子所对应的行为措施。所
以B优于C项。

D项:前半句很明显有错,排除此项。

E项:第二句话明显不能说明之前策略的吸引力,排除此项。

此题B项和C项在后半句有差异,考生们需要先思考选项差异,带
着选项差异再回原文核实哪个选项更合理,此题答案为B项。

例2 Historian: In the Drindian Empire, censuses were conducted


annually to determine the population of each village. Village census records
for the last half of the 1600's are remarkably complete. This very
completeness makes one point stand out; in five different years, villages
overwhelmingly reported significant population declines. Tellingly, each of
those five years immediately followed an increase in a certain Drindian tax.
This tax, which was assessed on villages, was computed by the central
government using the annual census figures. Obviously, whenever the tax
went up, villages had an especially powerful economic incentive to minimize
the number of people they recorded; and concealing the size of a village's
population from government census takers would have been easy. Therefore,
it is reasonable to think that the reported declines did not happen.

In the historian's argument, the two portions in boldface play which of


the following roles?

A. The first supplies a context for the historian's argument; the second
acknowledges a consideration that has been used to argue against the position
the historian seeks to establish.

B. The first presents evidence to support the position that the historian
seeks to establish; the second acknowledges a consideration that has been
used to argue against that position.

C. The first provides a context for certain evidence that supports the
position that the historian seeks to establish; the second is that position.

D. The first is a position for which the historian argues; the second is an
assumption that serves as the basis of that argument.

E. The first is an assumption that the historian explicitly makes in


arguing for a certain position; the second acknowledges a consideration that
calls that assumption into question.

解析: 第二个黑体句前有Therefore一词,所以很明显第二个黑
体句是全文的结论。第二个黑体句的内容和“villages overwhelmin
gly reported significant population declines”这句话呈相反的
逻辑关系。第一个黑体句是一个背景性信息,可当作事实来理解。两
个黑体句的逻辑关系是:基于黑体句1,后文经过论证得出“人口下
降”的观点,黑体句2反驳了“人口下降”的观点。但不能得出黑体
句2反驳了黑体句1,因为事实本身不能被反驳,只能反驳基于事实所
得出的观点或评价。

A项:后半句“第二句话和作者观点相反(against the positio


n the historian seeks to establish)”与原文不符合,错误。

B项:前半句“黑体句1支持作者想要确立的一个观点”与之前分
析的方向不一致,排除此项。

C项:此项和B项错误一样,排除。

D项:第一个黑体句不是立场(position),而是一个事实。前
半句错误。

E项:“黑体句1支持一个观点;黑体句2削弱之前的观点”并没
有找到错误,得出答案为E项。

有同学可能会问“事实”可以是“assumption”假设条件吗?

答案: 可以。

附:标准逻辑六大元素的详细描述

前提(premise)

属性:a fact or an opinion, a description, historical information,


statistical or numerical data, or a comparison of things, etc

信号词:because of, because, since, due to, the reason is, for, and as a
result of等。

中间性结论(intermediate conclusion)

属性:A premise supports a conclusion,and that conclusion then


supports a further conclusion. The first conclusion is called the intermediate
conclusion(also known as the secondary conclusion). The second
conclusion can be called the final conclusion to distinguish it from the
intermediate conclusion.
结论(conclusion)

属性:the author's main opinion or claim; can be in the form of a


prediction, a judgment of quality or merit, or a statement of causality.

信号词:therefore, thus, so, thereby, clearly, as a result, for this reason,


this demonstrate, they conclude that, and consequently等。

背景信息(background information)

属性:often present, but not always.

信号词:almost always fact-based; can be in almost any form:


historical information, numericalor other data, descriptions of plans or ideas,
definitions of words or concepts等。

与作者态度相反的前提和结论(counter-premise & counter-


conclusion or opposing conclusion)

属性:An opposing conclusion is simply a conclusion that goes against


the author's main conclusion. counter-premise即是支持counter-conclusion的
前提。

例1 Although the earliest surviving Greek inscriptions written in an


alphabet date from the eighth century B.C., a strong case can be made that
the Greeks actually adopted alphabetic writing at least two centuries
earlier. Significantly, the text of these earliest surviving Greek inscriptions
sometimes runs from right to left and sometimes from left to right. Now, the
Greeks learned alphabetic writing from the Phoenicians, and in the process
they would surely have adopted whatever convention the Phoenicians
were then using with respect to the direction of writing. Originally,
Phoenician writing ran in either direction, but by the eighth century B.C. it
had been consistently written from right to left for about two centuries.

In the argument given, the two portions in boldface play which of the
following roles?
A. The first is the position that the argument seeks to establish; the
second reports a discovery that has been used to support a position that the
argument opposes.

B. The first is the position that the argument seeks to establish; the
second presents an assumption on which the argument relies.

C. The first presents evidence that is used in support of the position that
the argument seeks to establish; the second presents an assumption on which
the argument relies.

D. The first is an objection raised against a position that the argument


opposes; the second is the position that the argument seeks to establish.

E. The first is an objection raised against a position that the argument


opposes; the second is evidence that has been used to support that position.

解析: 假设考生没有读懂此题。从形式和内容来看,并不存在
内容对立的情况。根据“要么逆接,要么顺接”的原则,可认为这两
句话是顺接的关系,带着这样的关系来看选项。

A项:opposes(反对)一词提示此项不正确,排除此项。

B项:不可说错,放着此项。

C项:此项和B项的差异在于前半句,B项前半句认为黑体句本身
就是观点,C项前半句认为黑体句是支持观点句的一个证据。通过第
一个黑体句之前的信息“a strong case can be made”得出第一个
黑体句是“观点”,因此B项更合理。

D、E项:D项和E项中opposes(反对)一词说明这两项都不对。

答案为B选项。

例2 Several of a certain bank's top executives have recently been


purchasing shares in their own bank. This activity has occasioned some
surprise, since it is widely believed that the bank, carrying a large number of
bad loans, is on the brink of collapse. Since the executives are well placed to
know their bank's true condition, it might seem that their share purchases
show that the danger of collapse is exaggerated. However, the available
information about the bank's condition is from reliable and informed sources,
and corporate executives do sometimes buy shares in their own company in a
calculated attempt to calm worries about their company's condition. On
balance, therefore, it is likely that the executives of the bank are
following this example.

In the argument given, the two boldfaced portions play which of the
following roles?

A. The first describes the circumstance the explanation of which is the


issue that the argument addresses; the second states the main conclusion of
the argument.

B. The first describes the circumstance the explanation of which is the


issue the argument addresses; the second states a conclusion that is drawn in
order to support the main conclusion of the argument.

C. The first provides evidence to defend the position that the argument
seeks to establish against opposing positions; the second states the main
conclusion of the argument.

D. The first provides evidence to support the position that the argument
seeks to establish; the second states a conclusion that is drawn in order to
support the argument's main conclusion.

E. Each provides evidence to support the position that the argument


seeks to establish.

解析: 文章可读成“现象+说法1 +however +说法2”。第一个


黑体句是一个现象,因此不可被削弱或支持,黑体句1支持了作者之
后削弱的一个说法(“it might seem that their share purchases
show that the danger of collapse is exaggerated”)。第二个
黑体句中有“therefore”(因此),提示该句是文章最终的结论。
因此,第二个黑体句是作者支持的,第一个黑体句支持了作者反对的
一个说法。

A项:找不出错,可先放着此项。

B项:第二个黑体句本身就是结论,此项的后半句出错。

C项:前半句“position that the argument seeks to establi


sh against opposing positions”提示此项错误。

D项:此项的后半句犯了和B项一样的错误,排除此项。

E项:此项和B项、D项犯的错误一样,第二个黑体句本身就是结
论,并不是支持文章结论的证据。

答案为A选项。
第七章 混合逻辑题练习

GRE逻辑

(扫码看视频)

1. Although 90 percent of the population believes itself to be well


informed about health care, only 20 percent knows enough about DNA. So
apparently at least 80 percent of the population does not know enough about
medical concepts to make well-informed personal medical choices or to make
good public policy decisions about health care.

The argument's reasoning is questionable because the argument fails to


demonstrate that

A. those people who can understand news stories about DNA are able to
make well-informed personal medical choices.

B. more than 20 percent of the population needs to be well informed


about health care for good public policy decisions about health care to be
made.

C. one's being able to make well-informed personal medical choices


ensures that one makes good public policy decisions about health care.
D. an understanding of DNA is essential to making well-informed
personal medical choices or to making good public policy decisions about
health care.

E. since 90 percent of the population believes itself to be well informed


about health care, at least 70 percent of the population is mistaken in that
belief.

2. During the 1980's, Japanese collectors were very active in the market
for European art, especially as purchasers of nineteenth-century Impressionist
paintings. This striking pattern surely reflects a specific preference on the part
of many Japanese collectors for certain aesthetic attributes they found in
nineteenth-century Impressionist paintings.

Which one of the following, if true, most strongly supports the


explanation above?

A. Impressionist paintings first became popular among art collectors in


Europe at the beginning of the twentieth century.

B. During the 1980's, the Japanese economy underwent a sustained


expansion that was unprecedented in the country's recent history.

C. Several nineteenth-century Impressionist painters adopted certain


techniques and visual effects found in Japanese prints that are highly
esteemed in Japan.

D. During the 1960s and the 1970s, the prices of nineteenth-century


Impressionist paintings often exceeded the prices of paintings by older
European masters.

E. During the 1980's, collectors from Japan and around the world
purchased many paintings and prints by well-known twentieth-century
Japanese artists.

3. Because of the recent recession in Country A most magazines


published there have experienced decreases in advertising revenue, so much
so that the survival of the most widely read magazines is in grave doubt. At
the same time, however, more people in Country A are reading more
magazines than ever before, and the number of financially successful
magazines in Country A is greater than ever.

Which one of the following, if true, most helps to resole the apparent
discrepancy in the information above?

A. Most magazines reduce the amount they charge for advertisement's


during a recession.

B. The audience for a successful television show far exceeds the


readership of even the most widely read magazine.

C. Advertising is the main source of revenue only for the most widely
read magazines; other magazines rely on circulation for their revenue.

D. Because of the recession, people in Country A have cut back on


magazine subscriptions and are reading borrowed magazines.

E. More of the new general interest magazines that were launched this
year in Country A have survived than survived in previous years.

4. The gray squirrel, introduced into local woodlands ten years ago,
threatens the indigenous population of an endangered owl species, because
the squirrels' habitual stripping of tree bark destroys the trees in which the
owls nest. Some local officials have advocated setting out poison for the gray
squirrels. The officials argue that this measure, while eliminating the
squirrels, would pose no threat to the owl population, since the poison would
be placed in containers accessible only to squirrels and other rodents.

Which one of the following, if true, most calls into question the officials'
argument?

A. One of the species whose members are likely to eat the poison is the
red squirrel, a species on which owls do not prey.

B. The owls whose nesting sites are currently being destroyed by the
gray squirrels feed primarily on rodents.
C. No indigenous population of any other bird species apart from the
endangered owls is threatened by the gray squirrels.

D. The owls that are threatened build their nests in the tops of trees, but
the gray squirrels strip away back from the trunks.

E. The officials' plan entails adding the poison to food sources that are
usually eaten by rodents but not by other animals.

Questions 5-6

Sales manager: Last year the total number of meals sold in our company
restaurants was much higher than it was the year before. Obviously
consumers find our meals desirable.

Accountant: If you look at individual restaurants, however, you find that


the number of meals sold actually decreased substantially at every one of our
restaurants that was in operation both last year and the year before. The
desirability of our meals to consumers has clearly decreased, given that this
group of restaurants—the only ones for which we have sales figures that
permit a comparison between last year and the year before—demonstrates a
trend toward fewer sales.

5. If the sales figures cited by the accountant and the sales manager are
both accurate, which one of the following must be true?

A. The company opened at least one new restaurant in the last two years.

B. The company's meals are less competitive than they once were.

C. The quality of the company's meals has not improved over the last
two years.

D. The prices of the company's meals have changed over the past two
years.

E. The market share captured by the company's restaurants fell last year.
6. Which one of the following, if true, most seriously calls into question
the accountant's argument?

A. The company's restaurants last year dropped from their menus most
of the new dishes that had been introduced the year before.

B. Prior to last year there was an overall downward trend in the


company's sales.

C. Those of the company's restaurants that did increase their sales last
year did not offer large discounts on prices to attract customers.

D. Sales of the company's most expensive meal contributed little to the


overall two-year sales increase.

E. Most of the company's restaurants that were in operation throughout


both last year and the year before are located in areas where residents
experienced a severe overall decline in income last year.

7. Problems caused by the leaching of pollutants from dumps and


landfills are worst in countries with an annual per capita economic output of
$4,000 to $5,000, and less severe for considerably poorer and considerably
richer countries. This is so because pollution problems increase during the
early stages of a country's industrial development but then diminish as
increasing industrial development generates adequate resources to tackle such
problems. Therefore, problems caused by such leaching in Country X, where
the annual per capita economic output is now $5,000, should begin to
diminish in the next few years.

Which one of the following is an assumption on which the argument


depends?

A. Within the next few years, Country X will impose a system of fines
for illegal waste disposal by its industrial companies.

B. Countries surrounding Country X will reduce the amount of pollution


that their factories release into the air and water.
C. Industrial development in Country X will increase in the next few
years.

D. Country X will begin the process of industrialization in the next few


years.

E. No other country with a similar amount of industrial development has


pollution problems that are as severe as those in Country X.

8. The stable functioning of a society depends upon the relatively long-


term stability of the goals of its citizens. This is clear from the fact that unless
the majority of individuals have a predictable and enduring set of aspirations,
it will be impossible for a legislature to craft laws that will augment the
satisfaction of the citizenry, and it should be obvious that a society is stable
only if its laws tend to increase the happiness of its citizens.

The claim that a society is stable only if its laws tend to increase the
happiness of its citizens plays which one of the following roles in the
argument?

A. It is the conclusion of the argument.

B. It helps to support the conclusion of the argument.

C. It is a claim that must be refuted if the conclusion is to be established.

D. It is a consequence of the argument.

E. It is used to illustrate the general principle that the argument


presupposes.

9. Astronauts who experience weightlessness frequently get motion


sickness. The astronauts see their own motion relative to passing objects, but
while the astronauts are weightless their inner ears indicate that their bodies
are not moving. The astronauts' experience is best explained by the
hypothesis that conflicting information received by the brain about the body's
motion causes motion sickness.

Which one of the following, if true, provides the strongest additional


support for the hypothesis above?

A. During rough voyages ship passengers in cabins providing a view of


the water are less likely to get motion sickness than are passengers in cabins
providing no view.

B. Many people who are experienced airplane passengers occasionally


get motion sickness.

C. Some automobile passengers whose inner ears indicate that they are
moving and who have a clear view of the objects they are passing get motion
sickness.

D. People who have aisle seats in trains or airplanes are as likely to get
motion sickness as are people who have window seats.

E. Some astronauts do not get motion sickness even after being in orbit
for several days.

10. In order to pressure the government of Country S to become less


repressive, some legislators in Country R want to ban all exports from R to S.
Companies in R that manufacture telecommunication equipment such as
telephones and fax machines have argued that exports of their products
should be exempted from the ban, on the ground that it is impossible for a
country to remain repressive when telecommunication equipment is widely
available to the population of that country.

Which one of the following is an assumption on which the argument


given by the manufacturers depends?

A. The government of S has recently increased the amount of


telecommunication equipment it allows to be imported into the country.

B. The telecommunication equipment that would be imported into S if


the exemption were to be granted would not be available solely to top
development officials in S.

C. A majority of the members of R's legislature do not favor exempting


telecommunication equipment from the ban on exports to Country S.

D. Of all exports that could be sent to Country S, telecommunication


equipment would be the most effective in helping citizens of S oppose that
country's repressive government.

E. Without pressure from Country R, the government of S would be able


to continue repressing its citizens indefinitely.

11. The average cable television company offers its customers 50


channels, but new fiber-optic lines will enable telephone companies to
provide 100 to 150 television channels to their customers for the same price
as cable companies charge for 50. Therefore, cable companies will be
displaced by the new companies within a few years.

Which of the following, if true, most helps to strengthen the argument?

A. The initial cost per household of installing new fiber-optic television


service will exceed the current cost of installing cable television service.

B. The most popular movies and programs on channels carried by cable


companies will also be offered on channels carried by the fiber-optic lines
owned by the telephone companies.

C. Cable television companies will respond to competition from the


telephone companies by increasing the number of channels they offer.

D. Some telephone companies own cable companies in areas other than


those in which they provide telephone services.

E. The new fiber-optic services offered by telephone companies will be


subject to more stringent governmental programming regulations than those
to which cable companies are now subject.

12. The only physical factor preventing a human journey to Mars has
been weight. Carrying enough fuel to propel a conventional spacecraft to
Mars and back would make even the lightest craft too heavy to be launched
from Earth. A device has recently been invented, however, that allows an
otherwise conventional spacecraft to refill the craft's fuel tanks with fuel
manufactured from the Martian atmosphere for the return trip. Therefore, it is
possible for people to go to Mars in a spacecraft that carries this device and
then return.

Which one of the following is an assumption on which the argument


depends?

A. The amount of fuel needed for a spacecraft to return from Mars is the
same as the amount of fuel needed to travel from Earth to Mars.

B. The fuel manufactured from the Martian atmosphere would not differ
in composition from the fuel used to travel to Mars.

C. The device for manufacturing fuel from the Martian atmosphere


would not take up any of the spaceship crew's living space.

D. A conventional spacecraft equipped with the device would not be


appreciably more expensive to construct than current spacecraft typically are.

E. The device for manufacturing fuel for the return to Earth weighs less
than the tanks of fuel that a conventional spacecraft would otherwise need to
carry from Earth for the return trip.

13. In 1712 the government of Country Y appointed a censor to prohibit


the publication of any book critical of Country Y's government; all new
books legally published in the country after 1712 were approved by a censor.
Under the first censor, one half of the book manuscripts submitted to the
censor were not approved for publication. Under the next censor, only one
quarter of the book manuscripts submitted were not approved, but the number
of book manuscripts that were approved was the same under both censors. If
the statements in the passage are true, which one of the following can be
properly concluded from them?

A. More books critical of Country Y's governments were published


before the appointment of the first censor than after it.

B. The first censor and the second censor prohibited the publication of
the same number of book manuscripts.

C. More book manuscripts were submitted for approval to the first


censor than to the second.

D. The second censor allowed some book manuscripts to the published


that the first censor would have considered critical of Country Y's
government.

E. The number of writers who wrote unpublished manuscripts was


greater under the first censor than under the second.

14. If the government increases its funding for civilian scientific


research, private patrons and industries will believe that such research has
become primarily the government's responsibility. When they believe that
research is no longer primarily their responsibility, private patrons and
industries will decrease their contributions toward research. Therefore, in
order to keep from depressing the overall level of funding for civilian
scientific research, the government should not increase its own funding.

Which one of the following is an assumption on which the argument


relies?

A. Governments should bear the majority of the financial burden of


funding for civilian scientific research.

B. Any increase in government funding would displace more private


funding for civilian scientific research than it would provide.

C. Private donations toward research are no longer welcomed by


researchers whose work receives government funding.

D. Civilian scientific research cannot be conducted efficiently with more


than one source of funding.

E. Funding for civilian scientific research is currently at the highest


possible level.

15. Dental researcher: Filling a cavity in a tooth is not a harmless


procedure: it inevitably damages some of the healthy parts of the tooth.
Cavities are harmful only if the decay reaches the nerves inside the tooth, and
many cavities, if left untreated, never progress to that point. Therefore,
dentists should not fill a cavity unless the nerves inside the tooth are in
imminent danger from that cavity.

Which one of the following principles, if valid, most strongly supports


the researcher's reasoning?

A. Dentists should perform any procedure that is likely to be beneficial


in the long term, but only if the procedure does not cause immediate damage.

B. Dentists should help their patients to prevent cavities rather than


waiting until cavities are present to begin treatment.

C. A condition that is only potentially harmful should not be treated


using a method that is definitely harmful.

D. A condition that is typically progressive should not be treated using


methods that provide only temporary relief.

E. A condition that is potentially harmful should not be left untreated


unless it can be kept under constant surveillance.

16. The axis of Earth's daily rotation is tilted with respect to the planet of
its orbit at an angle of roughly 23 degrees. That angle can be kept fairly
stable only by the gravitational influence of Earth's large, nearby Moon.
Without such a stable and moderate axis tilt, a planet's climate is too extreme
and unstable to support life. Mars, for example, has only very small moons,
tilts at wildly fluctuating angles, and cannot support life.

If the statements above are true, which one of the following must also be
true on the basis of them?

A. If Mars had a sufficiently large nearby moon, Mars would be able to


support life.

B. If Earth's Moon were to leave Earth's orbit, Earth's climate would be


unable to support life.

C. Any planet with a stable, moderate axis tilt can support life.

D. Gravitational influences other than moons have little or no effect on


the magnitude of the tilt angle of either Earth's or Mars's axis.

E. No planet that has more than one moon can support life.

17. Psychologist: Some astrologers claim that our horoscopes


completely determine our personalities, but this claim is false. I concede that
identical twins—who are, of course, born at practically the same time—often
do have similar personalities. However, birth records were examined to find
two individuals who were born 40 years ago on the same day and at exactly
the same time—one in a hospital in Toronto and one in a hospital in New
York. Personalities of these two individuals are in fact different.

Which one of the following is an assumption on which the


psychologist's argument depends?

A. Astrologers have not subjected their claims to rigorous


experimentation.

B. The personality differences between the two individuals cannot be


explained by the cultural difference between Toronto and New York.

C. The geographical difference between Toronto and New York did not
result in the two individuals having different horoscopes.

D. Complete birth records for the past 40 years were kept at both
hospitals.

E. Identical twins have identical genetic structures and usually have


similar home environments.

18. Modern navigation systems, which are found in most of today's


commercial aircraft, are made with low-power circuitry, which is more
susceptible to interference than the vacuum-tube circuitry found in older
planes. During landing, navigation systems receive radio signals from the
airport to guide the plane to the runway. Recently, one plane with low-power
circuitry veered off course during landing, its dials dimming, when a
passenger turned on a laptop computer. Clearly, modern aircraft navigation
systems are being put at risk by the electronic devices that passengers carry
on board, such as cassette players and laptop computers.

Which one of the following, if true, LEAST strengthens the argument


above?

A. After the laptop computer was turned off, the plane regained course
and its navigation instruments and dials returned to normal.

B. When in use all electronic devices emit electromagnetic radiation,


which is known to interfere with circuitry.

C. No problems with navigational equipment or instrument dials have


been reported on flights with no passenger-owned electronic devices on
board.

D. Significant electromagnetic radiation from portable electronic devices


can travel up to eight meters, and some passenger seats on modern aircraft are
located within four meters of the navigation systems.

E. Planes were first equipped with low-power circuitry at about the same
time portable electronic devices became popular.
第二部分 GRE阅读

第八章 阅读的整体解题思维
GRE阅读的文章全部是学术性议论文,不包括小说、散文和诗歌。
议论文由论点(thesis)和论据(detail)组成。论据本身又包括两种形
式:(1)论述,即对观点句的理论展开;(2)例子,即用实例来证明
观点句是正确的。基于GRE阅读文章的特殊性和出题特征,本章将从三
个角度整体剖析文章至题目的大致套路和应对方案,这三个角度为:
(1)如何读原文;(2)如何分析题目;(3)如何比较选项。

一、如何读原文

文章类型决定了出题方向,既然议论文由论点和论据组成,则文章
之后不是针对论点来出题,就是围绕细节(论据)来出题。细节题(围
绕论据出题)的题干往往会给考生定位词或相关的定位提示,等具体题
目考到文章某处细节时考生再去仔细阅读这部分即可,在一开始读文章
的时候无须读懂并记住所有细节。那么,读文章需要读出什么呢?答案
是主旨(thesis)。阅读任何文章,文章的主旨一定要读出来,这样如
果试题围绕主旨进行考查,则考生不需要回原文定位,直接用主旨来解
题即可。如果文章针对细节设题,则通过细节回原文定位即可。

总结:读文章的时候,读出“主旨”,如果看不懂“细节”或发现时间
不够的话,可跳过。
(一)何为主旨

主旨就是文章主要的论点。在读文章的时候,总结出每个自然段提
到的话题,然后把各个自然段的不同话题串在一起,即为全文脉络,全
文脉络的浓缩,即是文章的主旨。它类似文章的大框架,可以把全部文
章内容都概括起来。

掌握主旨或文章脉络,对于解题有何帮助?

1.解决主旨题,这是一种常见的阅读题型。

2.在细节题无法明确定位的时候,如果有选项针对主旨中的一个话
题,这个选项即可成为答案备选。

3.阅读文章的时候,时刻分清眼前所读的是主旨还是细节,避免读
着读着就不知文章在讲什么了。

(二)具体如何读文章

1.短篇、中篇文章的读法

短篇文章一共就一段话,中篇文章比短篇文章多几行,所以针对短
篇文章和中篇文章建议考生全部都读,在读的过程中分清句间逻辑关
系。任意两个句子之间只有三种逻辑关系:顺接、逆接、开启新话题。
为了在实际考试中节省时间,一开始就要搞清楚句间逻辑,做题的时候
尽量不要回原文重读。

2.长篇文章的常规读法

在实际考试中只有一篇长文章是计分的,这篇长文章往往会在第一
个“verbal section”(语文部分)出现,而第一个语文部分又决定了第二
个语文部分的出题难度,所以为了避免进入低分库,必须尽可能地以高
正确率拿下长文章。

实际考场上,考生往往因为时间压力导致第一个语文部分来不及做
完,会空着很多题或最后几分钟乱点选项。为了避免这种由于时间压力
导致的问题,建议考生先空着长文章阅读,先做其他部分,最后留8分
钟左右来读长篇文章和做后续4道题。

长篇文章的常规读法和读短篇、中篇文章一样,基本每句话都读,
同时注意出题点所在的语句。这些出题点包括但不限于托福听力记笔记
时应注意的出题点。比如,“因果关系”“转折关系”“让步关系”“一个自然
段中的最后一句话说了新话题”等。类似这样的出题点的总结在后文会
进行补充论述。如一句话中出现两个及以上出题点,则这句话所描述的
话题一定要记住,防止之后题目考到或选项提到时自己没有印象。

3.在读不懂或有时间压力时长篇文章的读法

如果备考GRE时间不充裕,或是英语能力没法在备考期内得到相对
快速的提升,建议用此读法来应对长篇文章。这种读不懂或有时间压力
时的读法可被称作“脉络性阅读法”,即大致知道文章的框架,或文章脉
络,当中的细节在一开始读文章的时候可跳过。考生可能会提出问
题:“如何跳过?是整句跳读还是整段跳读?”“哪些地方可跳,哪些地
方不可以跳?”关于这些问题的答案,请看下面的关于此方法的详细论
述。

(三)详细论述“脉络性阅读法”

既然阅读文章都是议论文,议论文由论点和论据(细节)组成,那
么出题机构不是针对论点(thesis)出题,就是针对细节(detail)出
题。GRE阅读题型可大致分为以下四种:

围绕论点出题:主旨题+作者态度题

围绕细节出题:细节题+“in order to”考题

看到一道试题后,考生就要思考此题是针对主旨还是细节设置的。
如果一道试题问得很详细,则大概率是细节题,可以通过题干中的关键
词回原文查找定位句,然后通过定位句解题。如果一道试题针对论点进
行考查,则此题不需要再定位,通过文章脉络来解题即可。由此分析可
见,读文章的目的就是找到文章的框架、文章的脉络、文章的主旨。
(注意:这里“框架”“脉络”“主旨”是同一个意思)
看到这里,同学们可能要提出问题了。

1.如何读出文章脉络

很简单,每读完一个自然段,把这个自然段说的那些不同的事件都
总结出来;在读完全文之后,把各段所讲的不同的事件串在一起,即为
文章脉络。可能某个自然段篇幅很长,但这并不代表所描述的事件也
多。比如,有的自然段脉络为“观点1+具体举例”,即使具体的举例篇幅
达到40~50行,那么这个自然段也就只说了一件事情,即“观点1”。

2.读出文章脉络和一般情况下读文章有何不同

一般情况下读文章,学生是匀速地把文章的全部信息读一遍。脉络
性阅读法是考生在时间不够或英语阅读能力不够而无法匀速地把文章通
读一遍的情况下,读文章、解题的应试思维。

脉络性阅读法的核心是抓住文章脉络,弄清每段话共说了哪些事,
也就是每段话一共有几个分论点,说了几个话题(一个话题就是一个分
论点)。如果发现是理论论证,或是具体例子,以句子为单位快速地阅
读或跳过细节即可。如果考到某个具体细节,可通过细节题的题干定位
词来找到文章中的细节句,然后再解题。

拿一篇考生非常头痛的文科长文章来当例子(例1):

(在GRE阅读中,我个人觉得对于考生而言阅读难度的排列顺序
是:文科长文章>文科短文章>理科长文章>理科短文章。文科长文章
往往用词略抽象,容易把简单话题复杂化。)

例1 Despite winning several prestigious literary awards of the day,


when it first appeared, Alice Walker's The Color Purple generated critical
unease over puzzling aspects of its compositions. In what, as one reviewer
put it, was “clearly intended to be a realistic novel,”many reviewers
perceived violations of the conventions of the realistic novel form, pointing
out variously that late in the book, the narrator protagonist Celie and her
friends are propelled toward a happy ending with more velocity than
credibility, that the letters from Nettie to her sister Celie intrude into the
middle of the main action with little motivation or warrant, and that the
device of Celie's letters to God is especially unrealistic in as much as it
forgoes the concretizing details that traditionally have given the epistolary
novel(that is,a novel composed of letters)its peculiar verisimilitude: the
ruses to enable mailing letters, the cache, and especially the letters received in
return.

Indeed, the violations of realistic convention are so flagrant that they


might well call into question whether The Color of Purple is indeed intended
to be a realistic novel, especially since there are indications that at least some
of those aspects of the novel regarded by viewers as puzzling may constitutes
its links to modes of writing other than Anglo-European nineteenth-century
realism. For example, Henry Louis Gates, Jr., has recently located the letters
to God within an African American tradition deriving from slave narrative, a
tradition in which the act of writing is linked to a powerful deity who
“speaks”through scripture and bestows literacy as an act of grace. For Gates,
the concern with finding a voice, which he sees as the defining feature of
African American literature, links Celie's letters with certain narrative aspects
of Zora Neale Hurston's 1937 novel Their Eyes Were Watching God , the
acknowledged predecessor of The Color Purple.

Gates's paradigm suggests how misleading it may be to assume that


mainstream realist criteria are appropriate for evaluating The Color Purple.
But in his preoccupation with voice as a primary element unifying both the
speaking subject and the text as a whole Gates does not elucidate many of the
more conventional structural features of Walker's novel. For instance, while
the letters from Nettie clearly illustrate Nettie's acquisition of her own voice,
Gates's focus on “voice”sheds little light on the place that these letters occupy
in the narrative or on why the plot takes this sudden jump into geographically
and culturally removed surroundings. What is needed is an evaluative
paradigm that, rather than obscuring such startling structural features(which
may actually be explicitly intended to undermine traditional Anglo-European
novelistic conventions), confronts them, thus illuminating the deliberately
provocative ways in which The Color Purple departs from the traditional
models to which it has been compared.

1. The author of the passage would be most likely to agree with which of
the following statements about the letters from Nettie to Celie?

A. They mark an unintended shift to geographically and culturally


removed surroundings.

B. They may represent a conscious attempt to undermine certain


novelistic conventions.

C. They are more closely connected to the main action of the novel than
is at first apparent.

D. They owe more to the tradition of the slave narrative than do Celie's
letters to God.

E. They illustrate the traditional concretizing details of the epistolary


novel form.

2. In the second paragraph, the author of the passage is primarily


concerned with

A. examining the ways in which The Color Purple echoes its


acknowledged predecessor, Their Eyes Were Watching God .

B. providing an example of a critic who has adequately addressed the


structural features of The Color Purple.

C. suggesting that literary models other than the nineteenth-century


realistic novel may inform our understanding of The Color Purple.

D. demonstrating the ineffectiveness of a particularly scholarly attempt


to suggest an alternative way of evaluating The Color Purple.

E. disputing the perceived notion that The Color Purple departs from
conventions of the realistic novel form.

3. According to the passage, an evaluative paradigm that confronts the


startling structural features of The Color Purple would accomplish which of
the following?

A. It would adequately explain why many reviewers of this novel have


discerned its connections to the realistic novel tradition.
B. It would show the ways in which this novel differs from its reputed
Anglo-European nineteenth-century models.

C. It would explicate the overarching role of voice in this novel.

D. It would address the ways in which this novel echoes the central
themes of Hurston's Their Eyes Are Watching God .

E. It would reveals ways in which these structural features serve to


parody novelistic conventions.

4. The author of the passage suggests that Gates is most like the
reviewers mentioned in the first paragraph in which of the following ways?

A. He points out discrepancies between The Color Purple and other


traditional epistolary novels.

B. He sees the concern with finding a voice as central to both The Color
Purple and Their Eyes Are Watching God .

C. He assumes that The Color Purple is intended to be a novel primarily


in the tradition of Anglo-American nineteenth-century realism.

D. He does not address many of the unsettling structural features of The


Color Purple.

E. He recognizes the departure of The Color Purple from traditional


Anglo-European realistic novel conventions.

读者可自行计时8~9分钟读文章和解题,体会下时间压力。

第一段分析:

①Despite winning several prestigious literary awards of the day, when


it first appeared, Alice Walker's The Color Purple generated critical unease
over puzzling aspects of its compositions.② In what, as one reviewer put it,
was “clearly intended to be a realistic novel,”many reviewers perceived
violations of the conventions of the realistic novel form, pointing out
variously that late in the book, the narrator protagonist Celie and her friends
are propelled toward a happy ending with more velocity than credibility, that
the letters from Nettie to her sister Celie intrude into the middle of the main
action with little motivation or warrant, and that the device of Celie's letters
to God is especially unrealistic in as much as it forgoes the concretizing
details that traditionally have given the epistolary novel(that is, a novel
composed of letters)its peculiar verisimilitude: the ruses to enable mailing
letters, the cache, and especially the letters received in return.

分析1: Despite之后接的“好事情”,可预测主句说一个“坏
事件”。

分析2: 第②句和第①句之间没有转折词,优先以“顺承关系”
去理解,即认为第②句是在说一件“坏事情”。

分析3: 抽象理论之后出现具体内容,则具体内容往往是对之前
抽象内容的举例。因此,第②句中从“Cellie”可以快速浏览完。也
就是说第一段读完之后,头脑里只需要记下“一本书违背了现实主义
小说传统”这一点。

第二段分析:

①Indeed, the violations of realistic convention are so flagrant that they


might well call into question whether The Color of Purple is indeed intended
to be a realistic novel, especially since there are indications that at least some
of those aspects of the novel regarded by viewers as puzzling may constitutes
its links to modes of writing other than Anglo-European nineteenth-century
realism. ②For example, Henry Louis Gates, Jr., has recently located the
letters to God within an African American tradition deriving from slave
narrative, a tradition in which the act of writing is linked to a powerful deity
who “speaks”through scripture and bestows literacy as an act of grace. ③For
Gates, the concern with finding a voice, which he sees as the defining feature
of African American literature, links Celie's letters with certain narrative
aspects of Zora Neale Hurston's 1937 novel Their Eyes Were Watching God ,
the acknowledged predecessor of The Color Purple.

分析1: “Indeed”这个词提示第①句和之前一段说的是相同的
话题,本段依然在说“这本书违背了现实主义小说传统”。

分析2: 第②句的“For example”和第③句的“For Gates”表


示这两句话都是细节描述。因此这两句话与第一段的话题是一样的,
依然是“一本书违背了现实主义小说标准”。

第三段分析:

①Gates's paradigm suggests how misleading it may be to assume that


mainstream realist criteria are appropriate for evaluating The Color
Purple.② But in his preoccupation with voice as a primary element unifying
both the speaking subject and the text as a whole Gates does not elucidate
many of the more conventional structural features of Walker's novel. ③For
instance, while the letters from Nettie clearly illustrate Nettie's acquisition of
her own voice, Gates's focus on “voice”sheds little light on the place that
these letters occupy in the narrative or on why the plot takes this sudden jump
into geographically and culturally removed surroundings. ④What is needed
is an evaluative paradigm that, rather than obscuring such startling structural
features(which may actually be explicitly intended to undermine traditional
Anglo-European novelistic conventions), confronts them, thus illuminating
the deliberately provocative ways in which The Color Purple departs from
the traditional models to which it has been compared.

分析1: 此段第①句还是在说一样的话题——“用现实主义观点
评论这书是不行的”。

分析2: 第②句有一个转折,但是这里的转折是在末段,不可能
在末段再出现观点性的转折,所以只能是细节转折,细节不需要关注
。第③句是举例,有时间、有精力的情况下能读就读,否则就快速浏
览。
分析3: 末句提出需要一个“evaluative paradigm”,直面这
些结构特征,指出这本书故意偏离现实主义。

至此,全文读完,没有仔细读细节,只读出了各段大致论点
——“这本书不是现实主义小说”。接下来,看如何就借助这点信息来解
题。

1. The author of the passage would be most likely to agree with which of
the following statements about the letters from Nettie to Celie?

A. They mark an unintended shift to geographically and culturally


removed surroundings.

B. They may represent a conscious attempt to undermine certain


novelistic conventions.

C. They are more closely connected to the main action of the novel than
is at first apparent.

D. They owe more to the tradition of the slave narrative than do Celie's
letters to God.

E. They illustrate the traditional concretizing details of the epistolary


novel form.

解析: 分析题目的目的是找到题目最终的考查点。面对细节题
,需要分辨出此题考查的是针对细节的改写,还是作者写这个细节的
目的,或是作者对于细节的评价。此题问“作者会认为一个细节如何
”,这个细节是“Nettie to Celie”。细节词首字母大写,在文章
中肯定很容易定位到。回原文发现是在第一段中出现的,第一段的分
论点和框架我们已经得知——“论点+三个具体细节”(即文章中三
个例子that happy ending... that... and that...;“Nettie to
Celie”是具体细节的一部分),所以此题要以“这本书和现实主义
小说不同”为方向去找正确选项。
A 项:此项和要找的事件没有交集,排除。

B 项:“undermine certain novelistic conventions”和我们


要找的“违背现实主义小说传统”是同一个方向,先放着此项,待对
比。

C 项:此项和要找的事件没有交集,排除。

D 项:此项和要找的事件没有交集,排除。

E 项:此项和要找的事件没有交集,排除。

只有B项和我们要找的答案方向一致,所以B项当选。

此外,在第一段中,作者的分论点没有提到除了B项之外的内容
,从这个角度分析,也只有B项可以选择。

2. In the second paragraph, the author of the passage is primarily


concerned with

A. examining the ways in which The Color Purple echoes its


acknowledged predecessor, Their Eyes Were Watching God.

B. providing an example of a critic who has adequately addressed the


structural features of The Color Purple.

C. suggesting that literary models other than the nineteenth-century


realistic novel may inform our understanding of The Color Purple.

D. demonstrating the ineffectiveness of a particularly scholarly attempt


to suggest an alternative way of evaluating The Color Purple.

E. disputing the perceived notion that The Color Purple departs from
conventions of the realistic novel form.

解析: 此题直接考查第二段的分论点,不需要回原文定位到某
处细节,直接用之前记得的主旨来解题即可。第二段和第一段是顺接
的关系,依然在描述“这本书和现实主义小说标准不吻合”的话题。
带着这样的答案方向看选项:

A项:此项和要找的事件没有交集,排除。

B项:无关,排除。

C项:暗示除了现实主义小说之外的一个文学形式可能和这本书
有联系,和要找的事件有点接近,放着此项。

D项:不沾边,排除。

E项:如果把disputing变成supporting,那么E项也正确,此项
写法和第二段的分论点方向正好相反,排除。

故答案为C项。

3. According to the passage, an evaluative paradigm that confronts the


startling structural features of The Color Purple would accomplish which of
the following?

A. It would adequately explain why many reviewers of this novel have


discerned its connections to the realistic novel tradition.

B. It would show the ways in which this novel differs from its reputed
Anglo-European nineteenth-century models.

C. It would explicate the overarching role of voice in this novel.

D. It would address the ways in which this novel echoes the central
themes of Hurston's Their Eyes Are Watching God.

E. It would reveals ways in which these structural features serve to


parody novelistic conventions.
解析: 此题问得很细,需要找到“evaluative paradigm that
confronts the startling structural features”相关的句子,带
着定位句描述的事件来找哪个选项与之有话题重合之处。

evaluative paradigm 在文章最后一句话中出现了,于是找到这


句话:What is needed is an evaluative paradigm that, rather
than obscuring such startling structural features(which may
actually be explicitly intended to undermine traditional Ang
lo-European novelistic conventions), confronts them, thus i
lluminating the deliberately provocative ways in which The C
olor Purple departs from the traditional models to which it
has been compared.

这句话是说“evaluative paradigm”描述这本书故意偏离传统
的写作方式,即这本书不属于现实主义小说。下面来看哪个选项和这
个话题一致。

浏览五个选项,A项的“connections to the realistic novel


tradition”和B项整体描述的意思与要找的话题沾边。对比这两项,
在A项中有reviewers,但定位句没有体现这个对象,而B项找不出错
误的地方,因此答案为B项。

4. The author of the passage suggests that Gates is most like the
reviewers mentioned in the first paragraph in which of the following ways?

A. He points out discrepancies between The Color Purple and other


traditional epistolary novels.

B. He sees the concern with finding a voice as central to both The Color
Purple and Their Eyes Are Watching God.

C. He assumes that The Color Purple is intended to be a novel primarily


in the tradition of Anglo-American nineteenth-century realism.

D. He does not address many of the unsettling structural features of The


Color Purple.

E. He recognizes the departure of The Color Purple from traditional


Anglo-European realistic novel conventions.

解析: 此题问“Gates和第一段提到的评论家(reviewers)有
何共性”。借由此题向大家介绍“先决条件解题思维”。何为“先决
条件”?譬如,小黑比小白更大方,因为小黑会借给你更多的书。通
过此事件得出小黑更大方的先决条件是什么?可以是“小白有书可借
你”。因为如果小白连书都没有,那么他不借给你书不能说他不大方
,所以通过借书多少可推断出“大方”的先决条件是“小白有书可借
你”。所谓“先决条件”即为前提,或是逻辑题中的假设条件(assu
mption)。“先决条件解题法”需要判断出在找到最终的答案前,必
须做什么事情,以及选项必须有哪些特征才能成为正确答案。

回到Gates和评论家(reviewers)的共性,他们必须在论述同一
个话题,只有这样才能进行对比,如果话题不同,根本没法产生对比
。所以只需要找到他们中更容易定位的那个即可。此题题干已经指出
评论家(reviewers)在第一段,所以非常容易定位。找到第一段中
定位句“many reviewers perceived violations of the conventio
ns of the realistic novel form, pointing out variously that
late ...”。定位句的意思还是“这本书违背了现实主义小说传统”
,于是带着此句找答案即可,只有E项正确。

如果不用“先决条件解题法”,考生可能会先思考Gates的想法,再思
考评论家(reviewers)的想法,然后结合两者的共性内容得出答案,这
样的步骤就复杂多了。类似的出题方式,可再见例2。

例2 By the sixteenth century, the Incas of South America ruled an


empire that extended along the Pacific coast and Andean highlands from
what is now Ecuador to central Chile. While most of the Incas were self-
sufficient agriculturists, the inhabitants of the highland basins above 9,000
feet were constrained by the kinds of crops they could cultivate. Whereas 95
percent of the principal Andean food crops can be cultivated below 3,000
feet, only 20 percent reproduce readily above 9,000 feet. Given this unequal
resource distribution, highland Incas needed access to the products of lower,
warmer climatic zones in order to enlarge the variety and quantity of their
foodstuffs. In most of the preindustrial world, the problem of different
resource distribution was resolved by long-distance trade networks over
which the end consumer exercised little control. Although the peoples of the
Andean highlands participated in such networks, they relied primarily on the
maintenance of autonomous production forces in as many ecological zones as
possible. The commodities produced in these zones were extracted,
processed, and transported entirely by members of a single group.

This strategy of direct access to a maximum number of ecological zones


by a single group is called vertical economy. Even today, one can see Andean
communities maintaining use rights simultaneously to pasturelands above
12,000 feet, to potato fields in basins over 9,000 feet, and to plots of warm-
land crops in regions below 6,000 feet. This strategy has two principal
variations. The first is “compressed verticality,”in which a single village
resides in a location that permits easy access to closely located ecological
zones. Different crop zones or pasturelands are located within a few days
walk of the parent community. Community members may reside temporarily
in one of the lower zones to manage the extraction of products unavailable in
the homeland. In the second variation, called the “vertical archipelago,”the
village exploits resources in widely dispersed locations, constituting a series
of independent production “islands.”In certain pre-Columbian Inca societies,
groups were sent from the home territory to establish permanent satellite
communities or colonies in distant tropical forests or coastal locations. There
the colonists grew crops and extracted products for their own use and for
transshipment back to their high-altitude compatriots. In contrast to the
compressed verticality system, in this system, commodities rather than people
circulated through the archipelago.

The passage suggests that as a way of addressing the problem of


different resource distribution in the preindustrial world, the practice of
vertical economy differed from the use of long-distance trade networks in
that vertical economy allowed

A. commodities to reach the end consumer faster.


B. a wide variety of agricultural goods to reach the end consumer.

C. a single group to maintain control over the production process.

D. greater access to commodities from lower, warmer climatic zones.

E. greater use of self-sufficient agricultural techniques.

此题问“vertical economy和long-distance trade的区别”。理论上,要


把这两者分别读完了之后,再思考差异,最后得出答案。但是这样操作
时间太久,实际考试中恐怕来不及,所以平时就需要练习在面临时间压
力的情况下如何解题,锻炼解题思维。

既然问两者的区别,那么必须在两者描述相同话题的时候才能进行
对比,所以只需要先找到关于其中一个对象的描述,与另外一个在相同
的话题中进行对比。

原文第一段中讲“long-distance trade”有三个特点:(1)很多人使
用;(2)一群人会参与但不会依靠它;(3)最终用户很少能控制。浏
览整个第二段,并没有描述“long-distance trade”相关的语句,所
以“vertical economy”想要和“long-distance trade”有差异,也只能在这三
个话题中出现。我们可先看看有没有选项描述的话题属于这三个中的任
何一个,有的话就放着再对比。浏览这五个选项,只有C项和第三个特
点沾边,所以答案只能是C项。

因为考场上时间压力很大,所以在每一个步骤中能省时间就尽量省
时间,用最高效的方式找到答案。

二、如何分析题目

时常听到学生做错某题之后说他读错了题目想问的点。文章由主旨
和细节组成,所以题目也只能分为主旨题和本质考主旨的题,或是细节
题和本质考细节的题。本质考主旨的题、本质考细节的题意思是指看起
来像是主旨题,其实是细节题,或看上去像细节题,本质是主旨题。大
家可先思考这个问题:如果题目将某个细节句标了高亮或给出其所在的
具体行数,那么此题答案一定在这个高亮或给出行数的句子中出现吗?
答案肯定是“不一定”。比如,题目问“作者对于原文第10行的一个细节
句的看法”,此题考的不是细节句的内容,而是作者的评价,作者的评
价可能在文章最后一段中才出现,距离第10行很远。因此,要看清题目
问的内容。

题目大致会如何考查学生?大致有哪些题型?包括但不限于以下六
种:

(1)主旨题。

(2)本质主旨题。

(3)细节改写题。

(4)细节目的题。

(5)别人对于细节的观点题。

(6)作者对于细节的观点题。

相信各位GRE考生都了解过标准化考试,主旨题和细节改写题此处
不赘述。上文介绍了本质主旨题,接下来详细讲解其他题型。

* 细节目的题:

此类题在题干中往往会有“in order to / primarily to”,让考生分析作


者写一句话的目的是什么。比如:

(1)The author of the passage quotes Urban and Sherzer most


probably in order to...

(2)In the passage, the mention of “Alabama, Arkansas, Mississippi,


and Tennessee”serves primarily to...

(3)The passage mentions the possible benefit to algae of nitrogen


falling down in the rain most likely to...

任何句子在文章中只可能有三种角色:论点、论述和举例。如果题
干所问的定位句子是个分论点,则作者写分论点的目的一定是要说明文
章的论点(主旨)本身。所以如果发现句子本身是观点则直接找选项,
找一个类似这个观点句的选项即可。如果句子本身是细节,作者写细节
的目的一定是说明定位句所在段落的分论点,找段落中的观点句即可。

例1 The constructivists gain acceptance by misrepresenting


technological determinism: technological determinists are supposed to believe,
for example, that machinery imposes appropriate forms of order on society.
The alternative to constructivism, in other words, is to view technology as
existing outside society, capable of directly influencing skills and work
organization. Clark refutes the extremes of the constructivists by both
theoretical and empirical arguments. Theoretically he defines “technology”in
terms of relationships between social and technical variables. Attempts to
reduce the meaning of technology to cold, hard metal are bound to fail, for
machinery is just scrap unless it is organized functionally and supported by
appropriate systems of operation and maintenance. At the empirical level
Clark shows how a change at the telephone exchange from maintenance-
intensive electromechanical switches to semi-electronic switching systems
altered work tasks, skills, training opportunities, administration, and
organization of workers. Some changes Clark attributes to the particular way
management and labor unions negotiated the introduction of the technology,
whereas others are seen as arising from the capabilities and nature of the
technology itself. Thus Clark helps answer the question: “When is social
choice decisive and when are the concrete characteristics of technologymore
important?”

The author of the passage uses the expression “are supposed


to”primarily in order to

A. suggest that a contention made by constructivists regarding


determinists is inaccurate.

B. define the generally accepted position of determinists regarding the


implementation of technology.

C. engage in speculation about the motivations of determinists.

D. lend support to a comment critical of the position of determinists.

E. contrast the historical position of determinists with their position


regarding the exchange modernization.

解析: 定位到第一句话的冒号之后,冒号之后的内容是对冒号
之前观点句的展开,所以选项如果和冒号之前的句子有意思重合或方
向一致,就先放着。

A项:冒号之前说“建构主义者错误地描述科技决定论”,此项
的意思“建构主义者是错误的”。在方向上,inaccurate呼应misrep
resent,此项可先保留。

B、C、D、E项:均没有一点话题重合,排除。

只有A项可选。

例2 An alternative explanation is defense from predation. Black bear


cubs use trees for defense, whereas brown bears and polar bears, which
regularly inhabit treeless environments, rely on aggression to protect their
cubs. Like brown bears and polar bears(and unlike other myrmecophagous
mammals, which are noted for their passivity), sloth bears are easily
provoked to aggression. Sloth bears also have relatively large canine teeth,
which appear to be more functional for fighting than for foraging. Like brown
bears and polar bears, sloth bears may have evolved in an environment with
few trees. They are especially attracted to food-rich grasslands; although few
grasslands persist today on the Indian subcontinent, this type of habitat was
once wide spread there. Grasslands support high densities of tigers, which
fight and sometimes kill sloth bears; sloth bears also coexist with and have
been killed by tree-climbing leopards,(52)and are often confronted and
chased by rhinoceroses and elephants, which can topple trees(53).
Collectively these factors probably selected against tree-climbing as a
defensive strategy for sloth bear cubs. Because sloth bears are smaller than
brown and polar bears and are under greater threat from dangerous animals,
they may have adopted the extra precaution of carrying their cubs. Although
cub-carrying may also be adoptive for myrmecophagous foraging, the
behavior of sloth bear cubs, which climb on their mother's back at the first
sign of danger, suggests that predation was a key stimulus.
The author mentions rhinoceroses and elephants(lines 52-53)
primarily in order to

A. explain why sloth bears are not successful foragers in grassland


habitats.

B. identify the predators that have had the most influence on the
behavior of sloth bears.

C. suggest a possible reason that sloth bear cubs do not use tree-
climbing as a defense.

D. provide examples of predators that were once widespread across the


Indian subcontinent.

E. defend the assertion that sloth bears are under greater threat from
dangerous animals than any other bear species.

解析: 此题问“作者提到犀牛和大象的目的是什么”。在原文
黑体处发现了定位词,请问大家,如果现在距离考试结束只有10秒钟
,如何解答此题?通读段落找到分论点的话,时间根本来不及。不管
细节是什么,也不管细节描述的内容有多长,细节一定是围绕此段的
分论点来写的。那么,一段话的分论点最可能出现的地方是哪里?往
往是段落开头。如果后文和段落开头没有转折词,则后文不管细节展
开得有多长,都得围绕段落开头来描述。读了此段的第一句话后,发
现第二句话开始都是具体描述,直至黑体处也没有出现转折词,所以
不管当中细节内容如何描写,都得围绕“defence”来描述。所以找
和“防御”沾边的选项即可,于是得出答案为C项。

例3 Some historians question the widely held belief that continually


improving education led to gradual African American empowerment in the
southern United States from the late nineteenth century through the mid-
twentieth century. They note that the development of Black educational
institutions in the segregated South was never rapid or steady: disparities
between Black and White schools sometimes grew in the early decades of the
twentieth century. And African Americans' educational gains did not bring
commensurate economic gains. Starting in the 1940s, even as Black and
White schools in the South moved steadily toward equality, Black
southerners remained politically marginalized and experienced systematic job
discrimination. Although Black schools had achieved near parity with White
schools in per capita spending and teachers' salaries by 1965, African
Americans' income still lagged behind that of Whites. Nonetheless,
educational progress did contribute toward economic and political
empowerment. African Americans' campaigns to support Black schools
fostered a sense of community, nurtured political determination, and often
increased literacy. More significantly, politically outspoken Black
newspapers achieved record circulation during the 1940s, just as the literacy
rate among African Americans approached 90 percent. Finally, the leadership
of the Civil Rights movement of the 1960s was composed largely of
graduates of Black colleges.

The author of the passage refers to the fact that “disparities between
Black and White schools sometimes grew in the early decades of the
twentieth century”most likely in order to

A. support the argument that the economic struggles of southern Blacks


were largely due to educational inequalities.

B. give an example of the differences between Black schools in the early


part of the twentieth century and Black schools starting in the 1940s.

C. illustrate the unevenness of the progress made by Black schools in the


southern United States.

D. help explain why Black remained politically marginalized and


experienced systematic job discrimination in the segregated South.

E. provide evidence that educational progress was a precondition for


economic progress in the segregated South.

解析: 定位到第二句话的冒号之后,冒号之后的内容是对冒号
之前观点句的展开,即冒号前后句子说的是同一个话题。于是找和“
教育对黑人没多大好处”的话题沾边的选项,先把这样的选项留下待
比较。

A项:经济不行的原因。

B项:黑人学校之间的差异。

C项:黑人学校发展有些不平均。

就A、B、C三项对比而言,C项和“教育对黑人没多大好处”更接
近,这三项里面先得出答案为C项。

D项:解释黑人政治上和工作上处于不利地位。

E项:教育过程是一个前提。

五个选项中说到“教育对黑人没多大好处”的选项有C项和D项,
回原文看定位句描述的话题是教育相关,不是政治和工作的话题,因
此C项更合理。

* 别人/作者观点细节题:

在读文章的过程中,如果有作者观点,在作者观点之处出题的可能
性近乎百分百。作者的观点会以两个方式体现:(1)文章内直接地、
明确地讲明作者观点;(2)通过正向的修饰语来体现出作者的观点,
这些修饰语往往是形容词或是副词。

例1 Mary Barton, particularly in its early chapters, is a moving


response to the suffering of the industrial worker in the England of the 1840s.
What is most impressive about the book is the intense and painstaking effort
made by the author, Elizabeth Gaskell, to convey the experience of everyday
life in working class homes. Her method is partly documentary in nature: the
novel includes such features as a carefully annotated reproduction of dialect,
the exact details of food prices in an account of a tea party, an itemized
description of the furniture of the Bartons' living room, and a
transcription(again annotated)of the ballad “The Oldham Weaver”. The
interest of this record is considerable, even though the method has a slightly
distancing effect.

问题:Which of the following best describes the author's attitude


toward Gaskell's use of the method of documentary record in Mary Barton?

A. Uncritical enthusiasm.

B. Unresolved ambivalence.

C. Qualified approval.

D. Resigned acceptance.

E. Mild irritation.

解析: 第二句话中“most impressive”已体现出作者的观点和


态度,所以在找作者态度的时候负面态度不可选,排除B和D项。在段
落的最后提到“distancing effect”,说明作者认为这本书还有些
不妥之处,故作者的评价是“大正小负”,排除A项。“qualified”
在GRE考试中等同于“limited”,意为“限制”,在填空部分也考查
过很多次。“resigned”不能翻译成“辞职”,此处考查该词在文学
作品中“听天由命,没有办法了”的含义,作者并没有表达“被迫地
接受”这层意思,排除D项。C项为此题答案。

作者的态度一共三种:正评价、负评价和混合评价。混合评价又分
为“大正小负”“大负小正”。如果一篇文章没有写出作者评价,则在主旨
题中动词“evaluate”开头的选项不可选:文章没有写出“作者评价”,肯定
无法评估(evaluate)。

切不可把别人的态度当作作者的态度并从他人态度出发去理解文章
或选项。

例2 Historians who study European women of the Renaissance try to


measure “independence,”“options,”and other indicators of the degree to
which the expression of women's individuality was either permitted or
suppressed. Influenced by Western individualism, these historians define a
peculiar form of personhood: an innately bounded unit, autonomous and
standing apart from both nature and society. An anthropologist, however,
would contend that a person can be conceived in ways other than as an
“individual.”In many societies a person's identity is not intrinsically unique
and self-contained but instead is defined within a complex web of social
relationships.

In her study of the fifteenth-century Florentine widow Alessandra


Strozzi, a historian who specializes in European women of the Renaissance
attributes individual intention and authorship of actions to her subject. This
historian assumes that Alessandra had goals and interests different from those
of her sons, yet much of the historian's own research reveals that Alessandra
acted primarily as a champion of her sons' interests, taking their goals as her
own. Thus Alessandra conforms more closely to the anthropologist's notion
that personal motivation is embedded in a social context. Indeed, one could
argue that Alessandra did not distinguish her personhood from that of her
sons. In Renaissance Europe the boundaries of the conceptual self were not
always firm and closed and did not necessarily coincide with the boundaries
of the bodily self.

1. The passage suggests that the historian mentioned in the second


paragraph would be most likely to agree with which of the following
assertions regarding Alessandra Strozzi?

A. Alessandra was able to act more independently than most women of


her time because she was a widow.

B. Alessandra was aware that her personal motivation was embedded in


a social context.

C. Alessandra had goals and interests similar to those of many other


widows in her society.

D. Alessandra is an example of a Renaissance woman who expressed


her individuality through independent action.
E. Alessandra was exceptional because she was able to effect changes in
the social constraints placed upon women in her society.

解析: 思考此题的B项是否正确?不正确。原文的“personal m
otivation is embedded in a social context”是在thus之后,一
个自然段中thus之后的观点一定是作者的观点而非别人的。所以B项
错误。

此题问历史学家的观点,全文就只有“This historian assumes


that Alessandra had goals and interests different from those
of her sons...”这句话在描述历史学家的观点,所以答案选D项。

GRE阅读拿到高分需要具备两方面的能力:文章框架提取能力;
定位能力。全文只有一处提到了历史学家的观点,其他都是历史学家
的调查研究或作者的观点,并不是历史学家的观点。

2. It can be inferred that the author of the passage believes which of the
following about the study of Alessandra Strozzi done by the historian
mentioned in the second paragraph?

A. Alessandra was atypical of her time and was therefore an


inappropriate choice for the subject of the historian's research.

B. In order to bolster her thesis, the historian adopted the


anthropological perspective on personhood.

C. The historian argues that the boundaries of the conceptual self were
not always firm and closed in Renaissance Europe.

D. In her study, the historian reverts to a traditional approach that is out


of step with the work of other historians of Renaissance Europe.

E. The interpretation of Alessandra's actions that the historian puts


forward is not supported by much of the historian's research.
解析: C项内容和文章第二段后文有内容重合,是否正确?不正
确。此题问的是作者的观点,C项中“the boundaries of the conce
ptual self were not always firm and closed in Renaissance Eu
rope”在文中末处出现过,但这是作者的观点,并不是“the histor
ian”的观点。

E项的“historian's interpretation not supported by resea


rch”对应句中的“yet”,答案为E项。

列举以上三道题的目的是告诉大家,在读原文、题干和选项时要看
清考查的是作者的观点还是别人的观点,不能理解错位。

三、如何比较选项

如果把GRE verbal考试中的一道题比喻成一个项目或一项任务,那
么我们从“把一件事情做成”的角度来分析“从五个选项中选出一个答
案”这一任务:

1. 任务目标:五选一。

2. 执行任务的已有“资源”包括:题目,选项,原文的框架内容,之
前的试题和选项,之后的试题和选项,做之前的试题已经读的文章内
容,常识。

3. 任务难度:在有限时间内选择相对最好的选项。

以上的任务分析向考生传递了如下信息:除了“看题,回原文找,
再看选项”这个固定的思维之外,还有其他的“资源”或达成任务的路径
可帮助我们完成目标。譬如,试题给的信息很长,则题目本身已经提示
考生答案和哪个信息点相关联。选项的排列顺序也会提供逻辑线索,帮
助我们得出答案。

最后,整个GRE考试阅读内容不可能与客观的常识相违背,可以通
过常识判断哪个选项错误概率小,或哪个选项内容更符合常识。(切记
是“常识”,即世界各地的人都有的知识,而不是学生自以为是主观认为
的内容。)

例1 题干本身的提示

The passage suggests that if judges are required to provide written


explanations for the legal reasoning underlying their decisions about cases,
then

A. judicial bias will be almost completely eliminated.

B. any faulty reasoning employed by judges can in principle be detected.

C. judges' written explanations will usually conceal their real reasoning.

D. the public perception of the impartiality of judiciary will improve.

E. judges will be motivated to recuse themselves when there is an


appearance of bias.

解析: 此题问“如果法官把推理过程写下来会如何”。如果不
回看原文,如何猜出一个正确概率最大的选项?

答案一定要和“将某个东西写下来”沾边。“把推理过程写下来
”就一定会“完全消除(completely eliminated)偏见”吗?不一
定。因此,出现绝对化词汇completely的A项出错概率有点高。在B和
C项的比较中,既然已经“写下来了”,是能被探测到还是会被隐藏
?哪个更可能?显然B项更可能。在D和E项的比较中,能直接和“写
下来”呼应的是D项“公平观念可提升”,故此题在未回看原文的时
候,可将答案锁定在B和D项中。此题如果回原文定位解题很容易,这
里为考生提供一个不同的解法。

例2 选项排列的逻辑性

Like Beethoven, Schoenberg worked in a constantly changing and


evolving musical style that acknowledged tradition while simultaneously
lighting out for new territory. This is true of the three different musical styles
through which Schoenberg's music evolved. He began in the late-Romantic
manner—music charged with shifting chromatic harmonies—that was
pervasive in his youth. People who enjoy the music of Brahms ought to love
Schoenberg's Verklaerte Nacht, and they usually do, once they get past the
fact that they are listening to a piece by Schoenberg.

Schoenberg later pushed those unstable harmonies until they no longer


had a tonal basis. He did this in part because in his view it was the next
inevitable step in the historical development of music, and he felt he was a
man of destiny; he also did it because he needed to express what he was
compelled to express.

Finally, he developed the 12-tone technique as a means of bringing a


new system of order to nontonal music and stabilizing it. In all three styles,
Schoenberg operated at an awe-inspiring level of technical mastery. As his
career progressed, his music became more condensed, more violent in its
contrasts, and therefore more difficult to follow.

It can be inferred from the passage that the author would be most likely
to agree with which one of the following statements about the relationships
between the three styles in which Schoenberg wrote?

A. Each successive style represents a natural progression from the


previous one.

B. Each successive style represents an inexplicable departure from the


previous one.

C. The second style represents a natural progression from the previous


one, but the third style represents an inexplicable departure from the second.

D. The second style represents a departure from the first, but the third
style represents a natural progression from the second.

E. The second style represents an inexplicable departure from the first,


but the third style represents a natural progression from the first.
解析: 如果按照标准的解题步骤来看,此题考查三种音乐形式
,对应文章的在三个自然段(如上文内容)。按标准解法去分析,先
读内容再做此题,虽可将选项理解得很透彻,但在实际考试中是无法
实现的。此题是文章的最后一题,在时间紧和体力不支的状态下,再
回原文读这么长的内容然后解题只存在理论上的可能。那么,如何在
时间紧又不能回读的情况下找出最佳答案呢?从某种角度来讲,GRE
考试就是一项考查解决问题能力的考试。要解决问题,就要想我们手
中一共有哪些资源。除了题目之外,还有选项的逻辑排列顺序。当作
答本题需要定位到文章多处内容时,先别直接回原文找,而是分析下
选项是否会给我们一些线索。

选项内容的分析:

A项:每种形式都是之前一种形式的递进。

B项:每种形式都和之前不一样。

C项:第三种和第二种不同。

D项:第二种和第一种不同。

E项:第二种和第一种不同。

这五个选项中有四个选项描述“一个形式和其他形式不同”,只
有A项描述“递进”。作者一般通过from、however、nevertheless、
deviate等词表示“不一样,转折,有差异”,所以我们要做的事情
就是确定这三个自然段中是否存在这些描述“不同”的词汇。如有的
话,看下是哪两种形式(style)之间发生转折,如没有这些词汇,
则答案就是A项。

本题没有找到表示转折的词汇,故答案为A。

例3 单选题的出题限制给考生的答案提示

“答案只有一个”的出题限制,让出题者在选项设置上提供了关于答
案的方向性线索。
例4 看题目和选项,找出答案

The primary purpose of the passage is to

A. reevaluate a controversial theory.

B. identify the flaws in a study.

C. propose a new method of historical research.

D. compare two contrasting analyses.

E. provide a fresh perspective.

解析: 此题是主旨题,则D项不正确,因为compare、contrast
、reconcile从来没有在GRE阅读的正确答案中出现过。

A、B、C项都在说内容,而E项说提供一个新角度。

如果A、B、C三项正确,则E项必然正确。相当于A、B、C是E项“
提供一个新角度”的内容,即“重新评估一个理论”“识别错误”“
提到一个方法”都可被认为是“新角度”,所以此题答案必然为E项

例5 看题目和选项,找出答案

The passage is primarily concerned with

A. comparing various measures used to assess the performance of the


United States economy in 1997.

B. providing evidence that the performance of the United States


economy in 1997 was similar to its performance between 1873 and 1973.

C. evaluating an argument concerning the performance of the United


States economy in 1997.
D. examining the consequences of a popular misconception about the
performance of the United States economy in 1997.

E. supporting an assertion made by journalists about the performance of


the United States economy in 1997.

解析: 此题是主旨题,排除含有compare的A项。

B项的大致内容是“1997年的经济类似1873—1973年”;C项的大
致内容是“评估1997年的经济”;D项的大致内容是“检验一个关于1
997年经济表现的错误概念”;E项的大致内容是“支持一个记者关于
1997年经济的说法”。

如之前所述,在看选项的时候要“挑错”,正确的内容去掉不看
,而是看是否存在错误的地方。那么问题来了,各位读者可思考下选
项有哪些地方不会错。要养成“解决问题,做成事情”的能力,想象
力特别重要。首先,选项如果回答了题目本身问到的事件,则这点不
会错。其次,选项中的“废话词汇”不会错,比如indicate、provid
ing、examining,因为这些词是引出一个选项中真正内容的词汇,这
些词汇本身没有具体内容,不存在出错的情况。再者,选项间都有的
内容不可能错,否则就不存在正确答案了。还有很多其他的信息不会
错,比如之前明确读到过的内容等。大家可开动脑筋自行补充完善。

此题答案为C项,因为四个选项都在谈“1997年的经济”,所以C
项不存在出错的情况。

例6 看题目和选项,找出答案

According to the passage, which of the following is true of the average


rate at which real GDP per capita grew in the twenty-four years immediately
before 1997?

A. It was less than it had been between 1873 and 1973 because only
forty percent of the population worked for wages between 1873 and 1973.
B. It was less than it had been between 1873 and 1973 because labor
productivity grew less between 1973 and 1997 than it had between 1873 and
1973.

C. It was less than it had been between 1873 and 1973 as a result of an
increase in the percentage of the population earning wages during these
years.

D. It was less than the average rate at which real GDP per capita grew
between 1873 and 1973.

E. It was less than the rate at which real GDP per capita grew in 1997.

解析: 这五个选项中,A、B、C、D项都是在描述1873—1973年
的事,E项描述的是1997年的事件。根据历年真题的统计,如果五个
选项中有四个选项说的是同一个话题,另外一个选项说的是其他话题
,则答案在四个选项中。就已有的试题统计来看,这一招在GRE和GMA
T考试中100%有效,因此去掉E项。

去掉“共有内容”之后,A项主要留下“forty percent of popu


lation”;B项主要留下“labor productivity”;C项主要留下“po
pulation earning wages”;D项主要留下“GDP per capita”。A、
B、C、D的四个选项都有内容留下。

例7 找出错概率小的选项

The passage suggests that the presence of G10 garnet in a kimberlite


pipe indicates that

A. the pipe in which the garnet is found has a 90% chance of containing
diamonds.

B. the levels of calcium and chrome in the pipe are conducive to


diamond formation.
C. the pipe passed through a diamond-stability field and thus may
contain diamonds.

D. any diamonds the pipe contains would not have come from the
diamond-stability field.

E. the pipe's temperature was so high that it oxidized any diamonds the
pipe might have contained.

解析: 如读了原文,答案会锁定在A和C项中。A项描述的是“有
90%概率包含钻石”,C项描述的为“可能包含钻石”。如何判断一个
选项出错概率大还是小?越是说得具体、说得明确、话题范围小的选
项,出错的概论就越大。如果A项“有90%的概率包括钻石”这一说法
能成立,则C项“可能包含钻石”这一说法必然成立,所以C项为答案

例8 As an element in the argument presented by the author of the


passage, the reference to Blue Clark's study of the Lone Wolf case serves
primarily to

A. point out that this episode in Native American history has received
inadequate attention from scholars.

B. support the contention of the author of the passage that the Lone Wolf
decision had a greater long-term impact than did the congressional action of
1871.

C. challenge the validity of the Supreme Court's decision confirming the


unlimited unilateral power of Congress in Native American affairs.

D. refute the argument of commentators who regard the congressional


action of 1871 as the end of the era of formal negotiation between the federal
government and Native American tribes.

E. introduce a view about the Lone Wolf decision that the author will
expand upon.

解析: 此题的五个选项在内容设置上有以下逻辑关系:A、B、C
、D这四个选项描述的是具体内容,而E项是一个抽象的概括性表达,
例如“view”和“author will expand upon”。不管什么样的内容
,都可以被称为“view”,以及不管作者是持支持还是否定的态度,
只要作者在后文提到了类似话题,都可以说作者之后会“expand upo
n”,因此就判断哪个选项出错可能性小而言,E项最合理。如有一个
选项是原文再现,则选这个选项。如有几个选项方向都和原文的事件
一致,并且这些选项都是改写的,并不是原文再现,那么就要优选出
错概率较小的选项。此招也适用于GRE逻辑和填空题。

四、诱惑选项设置与正确答案改写的“套路”

设置诱惑选项有些哪“套路”可使用?结合GMAT、GRE、LSAT这
三种思维类考试,我们总结出一些诱惑选项设置的类型,大家要注意避
坑。

诱惑(错误)选项设置的四大经典“套路”:(1)信息
混搭(混);(2)选项出现绝对化(偏);(3)选项与
原文内容相反(反);(4)无中生有(无)。

1. 信息混搭(混)

说明: 出题机构用不同事件的局部信息串成一个选项,使考生在
面对这些信息时感到似曾相识,从而做出错误的判断。

2. 选项出现绝对化(偏)

说明: 选项的绝对化程度和原文事件不一致时,不能选。类似的
绝对化词汇包括但不限于:“solely”,形容词最高
级,“only”和“unique”等描述“唯一”的词汇,“最××”之类的词汇。

除了“程度”之外,若选项和原文的“条件”不同也不能选。

例 The passage suggests that the atomic structure of glass is such that
glass will

A. behave as a liquid even though it has certain properties of solids.

B. be noticeably deformed by the force of its own weight over a period


of a few millennia.

C. behave as a solid even when it has reached glass transition


temperature.

D. flow downward under its own weight if it is heated to glass transition


temperature.

E. stop flowing only if the atoms are arranged in a fixed crystalline


structure.

如通过“atomic structure of glass”来定位题,定位句第一次出现是在


文章第一段(如下):

To glass researchers it seems somewhat strange that many people


throughout the world share the persistent belief that window glass flows
slowly downward like a very viscous liquid. Repeated in reference books, in
science classes, and elsewhere, the idea has often been invoked to explain
ripply windows in old houses. The origins of the myth are unclear, but the
confusion probably arose partly from a misunderstanding of the fact that the
atoms in glass are not arranged in a fixed crystal structure. In this respect, the
structure of liquid glass and the structure of solid glass are very similar, but
thermodynamically they are not the same. Glass does not have a precise
freezing point; rather, it has what is known as a glass transition temperature,
typically a range of a few hundred degrees Celsius. Cooled below the lower
end of this range, molten glass retains an amorphous atomic structure, but it
takes on the physical properties of a solid.
解析: 读了第一段的最后几句,是否可得出答案为A项?答案是
不能。第一段最后的内容是“Cooled below the lower end of this
range, molten glass retains an amorphous atomic structure, b
ut it takes on the physical properties of a solid.”,其中提
到“below the lower end of this range”这个条件范围,而A项缺
少个条件,故A项不可选。

3. 选项与原文内容相反(反)

说明: 选项所说内容和原文相反,不能选。

4. 无中生有(无)

说明: 选项出现文中所没有的新概念或新话题,不能选。

需要注意的是,在“无中生有”的诱惑选项设置思维中,最重要的
是“无关对比”这个概念。若选项出现了比较级,则比较对象和比较内容
有任何一项原文没有说过,选项即为“无关对比”。

例 Hotter and more massive than the Sun, stars called “stragglers”are
puzzling to astronomers because such rapidly burning stars would not be
expected to persist in ancient star clusters. Some researchers believe that the
typical blue stragglers formed when two ancient, lower-mass stars collide and
merge form more massive, hotter star. Peter Leonard theorizes alternatively
that in low density globular clusters, where mergers between single stars
occur too infrequently to account for the observed quantity of blue stragglers,
these stragglers are created instead by a group of stars. He suggests that a pair
of stars already orbiting each other presents a larger target for a third star or
another pair. Once this new grouping forms, close encounters between the
stars could prompt any two to merge as a blue straggler. Leonard's model
predicts that each blue straggler has a distant orbiting companion—as appears
true of many blue stragglers in the M67 cluster of the Milky Way galaxy.

Information presented in the passage suggests which of the following


about blue stragglers?
A. They originate from stars that are hotter and more massive than the
Sun.

B. They are burning more rapidly than other types of stars observed in
ancient star clusters.

C. They are older than most other types of stars within the same star
cluster.

D. They are less numerous in low-density globular clusters than are


pairs of stars.

E. They generally originate from the oldest stars among those found in
ancient star clusters.

解析: 此题问“blue stragglers”相关信息,但是整个段落好


几处都有“blue stragglers”,出题机构可针对其中任何一句话出
题。那么如何在一开始读文章的时候,就能预测某些句子中会有出题
点?通过分析历年试题可知,这些有出题点的地方类似托福听力需要
记笔记的地方,例如,“因果关系”“转折关系”“比较级”“感情
色彩强烈的词汇”“让步句”,以及“每段话中最后一句话如出现新
话题,则最后一句话会出题”等。

此段第一句话中的“because”提示这句话可能会被用来出题,
需要记住出题点所在句的话题。所以B项为答案。

此题的C项中就出现了原段落没有的比较内容(“老不老”的话
题),不可选。

正确答案“改写”的写法:

(1)近义词 /上义词。

说明: 通过同义词改写,选项重述了文章定位句的话题。
“上义词”是定位句话题中某个对象的上一层分类,比如“电脑”的上
义词可以是“高科技”,“人”的上义词可以是“动物”等。如原定位句的话
题是“余翔在操作电脑”,选项可改写成“某人在体验高科技”,这两句话
属于同一个话题。

(2)换角度。

说明: 通过描述定位句某对象的对立面来描述正确选项。譬如“学
生”的对立面是“老师”,“站”的对立面是“坐”。

例:定位句——“余翔是你们的GRE老师。”

A项:你们是余翔GRE课程的学生。

A项正确,考生不要以为“学生”在定位句中没出现过便直接排除此
项。“学生”是“老师”的对立面,一个话题的对立面和这个话题也沾边,
而沾边的选项不能排除。考生需要从逻辑去判断是否沾边,而不是词本
身。

(3)双重否定。

说明: 选项中出现和文章定位句话题相反的词,然后再加上否定
词,变成“双重否定”,这种情况下,其实选项和原定位句描述的还是同
一话题。

例 In 1994, a team of scientists led by David Mckay began studying the


meteorite ALH84001, which had been discovered in Antarctica in 1984. Two
years later, the McKay team announced that ALH84001, which scientists
generally agree originated on Mars, contained compelling evidence that life
once existed on Mars. This evidence includes the discovery of organic
molecules in ALH84001, the first ever found in Martian rock. Organic
molecules—complex, carbonbased compounds—form the basis for terrestrial
life. The organic molecules found in ALH84001 are polycyclic aromatic
hydrocarbons, or PAH's. When microbes die, their organic material often
decays into PAH's.

Skepticism about the McKay team's claim remains, however. For


example, ALH84001 has been on earth for 13,000 years, suggesting to some
scientists that its PAH's might have resulted from terrestrial contamination.
However, McKay's team has demonstrated that the concentration of PAH's
increases as one looks deeper into ALH84001, contrary to what one would
expect from terrestrial contamination. The skeptic's strongest argument,
however, is that processes unrelated to organic life can easily produce all the
evidence found by McKay's team, including PAH's. For example, star
formation produces PAH's. Moreover, PAH's frequently appear in other
meteorites, and no one attributes their presence to life processes. Yet
McKay's team notes that the particular combination of PAH's in ALH84001
is more similar to the combinations produced by decaying organisms than to
those originating from nonbiological processes.

The passage asserts which of the following about the claim that
ALH84001 originated on Mars?

A. It was initially proposed by the McKay team of scientists.

B. It is not a matter of widespread scientific dispute.

C. It has been questioned by some skeptics of the McKay team's work.

D. It has been undermined by recent work on PAH's.

E. It is incompatible with the fact that ALH84001 has been on Earth for
13,000 years.

解析: 题干问“ALH84001起源于火星”的相关信息。全文从第
二段起描述“质疑者”怀疑“火星上有生命”,并不是题目所问,所
以不能定位在第二段。在第一段中,和“ALH84001起源于火星”相关
的句子只有“Two years later, the McKay team announced that A
LH84001, which scientists generally agree originated on Mars
, contained compelling evidence that life once existed on Ma
rs”,这句话是说“科学家同意这起源于火星”,则第一段从此定位
句之后描述的都是“火星上有生命”的话题。那么,根据“科学家同
意这起源于火星”这点来看B项,“同意”的对立面是“争议”,即
对应“dispute”,然后在B项中加入一个not的否定词,形成“双重
否定”,符合正确答案设置思路,因此B项为答案。

A项错在McKay team所提出的并不是“这起源于火星”,而是“
火星上有生命”。

文章从第二段开始有一个怀疑人士质疑的话题,但质疑的对象是
“火星上有生命”这个说法,并非题干所指的“ALH起源于火星”这
一话题。所以C、D、E是无关选项。

(4)分类不同,属性不同

在GRE填空中,这招也可使用。对于不同的分类,默认其对应事件
的属性是不同的。比如,“过去和现在”的时间不同导致观点不同,“精
英和大众”的分层不同导致理论不和。

例1 A divide between aesthetic and technical considerations has played


a crucial role in mapmaking and cartographic scholarship. Since nineteenth
century cartographers, for instance, understood themselves as technicians
who did not care about visual effects, while others saw themselves as
landscape painters. That dichotomy structured the discipline of the history of
cartography. Until the 1980s, in what Blakemore and Harley called “the Old
is Beautiful Paradigm,”scholars largely focused on maps made before 1800,
marveling at their beauty and sometimes regretting the decline of the pre-
technical age. Early mapmaking was considered art while modern
cartography was located within the realm of engineering utility. Alpers,
however, has argued that this boundary would have puzzled mapmakers in
the seventeenth century, because they considered themselves to be visual
engineers.

It can be inferred from the passage that, beginning in the 1980s,


historians of cartography

A. placed greater emphasis on the beauty of maps made after 1800.

B. expanded their range of study to include more material created after


1800.
C. grew more sensitive to the way mapmakers prior to 1800 conceived
of their work.

D. came to see the visual details of maps as aesthetic objects rather than
practical cartographic aids.

E. reduced the attention they paid to the technical aspects of


mapmaking.

解析: 此题问“20世纪80年代之后的情况”,原文定位到“Unt
il the 1980s, in what Blakemore and Harley called ‘the Old
is Beautiful Paradigm,' scholars largely focused on maps mad
e before 1800, marveling at their beauty and sometimes regre
tting the decline of the pre-technical age”。定位句描述在20
世纪80年代之前,关注的焦点是“美”,而在文章一开始给我们了两
个分类——“美”和“实用”,所以在20世纪80年代之后,关注的就
是以“实用主义”为方向的事件。B项描述“1800年之后的素材”,
因为原定位句指出1800年前关注“美”,1800年之后关注“实用主义
”,所以B项和我们要找的答案方向一致,可先留下待比较。前文提
到了“美”和“实用主义”两个分类,后文的内容是针对前文的细节
展开。1800年前提到了“美”,即使文章没有说1800年之后是什么,
也可以通过文章的逻辑推测出是“实用主义”。

A:和要找的答案方向相反,排除。

C:继续关注1800年之前的作品,还是和“美”相关,和要找的
答案方向相反,排除。

D:无关。

E:降低关注“实用”方面,和要找的答案方向相反,排除。

答案为B项。

注: 如文章一开始没有给出两个分类,仅仅告知20世纪80年代之
前如何,则不可推出20世纪80年代之后的情况。因为从客观性逻辑来判
断,20世纪80年代之后的情况我们没法判断。但如果文章一开始给出两
个分类进行对比,作者支持一个,就等于反驳另外一个,即“分类不
同,属性不同”。

例2 Skepticism about the McKay team's claim remains, however. For


example, ALH84001 has been on earth for 13,000 years, suggesting to some
scientists that its PAH's might have resulted from terrestrial contamination.
However, McKay's team has demonstrated that the concentration of PAH's
increases as one looks deeper into ALH84001, contrary to what one would
expect from terrestrial contamination. The skeptic's strongest argument,
however, is that processes unrelated to organic life can easily produce all the
evidence found by McKay’ s team, including PAH's. For example, star
formation produces PAH's. Moreover, PAH's frequently appear in other
meteorites, and no one attributes their presence to life processes. Yet
McKay's team notes that the particular combination of PAH's in ALH84001
is more similar to the combinations produced by decaying organisms than to
those originating from nonbiological processes.

The passage suggests that the fact that ALH84001 has been on earth for
13,000 years has been used by some scientists to support which of the
following claims about ALH84001?

A. ALH84001 may not have originated from Mars.

B. ALH84001 contains PAH's that are the result of nonbiological


processes.

C. ALH84001 may not have contained PAH's when it landed on Earth.

D. The organic molecules found in ALH84001 are not PAH's.

E. The organic molecules found in ALH84001 could not be the result of


terrestrial contamination.

解析: 通过“13000年”和“科学家”定位到所给段落第二句话
,内容是“PAH's可能来自地球上的污染”。
PAH's可能来自地球上的污染,说明PAH's到达地球之前并不存在
(“到达地球之后”和到“到达地球之前”分类不同),得出答案为
C项。C项还用了“may”一词,提示此项的出错概率很小,增加了其
成为答案的可能性。may、relatively、probable、significance这
些虚词所在的选项成为答案的概率很大。
第九章 阅读中的题型介绍与详细解法

一、综述

GRE阅读不考记叙文,只考议论文。议论文由论点和论据组成,所
有GRE阅读考题都只围绕论点和论据出题,包含多选题。

1. 针对论点,有以下两种题型

主旨题+作者态度题

* 主旨题:针对全文性内容出题,类似给文章起标题、找到全文核
心观点等。

* 作者态度题:考查文章中作者针对某个对象的评价,要判断出是
正评价还是负评价。(一般而言,如果文章中没有出现作者评价,也不
会出作者态度题)

2. 针对论据,有以下两种题型

细节改写题+细节作用题(句子作用题)

* 细节改写题:针对定位句的局部细节内容出题,考查考生是否理
解题目,从而能准确定位,以及是否理解细节句本身所述的内容。

* 细节作用题:考查考生是否理解某个细节句(detail)的功能。细
节句的功能就是支持一个分论点,所以细节作用题并不考查细节句本身
的内容,而是考查其功能。做此类题时要读定位句的上下句,查看此细
节是针对哪个分论点写的。同时,如果定位的某个句子本身就是一段话
的分论点,则这句话之后的内容都是围绕此分论点展开的,找到这个分
论点所描述的话题即可找到答案,不需要再读定位句的上下句。

GRE阅读题型的图谱:
二、各类题型的详细解法

(一)主旨题

1. 出题方式举例

The passage is primarily concerned with discussing the...

Which of the following best describes the main idea of the passage?

The primary purpose of the passage is to...

Which of the following best characterizes the organization of the


passage as a whole?

The passage is primarily concerned with doing which of the following?

2. 出题特征

√ 考查全文性内容,并不是某个局部信息。

√ 没法像细节题一样去定位文章具体某处。
√ 题目提问的对象是文章通篇都在讲述的。

3. 余翔老师的GRE主旨题解法秘技

√ 看到包含contrast、compare、reconcile的选项,直接排除。

√ 细节不能是答案,找能给文章做标题的选项。

√ 选项描述得越虚越好。

√ 含有比较级“than”的选项往往不是答案。

√ 文章篇幅50%处及之后描述的话题当作主旨题答案方向。

√ 如果文章中有观点转折,在主旨题中要体现这个转折。

1 看到包含contrast、compare、reconcile的选项,直接排除

解读: 在所有的美研考试(GRE、GMAT、LSAT)中,包含上述三
个词的选项成为主旨题答案的题目不超过3道。据目前已经有的题库
,GMAT中有1题,LSAT中有2题,GRE中没有出现过。

如果一篇文章的主旨题答案中有contrast、compare,则每段话
都必须提到不同的人或组织对于相同的对象有不同的观点,类似这样
的文章太罕见了。美研考试的文章一般是“新老观点对比型”“现象
解释型”“正反观点对比型”“问题解决型”等。因此,考试时要牢
记“看到包含contrast、compare、reconcile的选项,直接排除”这
一技法。

2 细节不能是答案,找能给文章做标题的选项

解读: 可以将主旨题想成是给文章起一个标题,因此必须找到
一个可以概括全文内容的选项。也就是说,全文不可能针对某处细节
来谈,因此细节不能是主旨题答案。第四项原则“含有比较级‘than
’的选项往往不是答案”也可以从此处进行理解。除了可以把主旨题
想成是给文章找一个标题之外,也可以把主旨题想成是在文章的最后
一段话下面再写一个总结句。这个总结句的描述对象必须是前文各段
话题或多或少都提到过的对象。

3 选项描述得越虚越好

解读: 要找描述范围大,或者出错概率小的选项。如果一个选
项描述得比较虚,则这个选项本身出错概率就很小,作答主旨题尤其
要遵循“找虚”的思维。既然不能选细节,就反过来找描述范围大、
话题包含性很强的选项。

4 含有比较级“than”的选项往往不是答案

解读: 是第二项原则的具体应用。除了主旨题,细节题的选项
中如果出现“than”也要谨慎处理。切记一条,选项出现than表示此
项出错概率比较大。但也不是说含有“than”的选项就不能成为答案
,只是其出错概率大,考生做题时要更加小心仔细对待此项。

5 文章篇幅50%处及之后描述的话题当作主旨题答案方向

解读: 此原则的具体应用是“如果一篇文章由两段话组成,并
且两段话的篇幅(字数)差不多,则以第二段话描述的话题作为主旨
题答案的查找方向”。就像之前描述的,可将主旨题想象成在全文最
后一段话的下方再写一个总结句。既然是在最后一段话后面再写内容
,则要考虑全文后半部分所说的话题。可将文章前半部分描述的内容
理解为“背景”或“铺垫”,以文章篇幅50%处及之后描述的话题作
为主旨题的答案方向,例如,文章有90行,则根据45~90行的话题内
容去找答案。

当然,如果文章是三段话,前两段说的是同一个话题,第三段用
很小的篇幅(比如很短的1~2句话)说了一个新话题,则不能依文章
最后一段话来找主旨题答案。依次类推,不管文章有几段,考生须通
过判断“文章篇幅50%处及之后大体在描述什么话题”来找主旨题答
案。

6 如果文章中有观点转折,在主旨题答案中要体现这个转折

解读: 文章由“观点(论点)”和“细节”组成,“观点转折
”即一个转折句本身在文章中的功能是“论点”,在这个转折句之后
会有细节句对此进行展开。GRE的文章往往会对旧观点或大众观点进
行取非,以体现“科学的进步需要打破常规”的思想,且文章后面部
分主要描述取非(转折后)正确的理由或例子。类似这样的“新旧观
点对比型”文章,在找主旨题答案时优先找含“argument against..
.”“dispute...”等内容的选项。当然,类似“discuss”这样正反
态度都能描述的词汇也可以是答案,因为不管是转折还是支持,都属
于“discuss”。考生切记不可仅凭借一个词来判断选项对错,还是
要看选项整体的意思,只是在时间紧迫的情况下,可以优先考虑某些
选项。

(二)细节改写题

1. 出题特征:针对文章中的某处细节出题,并非针对全文性内容
出题

“细节改写题”中“改写”的意思是指选项会针对文章细节,以不同写
法重述原文章的话题。
解读: 选项的“不同写法”包括但不限于以下几种:(1)变论
述;(2)变角度;(3)变是非。

(1)变论述:指选项把原文中的某些动词、形容词、副词、名词
变换成其同义词、近义词,或其上义词。例如,文章中细节句是“余翔
是你的GRE老师”,选项是“余翔是你的英语老师”,此项可选。因为
GRE可以被认为是英语考试或逻辑考试的一种。若选项为“余翔辅导你
某个课程”,此项也正确。因为从名词“老师”变成动词“辅导”,不过是
相同意思的词性转换,只要话题一致即可选。

(2)变角度:指选项换了一种角度去描述原文细节句。例如,原
文是“余翔是你的GRE老师”,选项是“我是余翔的学生”,此项可选。因
为“老师—学生”只是角度换了,但描述的还是相同话题,为正确的改
写,切不可认为“学生”这个词在原文中没出现过而不选此项。GRE考试
和语言类的托福或雅思考试相比,有一个显著特点,即GRE考试是“以
所读到的文章内容为基础看是否可以接受选项的说法”,而并不是找用
词和原文一模一样的选项。根据逻辑,即便选项内容和原文千差万别,
但只要两者话题差不多,或选项话题是原文话题的一种前提假设,我们
都可以接受。例如,原文是“余翔在教你GRE”,选项是“你和余翔在互
动过程中”,即使选项和原文并没有在说同一件事情,但我们从逻辑上
可以接受选项的说法,这也属于判断GRE答案的一种方法,毕竟一个在
教,一个听,的确体现了“互动过程”。只要选项以原文内容为基础且从
逻辑上可接受,选项即可成为答案或答案备选。

(3)变是非:类似双重否定的一种改写方式。例如,原文是“余翔
是你的GRE老师”,把“老师”改成其对立面“学生”,变成“余翔是你的
GRE学生”,这句话肯定和原句是相反的。但如果再加一个“取反”的词
汇,变成双重否定,即“余翔不是你的GRE学生”,这样的描述即为正
确。因为“余翔是你的GRE老师”,那当然“不是你的GRE学生”,这种说
法符合双重否定的出题思维。

例 In 1994, a team of scientists led by David Mckay began studying the


meteorite ALH84001, which had been discovered in Antarctica in 1984. Two
years later, the McKay team announced that ALH84001, which scientists
generally agree originated on Mars, contained compelling evidence that life
once existed on Mars. This evidence includes the discovery of organic
molecules in ALH84001, the first ever found in Martian rock. Organic
molecules—complex, carbonbased compounds—form the basis for terrestrial
life. The organic molecules found in ALH84001 are polycyclic aromatic
hydrocarbons, or PAH's. When microbes die, their organic material often
decays into PAH's.

Skepticism about the McKay team's claim remains, however. For


example, ALH84001 has been on earth for 13,000 years, suggesting to some
scientists that its PAH's might have resulted from terrestrial contamination.
However, McKay's team has demonstrated that the concentration of PAH's
increases as one looks deeper into ALH84001, contrary to what one would
expect from terrestrial contamination. The skeptic's strongest argument,
however, is that processes unrelated to organic life can easily produce all the
evidence found by McKay' s team, including PAH's. For example, star
formation produces PAH's. Moreover, PAH's frequently appear in other
meteorites, and no one attributes their presence to life processes. Yet
McKay's team notes that the particular combination of PAH's in ALH84001
is more similar to the combinations produced by decaying organisms than to
those originating from nonbiological processes.

The passage asserts which of the following about the claim that
ALH84001 originated on Mars?

A. It was initially proposed by the McKay team of scientists.

B. It is not a matter of widespread scientific dispute.

C. It has been questioned by some skeptics of the McKay team's work.

D. It has been undermined by recent work on PAH's.

E. It is incompatible with the fact that ALH84001 has been on Earth for
13,000 years.

解析: 此题定位到“which scientists generally agree orig


inated on Mars”,因为别的句子都没有描述“起源于火星”的事情
。以此句为基础,得出答案为B项。此项即是双重否定,为“变是非
”的答案描述方式。

如果疑惑为什么C、D、E项不能选,请再看下题目所问。文章之
后描述的怀疑对象是“火星上有生命”,并不是题干中所讲的“起源
于火星”。

2.解题策略

思考角度1: 分辨题目是不是细节题。一般如问得很细,非考查全
文性内容的题目即为细节题。常见的出题特征是题目中含有“according
to the passage”“文章中某个细节内容暗示(suggest)”之类可以明确定位
到文章某处的信息。

注意: 如果题目针对某个对象出题,然而这个对象在文章中多处
提及导致考生无法定位到某处,这样的考题不是细节题。细节题的答案
只针对文章的某句或某几句话,而非全文范围内多处内容。

解读: 出题者可能会把本质考查主旨或作者主要观点的题目出
成细节题的样子,以此来误导考生。

例 (第一段)现象……

...(尾句)Reaction to this phenomenon within Western


anthropological circles is sharply divided.

(第二段)One faction, led by anthropologist James Weiner, sees the


proliferation of video and television as the final assault of Western values on
indigenous cultures. Weiner argues that(细节1). He believes that(细节
2). Thus,Weiner concludes,(细节3). Moreover, he maintains that(细节
4).

(第三段)But Weiner's opponents contend that his views betray a


certain nostalgia for the idea of the “noble savage.”One such opponent,
anthropologist Faye Ginsburg...(细节5). Unlike Weiner, Ginsburg
maintains that(细节6).

(第四段)The Brazilian field work of anthropologist Terence Tumer,


who studies the relationshipbetween traditional Kayapo culture and Kayapo
videotapes, lends credence to Ginsburg's position.Primarily an oral society,
the Kayapo...(文章结束)

Which one of the following is most analogous to the Kayapo's use of


video to document ceremonial performances, as that use is described in the
last paragraph?

A. As various groups have emigrated to North America, they have


brought their culinary traditions with them and thereby altered the culinary
practices of North America.

B. In the 1940s, Latin American composers incorporated African


American inspired jazz instrumentation and harmonies into their music but
remained faithful to the traditions of Latin American music.

C. Some writers are predicting that the interactive nature of the Internet
will fundamentally reshape fiction, and they are already producing narratives
that take advantage of this capacity.

D. In the late 1980s, some fashion designers produced lines of various


articles of clothing that imitated fashions that were current in the 1920s and
1930s.

E. Early in the twentieth century, some experimental European artists


rejected the representational traditions of Western painting and began to
produce works inspired by surrealist literature.

解析: 文章仅保留分论点相关的句子,细节部分在实际考试中
可略读或快速阅读。如果后文是针对前文内容的细节展开,则后文所
述内容和之前的分论点方向大致相同。

第2段中细节1~4讨论的话题是一样的,即“Weiner认为科技对土
著文化是一种入侵”。第3段开头的转折词提示后文观点和前文内容
不同,可按照“科技对文化不是一种入侵”的方向再继续读下去。

提示:要做到带着预测读文章,而不是读完文章之后再总结,这
是文章快速阅读的一种方式。

第4段发现具体的例子Kayapo,例子一定是针对之前的论点展开
的,所以即使没有读Kayapo例子的具体内容也能推测出此处大致说的
话题是“科技对于土著文化没有影响”。

此题题目问的是Kayapo的例子内容,看上去是细节题,但是Kaya
po的例子就是要证明之前第3段转折词之后的观点是对的而展开的细
节描写。所以根据第3段转折后的观点来找答案即可。因此,此题看
上去是考细节,实质考的是主旨观点。

B项的“remained faithful to the traditions”和“文化没有


被影响”是同一个方向的,此项为题目答案。

补充实际解题过程中的三个技巧:

(1)按照预测的答案的话题(事件)方向看选项,这样解题思路
就不会被选项的内容带偏。

(2)选项只要符合预测的答案(事件)的方向或话题部分沾边,
即可先放着待比较。

(3)部分沾边、方向符合的选项不一定完全正确,但要找的不是
百分之百正确的选项,只要这个选项比其他选项更好即可。像此题只有
B项沾边,其他选项和“文化不会受影响”这一话题没有任何交集,所以
找相对最佳的答案即可,不可用完美主义思想找GRE阅读题的答案。

思考角度2: 在确定题型为细节题后,如题目没有“primary to/in


order to”等表示目的的词汇,则此题为“细节改写题”。

解读: 如前文所述,细节题有两种出题类型——细节改写题,
细节作用题。如题目本身包含“为了,目的”之类的词汇,则为细节
作用题。细节作用题和细节改写题在解题步骤上有不同,故要看清题
目真正考查的方向。

两种题型举例如下:

细节改写题:

(1)According to the passage, most pre-1990 literature on businesses'


use of IT included which of the following?

(2)It can be inferred from the passage that when aspen groves
experience a “disturbance”, such a disturbance...

(3)Which of the following, if true, would most weaken the author's


argument in lines 18-26(“Furthermore… sloth bear cubs”)?

(4)The passage suggests that the presence of G10 garnet in a


kimberlite pipe indicates that...

(5)According to the passage, Gurney refined the use of ilmenites in


prospecting for diamonds in which of the following ways?

细节作用题:

(1)The author of the passage compares the relationship between an


organism's biological clock and its environment to the relation between a
wristwatch and its owner most probably in order to...

(2)As an element in the argument presented by the author of the


passage, the reference to Blue Clark's study of the Lone Wolf case serves
primarily to...

(3)In the first paragraph, the author of the passage mentions a


contention that would be made by an anthropologist most likely in order to...

(4)The author of the passage mentions “empirical evidence”(line


10)primarily in order to...
(5)The author of the passage discusses productivity in the retail
industry in the first paragraph primarily in order to...

技巧: 如题目没有问句子功能或无“primary to / in order to”,一般


就是“细节改写题”。

思考角度3: 思考题目是考查细节本身,还是作者或文章中其他某
个人对于细节的看法或评价。如果是考查“评价”,则要找作者或第三方
的观点,而非定位到细节句本身。

解读: 对题目的解读不同,考生定位的句子也不同。如果定位
不正确,就会解答错误。实际解题时,确认是细节题前,再看下此题
是否在问“文章认为或作者认为”,如果是,需要结合作者的观点来
找答案。

例 In her study of the fifteenth-century Florentine widow Alessandra


Strozzi, a historian who specializes in European women of the Renaissance
attributes individual intention and authorship of actions to her subject. This
historian assumes that Alessandra had goals and interests different from those
of her sons, yet much of the historian's own research reveals that Alessandra
acted primarily as a champion of her sons' interests, taking their goals as her
own. Thus Alessandra conforms more closely to the anthropologist's notion
that personal motivation is embedded in a social context. Indeed, one could
argue that Alessandra did not distinguish her personhood from that of her
sons. In Renaissance Europe the boundaries of the conceptual self were not
always firm and closed and did not necessarily coincide with the boundaries
of the bodily self.

The passage suggests that the historian mentioned in the second


paragraph would be most likely to agree with which of the following
assertions regarding Alessandra Strozzi?

A. Alessandra was able to act more independently than most women of


her time because she was a widow.
B. Alessandra was aware that her personal motivation was embedded in
a social context.

C. Alessandra had goals and interests similar to those of many other


widows in her society.

D. Alessandra is an example of a Renaissance woman who expressed


her individuality through independent action.

E. Alessandra was exceptional because she was able to effect changes in


the social constraints placed upon women in her society.

解析: 此题不是问“Alessandra Strozzi”的观点,而是问历


史学家(historian)的观点。

此题不可选B项。B项错在不是“Alessandra意识到”,其后是作
者的观点。

答案为D项,历史学家的观点只有这句话:This historian assu


mes that Alessandra had goals and interests different from t
hose of her sons, yet much of the historian's own research r
eveals that Alessandra acted primarily as a champion of her
sons' interests, taking their goals as her own.原文此句后的
内容都是作者的观点,并不是历史学家的观点,因此定位句后的内容
均不可成为答案。

思考角度4: 确定是“细节改写题”后,可用题目中关键词(又
称“定位词”)去找文章的出题句(又称“定位句”)。

确定关键词:

第一,尽量用两个及以上的词汇去定位,因为仅仅用一个词定位容
易犯“模糊定位”的错误。

第二,优先用名词、数字或特殊词汇(比如大写词汇)当定位词,
这样方便查找。

补充:优先采用名词,并不是指不能用动词或形容词来定位。

第三,忌用文中通篇都有的词汇当作定位词,以免无法确定应该定
位到何处。

例 Over the last 150 years, large stretches of salmon habitat have been
eliminated by human activity: mining, livestock grazing, timber harvesting,
and agriculture as well as recreational and urban development. The numerical
effect is obvious: there are fewer salmon in degraded regions than in pristine
ones; however, habitat loss also has the potential to reduce genetic diversity.
This is most evident in cases where it results in the extinction of entire
salmon populations. Indeed, most analysts believe that some kind of
environmental degradation underlies the demise of many extinct salmon
populations. Although some rivers have been recolonized, the unique genes
of the original populations have been lost.

Large-scale disturbances in one locale also have the potential to alter the
genetic structure of populations in neighboring areas, even if those areas have
pristine habitats. Why? Although the homing instinct of salmon to their natal
stream is strong, a fraction of the fish returning from the sea(rarely more
than 15 percent)stray and spawn in nearby streams. Low levels of straying
are crucial, since the process provides a source of novel genes and a
mechanism by which a location can be repopulated should the fish there
disappear. Yet high rates of straying can be problematic because misdirected
fish may interbreed with the existing stock to such a degree that any local
adaptations that are present become diluted. Straying rates remain relatively
low when environmental conditions are stable, but can increase dramatically
when streams suffer severe disturbance. The 1980 volcanic eruption of
Mount Saint Helens, for example, sent mud and debris into several tributaries
of the Columbia River. For the next couple of years, steelhead trout(a
species included among the salmonids)returning from the sea to spawn
were forced to find alternative streams. As a consequence, their rates of
straying, initially 16 percent, rose to more than 40 percent overall.

Although no one has quantified changes in the rate of straying as a result


of the disturbances caused by humans, there is no reason to suspect that the
effect would be qualitatively different than what was seen in the aftermath of
the Mount Saint Helens eruption. Such a dramatic increase in straying from
damaged areas to more pristine streams results in substantial gene flow,
which can in turn lower the overall fitness of subsequent generations.

It can be inferred from the passage that the occasional failure of some
salmon to return to their natal streams in order to spawn provides a
mechanism by which

A. pristine streams that are near polluted streams become polluted


themselves.

B. the particular adaptations of a polluted stream's salmon population


can be preserved without dilution.

C. the number of salmon in pristine habitats decreases relative to the


number in polluted streams.

D. an environmentally degraded stream could be recolonized by new


salmon populations should the stream recover.

E. the extinction of the salmon populations that spawn in polluted


streams is accelerated.

解析: 全文都在讲三文鱼(salmon),因此不可用“三文鱼”
来当定位词。

此题有专有名词natal streams,可选择其作定位词,不可用ret
urn/failure这样表示动作的词汇作定位词,因为这些词汇很容易被
改写成同义词或近义词,增加定位难度。

此题最终问的是mechanism,所以定位词确定为两个。

找到natal steam所在的句子“Although the homing instinct


of salmon to their natal stream is strong, a fraction of the
fish returning from the sea(rarely more than 15 percent)st
ray and spawn in nearby streams.”,但没有找到mechanism,这
表示还不能解题,所以继续往下读,一直读到可以解答问题的时候,
即停止查找。“Low levels of straying are crucial, since the
process provides a source of novel genes and a mechanism by
which a location can be repopulated should the fish there di
sappear.”这句话中有mechanism,因此此题的答案在“by which”
之后的句子中找。哪个选项和“a location can be repopulated sh
ould the fish there disappear”是一个话题,该项即为答案。

答案为D项。

思考角度5: 确定关键词之后,找其在文章中第一次出现的位置。

解读: GRE阅读考试中,如果不算加试,会碰到9篇文章。其中8
篇是只有一段话的短篇文章和篇幅略长一点儿的中篇文章,真正由3~
4段话构成的长篇文章只占一篇。因为大多数文章篇幅有限,所以定
位关键词在文章中第一次出现的位置即可。

从已有的美研考试题库来判断,即使在长篇文章中答案往往也出现
在关键词第一次出现的位置上,低于5%的试题答案在关键词非第一次
出现的位置上。

如果不定位到关键词第一次出现的语句上,该往前文找还是往后文
找关键词再次出现的句子?应采取“定位于关键词第一次出现的语句”,
如其本身和其上下句也不能回答问题,则往下文找其在文章中第二次出
现的地方。

思考角度6: 读关键词所在的一整句话,思考此句是否回答了问
题。

解读: 对于大多数考题,定位句本身即可回答问题,但并不是
所有细节改写题都是这样的。要具体题目具体分析,判断清楚定位句
是否回答了问题。如没有回答,先看定位句的上下句,然后再找关键
词在文章中第二次出现的句子。这里尤其需要考生注意的是,不是立
刻找关键词下一次出现的句子,而是先看定位句的上下句是否回答了
问题。

3.出题陷阱

除了题目本身有陷阱外,在定位词的确定、查找,以及文章的读法
和选项的对比过程中都有出题机构设置的“坑”。这里以“出题陷阱”举
例。比如题目问“作者对于第2段第4行的某个名词持有什么看法”。这题
的陷阱是考生直接找到第2段第4行,但这样的解题切入点就“入坑”了。
因为题目并没有问细节的内容,而是作者的观点。一般而言,文章越往
后越会体现作者最终的观点,所以应该在文章靠后的地方找作者观点,
带着“最终观点”的内容来找选项。换个角度来看,类似这样的题根本不
需要定位,因为作者观点就是全文主旨,我们读文章的目的就是要读出
主旨,所以凭借主旨直接找选项即可,因为主旨反映了作者的最终观
点。

为了避免“入坑”,需要看清题目是问“细节本身”、“作者对于细节
的观点”,还是“细节的目的”。题型不同,解法会不一样。

4.余翔秘技

(1)如定位句本身可解题,则不用思考定位句之外的内容。

例 ... A potential objection is that the reasoning given by the judge,


however legally adequate, may not be the judge's real reasoning, thus
allowing for the presence of undetected bias. However, as long as a
knowledgeable observer cannot find any fault with the legal reasoning
provided, then there are no grounds for complaint. Under the law, a right of
recourse arises only if harm accrues. If a judge who had no improper
considerations in mind could have reached the same conclusion for the
reasons stated by a judge who had hidden reasons in mind, then there is no
harm on which to base a complaint.

The author of the passage regards the legal principle that “a right of
recourse arises only if harm accrues”as
A. an established principle of law.

B. part of the definition of the function of the law.

C. a tool for judges to disguise their real reasoning.

解析: 此题定位到黑体句,根据“优选和定位句本身话题沾边
的选项”这一做题方法得出答案为A项。

(2)不可只用1~2个词来判断选项正确与否,要看选项整体意思。

(3)除了文章本身给出的答案提示外,还有其他“提示点”。

① 题目给出的提示。

解读: 一般做题流程是从题目至原文,再到选项。但是,只要
我们能从五个选项中找到一个正确概率最大的选项,即完成了任务。
因此,无须每道题都遵循标准步骤。

阅读解题的三个要素:题目、选项和文章。三者之中,题目最重
要。题目告诉我们大致定位方向,题目本身可能也包含了正确答案的话
题。

提示: 找选项最终的目的是回答问题,所以如果题目本身写得很
具体,已经包含了某个话题,则答案一定和这个话题沾边。

例 Regulations will not allow a pesticide that is toxic to humans to be


used inside houses unless the pesticide will dissipate completely from the air
within eight hours after its application.One test that pesticide manufacturers
standardly use to determine how quickly anti-termite pesticides dissipate
involves spraying the pesticides on the walls of room-sized plywood boxes
and then timing its dissipation.
Which of the following would it be most useful to know in order to
evaluate whether a dissipation time of just under eight hours on the
manufacturers' test indicates that an anti-termite pesticide that is toxic to
humans obeys regulations for use in houses?

A. Whether anti-termite pesticides dissipate more slowly in furnished


rooms than in plywood boxes.

B. Whether people who apply anti-termite pesticide standardly wear


protective equipment that prevents them from being exposed to the pesticide.

C. Whether people whose house is being treated with anti-termite


pesticide generally know that they should remain out of their house during
the hours immediately after the pesticide's application.

D. Whether there are anti-termite pesticides that are toxic to humans


that, when subjected to the manufacturer's test, dissipate completely from the
air in the boxes in well under eight hours.

E. Whether anti-termite pesticides that are not toxic to humans tend to


take longer to dissipate than those that are toxic.

解析: 此题题干中包含了“实验室和家庭”类比关系的话题,
正确答案一定是和此话题最相关的选项。下面找与“实验室和家庭”
相关的选项。

A项:“room与plywood boxes”的关系。

B项:“人们是否穿保护装备”。

C项:“人们是否知道他们应该在户外”。

D项:“是否有一种对人有害的杀虫剂能在8小时内完全挥发”。

E项:“是否有无毒的杀虫剂”。

以话题提炼的方式把五个选项读完之后会发现,只有A项的“ply
wood boxes”和“实验室”有关系,进而可确定只有第一个选项和要
选的答案相关联,故答案为A项。如果不确定“plywood boxes”是否
和“实验室”有关,则只需要回到原文查找实验里面是否有plywood
boxes即可,此验证过程比读完全文要快速、简单多了。

② 选项给出的提示。

解读: 如果阅读考查的题型为简答题,那就没有任何技巧可用
了。对于选择题而言,选项排列的逻辑关系也能帮助我们确定答案的
范围,或发现查找答案的方向。

例1 Which of the following statements about changes in passengers' use


of urban rail systems can be inferred from the passage?

A. The majority of rail passengers in the 1950s had previously been


highway users.

B. During the 1950s, fewer highway users became rail passengers than
vice versa.

C. During the 1950s, rail ridership declined more slowly than had been
anticipated.

D. The changes in rail ridership in the 1950s were result of changes in


urban population density.

E. The changes in rail ridership in the 1950s occurred independently of


changes in the capacity of the transportation system as a whole.

解析: 五个选项中都含有“1950s”,则可通过“1950s”这个
线索词回到原文,查看有“1950s”的句子。若有选项和读到的话题
一致,即为备选答案。
例2 It can be inferred from the passage that the author would be most
likely to agree with which one of the following statements about the
relationships between the three styles in which Schoenberg wrote?

A. Each successive style represents a natural progression from the


previous one.

B. Each successive style represents an inexplicable departure from the


previous one.

C. The second style represents a natural progression from the first, but
the third style represents an inexplicable departure from the second.

D. The second style represents an inexplicable departure from the first,


but the third style represents a natural progression from the second.

E. The second style represents an inexplicable departure from the first,


but the third style represents a natural progression from the first.

解析: 此题的常规性做法是回原文读相关内容,再分析每个选
项。这种方式显然没法在实际考试中运用,因为太耗时间了。

当读完题目感觉迷茫时,可看一下选项,通过选项间的共同点、
差异点或排列的逻辑关系找出正确答案的线索。

此题后四个选项都在描述某个风格和之前风格有哪些不同(depa
rture),只有A项讲这些风格是顺接的关系。

文章中如果要描述两者之间的关系“断开”,则必然要用from、
departure、deviate这样的转折词,所以考生要做的便是回到原文大
致定位的地方找这些可能表示“不同,转折,断开”的词汇,如果找
到则仔细看是哪个与哪个“断开”。如果找不到,则答案便是A项。

③ 找出错概率最小的选项。
④ 找正确概率最大的选项。

解读: 出错概率小是指选项描述得非常模糊、宽泛,缺少实际
具体内容,或者通过常识无法判定选项是错误的,又或某个选项的表
达有一种说法带有余地、语气不是非常绝对的感觉。反之,选项描述
得越具体,语气越强烈,含有比较关系等,出错概率越大。比如,“
这场战役中数千人阵亡”与“这场战役中2 800人阵亡”这两句话中
,哪句话出错概率大?显然第二句话说得更明确,越明确的表达出错
概率就越大。

例1 It can be inferred from the passage that the author would be most
likely to agree with which one of the following?

A. There is evidence that objects similar to wampum were used for


symbolic representation by other peoples in addition to the Haudenosaune.

B. The Europeans who first came in contact with the Haudenosaune


insisted on using wampum as a form of currency in spite of their awareness
of its true significance.

C. There is evidence that Europeans who came in contact with the


Haudenosaune adopted some long-standing Haudenosaune uses of wampum.

D. A long-term peaceful association among the groups that formed the


Haudenosaune Confederacy was an important precondition for the use of
wampum as a means of communication.

E. Present day interpretations of the significance of some of the symbols


used in wampum belts are not conclusive.

解析: “批评性思维”在考试中的应用是指,只要选项的某些
信息没有错,即可放着待选。也就是说,通过常识不能排除的选项可
作为答案备选。
目前能够盖棺定论地被我们描述成研究透的科学理论少之又少,
所以E项“某些符号没有被充分地解释”出错概率极小。

例2 The passage suggests that if urban rail systems had received


adequate funding in the 1950s, which of the following would have been most
likely to have occurred?

A. The number of highway users would have declined sharply.

B. The population density of urban areas would have increased


significantly.

C. Rail transportation's ridership would have declined more slowly than


it had prior to 1950.

D. The volume of rail ridership would have increased faster than


planners anticipated.

E. The volume of automobile traffic would have increased less rapidly


than it did.

解析: 此题如果A和E两项都符合答案的方向,选哪个?

A项提到“declined sharply”,E项提到“increased less rap


idly”,哪一个选项的说法相对委婉,说法有保留的余地?显然是E
项。所以此题E项出错概率更小。

例3 The passage suggests that the presence of G10 garnet in a kimberlite


pipe indicates that

A. the pipe in which the garnet is found has a 90% chance of containing
diamonds.

B. the levels of calcium and chrome in the pipe are conducive to


diamond formation.

C. the pipe passed through a diamond-stability field and thus may


contain diamonds.

D. any diamonds the pipe contains would not have come from the
diamond-stability field.

E. the pipe's temperature was so high that it oxidized any diamonds the
pipe might have contained.

解析: 此题A和C项中都描述“含有钻石”的事情。在没有读文
章的情况下,A和C项比其他选项成为答案的概率更大,因为出题机构
围绕正确答案来设一个诱惑选项的概率大于创造一个和正确答案完全
无关的选项。

A项描述“有90%的概率会含有钻石”,C项表示“可能含有钻石
”,自然C项正确。尤其注意may的用法,最终的真实情况“含有钻石
”或“不含有钻石”都不影响“可能含有钻石”这个说法的正确性。
所以如果有选项含有relatively、may、probably等表示相对性的词
汇,则这个选项错误概率会大大降低。

除上述提示之外,还有“之前的试题给出的提示”以及“之后的试题
给出的提示”等其他正确答案的提示线索。

GRE阅读,尤其是长篇文章阅读,考查考生短时间记忆力,具体要
记忆的地方如下:

第一,做完所有试题前,文章的主旨或大致脉络不可忘记。

解读: 遇到主旨题和不容易定位的细节题时,第一步就是思考
文章的脉络。
第二,做完一道试题前,题目精准提问的地方不可忘记。

解读: 不可因为读了选项而把题目所问之处下意识地修改了。

第三,定位句所说的话题不可忘记。

解读: 不可因为读选项把文章定位句所讲的话题给修改或忘记
了。

第四,对于含有出题点的句子,在阅读文章时需多加留意,避免选
项出现对应话题时自己的脑中没有一点印象。

特别是当文章特别难,靠理解能力不能解答试题时,只有靠记忆力
来做题。

(4)常考出题点

读文章时要注意哪些段落句子在之后可能会出题,不出题的语句可
以读得相对快一些。常见出题点包含以下这些:

① 转折关系:比如句子中包含however、nevertheless、yet等。

② 让步关系:比如句子中包含while、although、though、even
though、despite、albeit等。

③ 因果关系:比如句子中包含in that、because、because of、due


to、thanks to等。

④ 比较关系:常见的有“A than B”逻辑关系的比较句。

⑤ 段落末句出现新话题。

⑥ 强调句:比如句中含有表示“唯一”的词汇(solely、only等),
或最高级表达(the most the best等),或褒贬色彩明显的修饰语
(unfortunately等)。

⑦ 括号中的内容。

* 如果一句话中出现两个及以上出题点,则这句话被考到的概率很
高。

读这段话时,画线部分有一个让步关系的出题点,此句话说的
例1
话题要留意一下。

In the fourteenth and fifteenth centuries, many Western Pueblo


settlements in what is now the southwestern United States may have
possessed distinctly hierarchical organizational structures. These
communities' agricultural systems―which were “intensive”in the use of
labor rather than “extensive”in area―may have given rise to political
leadership that managed both labor and food resources. That formal
management of food resources was needed is suggested by the large size of
storage spaces located around some communal Great Kivas(underground
ceremonial chambers). Though no direct evidence exists that such spaces
were used to store food, Western Pueblo communities lacking sufficient
arable land to support their populations could have preserved the necessary
extra food, including imported foodstuffs, in such apparently communal
spaces.

画线句里面有一个“by contrast”的对比关系,但是考生需要把整
例2
句话都看完,这样才能看出对比之处在哪里。

Until recently, zoologists believed that all species of phocids(true


seals), a pin-nipped family, use a different maternal strategy than do
otariids(fur seals and sea lions), another pinniped family. Mother otariids
use a foraging strategy. They acquire moderate energy stores in the form of
blubber before arriving at breeding sites and then fast for 5 to 11 days after
birth. Throughout the rest of the lactation(milk production)period, which
lasts from 4 months to 3 years depending on the species, mother otariids
alternately forage at sea, where they replenish their fat stores, and nurse their
young at breeding sites. Zoologists had assumed that females of all phocids
species, by contrast, use a fasting strategy in which mother phocids, having
accumulated large energy stores before they arrive at breeding sites, fast
throughout the entire lactation period, which lasts from 4 to 50 days
depending on the species. However, recent studies on harbor seals, a phocids
species, found that lactating females commenced foraging approximately 6
days after giving birth and on average made 7 foraging trips during the
remainder of their 24-day lactation period.

括号中的内容不可跳过,而是要多留心注意,因为题目常常会
例3
考到括号里面的内容。要记住:举例之后的内容可快速查或省略不
看,但括号里面的内容必须看。

Folk blues and classic blues are distinguished from one another by
differences in performers(local talents versus touring professionals),
patronage(local community versus mass audience), creation(improvised
versus composed), and transmission(oral versus written).

(三)细节作用题

除了考查和文章脉络或文章主旨相关的题目之外,阅读部分考查较
多的题目就是细节题。前面谈到了细节改写题的相关技巧,接下来将详
细讲述细节作用题的解题技巧。

1. 细节作用题的判定标准

在题目中看到“in order to/primarily to”等表示“目的”的词汇,基本可


判定此题为细节作用题。

The author of the passage quotes Urban and Sherzer most probably in
order to

The passage mentions the possible benefit to algae of nitrogen falling


down in the rain most likely in order to

The primary purpose of the third paragraph of the passage is to

In the passage, the mention of “Alabama, Arkansas, Mississippi, and


Tennessee”serves primarily to
The final sentence of the passage primarily serves to do which of
following?

2. 细节作用题的特征

教学界通常叫的细节作用题这个表达不精确,应该叫句子作用题,
因为所考查的画线句不一定是细节句,画线句完全可以是一个描述分论
点的句子。如果画线句是细节句,则要找到细节句所描述的分论点,用
分论点的内容来找答案;如果画线句本身是分点句,就用定位句本身来
当答案的方向找选项即可。

除了考句子外,出题机构还会考查整个段落的写作目的、段落间的
逻辑关系。假设在一个段落中有一个句子特别长,那么这个句子自然可
以独立成为一个段落。因此段落与段落间的关系完全等同于句子与句子
间的逻辑关系,即“顺接”“逆接”“新话题”。因此考查段落目的的时候,
需要借助全文脉络来分析这个段落的作用,判断其是针对前面一段的细
节展开,还是与前文形成转折关系,或是描述了一个新话题。

3. 细节作用题与细节改写题在做题过程中的区别

解答细节改写题时,如果定位句本身可以解题的话,优先用定位句
本身的内容找答案,不要看定位句之外的句子。细节作用题考查的定位
句一般是关于一个细节的,所以要分析定位句与前后句的逻辑关系,找
到定位句所描述的分论点句子。也就是说,改写题主要看定位句本身,
作用题(目的题)主要分析定位句和前后句的逻辑关系。

4. 试题演练

例1 During the later eighteenth century, wealthy gentlemen exchanged


the velvets and satins long in fashion for somber woolen suits. Frantz
contends that this change reflected deeper cultural changes.The value once
placed on men's expressiveness, reflected in Mackenzie's novel The Man of
Feeling (1771), gave way to a preference for emotional restraint. In
Austen's novels, the heroine often struggles to glimpse the true nature of hero
beneath his reserved exterior.

The author of the passage mentions The Man of Feeling(1771)in


order to

A. contrast Mackenzie's reasons for writing novels with those of Austen.

B. introduce evidence regarding the influence of particular writers on


Austen.

C. corroborate a claim that a convention of masculine behavior changed


during Austen's lifetime.

D. suggest that Austen's novels were more reflective of their historical


context than Mackenzie's had been.

E. challenge a particular misconception about the modes of behavior


common among gentlemen in the later eighteenth century.

解析: 题目中有一个数字,所以很容易识别定位词在原文的位
置——“The value once placed on men's expressiveness, refle
cted in Mackenzie's novel The Man of Feeling(1771), gave
way to a preference for emotional restraint.”,“reflected
in Mackenzie's novel 修饰“man's expressiveness”,此句话的
内容是“价值观有些改变”,根据句意来看选项,只有C项提到了“
男性有变化”这样的话题,其他选项关联度很小。

例2 Some attine ants carry vegetation into their nests and add fungal
material, thereby creating “gardens”in which fungal food for the ants grow.
Because the ants play the behaviorally active role, it seems compelling to say
that they cultivate and control the passive fungi. But even if that is true, the
symbiotic association has existed for so long that cultivar traits may have
evolved in the fungi that benefit the fungi but not necessarily the ants.
Furthermore, many microorganisms have developed sophisticated
mechanisms to manipulate the physiology and behavior of their symbiotic
animals. It is not implausible, therefore, that the ants' fungi have evolved
chemical and physiological schemes that alter ant behavior to serve the
fungus' reproductive interests, possibly even compromising the reproductive
interests for the ant hosts.

The passage points out which of the following in order to explain the
appeal of the notion that some attine ants cultivate and control fungus?

A. The ants play the behaviorally active roles in the symbiotic


relationship.

B. The ants purposefully carry vegetation into their nests for their own
consumption.

C. Fungus is physiologically a passive substance.

D. Other organisms are able to manipulate their symbiotic partners.

E. The chemical and physiological properties of the fungus are subject


to ant influence.

解析: 题干中的关键词为“ants cultivate and control fung


us”,其在文中第一次出现的位置是在第二句话“Because the ants
play the behaviorally active role, it seems compelling to sa
y that they cultivate and control the passive fungi.”,这句
话本身就是一个论点,根据此句内容来筛选答案。

A项和原文定位句明显有部分交集,可先放着此项。B项和定位句
描述方向一致,也可以先放着此项,待比较。C项和定位句的内容也
有部分相关性,可以放着。其他生物和原文中描述的“蚂蚁”“真菌
”无关,排除D项。化学和生理学特征与蚂蚁的关系并不是定位句的
内容,排除E项。

A、B、C三项内容和定位句都沾边,此时要看清楚问题。题目说
的是“The passage points out which of the following”(文章
指出下面哪个),既然题目中已经暗示了“文章指出”,则表示“方
向正确,但文章没有明确指出的信息”不是此题要问的内容。A、B、
C三项中哪一个选项原文中提过?只有A项。
例3 Although the passenger pigeons, now extinct, were abundant in
eighteenth- and nineteenth-century America, archaeological studies at
twelfth-century Cahokian sites in the present day United States examined
household food trash and found that traces of passenger pigeon were quite
rare. Given that the sites were close to a huge passenger pigeon roost
documented by John James Audubon in the nineteenth century and that
Cahokians consumed almost every other animal protein source available, the
archaeologists conducting the studies concluded the passenger pigeon
population had once been very limited before increasing dramatically in post-
Columbian America. Other archaeologists have criticized those conclusions
on the grounds that passenger pigeon bones would not be likely to be
preserved. But all the archaeological projects found plenty of bird bones and
even tiny bones from fish.

The author of the passage mentions “tiny bones from fish”primarily in


order to

A. explain why traces of passenger pigeon are rare at Cahokian sites.

B. support a claim about the wide variety of animal proteins in the


Cahokian diet.

C. provide evidence that confirms a theory about the extinction of the


passenger pigeon.

D. cast doubt on the conclusion reached by the archaeologists who


conducted the studies discussed in the passage.

E. counter an objection to an interpretation of the data obtained from


Cahokian sites.

解析: 由题目中的“骨头”定位到原文最后一句话,因为“in
order to”题型需要关注句子与句子间的逻辑关系,所以需要往前读
。前句“鸽子骨头不会被保存”和定位句“鱼骨头可以保存”构成一
个转折的逻辑关系。根据这样的逻辑结构来看选项:只有D和E两项有
转折关系,排除A、B、C三项。
在课堂中学生也会问D和E项该如何进行对比。按照一贯的做法,
对比选项时先要找到选项间的差异,D项削弱的是“做研究的考古学
家”的结论。原文中提到“鱼骨头”是在最后一句话,其驳斥的对象
是其他考古学家(other archaeologists),所以D项错误。E项削弱
了一个解释。明显原文最后一句话削弱的肯定不是做研究的人,因此
E项为答案。

例4 Many scholars have argued that government investment in


manufacturing in the southern United States during the Second World War
spurred a regional economic boom that lasted into the postwar period. But
much of this investment went to specialized plants, many of them unsuitable
for postwar production. Large-scale, wartime government funding led to a
massive increase in the number and scale of munitions facilities. By the war's
end, 216 munitions establishment costing more than $3.5 billion had been
built, many of them located in the south.Indeed, according to one estimate,
more than 70 percent of federally financed manufacturing construction
capital in Alabama, Arkansas, Mississippi, and Tennessee went into
munitions plants.

In the passage, the mention of “Alabama, Arkansas, Mississippi, and


Tennessee”serves primarily to

A. suggest that some states were better than others at anticipating


postwar economic needs.

B. identify evidence used to support a view held by scholars mentioned


at the beginning of the passage.

C. suggest that federal investment in some kinds of manufacturing was


excessive.

D. identify the states that received the largest allocations of federal


funds.

E. provide information to support a point about the nature of


government investment made earlier in the paragraph.
解析: 定位到文章末句“Indeed, according to one estimate
, more than 70 percent of federally financed manufacturing c
onstruction capital in Alabama, Arkansas, Mississippi, and T
ennessee went into munitions plants.”,此处出现递进关系的逻
辑提示词“Indeed”,因此我们要往前再看一句话,末句和倒数第二
、三句都是针对“But much of this investment went to speciali
zed plants, many of them unsuitable for postwar production.
”这句话展开的细节描写,所以要以“But”后的论点为方向来筛选
选项。

C和E项都相关,对比此两项。

E项没有错,C项中“excessive”是对原文的改写,根据“找一
个出错概率小的选项”的做题思路,E项更好。

三、GRE阅读考试的根本思维研究

下面,通过讨论以下三个话题,加深考生对GRE阅读考试的理解,
使其学会自我调整,达到事半功倍的备考效果。

1. GRE阅读和托福或雅思的阅读有什么区别?

2. 如何理解“解决问题的能力”?

3. 何为“出错概率小的选项”?

1. GRE阅读和托福或雅思的阅读有什么区别?

托福和雅思考试是针对母语为非英语的考生设计的测试英语熟悉度
的考试。GRE、GMAT、LSAT是目前世界上主流的研究生考试,是美
国本土学生也需要参加的考试,其考试目标自然不仅仅是测试学生的英
语能力。从我14年的授课经验和对考试的理解来看,GRE、GMAT、
LSAT考试的阅读部分主要考查以下能力:① 问题识别能力;② 信息搜
索和提取能力;③ 决策能力;④ 灵活应变能力;⑤ 短时间记忆能力。
这五大能力在研究生阶段学习和之后的工作中极其重要,可以这么说,
和思维相关的这五大能力的差异决定了人们能取得的成就上限的差距。
要在美研考试中获得高分,光努力是不够的,还需要学会思维转换,识
别和分析考试的规律。

练习: 3分钟内记住以下数字及对应的图像

学生们肯定会觉得这些数字和图形很难记住,死记硬背肯定会消耗
很多时间和精力。但如果把此图变成下面这样,大家是不是就能很快地
记住了?
不仅是考试,在研究生阶段的学习和毕业后的工作中,只靠努力是
不能把事情做成的,一定要分析思考事物的内部规律,靠“思维”取胜。

下面用企业的营销调研活动为例,让大家进一步理解上述提到的五
大能力:① 问题识别能力;② 信息搜索和提取能力;③ 决策能力;④
灵活应变能力;⑤ 短时间记忆能力。

营销中调研的步骤为:①确定问题和调研内容;②编辑调研计划;
③收集信息;④分析信息;⑤展示调研结果;⑥制定营销决策;⑦具体
执行。

解读: ①确定问题和调研内容

在这七个步骤中,第一步“确定问题和调研内容”和GRE阅读中
的“问题识别能力”相通,需要以客观的态度对存在的问题加以分辨,并
进行仔细思考。例如,工商银行开设的网点多,而平安银行开设的网点
少,那么网点多是优势还是缺点?乍一看似乎是优势,但优势都是建立
在成本之上的,平安银行用开网点的成本去补贴客户,这样“网点少”就
不是问题反而是一个优势了。在阅读考试中,需要判断真正要考查什
么。

演练1: The passage suggests that, if the criteria discussed in lines 16–
32 were the only criteriafor establishing a reservation's water rights, which of
the following would be true?

问题: 此题是否在问“16~32行”的内容?

解答: 如果将“16~32行”的内容视作答案方向来寻找选项的话
,会发现没有一项正确。事实上,此题问的并不是“16~32行”的内
容,因此要思考全文的脉络,看看前后文是否有与“16~32行”的标
准对立或是一致的情况发生。

演练2: The passage suggests which of the following about the


accuracy of the commuter diatom's biological clock?

问题: 此题是否在问“biological clock”?

解答: 此题问的是“生物钟的准确性”,并不是“生物钟”本
身。定位词如果确定错误,则会导致文章中定位行数的错误,从而也
就无法正确解答此题。

演练3: The author's explanation of how deep events occur would be


most weakened if which of the following were discovered to be true?

问题: 此题是否定位“deep events”?

解答: 此题问的不是“deep events”本身,而是“how deep e


vents occur”,要思考全文的框架,然后定位到“deep events”产
生原因的段落。因此可优先迅速浏览段落中含有“因果”逻辑关系词
汇的句子。

从心理学角度分析,导致“问题识别”环节出错的原因可以大致概括
为以下两点:

第一,选择性注意:考生会下意识地去掉自己主观上以为不重要的
信息。

第二,选择性扭曲:考生在解读信息时会有先入为主的倾向,对类
似出题方向的固有认知难以打破。

为了避免上述的问题,同学们要切记:每一道试题都是一道全新
的试题,出题方式非常可能和过去的题目不同,要把过去的固定套路
都忘记,从题目本身入手,来分析此题到底在考查什么。

解读: ②编辑调研计划;③收集信息

这两个步骤和GRE阅读中的“确定定位词+找到定位句”相关。哪个
词可以做定位词?如何找到定位句?定位句读到哪里结束?这些需要信
息处理能力。

在实际工作中,我们会面对大量纷杂的信息,但因为时间和经费有
限,我们不可能逐一处理,在众多信息源和大量信息内容中挑出能解决
眼前问题的资讯,就是信息处理能力的关键。

下面我们从必要性思维的角度来探讨信息处理能力。如果对获取的
所有信息都加以处理,理解清晰,然后再解决问题,这就叫充分性思
维,一般在实际工作中无法运用。用必要性思维解决问题可以这么理
解:解决一个问题前,先思考如果要解决这个问题,还有哪些问题是不
解决不行的,先把与必要条件相关的问题或前提性问题解决了。在实际
工作中,往往在解决前提性问题的过程中,最终的问题也会迎刃而解。
比如,企业CEO如果想把企业搞好,就必须要懂如何进行内部管理,了
解财务、法律、营销、产品创新等知识,以及如何和渠道商打交道,如
何开拓商业新模式,甚至还要懂得易经、道德经等,这样的思考就体现
了充分性思维。换必要性思维来分析下如何将企业经营好。企业经营得
好的前提是“企业得活着”,“企业活着”的前提是要有利润。所以按照必
要性思维分析,一个企业要把时间用于能带来利润的事情,管理、财务
等都是辅助。能带来利润的事情要么是减少成本,要么是提升营业额,
如此就可以得出“轻资产”等减少成本的策略。商业大师彼得·德鲁克说
过,“企业经营只有两个核心:创新和营销。”这两个方面正好和增加利
润有关。因此,要更高效地解决问题,可借助必要性思维。

在阅读考试中,定位句读到哪里结束有一个标准,读到可以解题之
处就无须再读了。

例1 But some observers questioned why, if IT had conferred economic


value, it did not produce direct competitive advantages for individual firms.
Resource-based theory offers an answer, asserting that, in general, firms gain
competitive advantages by accumulating resources that are economically
valuable, relatively scarce, and not easily replicated. According to a recent
study of retail firms, which confirmed that IT has become pervasive and
relatively easy to acquire, IT by itself appeared to have conferred little
advantage. In fact, though little evidence of any direct effect was found, the
frequent negative correlations between IT and performance suggested that IT
had probably weakened some firms' competitive positions. However, firms'
human resources, in and of themselves, did explain improved performance,
and some firms gained IT-related advantages by merging IT with
complementary resources, particularly human resources. The findings support
the notion, founded in resource-based theory, that competitive advantages do
not arise from easily replicated resources, no matter how impressive or
economically valuable they may be, but from complex, intangible resources.

The passage suggests that proponents of resource-based theory would be


likely to explain IT's inability to produce direct competitive advantages for
individual firms by pointing out that

A. IT is not a resource that is difficult to obtain.

B. IT is not an economically valuable resource.

C. IT is a complex, intangible resource.

D. economic progress has resulted from IT only in the service sector.


E. changes brought about by IT cannot be detected by macroeconomic
measures.

解析: 此题要找一个选项从“resourced-based theory”角度


解释为什么IT不能带来竞争优势,因此肯定要以“resource-based t
heory”为关键词查找。

原文第二句:“Resource-based theory offers an answer, as


serting that, in general, firms gain competitive advantages
by accumulating resources that are economically valuable, re
latively scarce, and not easily replicated.”请大家思考下,
读完这句能看选项了吗?答案是不行。因为这句话并没有回答“IT为
什么不能带来竞争优势”,这句话里面连IT都没谈。所以要继续读下
去,读到可以解释“inability”为止。

定位句后一句话“According to a recent study of retail fi


rms, which confirmed that IT has become pervasive and relati
vely easy to acquire, IT by itself appeared to have conferre
d little advantage.”,这句话中提到IT变得“pervasive and eas
y to acquire”,解释了IT不能带来竞争优势(conferred little a
dvantage)的理由。既然此句已经回答了问题,那么就可以不再读了
,按照“IT常见、易得”的答案方向去看选项。

只有A项和分析的答案方向一致,答案为A项。

注: “如何找到定位句”&“定位句读到哪里结束”均是信息处理能力
的应用。

例2 必要性思维在阅读中的实际运用

According to a theory advanced by researcher Paul Martin, the wave of


species extinctions that occurred in North America about 11,000 years ago, at
the end of the Pleistocene era, can be directly attributed to the arrival of
humans, i.e., the Paleoindians, who were ancestors of modern Native
Americans. However, anthropologist Shepard Krech points out that large
animal species vanished even in areas where there is no evidence to
demonstrate that Paleoindians hunted them. Nor were extinctions confined to
large animals: small animals, plants, and insects disappeared, presumably not
all through human consumption. Krech also contradicts Martin's exclusion of
climatic change as an explanation by asserting that widespread climatic
change did indeed occur at the end of the Pleistocene. Still, Krech attributes
secondary if not primary responsibility for the extinctions to the Paleoindians,
arguing that humans have produced local extinctions elsewhere. But,
according to historian Richard White, even the attribution of secondary
responsibility may not be supported by the evidence. White observes that
Martin's thesis depends on coinciding dates for the arrival of humans and the
decline of large animal species, and Krech, though aware that the dates are
controversial, does not challenge them; yet recent archaeological discoveries
are providing evidence that the date of human arrival was much earlier than
11,000 years ago.

Which of the following, if true, would most weaken Krech's objections


to Martin's theory?

A. Further studies showing that the climatic change that occurred at the
end of the Pleistocene era was even more severe and widespread than was
previously believed.

B. New discoveries indicating that Paleoindians made use of the small


animals, plants, and insects that became extinct.

C. Additional evidence indicating that widespread climatic change


occurred not only at the end of the Pleistocene era but also in previous and
subsequent eras.

D. Researchers' discoveries that many more species became extinct in


North America at the end of the Pleistocene era than was previously believed.

E. New discoveries establishing that both the arrival of humans in North


America and the wave of Pleistocene extinctions took place much earlier than
11,000 years ago.
解析: 题目要削弱“Krech's objection to Martin theory”
。不管答案如何削弱Krech,肯定要和Krech的说法是一个话题才行,
所以先找到哪些选项和Krech描述的话题相同。【和定位句描述的是
同一个话题的选项才能是答案,要先找到描述相同话题的选项——必
要性思维的应用】

Krech在文章中的反驳(objection)有三个角度,分别是“大动
物”“小动物和植物等”“气候”。(定位句:However, anthropol
ogist Shepard Krech points out that large animal species van
ished even in areas where there is no evidence to demonstrat
e that Paleoindians hunted them. Nor were extinctions confin
ed to large animals: small animals, plants, and insects disa
ppeared, presumably not all through human consumption. Krech
also contradicts Martin's exclusion of climatic change as an
explanation by asserting that widespread climatic change did
indeed occur at the end of the Pleistocene.)

选项中与Krech描述相同话题的有A、B、C项。D项描述“动物多
少”的话题,E项描述“到达时间”的话题,话题与Krech所说内容不
一致。题目问的是削弱(weaken),所以要找一个和文章内容相反的
选项。A和C项说的“气候发生变化”和原文的事件态度一致,并不能
起到反驳作用,所以只有B项可选

2. 如何理解“解决问题的能力”?

解读: ④分析信息;⑤展示调研结果;⑥制定营销决策

这三个步骤对应了GRE阅读中的“不带主观色彩,客观对待信息的
能力”“灵活应变解决问题能力”“选项选择的决策思维”。在商界,有这
么一句话:“正面思考反映事物的普遍规律,而逆向思考反映事物的本
质规律。”对于GRE考试而言这句话可以这样理解:只靠努力,按照固
有的解题顺序去解题,并不能产生高效的结果。要学会灵活应变,合理
进行逆向思考,学会利用想象力,这才是解决问题的关键。考生应把整
个GRE备考过程当作提升思维、学习解题能力的过程,这对未来有好
处。

如何在GRE阅读备考中“灵活应变”?

如果一个考生英语能力过关,并且有大量的时间来备考,那么他采
取任何备考方式都可以。但如果他只有1~2个月的时间,在托福90分的
基础上希望GRE能考到330分,那么采取一般的方法来复习肯定达不到
目标。针对这样的学生,此刻的问题就变成“在不太理解阅读,以及英
语基础不好导致阅读速度慢,读完再解题时间肯定不够的情况下,如何
搞定阅读”。

这里汇总一下之前提过的与灵活应变相关的重要思维:

第一,分析哪些信息不能解答此题,不用读。

第二,如果题目和作者相关,则除了定位句,还要思考作者态度。

第三,如果题目自带很多细节,则可直接找与该细节话题相同的选
项。

第四,不仅是原文内容可解题,选项的内容及逻辑排列顺序也会提
供解题线索。

第五,前面已经出的试题会给出解题线索。

第六,之后的题会给出解题线索。

第七,无法定位时,思考文章的主旨。

第八,文章中分论点需要关注,其他细节性信息可快速浏览。

第九,站在出题机构的角度来思考。

第十,所总结的解题套路都只是参考,考试时不要受限于固定的解
题步骤。

何为GRE阅读中的“决策能力”?
在生活和工作中,会面临各种选择。为什么有人判断力好
(decisive),而有人作决定时犹犹豫豫呢?好的决策思考体系对事业
发展大有裨益,在GRE阅读考试中要学会运用决策思维系统。

“决策思维系统”的四大标准:

第一,不追求何为“对”,但要确定何为“坏”,先排除一定不可取的
选项。

第二,只选择相对而言最好的选项,而不是追求最完美的选项。

第三,只要部分合理,该选项就先放着和其他选项进行对比。

第四,对比选项时,追求出错概率小的。

例1假设你是麦肯锡的一位商业咨询顾问,巴黎市政府找到麦肯锡
要针对增加卢浮宫的游客人数设计方案。你面前有五种方案:A. 开发
新兴市场(比如让东亚的游客访问卢浮宫);B. 开发曾经的法属殖民地
(比如让北非的旅客了解法国文化);C. 让品牌年轻化(比如邀请新一
代年轻人进入卢浮宫学习);D.功能升级(比如可以在卢浮宫里喝咖
啡、吃饭等);E. 把原有人群根据兴趣和意识形态进行细分(比如开发
圣经文化路线来吸引对应人群)。你如何给出对巴黎市政府的建议?

按照之前描述过的思维方式,来看这五个选项。前四个都是开发新
市场或是加入新功能元素,而第五个是在原客户中进行再开发。如果市
政府的预算和时间充裕,可研究前四个方案,因为开发新市场就意味着
再投入成本和时间等待效果出现。如果预算和时间紧张,则可采取第五
个方案,因为第五个方案是对原来的顾客群体进行细分,投入的资源成
本和时间最少。所以就出错概率小而言,可选择第五个方案。

在真实的场景中,巴黎市政府也的确选择了第五个方案,使得参观
人数在第二年增加了25%。

例2 The primary purpose of the passage is to

A. reevaluate a controversial theory.

B. identify the flaws in a study.


C. propose a new method of historical research.

D. compare two contrasting analyses.

E. provide a fresh perspective.

解析: 主旨题中,D项肯定不正确,因为含compare、contrast
的选项不会成为主旨题的答案。

A、C项很明显是说了一个新内容,而B项和E项比起来,E项描述
范围大,犯错的可能性更小,因此E项更好。

不管什么样的内容,都可以说成是“提出新的角度”,所以E项
是出错概率最小的选项,得出答案为E项。

例3 The passage suggests that the presence of G10 garnet in a kimberlite


pipe indicates that

A. the pipe in which the garnet is found has a 90% chance of containing
diamonds.

B. the pipe passed through a diamond-stability field and thus may


contain diamonds.

解析: A项是“有90%的概率包含钻石”,B项是“可能有钻石”
,显而易见,B项的出错概率小多了,答案为B项。

* 短时间记记力

在组织行为学领域中,有一个著名的“狄德罗的睡袍”的故事。有人
送了狄德罗一件睡袍,他在家里穿着这件睡袍踱来踱去。突然他意识到
他家的沙发、地毯等家具和睡袍的风格不符,于是换了整套家具。在换
了整套家具之后,他又发现他家太小,和家具不符,于是又换了更大的
别墅。从一件小小的睡袍到最后换了大别墅,说明人们的行为目的会因
为新的信息刺激而改变。

在做GRE阅读题的过程中,不能因为读了定位句而修改了题目所问
的内容,也不能因为读了选项而修改了定位句的内容,以下四点在做题
过程中需要牢记:

第一,做完所有试题前,文章的主旨或大致脉络不可忘记;

第二,做完一道试题前,题目精准所问的地方不可忘记;

第三,定位句所说的话题不可忘记;

第四,对于含有出题点的句子,在阅读文章时需多加留意。

3. 何为“出错概率小的选项”?

在职场上,职位越高说话越要慎重,不可以透露公司机密或重要数
据,因为竞争对手听到信息之后可能会采取对应措施。所以针对外界的
提问,职位越高的人越不可以说真话,但是也不能假话,那么他能说什
么?只能说空话。

所谓空话即不能认为其有错,说了和没有说差不多,或内容特别
虚,根本没有具体实质性内容的话语。比如,“针对这个问题,我们会
采取相对应的方案”“这个制度有其优点,也有其缺点”“我们对于这个研
究还不够充分,可以再进一步探索”“对于这个问题我们有自己的态
度”“对一些人而言,这个问题有难度”“这个问题他可能是对的”。这些
话并没有传递给听众任何有效信息。

类似的表达在GRE阅读题选项中出现,错误概率小,考生可优先选
择。切记当几个选项感觉都正确时,找一个出错概率小的选项。
第十章 短文章和长文章的实战练习&易错
题和难题汇总练习

【练习与解析】1

From 1910 to 1913, women suffragists in the United States organized


annual parades—activity traditionally conducted by men to proclaim
solidarity in some cause—not only as a public expression of suffragist
solidarity but also a conscious transgression of the rules of social order:
women's very presence in the streets challenged traditional notions of
femininity and restrictions on women's conduct. While recognizing the
parade's rhetorical force as a vehicle for social change, scholars have recently
begun to examine its drawbacks as a form of protest. Lumsden characterizes
the American suffrage parade as a “double-edged sword”, arguing that
women's efforts to proclaim their solidarity left them open to patronizing
commentary from press and public and to organized opposition from
antisuffragists.

1. It can be inferred from the passage that men's and women's parades
were similar in that both

A. were employed as rhetorical vehicles for social change.

B. were regarded as violating contemporary standards of public


decorum.

C. made participants vulnerable to organized opposition.

D. were largely ineffective as forms of protest.

E. were intended by their participants as public declarations of


solidarity.
解析: 此题问“男性游行”和“女性游行”的相似性。文章绝
大篇幅都在讲“女性”,所以用“men's parades”去定位更合理。
定位到文章第一句话“... conducted by men to proclaim solidar
ity in some cause ...”,于是得出答案为E项。B项针对“女性的
游行”,不适用于“男性”,故排除。其他三个选项完全和定位句的
话题无关。

做题技巧: 具备“定位能力+选项对比能力”,基本上就搞定了
阅读题。

2. The passage suggests which of the following about proponents of the


“rules of social order”?

A. They frowned upon public displays such as parades.

B. They had ulterior motives for objecting to women's participation in


suffrage parades.

C. They formed the core of the organized opposition to women suffrage.

D. They believed that it was unfeminine for women to march in suffrage


parades.

E. They supported women's rights to vote but disapprove some of the


methods that suffragists employed to gain that right.

解析: 很细节的问法,需要找关键词定位。通过“rules of so
cial order”定位到第一句话,需要读完第一句话的所有内容,直到
句号为止,因为这句话中的任何一处信息都可能被用来出题。

A项:学生可能会选A。但是A项不对,因为不是反对游行本身,
而是反对“女性参与游行”。

B项:大方向是“反驳女性”,比A项好些,可放着待比较。
C项:“core”无中生有,不太好。

D项:定位句的内容再现,和B项比起来表达更直接,意思和原文
更接近,目前最佳。

E项:出现新话题,不太好

所以答案为D项。

选项只要“部分沾边”或“方向一致”即可放着待
做题技巧:
比较,答案是比较后得出的结果。

【练习与解析】2

Although the passenger pigeons, now extinct, were abundant in


eighteenth- and nineteenth-century America, archaeological studies at
twelfth-century Cahokian sites in the present day United States examined
household food trash and found that traces of passenger pigeon were quite
rare. Given that the sites were close to a huge passenger pigeon roost
documented by John James Audubon in the nineteenth century and that
Cahokians consumed almost every other animal protein source available, the
archaeologists conducting the studies concluded the passenger pigeon
population had once been very limited before increasing dramatically in post-
Columbian America. Other archaeologists have criticized those conclusions
on the grounds that passenger pigeon bones would not be likely to be
preserved. But all the archaeological projects found plenty of bird bones and
even tiny bones from fish.

1. The author of the passage mentions “tiny bones from fish”primarily in


order to

A. explain why traces of passenger pigeon are rare at Cahokian sites.

B. support a claim about the wide variety of animal proteins in the


Cahokian diet.
C. provide evidence that confirms a theory about the extinction of the
passenger pigeon.

D. cast doubt on the conclusion reached by the archaeologists who


conducted the studies discussed in the passage.

E. counter an objection to an interpretation of the data obtained from


Cahokian sites.

解析: 此题定位到最后一句话“But all the... even tiny bo


nes from fish.”。“in order to”考题最核心的解题步骤就是思
考定位句和上下句之间的逻辑关系。定位句是反驳之前一句话的说法
,于是带着“反驳一群人的说法”的方向来看选项。

A、B、C项:均不是反驳的内容。

D和E项乍一看都像是反驳,于是要比较两项的差异。将反驳的共
性内容去掉之后,D项削弱的是“做研究的考古学家”的结论。原文
中提到“鱼骨头”是在最后一句话,其驳斥的对象是“其他考古学家
”(other archaeologists),所以D项错误。E项反驳的对象是一个
“解释”。于是返回原文看倒数第二句话,判断驳斥的内容。倒数第
二句话提到“其他考古学家”(Other archaeologists),这肯定和
“进行研究”(conducted the studies)的不是同一拨人,所以得
出答案为E项。

做题技巧: 先对备选项进行比较,判断差异,再回看原文。

2. Which of the following, if true, would most call into question the
reasoning of “the archaeologists conducting the studies”?

A. Audubon was unable to correctly identify twelfth-century Cahokian


sites.

B. Audubon made his observations before passenger pigeon populations


began to decline.
C. Passenger pigeons would have been attracted to household food trash.

D. Archaeologist have found passenger pigeon remains among food


waste at eighteenth-century human settlements.

E. Passenger pigeons tended not to roost at the same sites for very many
generations.

解析: 此题是阅读题中的逻辑题,可采用相差的技巧来解题。
首先,要找到原文中的推理链。

定位句的推理链是“鸽子可能被吃了——鸽子数量少”。出题机
构可能会从推理链的“前提”“结论”“推理过程”这三个角度中的
任何一个角度来出题。但根据现有的试题统计结果,做削弱题时对结
论进行“取非”,往往就能找到答案了。此题的E项表示“鸽子不会
在同一个地方停留很久”,表明鸽子数量并不少,只是鸽子会飞走而
已,削弱了结论,故答案为E项。

【练习与解析】3

Hotter and more massive than the Sun, stars called “stragglers”are
puzzling to astronomers because such rapidly burning stars would not be
expected to persist in ancient star clusters. Some researchers believe that the
typical blue stragglers formed when two ancient, lower-mass stars collide and
merge form more massive, hotter star. Peter Leonard theorizes alternatively
that in low density globular clusters, where mergers between single stars
occur too infrequently to account for the observed quantity of blue stragglers,
these stragglers are created instead by a group of stars. He suggests that a pair
of stars already orbiting each other presents a larger target for a third star or
another pair. Once this new grouping forms, close encounters between the
stars could prompt any two to merge as a blue straggler. Leonard's model
predicts that each blue straggler has a distant orbiting companion-as appears
true of many blue stragglers in the M67 cluster of the Milky Way galaxy.
1. The reference to a “larger target”serves primarily to suggest why a

A. blue straggler would be more likely to collide and merge with another
star than would be a lower-mass star.

B. pair of stars would be more likely to encounter other stars than would
the typical blue straggler.

C. pair of stars would be more likely to interact with other stars than
would a single star.

D. blue straggler would be more likely to interact with a pair of stars


than it would with a third star.

E. third star would be more likely to encounter a pair than it would to


encounter a blue straggler.

解析: 找到题干所问的句子后,发现定位句是在说“一个新的
蓝离散星(blue stragglers)的产生形式与之前的不同”。带着这
样的方向来看选项。

五个选项中,A、B、D、E项都犯了一个共同的错误。定位句的话
题是蓝离散星(blue stragglers)形成的过程,既然是过程,就无
法和“blue stragglers”本身进行对比,所以答案只有选C项。

2. Information presented in the passage suggests which of the following


about blue stragglers?

A. They originate from stars that are hotter and more massive than the
Sun.

B. They are burning more rapidly than other types of stars observed in
ancient star clusters.

C. They are older than most other types of stars within the same star
cluster.
D. They are less numerous in low-density globular clusters than are
pairs of stars.

E. They generally originate from the oldest stars among those found in
ancient star clusters.

解析: 全文都在讲蓝离散星(blue stragglers),没法定位。


所以在读原文的时候,对于可能有出题点的句子要多加留意,加深记
忆。此题在其他章节有做分析,答案为B项。

记忆力是阅读考试考查的能力之一。做阅读部分时,大致要记哪
些地方?

长文章的“脉络”——没法定位的题,先靠“脉络”来解答。

题目所问的点——避免读选项时把题目所问之处给改了。

定位句的话题——避免读选项时把定位句内容给改了。

文章出题点所在句子——边读文章应记忆,避免选项出现对应话
题时脑中没有印象。

3. The passage cites which of the following as evidence undermining the


theory presented in the second sentence?

A. a discrepancy between the number of mergers between single stars in


certain low-density globular clusters and that in other low-density globular
clusters

B. a discrepancy between the heat and mass of blue stragglers formed by


one type of process and the heat and mass of blue stragglers formed by
another type of process

C. a discrepancy between the frequency of star mergers in low-density


globular clusters and those in high-density globular clusters
D. a discrepancy between the amount of heat and mass of ancient single
stars and that of blue stragglers

E. a discrepancy between the number of mergers between single stars in


certain star clusters and the number of blue stragglers in those clusters

解析: 此题问的是“文章引用了哪个证据来削弱一个理论”,
既然是“引用”,则文章中肯定提及答案了。根据题目中的“underm
ining”可知,文章中哪句话有转折即为此题的答案方向。

文章中对于旧观点的反驳依据是“where mergers between sing


le stars occur too infrequentlyto account for the observed q
uantity of blue stragglers”,和此句形成最直接呼应的是E项,
得出答案为E项。

在思维类考试中要想取得高分,就不能靠回忆过去
做题技巧:
的思维模式做题。

【练习与解析】4

Some historians question the widely held belief that continually


improving education led to gradual African American empowerment in the
southern United States from the late nineteenth century through the mid-
twentieth century. They note that the development of Black educational
institutions in the segregated South was never rapid or steady: disparities
between Black and White schools sometimes grew in the early decades of the
twentieth century. And African Americans' educational gains did not bring
commensurate economic gains. Starting in the 1940s, even as Black and
White schools in the South moved steadily toward equality, Black
southerners remained politically marginalized and experienced systematic job
discrimination. Although Black schools had achieved near parity with White
schools in per capita spending and teachers' salaries by 1965, African
Americans' income still lagged behind that of Whites. Nonetheless,
educational progress did contribute toward economic and political
empowerment. African Americans' campaigns to support Black schools
fostered a sense of community, nurtured political determination, and often
increased literacy. More significantly, politically outspoken Black
newspapers achieved record circulation during the 1940s, just as the literacy
rate among African Americans approached 90 percent. Finally, the leadership
of the Civil Rights movement of the 1960s was composed largely of
graduates of Black colleges.

1. The author of the passage refers to the fact that “disparities between
Black and White schools sometimes grew in the early decades of the
twentieth century”most likely in order to

A. support the argument that the economic struggles of southern Blacks


were largely due to educational inequalities.

B. give an example of the differences between Black schools in the early


part of the twentieth century and Black schools starting in the 1940s.

C. illustrate the unevenness of the progress made by Black schools in the


southern United States.

D. help explain why Black remained politically marginalized and


experienced systematic job discrimination in the segregated South.

E. provide evidence that educational progress was a precondition for


economic progress in the segregated South.

解析: 在“Nonetheless”之前,文章都在描述“教育对黑人没
有好处”这个话题。此题所问之处就是在Nonetheless之前,因此不
管题目问的是“disparities”,还是“Black educational institu
tions”或是“Black southerners”,这些细节都在说明同一个话题
,寻找以“教育对黑人没好处”为话题方向的选项即可。

C项的“unevenness... by Black school”与要找的答案方向一


致,此项合理。
其他选项都和要找的答案话题无关。

2. Which of the following best describes the purpose of the highlighted


sentence?

A. It clarifies a point introduced in the preceding sentence.

B. It reiterates a point introduced in the first sentence of the passage.

C. It questions the accuracy of some of the evidence used to support the


argument of the historians.

D. It introduces a perspective that runs contrary to the view of those who


endorse the “belief.”

E. It qualifies the interpretation made by the historians.

解析: 黑体句的目的非常直接,就是削弱之前的一个说法,即
反驳了文章的第一句话。

A项:没有体现削弱的方向,不太好。

B项:变成顺接了,方向和要找的答案相反。

C项:质疑的对象是文章第一句话,即一个“说法”,并不是质
疑“证据”。

D项:文章第一句话就是“削弱一个观点(belief)”,要找的
答案应是驳斥第一句话的。此项说法相反。

E项:“qualify”在这里是“limit”的意思(GRE填空中有不少
题考查qualify的这层含义)。

答案为E项。
3. The passage suggests which of the following about the Civil Rights
movement?

A. It gave rise to a rapid increase in the number of Black newspapers


published in the South.

B. Its political effectiveness was greatly enhanced by the increased


circulation of Black newspapers.

C. Its leadership benefited from improvements in education for African


Americans.

D. It was the force primarily responsible for increasing per capita


spending in Black schools.

E. It was responsible for changing many historians' view of the relation


between education and African American empowerment.

解析: “Civil Rights movement”在“Nonetheless...”这句


话之后,话题是“教育对黑人有好处”。以此为话题方向来找选项。

A项:黑人报纸增加。

B项:政治效率提升。

C项:对黑人教育有好处——直接且明确地与话题相关。

D项:增加人均花费。

E项:改变历史学家的观点。

只有一个选项沾边,答案为C。

如遇到几个选项方向都对,则优先找一个自己不需
做题技巧:
要思考,也不改变原文内容的选项。
【练习与解析】5

“Blues is for singing,” writes folk musicologist Paul Oliver, and “is not
a form of folk song that stands up particularly well when written down.”
A poet who wants to write blues can attempt to avoid this problem by
poeticizing the form—but literary blues tend to read like bad poetry rather
than like refined folk song. For Oliver, the true spirit of the blues inevitably
eludes the self-conscious imitator. However, Langston Hughes, the first
writer to grapple with these difficulties of blue poetry, in fact succeeded in
producing poems that capture the quality of genuine, performed blues while
remaining effective as poems. In inventing blues poetry, Hughes solved two
problems: first, how to write blues lyrics in such a way that they work on the
printed page, and second, how to exploit the blues form poetically without
losing all sense of authenticity.

There are many styles of blues, but the distinction of importance to


Hughes is between the genres referred to as “folk blues”and “classic
blues.”Folk blues and classic blues are distinguished from one another by
differences in performers(local talents versus touring professionals),
patronage(local community versus mass audience), creation(improvised
versus composed), and transmission(oral versus written). It has been a
commonplace among critics that Hughes adopted the classic blues as the
primary model for his blues poetry, and that he writes his best blues poetry
when he tries least to imitate the folk blues. In this view, Hughes' attempts to
imitate the folk blues are too self-conscious, too determined to romanticize
the African American experience, too intent on reproducing what he takes to
be the quaint humor and naive simplicity of the folk blues to be successful.

But a more realistic view is that by conveying his perceptions as a folk


artist ought to—through an accumulation of details over the span of his blues
oeuvre, rather than by overloading each poem with quaintness and naivety-
Hughes made his most important contributions to the genre. His blues poems
are in fact closer stylistically to the folk blues on which he modeled them
than to the cultivated classic blues. Arnold Rampersad has observed that
virtually all of the poems in the 1927 collection in which Hughes essentially
originated blues poetry fall deliberatively within the “range of utterance”of
common folk. This surely applies to “Young Gal's Blues,”in which Hughes
avoids the conventionally “poetic”language and images that the subjects of
death and love sometimes elicit in his ordinary lyric poetry. To see what
Hughes' blues poetry might have been like if he had truly adopted the classic
blues as his model, one need only look to “Golden Brown Blues,”a song lyric
Hughes wrote for composer W.C. Handy. Its images, allusions, and diction
are conspicuously remote from the common “range of utterance.”

1. The primary purpose of the passage is to

A. describe the influence of folk and classic blues on blues poetry.

B. analyze the effect of African American culture on blues poetry.

C. demonstrate that the language used in Hughes' blues poetry is


colloquial.

D. defend Hughes' blues poetry against criticism that it is derivative.

E. refute an accepted view of Hughes' blues poetry style.

2. The author of the passage uses the highlighted quotation primarily to

A. indicate how blues poetry should be performed.

B. highlight the difficulties faced by writers of blues poetry.

C. support the idea that blues poetry is a genre doomed to fail.

D. illustrate the obstacles that blues poetry is unable to overcome.

E. suggest that written forms of blues are less authentic than sung blues.

3. It can be inferred from the passage that, as compared with the


language of “Golden Brown Blues,”the language of “Young Gal's Blues”is

A. more colloquial.

B. more melodious.
C. marked by more allusions.

D. characterized by more conventional imagery.

E. more typical of classic blues song lyrics.

4. According to the passage, Hughes' blues poetry and classic blues are
similar in which of the following ways?

A. Both are improvised.

B. Both are written down.

C. Both are intended for the same audience.

D. Neither uses colloquial language.

E. Neither is professionally performed.

下面对这篇文章逐段分析。

【第一段】

①“Blues is for singing,”writes folk musicologist Paul Oliver, and “is


not a form of folk song that stands up particularly well when written
down.”②A poet who wants to write blues can attempt to avoid this problem
by poeticizing the form—but literary blues tend to read like bad poetry rather
than like refined folk song. ③For Oliver, the true spirit of the blues
inevitably eludes the self-conscious imitator. ④However, Langston Hughes,
the first writer to grapple with these difficulties of blue poetry, in fact
succeeded in producing poems that capture the quality of genuine, performed
blues while remaining effective as poems. ⑤In inventing blues poetry,
Hughes solved two problems: first, how to write blues lyrics in such a way
that they work on the printed page, and second, how to exploit the blues form
poetically without losing all sense of authenticity.

分析: ①和③句话题相同——“‘blues'不能被写下来”。通
过④句开头的“However”,可先预测后文讲述“布鲁斯(blues)可
以被写下来”。(边预测边阅读)第④和⑤句果然写了“布鲁斯(bl
ues)被休斯(Hughes)这个人写下来了”。

第一段读完之后,发现就说了两件事情:“之前布鲁斯不能被写
下来”+“休斯把布鲁斯写下来了”。

阅读技巧: (1)每读完一段,思考下这段一共说了多少不同的
话题。(2)在实际考试中,将每段内容归纳得越简洁越好。实际
考试中不可能读得很透彻。

【第二段】

There are many styles of blues, but the distinction of importance to


Hughes is between the genres referred to as “folk blues”and “classic
blues.”Folk blues and classic blues are distinguished from one another by
differences in performers(local talents versus touring professionals),
patronage(local community versus mass audience), creation(improvised
versus composed), and transmission(oral versus written). It has been a
commonplace among critics that Hughes adopted the classic blues as the
primary model for his blues poetry, and that he writes his best blues poetry
when he tries least to imitate the folk blues. In this view, Hughes' attempts to
imitate the folk blues are too self-conscious, too determined to romanticize
the African American experience, too intent on reproducing what he takes to
be the quaint humor and naive simplicity of the folk blues to be successful.

分析: 假设在考场中时间紧或有些句子读不懂,导致细节掌握
得不多,只读出了大致的话题。

第二段前两句介绍了民谣布鲁斯(folk blues)和古典布鲁斯(
classic blues)的区别,之后讲“休斯(Hughes)的作品属于民谣
布鲁斯(folk blues)”。
即使是跳读的地方,也要清楚大致的话题方向,避
阅读技巧:
免试题考到相关话题不知定位到何处。

【第三段】

But a more realistic view is that by conveying his perceptions as a folk


artist ought to—through an accumulation of details over the span of his blues
oeuvre, rather than by overloading each poem with quaintness and naivety—
Hughes made his most important contributions to the genre. His blues poems
are in fact closer stylistically to the folk blues on which he modeled them
than to the cultivated classic blues. Arnold Rampersad has observed that
virtually all of the poems in the 1927 collection in which Hughes essentially
originated blues poetry fall deliberatively within the “range of utterance”of
common folk. This surely applies to “Young Gal's Blues,”in which Hughes
avoids the conventionally “poetic”language and images that the subjects of
death and love sometimes elicit in his ordinary lyric poetry. To see what
Hughes' blues poetry might have been like if he had truly adopted the classic
blues as his model, one need only look to “Golden Brown Blues,”a song lyric
Hughes wrote for composer W.C. Handy. Its images, allusions, and diction
are conspicuously remote from the common “range of utterance.”

分析: 文段一开始出现转折,之后内容表示休斯(Hughes)的
作品更接近民谣布鲁斯(folk blues),之后又描述了一些作品名字
。在Arnold Rampersad之后没有转折词,而这本身就是一个示例,所
以可以断定最后一段的脉络为“休斯的作品属于民谣布鲁斯(folk b
lues)”。

在实际考场中,尽量读出细节,如没法做到,则需
阅读技巧:
要把此段大致框架或脉络读出。

下面逐题解析。

1. The primary purpose of the passage is to

A. describe the influence of folk and classic blues on blues poetry.


B. analyze the effect of African American culture on blues poetry.

C. demonstrate that the language used in Hughes' blues poetry is


colloquial.

D. defend Hughes' blues poetry against criticism that it is derivative.

E. refute an accepted view of Hughes' blues poetry style.

解析: 如文章中读到观点“转折”,则要在主旨题答案中体现
出这个“转折点”。

A项:乍一看,没有转折,再看看有没有更好的选项。

B项:文章不是在说“African American culture”,而是在描


述一个人的作品,此项不太好。

C项:“语言是口语化的”,此项和要找的答案方向差得太远,
直接排除。

D项:“作品是不是原创”,此项话题和要找的答案话题差太远
,不选。

E项:“反驳一个被接受的说法”,此项为唯一符合答案方向的
选项。

故正确答案为E项。

2. The author of the passage uses the highlighted quotation primarily to

A. indicate how blues poetry should be performed.

B. highlight the difficulties faced by writers of blues poetry.

C. support the idea that blues poetry is a genre doomed to fail.


D. illustrate the obstacles that blues poetry is unable to overcome.

E. suggest that written forms of blues are less authentic than sung blues.

解析: 根据quotation一词定位到第一段第一句话(“Blues is
for singing,”writes folk musicologist Paul Oliver, and “is
not a form of folk song that stands up particularly well whe
n written down.”)。此句可读成“布鲁斯写下来不行”。带着这
话题来看选项。

A项读成“布鲁斯应该如何被表演”。

B项读成“强调了一个困难”。

在A和B选项对比时,显然B项比A选项符合原句内容,B项优于A项

C项读成“支持了一个观点——布鲁斯诗歌注定会失败”。

D项读成“描述一个不可克服的障碍”。

C项和D项中的“注定失败”与“不可克服的障碍”均与原文后句
的“一个人克服了这样困难,成功了写了布鲁斯诗歌的内容”冲突。
所以此两项均不可选。

E项读成“写下来的布鲁斯不正宗”。

和B项比较起来,显然B项和原定位句是一个话题。

故答案为B项。

3. It can be inferred from the passage that, as compared with the


language of “Golden Brown Blues,”the language of “Young Gal's Blues”is

A. more colloquial.
B. more melodious.

C. marked by more allusions.

D. characterized by more conventional imagery.

E. more typical of classic blues song lyrics.

解析: 此题问“Young Gal's Blues”的语言形式如何,因为读


原文的时候基本跳过了最后一段的细节,所以做此题时应结合选项寻
找答案方向。此题的五个选项都是在描述具体特征。于是,先定位到
Young Gal's Blues所在的地方,看看有没有类似选项特征的描述。

定位到第三段第四句:This surely applies to “Young Gal's


Blues,”in which Hughes avoids the conventionally “poetic”
language and images that the subjects of death and love some
times elicit in his ordinary lyric poetry. 此句和上下句都和
选项描述的“特征”无关,如何是好?牢记阅读解题思维:“当没法
定位的时候,思考文章脉络。”“YGB”在文中最后一段靠后,此处
的话题是休斯的作品属于民谣布鲁斯(folk blues)。所以民谣布鲁
斯有什么特征,YGB就会有什么特征。在第二段的括号中读到了“fol
k blues”(民谣布鲁斯)与“classic blues”(古典布鲁斯)的特
征区别—(local talents versus touring professionals,local
community versus mass audience, improvised versus composed,
and oral versus written),因此folk blues(民谣布鲁斯)中的o
ral(口头上)的特征可与选项A 对应,则此题答案为A。

4. According to the passage, Hughes' blues poetry and classic blues are
similar in which of the following ways?

A. Both are improvised.

B. Both are written down.

C. Both are intended for the same audience.


D. Neither uses colloquial language.

E. Neither is professionally performed.

解析: 题目问两个对象在哪方面是类似的,我们只需要找到一
个相对熟悉的对象,然后通过它的属性先筛选选项。classic blues
(古典布鲁斯)的特征在第2段的括号中有描述(professionals,ma
ss audience, composed,written),所以先带着此描述来看选项。

A项读成:“即兴创作”。此项和“composed(创作好的)”的
特征冲突,不可选。

B项读成:“写下来”。没错,可放着待比较。

C项读成:“一样的观众”。不可排除,先放着待比较。

D项读成:“都没有使用口头语言”。不可排除,先放着。

E项读成:“都不是职业的”。此项与原文“professionals”(
职业的)的信息冲突,排除此项。

剩下B、C、D三项如何对比?再思考Hughes的相关内容。在文章
的第一段即写到Hughes的贡献是把布鲁斯写了下来。

所以答案为B。

【练习与解析】6

In February 1848 the people of Paris rose in revolt against the


constitutional monarchy of Louis-Philippe. Despite the existence of excellent
narrative accounts, the February Days, as this revolt is called, have been
largely ignored by social historians of the past two decades. For each of the
three other major insurrections in nineteenth-century Paris—July 1830, June
1848, and May 1871—there exists at least a sketch of participants'
backgrounds and an analysis, more or less rigorous, of the reasons for the
occurrence of the uprisings. Only in the case of the February Revolution do
we lack a useful description of participants that might characterize it in the
light of what social history has taught us about the process of revolutionary
mobilization.

Two reasons for this relative neglect seem obvious. First, the
insurrection of February has been overshadowed by that of June. The
February Revolution overthrew a regime, to be sure, but met with so little
resistance that it failed to generate any real sense of historical drama. Its
successor, on the other hand, appeared to pit key socioeconomic groups in a
life-or-death struggle and was widely seen by contemporary observers as
marking a historical departure. Through their interpretations, which exert a
continuing influence on our understanding of the revolutionary process, the
impact of the events of June has been magnified, while, as an unintended
consequence, the significance of the February insurrection has been
diminished. Second, like other “successful”insurrections, the events of
February failed to generate the most desirable kinds of historical records.
Although the June insurrection of 1848 and the Paris Commune of 1871
would be considered watersheds of nineteenth-century French history by any
standard, they also present the social historian with a signal advantage: these
failed insurrections created a mass of invaluable documentation as a by-
product of authorities' efforts to search out and punish the rebels.

Quite different is the outcome of successful insurrections like those of


July 1830 and February 1848. Experiences are retold, but participants
typically resume their daily routines without ever recording their activities.
Those who played salient roles may become the objects of highly
embellished verbal accounts or in rare cases, of celebratory articles in
contemporary periodicals. And it is true that the publicly acknowledged
leaders of an uprising frequently write memoirs. However, such documents
are likely to be highly unreliable, unrepresentative, and unsystematically
preserved, especially when compared to the detailed judicial dossiers
prepared for everyone arrested following a failed insurrection.

As a consequence, it may prove difficult or impossible to establish for a


successful revolution a comprehensive and trustworthy picture of those who
participated, or to answer even the most basic questions one might pose
concerning the social origins of the insurgents.

1. With which of the following statements regarding revolution would


the author most likely agree?

A. Revolutionary mobilization requires a great deal of planning by


people representing disaffected groups.

B. The objectives of the February Revolution were more radical than


those of the June insurrection.

C. The process of revolutionary mobilization varies greatly from one


revolution to the next.

D. Revolutions vary greatly in the usefulness of the historical records


that they produce.

E. As knowledge of the February Revolution increases, chances are


good that its importance will eventually eclipse that of the June insurrection.

2. Which of the following is the most logical objection to the claim


made in the last paragraph?

A. The February Revolution of 1848 is much less significant than the


July insurrection of 1830.

B. The backgrounds and motivations of participants in the July


insurrection of 1830 have been identified, however cursorily.

C. Even less is known about the July insurrection of 1830 than about the
February Revolution of 1848.

D. Historical records made during the July insurrection of 1830 are less
reliable than those made during the May insurrection of 1871.

E. The importance of the July insurrection of 1830 has been magnified


at the expense of the significance of the February Revolution of 1848.
3. The purpose of the second paragraph is to explain why

A. the people of Paris revolted in February 1848 against the rule of


Louis-Philippe.

B. there exist excellent narrative accounts of the February Days.

C. the February Revolution met with little resistance.

D. a useful description of the participants in the February Revolution is


lacking.

E. the February Revolution failed to generate any real sense of historical


drama.

4. It can be inferred from the passage that the author considers which of
the following essential for understanding a revolutionary mobilization?

A. A comprehensive theory of revolution that can be applied to the


major insurrections of the nineteenth century.

B. Awareness of the events necessary for a revolution to be successful.

C. Access to narratives and memoirs written by eyewitnesses of a given


revolution.

D. The historical perspective provided by the passage of a considerable


amount of time.

E. Knowledge of the socioeconomic backgrounds of a revolution's


participants.

下面对这篇文章分段讲解。

【第一段】

In February 1848 the people of Paris rose in revolt against the


constitutional monarchy of Louis-Philippe. Despite the existence of excellent
narrative accounts, the February Days, as this revolt is called, have been
largely ignored by social historians of the past two decades. For each of the
three other major insurrections in nineteenth-century Paris—July 1830, June
1848, and May 1871—there exists at least a sketch of participants'
backgrounds and an analysis, more or less rigorous, of the reasons for the
occurrence of the uprisings. Only in the case of the February Revolution do
we lack a useful description of participants that might characterize it in the
light of what social history has taught us about the process of revolutionary
mobilization.

分析: 首段描述的事件:

事件1:二月革命被忽略。

事件2:缺乏二月革命参与者的信息,而借助参与者的信息我们
可以描述革命过程。

不要因为有不懂的词而产生焦虑。我们只要能猜出
阅读技巧:
这几行大致在讲什么话题即可。

【第二段】

Two reasons for this relative neglect seem obvious. First, the
insurrection of February has been overshadowed by that of June. The
February Revolution overthrew a regime, to be sure, but met with so little
resistance that it failed to generate any real sense of historical drama. Its
successor, on the other hand, appeared to pit key socioeconomic groups in a
life-or-death struggle and was widely seen by contemporary observers as
marking a historical departure. Through their interpretations, which exert a
continuing influence on our understanding of the revolutionary process, the
impact of the events of June has been magnified, while, as an unintended
consequence, the significance of the February insurrection has been
diminished. Second, like other “successful”insurrections, the events of
February failed to generate the most desirable kinds of historical records.
Although the June insurrection of 1848 and the Paris Commune of 1871
would be considered watersheds of nineteenth-century French history by any
standard, they also present the social historian with a signal advantage: these
failed insurrections created a mass of invaluable documentation as a by-
product of authorities' efforts to search out and punish the rebels.

分析: 第二段的话题:二月革命被忽略的原因。

看到“First”,接下来一定是原因的具体展开,然后扫看到第
二段末尾,发现是很具体的内容,说明是细节,则整个第二段都是在
陈述“两个理由”。如时间够,可仔细阅读,如时间不够,从First
开始可全部跳过。因为第二段所谈论的话题已经十分清晰,如题目考
到具体原因,再返回第二段细读即可。

阅读时首先要找出文章框架,具体细节待题目涉及
阅读技巧:
的时候再通过定位词返回原文。这样可节省时间和脑力。

【第三、四段】

Quite different is the outcome of successful insurrections like those of


July 1830 and February 1848. Experiences are retold, but participants
typically resume their daily routines without ever recording their activities.
Those who played salient roles may become the objects of highly
embellished verbal accounts or in rare cases, of celebratory articles in
contemporary periodicals. And it is true that the publicly acknowledged
leaders of an uprising frequently write memoirs. However, such documents
are likely to be highly unreliable, unrepresentative, and unsystematically
preserved, especially when compared to the detailed judicial dossiers
prepared for everyone arrested following a failed insurrection.

As a consequence, it may prove difficult or impossible to establish for a


successful revolution a comprehensive and trustworthy picture of those who
participated, or to answer even the most basic questions one might pose
concerning the social origins of the insurgents.
解析: (假设一位英语水平不高的学生读文章)第三段的话题

事件1:两个起义结果不同。

事件2:和“参与者”有关的事情,具体没太读懂。

第四段的话题:

事件:没有参与者信息,就没法得到一个完整的关于起义的群像

下面逐个解析试题。

1. With which of the following statements regarding revolution would


the author most likely agree?

A. Revolutionary mobilization requires a great deal of planning by


people representing disaffected groups.

B. The objectives of the February Revolution were more radical than


those of the June insurrection.

C. The process of revolutionary mobilization varies greatly from one


revolution to the next.

D. Revolutions vary greatly in the usefulness of the historical records


that they produce.

E. As knowledge of the February Revolution increases, chances are


good that its importance will eventually eclipse that of the June insurrection.

解析: 题目问作者对于“revolution”(革命)的观点,则此
题可结合文章的脉络一起思考答案。
A项读成“革命要求计划”。

B项读成“目标更激进”。

C项读成“革命过去会变化”。

D项读成“革命变化与历史记录的有用性”之间的关系。

E项读成“如果二月革命相关的知识多了,则二月革命的重要性
会遮住六月起义”。

文章说了很多次关于“参与者信息”与“革命”相关的事件,所
以D项更好。

2. Which of the following is the most logical objection to the claim


made in the last paragraph?

A. The February Revolution of 1848 is much less significant than the


July insurrection of 1830.

B. The backgrounds and motivations of participants in the July


insurrection of 1830 have been identified, however cursorily.

C. Even less is known about the July insurrection of 1830 than about the
February Revolution of 1848.

D. Historical records made during the July insurrection of 1830 are less
reliable than those made during the May insurrection of 1871.

E. The importance of the July insurrection of 1830 has been magnified


at the expense of the significance of the February Revolution of 1848.

解析: 问题问最后一段的“objection”(反驳),相当于削弱
最后一段话中的信息。最后一段的话题有“并不知道参与者信息”,
把这个事件取反即得到“知道参与者信息”,所以B选项即为答案。
3. The purpose of the second paragraph is to explain why

A. the people of Paris revolted in February 1848 against the rule of


Louis-Philippe.

B. there exist excellent narrative accounts of the February Days.

C. the February Revolution met with little resistance.

D. a useful description of the participants in the February Revolution is


lacking.

E. the February Revolution failed to generate any real sense of historical


drama.

解析: 第二段一开始即说到了二月革命被忽略的原因。带着这
个事件找选项,即得到D项。

4. It can be inferred from the passage that the author considers which of
the following essential for understanding a revolutionary mobilization?

A. A comprehensive theory of revolution that can be applied to the


major insurrections of the nineteenth century.

B. Awareness of the events necessary for a revolution to be successful.

C. Access to narratives and memoirs written by eyewitnesses of a given


revolution.

D. The historical perspective provided by the passage of a considerable


amount of time.

E. Knowledge of the socioeconomic backgrounds of a revolution's


participants.
解析: 此题又是考作者的观点,所以可优先思考文章的脉络,
哪个选项与文章脉络中的事件沾边,这个选项可重点分析。文章中反
复强调了“参与者信息”与“起义”的关系,所以此题的E项更好。

【练习与解析】7

Although recent years have seen substantial reductions in noxious


pollutants from individual motor vehicles, the number of such vehicles has
been steadily increasing, consequently, more than 100 cities in the United
States still have levels of carbon monoxide, particulate matter, and
ozone(generated by photochemical reactions with hydrocarbons from
vehicle exhaust)that exceed legally established limits. There is a growing
realization that the only effective way to achieve further reductions in vehicle
emissions—short of a massive shift away from the private automobile—is to
replace conventional diesel fuel and gasoline with cleaner-burning fuels such
as compressed natural gas, liquefied petroleum gas, ethanol, or methanol.

All of these alternatives are carbon-based fuels whose molecules are


smaller and simpler than those of gasoline. These molecules burn more
cleanly than gasoline, in part because they have fewer, if any, carbon-carbon
bonds, and the hydrocarbons they do emit are less likely to generate ozone.
The combustion of larger molecules, which have multiple carbon-carbon
bonds, involves a more complex series of reactions. These reactions increase
the probability of incomplete combustion and are more likely to release
uncombusted and photochemically active hydrocarbon compounds into the
atmosphere. On the other hand, alternative fuels do have drawbacks.
Compressed natural gas would require that vehicles have a set of heavy fuel
tanks—a serious liability in terms of performance and fuel efficiency—and
liquefied petroleum gas faces fundamental limits on supply.

Ethanol and methanol, on the other hand, have important advantages


over other carbon-based alternative fuels: they have a higher energy content
per volume and would require minimal changes in the existing network for
distributing motor fuel. Ethanol is commonly used as a gasoline supplement,
but it is currently about twice as expensive as methanol, the low cost of
which is one of its attractive features. Methanol's most attractive feature,
however, is that it can reduce by about 90 percent the vehicle emissions that
form ozone, the most serious urban air pollutant.

Like any alternative fuel, methanol has its critics. Yet much of the
criticism is based on the use of “gasoline clone”vehicles that do not
incorporate even the simplest design improvements that are made possible
with the use of methanol. It is true, for example, that a given volume of
methanol provides only about one-half of the energy that gasoline and diesel
fuel do; other things being equal, the fuel tank would have to be somewhat
larger and heavier. However, since methanol-fueled vehicles could be
designed to be much more efficient than “gasoline clone”vehicles fueled with
methanol, they would need comparatively less fuel. Vehicles incorporating
only the simplest of the engine improvements that methanol makes feasible
would still contribute to an immediate lessening of urban air pollution.

1. According to the passage, incomplete combustion is more likely to


occur with gasoline than with an alternative fuel because

A. the combustion of gasoline releases photochemically active


hydrocarbons.

B. the combustion of gasoline involves an intricate series of reactions.

C. gasoline molecules have a simple molecular structure.

D. gasoline is composed of small molecules.

E. gasoline is a carbon-based fuel.

2. Which of the following most closely parallels the situation described


in the first sentence of the passage?

A. Although a town reduces its public services in order to avoid a tax


increase, the town's tax rate exceeds that of other towns in the surrounding
area.
B. Although a state passes strict laws to limit the type of toxic material
that can be disposed of in public landfills, illegal dumping continues to
increase.

C. Although a town's citizens reduce their individual use of water, the


town's water supplies continue to dwindle because of a steady increase in the
total population of the town.

D. Although a country attempts to increase the sale of domestic goods


by adding a tax to the price of imported goods, the sale of imported goods
within the country continues to increase.

E. Although a country reduces the speed limit on its national highways,


the number of fatalities caused by automobile accidents continues to increase.

3. It can be inferred from the passage that a vehicle specifically designed


to use methanol for fuel would

A. be somewhat lighter in total body weight than a conventional vehicle


fueled with gasoline.

B. be more expensive to operate than a conventional vehicle fueled with


gasoline.

C. have a larger and more powerful engine than a conventional vehicle


fueled with gasoline.

D. have a larger and heavier fuel tank than a “gasoline clone”vehicle


fueled with methanol.

E. average more miles per gallon than a “gasoline clone”vehicle fueled


with methanol.

4. The passage suggests which of the following about air pollution?

A. Further attempts to reduce emissions from gasoline-fueled vehicles


will not help lower urban air-pollution levels.

B. Attempts to reduce the pollutants that an individual gasoline-fueled


vehicle emits have been largely unsuccessful.

C. Few serious attempts have been made to reduce the amount of


pollutants emitted by gasoline-fueled vehicles.

D. Pollutants emitted by gasoline-fueled vehicles are not the most


critical source of urban air pollution.

E. Reductions in pollutants emitted by individual vehicles have been


offset by increases in pollution from sources other than gasoline-fueled
vehicles.

下面对这篇文章进行逐段分析。

【第一段】

Although recent years have seen substantial reductions in noxious


pollutants from individual motor vehicles, the number of such vehicles has
been steadily increasing, consequently, more than 100 cities in the United
States still have levels of carbon monoxide, particulate matter, and
ozone(generated by photochemical reactions with hydrocarbons from
vehicle exhaust)that exceed legally established limits. There is a growing
realization that the only effective way to achieve further reductions in vehicle
emissions—short of a massive shift away from the private automobile—is to
replace conventional diesel fuel and gasoline with cleaner-burning fuels such
as compressed natural gas, liquefied petroleum gas, ethanol, or methanol.

分析: 第一句就是让步句,让步句是出题点,须记住第一句的
话题——单车排放的有害污染物大幅减少,但车辆总量导致污染还是
大。第二句中有一个only,表示此句也可能出题。第二句是在说唯一
有效的减排方式是用清洁燃料替代传统燃料。第一段读完,记住上述
两件事情。

【第二段】
All of these alternatives are carbon-based fuels whose molecules are
smaller and simpler than those of gasoline. These molecules burn more
cleanly than gasoline, in part because they have fewer, if any, carbon-carbon
bonds, and the hydrocarbons they do emit are less likely to generate ozone.
The combustion of larger molecules, which have multiple carbon-carbon
bonds, involves a more complex series of reactions. These reactions increase
the probability of incomplete combustion and are more likely to release
uncombusted and photochemically active hydrocarbon compounds into the
atmosphere. On the other hand, alternative fuels do have drawbacks.
Compressed natural gas would require that vehicles have a set of heavy fuel
tanks—a serious liability in terms of performance and fuel efficiency—and
liquefied petroleum gas faces fundamental limits on supply.

分析: (模拟英语水平不太好的学生读第二段)第二段乍一看
是在描述污染原理,之后好像在描述“使用替代(alternative)燃
料的车辆”也有缺点。考虑到可能读不懂细节,所以不用看了,记住
这些话题更重要。

【第三段】

Ethanol and methanol, on the other hand, have important advantages


over other carbon-based alternative fuels: they have a higher energy content
per volume and would require minimal changes in the existing network for
distributing motor fuel. Ethanol is commonly used as a gasoline supplement,
but it is currently about twice as expensive as methanol, the low cost of
which is one of its attractive features. Methanol's most attractive feature,
however, is that it can reduce by about 90 percent the vehicle emissions that
form ozone, the most serious urban air pollutant.

分析: (模拟英语水平不高的读第三段)第三段说的是燃料E和
M有各自的优势。然后大量细节信息估计在描述具体优势。
阅读技巧: 记住文章话题比理解细节重要。

【第四段】

Like any alternative fuel, methanol has its critics. Yet much of the
criticism is based on the use of “gasoline clone”vehicles that do not
incorporate even the simplest design improvements that are made possible
with the use of methanol. It is true, for example, that a given volume of
methanol provides only about one-half of the energy that gasoline and diesel
fuel do; other things being equal, the fuel tank would have to be somewhat
larger and heavier. However, since methanol-fueled vehicles could be
designed to be much more efficient than “gasoline clone”vehicles fueled with
methanol, they would need comparatively less fuel. Vehicles incorporating
only the simplest of the engine improvements that methanol makes feasible
would still contribute to an immediate lessening of urban air pollution.

分析: (模拟英语水平不太好的学生读第四段)第四段讲燃料M
也有缺点并举例。最后提到汽车改进引擎之后能减少污染。

读完全文之后,总结下文章的框架(脉络):开篇指出治理车辆
污染唯一有效的方式是推广使用清洁燃料的车辆,后几段具体介绍污
染是如何产生的,以及其他燃料的优势和弊端。

假设一个学生英语能力很有限,文章只能读成这种程度,来看下
文如何解题。

下面逐个解答试题。

1. According to the passage, incomplete combustion is more likely to


occur with gasoline than with an alternative fuel because

A. the combustion of gasoline releases photochemically active


hydrocarbons.

B. the combustion of gasoline involves an intricate series of reactions.


C. gasoline molecules have a simple molecular structure.

D. gasoline is composed of small molecules.

E. gasoline is a carbon-based fuel.

解析: 用“incomplete combustion”和“gasoline”来定位,


找到第二段的定位句“These reactions increase the probability
of incomplete combustion and are more likely to release unc
ombusted and photochemically active hydrocarbon compounds in
to the atmosphere.”。这个定位句提示我们是“这些反应(These
reactions)导致的不充分燃烧”。所以要往上再读一句,找到“反
应”(reactions)的具体内容。之前一句的内容是“The combustio
n of larger molecules, which have multiple carbon-carbon bon
ds, involves a more complex series of reactions.”。哪个选项
和这个定位句“部分相应”或“方向一致”,即可作为备选项。

B项是唯一和前句沾边的选项,答案为B项。

2. Which of the following most closely parallels the situation described


in the first sentence of the passage?

A. Although a town reduces its public services in order to avoid a tax


increase, the town's tax rate exceeds that of other towns in the surrounding
area.

B. Although a state passes strict laws to limit the type of toxic material
that can be disposed of in public landfills, illegal dumping continues to
increase.

C. Although a town's citizens reduce their individual use of water, the


town's water supplies continue to dwindle because of a steady increase in the
total population of the town.

D. Although a country attempts to increase the sale of domestic goods


by adding a tax to the price of imported goods, the sale of imported goods
within the country continues to increase.

E. Although a country reduces the speed limit on its national highways,


the number of fatalities caused by automobile accidents continues to increase.

解析: 文章第一句话描述的是“个体和总量的关系”,纵观五
个选项,只有C项直接描述了“individual”和“total population
”,最直观地反映出第一句话的话题。其他选项和C项比起来较容易
排除。

3. It can be inferred from the passage that a vehicle specifically designed


to use methanol for fuel would

A. be somewhat lighter in total body weight than a conventional vehicle


fueled with gasoline.

B. be more expensive to operate than a conventional vehicle fueled with


gasoline.

C. have a larger and more powerful engine than a conventional vehicle


fueled with gasoline.

D. have a larger and heavier fuel tank than a “gasoline clone”vehicle


fueled with methanol.

E. average more miles per gallon than a “gasoline clone”vehicle fueled


with methanol.

解析: 此题问如果用“vehicle”来当定位词,够吗?答案是不
够。定位的时候必须要用两个及以上的词汇来当作定位词。此题可用
“vehicle, design, methanol”这些词定位,定位到最后一段中的
“However, since methanol-fueled vehicles could be designed
to be much more efficient than ‘gasoline clone’ vehicles f
ueled with methanol, they would need comparatively less fuel
.”。如果仅用“vehicle”来定位,则可能会定位到前面几行,从而
做错此题。

以此题定位句中提到的“更有效率”为话题方向寻找答案。

A项:“更轻”。

B项:“更贵”。

C项:“更大更有力的引擎”。

D项:“更大更重的油箱”。

E项:“行驶更多的里程”。

哪个选项是不用思考就可判断出与“更有效率”这个话题相关的
?更轻和更大更有力的引擎不一定能使效率更高,但是“更多的行驶
里程”直接反映了“效率”。故答案为E项。

4. The passage suggests which of the following about air pollution?

A. Further attempts to reduce emissions from gasoline-fueled vehicles


will not help lower urban air-pollution levels.

B. Attempts to reduce the pollutants that an individual gasoline-fueled


vehicle emits have been largely unsuccessful.

C. Few serious attempts have been made to reduce the amount of


pollutants emitted by gasoline-fueled vehicles.

D. Pollutants emitted by gasoline-fueled vehicles are not the most


critical source of urban air pollution.

E. Reductions in pollutants emitted by individual vehicles have been


offset by increases in pollution from sources other than gasoline-fueled
vehicles.
解析: 此题首先要思考文中哪里出现了“空气污染”(air pol
lution)。回顾文章,除了第一段之外,其他几段都在描述不同的燃
料各自有哪些好处和坏处,因此有关“空气污染”的信息要在第一段
中找。首段一共只说了两个话题,分别是“个体行,总体不行”以及
“只有使用替代燃料(alternative fuel)的车行,传统车不行”。
带着这两个话题来看选项。

A项:“继续减少传统车排放对减少空气污染无效”。第一段的
“only”提示我们,只有替代燃料(alternative fuel)这一招才有
效,其他方法都无效。所以此项可作为答案的备选。

B项:“减少传统车单车的污染排放,失败了”。根据首段第一
个话题“个体行,而总体不行”可知,此项所述内容和原文相反,排
除。

C项:“关于减少污染的尝试很少”。此项没法判断,可能有过
尝试,但是尝试后的效果不佳,所以只介绍了唯一有效的方式是采用
替代燃料(alternative fuel),故排除此项。

D项:“传统汽油车造成的污染不是最核心的污染来源”。第一
段里面并没有谈论此话题,排除。

E项:“单车污染物的减少已经被其他方面污染物的增加所抵消
”。第一段只说了“个体和总量”的关系,并没有说“个体和其他”
的关系,排除此项。

答案为A项。

扫码看余翔亲授视频
GRE阅读特训

练习1

The importance of the Bill of Rights in twentieth-century United States


law and politics has led some historians to search for the “original
meaning”of its most controversial clauses. This approach, known as
“originalism”, presumes that each right codified in the Bill of Rights had an
independent history that can be studied in isolation from the histories of other
rights, and its proponents ask how formulations of the Bill of Rights in 1791
reflected developments in specific areas of legal thinking at that time. Legal
and constitutional historians, for example, have found originalism especially
useful in the study of provisions of the Bill of Rights that were innovative by
eighteenth-century standards, such as the Fourth Amendment's broadly
termed protection against “unreasonable searches and seizures”. Recent calls
in the legal and political arena for a return to a “jurisprudence of original
intention”, however, have made it a matter of much more than purely
scholarly interest when originalism insist that a clause's true meaning was
fixed at the moment of its adoption, or maintain that only those rights
explicitly mentioned in the United States Constitution deserve constitutional
recognition and protection. These two claims seemingly lend support to the
notion that an interpreter must apply fixed definitions of a fixed number of
rights to contemporary issues, for the claims imply that the central problem of
rights in the Revolutionary era was to precisely identify, enumerate, and
define those rights that Americans felt were crucial to protecting their liberty.

Both claims, however, are questionable from the perspective of a strictly


historical inquiry, however sensible they may seem from the vantage point of
contemporary jurisprudence. Even though originalists are correct in claiming
that the search for original meaning is inherently historical, historians would
not normally seek to determine exactly what a specific clause or right meant
when the Bill of Rights was adopted in 1791, because historians would not
normally feel compelled to support attempts to make that “original
meaning”binding today. The strictly historical purpose for an inquiry into the
original meaning of specific rights would be to determine why a particular
clause was adopted and to establish a baseline from which its subsequent
evolution could be traced and evaluated.

Because of its proponents' pressing need to find determinate meaning at


a fixed historical moment, originalism cannot capture everything that was
dynamic and creative—thus uncertain and problematic—in Revolutionary
constitutionalism, nor can it easily accommodate the diversity of views that
explains why the debates of the Revolutionary era were so lively. A strictly
historical approach, on the other hand, makes it clear that the framers and
ratifiers of the Bill of Rights were struggling with complex questions, the
novelty of which had carried them away from the received wisdom of their
time and was forcing their ideas about rights and the protection of those
rights to continuallyevolve.

1. The primary purpose of the passage is to

A. reconcile opposing views of a modern-day political and legal issue.

B. introduce an innovative approach to a particular legal and political


issue.

C. criticize the application of a certain scholarly methodology to


contemporary legal issues.

D. trace the origin and development of a certain scholarly methodology.

E. explain complementary scholarly approaches to a historical question.

2. It can be inferred from the passage that a jurisprudence of original


intention is based on which of the following assumptions about the Bill of
Rights?

A. Its framers and ratifiers sought to protect individual rights in as many


situations as possible by describing each right in broad terms.

B. Its framers and ratifiers originally intended the rights enumerated in


the various individual clauses to be interpreted in relation to one another.

C. Each clause has a meaning that can be determined by studying its


history and can be applied to contemporary issues.

D. Each right reflects the diversity of views that its framers held about
individual rights.

E. A study of interpretations of the Bill of Rights suggests that the Bill


can legitimately be read in more than one way.

3. The passage suggests that a historian conducting a strictly historical


inquiry would make which of the following assumptions when studying the
Bill of Rights?

A. The framers of the Bill of Rights sought to define each right in strict
and narrow terms.

B. The results of historical inquiry into the true meaning of its clauses
must be applied to contemporary issues.

C. Developments in thinking about individual rights ended after the


codification of those rights.

D. It is possible to determine why a particular clause was included in the


Bill of Rights.

E. Legislators of the Revolutionary era were preoccupied with defining


and enumerating those rights that were crucial to individual liberty.

4. Which of the following historical documents, if they existed, would


most strengthen the author's characterization of Revolutionary
constitutionalism?
A.Placards from 1791 urging people to ratify the Bill of Rights because
it explicitly mentions all rights deserving of constitutional protection.

B.Personal letters of a framer of the Bill of Rights complaining about his


colleagues' failure to reach consensus about which rights to protect and how
to protect them.

C.Minutes of a meeting during which the precise wording of a right was


worked out in order to ensure that the right had a single meaning.

D.The diary of a framer of the Bill of Rights that details a discussion


concerning why one particular clause should be included in the Bill of Rights.

E.Newspaper editorials asserting that the framers of the Bill of Rights


failed to develop creative or innovative ideas about rights.

练习2

In the late nineteenth century, art critics regarded seventeenth-century


Dutch paintings as direct reflections of reality. The paintings were discussed
as an index of the democracy of a society that chose to represent its class,
action, and occupations exactly as they were, wide-ranging realism was seen
as the great accomplishment of Dutch art. However, the achievement of more
recent study of Dutch art has been the recovery of the fact that such paintings
are to be taken as symbolizing mortality, the renaissance of earthly life, and
the power of God, and as message that range from the mildly moralizing to
the firmly didactic.

How explicit and consistent the symbolizing process was intended to be


is a much thornier matter, but anyone who has more familiarity than a
passing acquaintance with Dutch literature or with the kinds of images used
in illustrated books(above all emblem books)will know how much less
pervasive was the habit of investing ordinary objects than of investing scenes
with meaning that go beyond their surface and outward appearance. In the
mid-1960s, Eddy de Jongh published an extraordinary array of material—
especially from the emblem books and vernacular literature—that confirmed
the unreliability of taking Dutch pictures at surface value alone.
The major difficulty, however, with the findings of critics such as de
Jongh is that it is not easy to assess the multiplicity of levels in which Dutch
viewers interpreted these pictures. De Jongh's followers typically regard the
pictures as purely symbolic. Not every object within Dutch paintings need to
be interpreted in terms of the gloss given to its equivalent representation in
the emblem books. Not every foot warmer is to be interpreted in terms of the
foot warmer in Rowmer Visscher's Sinnepoppen of 1614, not every bridle is
an emblem of restraint(though many were indeed just that).

To maintain as Brown does, that the two children in Netscher's painting


A Lady Teaching a Child to Read stand for industry and idleness is to fail to
understand that the painting has a variety of possible meanings, even though
the picture undoubtedly carriers unmistakable symbolic meanings, too.
Modern Art historians may well find the discovery of parallels between a
painting and a specific emblem exciting; they may, like seventeenth-century
viewers, search for the double that lie behind many paintings. But
seventeenth-century response can hardly be reduced to the level of formula.
To suggest otherwise is to imply a laboriousness of mental process that may
well characterize modern interpretations of seventeenth-century Dutch Art,
but that was, for the most part, not characteristic in the seventeenth century.

1. The passage is primarily concerned with which of the following?

A. Reconciling two different points of view about how art reflects.

B. Criticizing a traditional method of interpretation.

C. Tracing the development of an innovative scholarly approach.

D. Describing and evaluating a recent critical approach.

E. Describing a long-standing controversy and how it was resolved.

2. The author of the passage mentions bridles in the highlighted portion


of the passage most likely in order to

A. suggest that restraint was only one of the many symbolic meanings
attached to bridles.
B. provide an example of an everyday, physical object that was not
endowed with symbolic meaning.

C. provide an example of an object that modern critics have endowed


with symbolic meaning different from the meaning assigned to it by
seventeenth-century Dutch artists.

D. provide an example of an object with symbolic meaning that was not


always used as a symbol.

E. provide an example of an everyday object that appears in a significant


number of seventeenth century Dutch paintings.

3. Which of the following best describes the function of the last


paragraph of the passage?

A. It provides specific applications of the critical approach introduced in


the preceding paragraph.

B. It present a caveat about the critical approach discussed in the


preceding paragraph.

C. It presents the research on which a theory presented in the preceding


paragraph is based.

D. It refutes a theory presented in the preceding paragraph and


advocates a return to a more traditional approach.

E. It provides further information about the unusual phenomenon


described in the preceding paragraph.

4. The passage suggests which of the following about emblem books in


seventeenth-century Holland?

A. They confirm that seventeenth century Dutch painting depict some


objects and scenes rarely found in daily life.

B. They are more useful than vernacular literature in providing


information about the symbolic content of seventeenth-century Dutch
painting.

C. They have been misinterpreted by art critics, such as de Jongh, who


claim seventeenth- century Dutch paintings contain symbolic meaning.

D. They are not useful in interpreting seventeenth-century Dutch


landscape painting.

E. They contain material that challenges the assumptions of the


nineteenth-century critics about seventeenth-century Dutch painting.

练习3

A divide between aesthetic and technical considerations has played a


crucial role in mapmaking and cartographic scholarship. Since nineteenth
century cartographers, for instance, understood themselves as technicians
who did not care about visual effects, while others saw themselves as
landscape painters. That dichotomy structured the discipline of the history of
cartography. Until the 1980s, in what Blakemore and Harley called “the Old
is Beautiful Paradigm,”scholars largely focused on maps made before 1800,
marveling at their beauty and sometimes regretting the decline of the pre-
technical age. Early mapmaking was considered art while modern
cartography was located within the realm of engineering utility. Alpers,
however, has argued that this boundary would have puzzled mapmakers in
the seventeenth century, because they considered themselves to be visual
engineers.

1. According to the passage, Alpers would say that the assumptions


underlying the “paradigm”were

A. inconsistent with the way some mapmakers prior to 1800 understand


their own work.

B. dependent on a seventeenth-century conception of mapmaking visual


engineering.

C. unconcerned with the difference between the aesthetic and technical


questions of mapmaking.

D. insensitive to divisions among cartographers working in the period


after 1800.

E. supported by the demonstrable technical superiority of mapmaking


made after 1800.

2. It can be inferred from the passage that, beginning in the 1980s,


historians of cartography

A. placed greater emphasis on the beauty of maps made after 1800.

B. expanded their range of study to include more material created after


1800.

C. grew more sensitive to the way mapmakers prior to 1800 conceived


of their work.

D. came to see the visual details of maps as aesthetic objects rather than
practical cartographic aids.

E. reduced the attention they paid to the technical aspects of


mapmaking.

练习4

In 1919 Britain experienced its largest ever reduction in industrial


working hours, to 48 per week. In Dowie's view the 48-hour week played a
central role in Britain's poor economic performance during the 1920s. Dowie
argued that the reduction, together with rapid wage growth, drove up prices.
However, Greasly and Oxley found that the First World War(1914—
1918)constituted a more powerful negative macroeconomic shock to
Britain's competitiveness. And Scott argues that Dowie's thesis ignores
considerable evidence that hourly productivity improves when hours are
reduced from a high base level. Crucially, Dowie's thesis does not
acknowledge that hours were reduced to around 48 hours a week for
industrial workers in most industrialized nations at this time so far—
undermining any potential impact of reduced hours on industrial productivity
relative to other nations.

1. Which of the following, if true, provides the strongest support for


Scott's argument?

A. Companies have generally found that part-time employees are less


productive than full-time ones.

B. When the total number of hours worked at a company increases


owing to the addition of more employees, the usual result is improved
productivity at the company.

C. When the total hours worked by all employees per week in two
companies are equivalent, hourly productivity tends to be equivalent as well.

D. Companies whose employees usually work a high number of hours


tend to have greater total costs than do similar companies whose employees
work fewer hours.

E. Companies have found that total output per employee is not


necessarily changed by reductions in the number of hours worked per
employee.

2. It can be inferred from the passage that in the view of Greasly and
Oxley

A. a reduced workweek was ultimately beneficial to employees.

B. the economic effects of the reduction in working hours in 1919 were


brief.

C. Britain became less economically competitive in the 1920s.

D. reduced working hours were the primary cause of the economic


changes observed by Dowie.

E. the changes in economic performance in Britain in the 1920s were


unforeseen.
第十一章 补充文章练习与讲解
练习1

According to a theory advanced by researcher Paul Martin, the wave of


species extinctions that occurred in North America about 11,000 years ago, at
the end of the Pleistocene era, can be directly attributed to the arrival of
humans, i.e., the Paleoindians, who were ancestors of modern Native
Americans. However, anthropologist Shepard Krech points out that large
animal species vanished even in areas where there is no evidence to
demonstrate that Paleoindians hunted them. Nor were extinctions confined to
large animals: small animals, plants, and insects disappeared, presumably not
all through human consumption. Krech also contradicts Martin's exclusion of
climatic change as an explanation by asserting that widespread climatic
change did indeed occur at the end of the Pleistocene. Still, Krech attributes
secondary if not primary responsibility for the extinctions to the Paleoindians,
arguing that humans have produced local extinctions elsewhere. But,
according to historian Richard White, even the attribution of secondary
responsibility may not be supported by the evidence. White observes that
Martin's thesis depends on coinciding dates for the arrival of humans and the
decline of large animal species, and Krech, though aware that the dates are
controversial, does not challenge them; yet recent archaeological discoveries
are providing evidence that the date of human arrival was much earlier than
11,000 years ago.

1. Which of the following is true about Martin's theory, as that theory is


described in the passage?

A. It assumes that the Paleoindians were primarily dependent on hunting


for survival.

B. It denies that the Pleistocene species extinctions were caused by


climate change.
C. It uses as evidence the fact that humans have produced local
extinctions in other situations.

D. It attempts to address the controversy over the date of human arrival


in North America.

E. It admits the possibility that factors other than the arrival of humans
played a role in the Pleistocene extinctions.

解析: 由题目中的“Martin's theory”定位到文章首句,首句


的话题大致是“人类导致动物灭绝”。假设学生没有通读全文,根据
这一话题判断哪个选项当选。

A项:“人类依靠打猎生存”。这项描述的话题是“人类生存的
方式”,和“人类导致动物灭绝”没有必然联系。

B项:“否定了气候导致动物灭绝”。“人类导致动物灭绝”侧
面削弱了其他因素导致动物灭绝的可能性,B项比A项好些。

C项:“人类在其他场合导致动物灭绝”。“其他场合(other s
ituations)”是原句未提到的新概念,属于无中生有,不可选。

D项:“解决了日期的争议”。根据第一句话的内容,得不出和
“人类到达日期”相关的话题,此项属于无关选项。

E项:“人类之外的其他因素导致动物灭绝”。此项和原定位所
述内容正好相反,排除。

正确答案为B项。

若学生通读了全文,由“Krech also contradicts Martin's ex


clusion of climatic change as an explanation”这句话可知“Ma
rtin确实排除了气候这个解释”,依然可以得出答案为B项。

2. Which of the following, if true, would most weaken Krech's


objections to Martin's theory?

A. Further studies showing that the climatic change that occurred at the
end of the Pleistocene era was even more severe and widespread than was
previously believed.

B. New discoveries indicating that Paleoindians made use of the small


animals, plants, and insects that became extinct.

C. Additional evidence indicating that widespread climatic change


occurred not only at the end of the Pleistocene era but also in previous and
subsequent eras.

D. Researchers' discoveries that many more species became extinct in


North America at the end of the Pleistocene era than was previously believed.

E. New discoveries establishing that both the arrival of humans in North


America and the wave of Pleistocene extinctions took place much earlier than
11,000 years ago.

解析: 题目问哪项能“削弱Krech对Martin的理论的反驳”。出
题人可能针对任何一条反驳进行削弱(weaken),所以在记原文话题
的时候,需要把Krech驳斥的话题都记住。

通过大写的Krech定位到原文中反驳的相关内容——“anthropol
ogist Shepard Krech points out that large animal species van
ished even in areas where there is no evidence to demonstrat
e that Paleoindians hunted them(事件1). Nor were extinctio
ns confined to large animals: small animals, plants, and ins
ects disappeared, presumably not all through human consumpti
on.(事件2)Krech also contradicts Martin's exclusion of cli
matic change as an explanation by asserting that widespread
climatic change did indeed occur at the end of the Pleistoce
ne.”(事件3)

正确答案必须和提到的三个事件相关。由此,可排除D和E项。(
D项的“种类多少”和E项的“人类到达日期”不属于这三个事件。)

既然题目问削弱的方向,那么最直接的方式是在原句谓语后或主
语前加上否定词。原定位句提到“气候变化”的话题,我们要找的答
案方向就必须没有提到“气候变化”,A和C项的内容因为和我们要找
的事件方向相反,排除。最终得出答案为B项。

3. In the last sentence of the passage, the author refers to “recent


archaeological discoveries”(line 14)most probably in order to

A. refute White's suggestion that neither Maritn nor Krech adequately


account for Paleoindians' contributions to the Pleistocene extinctions.

B. cast doubt on the possibility that a more definitive theory regarding


the causes of the Pleistocene extinctions may be forthcoming.

C. suggest that Martin's, Krech's, and White's theories regarding the


Pleistocene extinctions are all open to question.

D. call attention to the most controversial aspect of all the current


theories regarding the Pleistocene extinctions.

E. provide support for White's questioning of both Martin's and Krech's


positions regarding the role of Paleoindians in the Pleistocene extinctions.

解析: 作答“in order to”考题时,要思考定位句本身是围绕


哪句话来谈的。换言之,要思考定位句和上下句共同说了什么话题,
这个话题才是找答案的方向。

回原文定位“recent archaeological discoveries”所在的句


子及上句:

But, according to historian Richard White, even the attr


ibution of secondary responsibility may not be supported by
the evidence. White observes that Martin's thesis depends on
coinciding dates for the arrival of humans and the decline o
f large animal species, and Krech, though aware that the dat
es are controversial, does not challenge them; yet recent ar
chaeological discoveries are providing evidence that the dat
e of human arrival was much earlier than 11,000 years ago.

定位词所在的句子是“But”所在句展开的细节,用于反驳之前K
rech和Martin的说法。根据话题来筛选选项。

A项:原文White反驳Krech和Martin这两个人,而不是反驳White
自己。

B项:和定位句上句的话题无关,排除此项。

C项:不能反驳White本身,排除此项。

D项:并没有在定位句附近找到最高级的说法,而此项出现了最
高级,不太合适。

E项:此项支持White,反驳Martin和Krech,描述的内容和要找
的答案方向一致,且前几项方向都不正确,只能选E项。

做题技巧: 先判断答案的方向再看选项,这样思维不会被选项
内容带偏。

练习2

Jon Clark's study of the effect of the modernization of a telephone


exchange on exchange maintenance work and workers is a solid contribution
to a debate that encompasses two lively issues in the history and sociology of
technology: technological determinism and social constructivism. Clark
makes the point that the characteristics of a technology have a decisive
influence on job skills and work organization. Put more strongly, technology
can be a primary determinant of social and managerial organization. Clark
believes this possibility has been obscured by the recent sociological fashion,
exemplified by Braverman's analysis, that emphasizes the way machinery
reflects social choices. For Braverman, the shape of a technological system is
subordinate to the manager's desire to wrest control of the labor process from
the workers. Technological change is construed as the outcome of
negotiations among interested parties who seek to incorporate their own
interests into the design and configuration of the machinery. This position
represents the new mainstream called social constructivism.

The constructivists gain acceptance by misrepresenting technological


determinism: technological determinists are supposed to believe, for example,
that machinery imposes appropriate forms of order on society. The alternative
to constructivism, in other words, is to view technology as existing outside
society, capable of directly influencing skills and work organization. Clark
refutes the extremes of the constructivists by both theoretical and empirical
arguments. Theoretically he defines “technology”in terms of relationships
between social and technical variables. Attempts to reduce the meaning of
technology to cold, hard metal are bound to fail, for machinery is just scrap
unless it is organized functionally and supported by appropriate systems of
operation and maintenance. At the empirical level Clark shows how a change
at the telephone exchange from maintenance-intensive electromechanical
switches to semi-electronic switching systems altered work tasks, skills,
training opportunities, administration, and organization of workers. Some
changes Clark attributes to the particular way management and labor unions
negotiated the introduction of the technology, whereas others are seen as
arising from the capabilities and nature of the technology itself. Thus Clark
helps answer the question: “When is social choice decisive and when are the
concrete characteristics of technology more important?”

1. The primary purpose of the passage is to

A. advocate a more positive attitude toward technological change.

B. discuss the implications for employees of the modernization of a


telephone exchange.

C. consider a successful challenge to the constructivist view of


technological change.
D. challenge the position of advocates of technological determinism.

E. suggest that the social causes of technological change should be


studied in real situations.

解析: 作答主旨题不需要回原文定位。如果文章两段话,每段
话说不同的话题,并且两段话篇幅差不多,则要以第二段所述内容为
思考的依据。

第二段是对“社会建造主义”的批判。根据话题找答案,只有C
项符合。

A项:批评“社会建造主义”是间接地支持“科技主义”,但A项
肯定不如C项接近答案的方向,并且A项中more的说法属于无中生有,
排除此项。

B项:和第二段框架信息无关。

D项:全文的态度是支持“科技主义”,此项和主态度相反,排
除此项。

E项:和第二段的话题无关,排除。

正确答案为C项。

2. The information in the passage suggests that Clark believes that


which of the following would be true if social constructivism had not gained
widespread acceptance?

A. Businesses would be more likely to modernize without considering


the social consequences of their actions.

B. There would be greater understanding of the role played by


technology in producing social change.
C. Businesses would be less likely to understand the attitudes of
employees affected by modernization.

D. Modernization would have occurred at a slower rate.

E. Technology would have played a greater part in determining the role


of business in society.

解析: 题目问得比较细,是需要定位的细节题。

此题通过“gained widespread acceptance”即可很容易地定位


到第二段首句:The constructivists gain acceptance by misrepr
esenting technological determinism: technological determinis
ts are supposed to believe, for example, that machinery impo
ses appropriate forms of order on society.这句话描述的话题是
“科技”和“社会”的关系,找提到类似话题的选项。

A项:“商业”和“社会”的关系。“商业”这个概念是原定位
句中没有的,此项不太好。

B项:“科技”和“社会”的关系。符合要找的答案方向,可先
放着。

C项:“商业”和“员工”的关系。此项无关,排除。

D项:此项内容和定位句的内容相差太远,直接排除。

E项:“科技”和“商业”的关系,排除。

得出答案为B项。

可将定位句和选项读成“A与B的关系”。这里的A与
做题技巧:
B是句子中心,且优先是名词,若没有名词,可再思考动词或其他
内容。用概括中心的方式将长句子浓缩成“短话题”,方便判断选
项和定位句是否在描述同一个话题,同时方便做题过程中加以记
忆。

3. The author of the passage uses the expression “are supposed to”in line
14 primarily in order to

A. suggest that a contention made by constructivists regarding


determinists is inaccurate.

B. define the generally accepted position of determinists regarding the


implementation of technology.

C. engage in speculation about the motivations of determinists.

D. lend support to a comment critical of the position of determinists.

E. contrast the historical position of determinists


with their position regarding the exchange modernization.

解析: “in order to”考题又来了。要看定位句是论点还是论


据,如定位句是论点,就找和定位句话题类似的选项;如定位句是论
据,须找到此论据针对的论点,并将论点的内容作为答案的方向。

此题还是定位到第二段首句。第二段首句描述的话题是“Clark
反驳社会建造主义,并且认为科技对社会有影响力”。只要选项和这
个话题部分相关或方向一致,都可以作为备选项。

A项:此项的话题是“社会建造主义是错误的”,和原定位句的
“misrepresenting”沾边,只要部分相关即可先放着待比较。

B项:“定义一个广泛被接受的观点。”定位句中不存在“gener
ally accepted position”这个概念,排除此项。

C项:此项一看即与话题无关。

D项:和定位句无关。

E项:和定位句并无意思上交集,排除此项。
得出答案为A项。

不要追求完美的答案,选项部分沾边或方向一致即
做题技巧:
可放着待比较。有时GRE的答案是比较后得出的结果,不是这个选
项好,而是和其他选项比起来,只有这个选项相对合理,所以选择
了这项为答案。

4. Which of the following statements about the modernization of the


telephone exchange is supported by information in the passage?

A. The new technology reduced the role of managers in labor


negotiations.

B. The modernization was implemented without the consent of the


employees directly affected by it.

C. The modernization had an impact that went significantly beyond


maintenance routines.

D. Some of the maintenance workers felt victimized by the new


technology.

E. The modernization gave credence to the view of advocates of social


constructivism.

解析: 题目问“关于电话交换机的现代化(modernization of
the telephone exchange),哪个选项被全文支持”。此题考查的是
全文性内容,需要结合文章的脉络来思考。可以定位到文章的首段首
句,后文都是由第一句话而引出的内容(从第一句话引出了两个“主
义”)。

结合脉络,比较选项,寻找答案。

A项:文章脉络并没有涉及此项的话题,A项不太好。
B项:此项话题并没有出现过,不太好。

C项:此项说的是“现代化有一个影响力,这个影响力超越了例
行维护(maintenance routines)”。根据文章脉络可知,由“现代
化(modernization)”引出了两个主义,后文扩展了这两个主义的
细节。“有一个影响力”说明不管是什么样的影响力,只要这个影响
力在例行维护之外即可。“引出了两个主义”肯定属于例行维护之外
,故此项不能说错,先放着待比较。

A、B、C三项中找一个正确答案,概率最大的是C项,因为C项至
少符合文章的逻辑顺序。

D项:文章的态度是支持“科技主义”,此项描述的事件方向和
主态度相反,不太好。

E项:文章整个第二段在反驳“社会建造主义”,此项支持社会
建造主义,和文章主态度相反,排除此项。

得出答案为C项。

作答需要从全文去思考答案的问题时,如发现一个
做题技巧:
选项描述的事件和文章脉络一致,则优选这个选项。

练习3

In the fourteenth and fifteenth centuries, many Western Pueblo


settlements in what is now the southwestern United States may have
possessed distinctly hierarchical organizational structures. These
communities' agricultural systems―which were “intensive”in the use of
labor rather than “extensive”in area―may have given rise to political
leadership that managed both labor and food resources. That formal
management of food resources was needed is suggested by the large size of
storage spaces located around some communal Great Kivas(underground
ceremonial chambers). Though no direct evidence exists that such spaces
were used to store food, Western Pueblo communities lacking sufficient
arable land to support their populations could have preserved the necessary
extra food, including imported foodstuffs, in such apparently communal
spaces.

Moreover, evidence of specialization in producing raw materials and in


manufacturing ceramics and textiles indicates differentiation of labor within
and between communities. The organizational and managerial demands of
such specialization strengthen the possibility that a decision making elite
existed, an elite whose control over labor, the use of community surpluses,
and the acquisition of imported goods would have led to a concentration of
economic resources in their own hands. Evidence for differential
distribution of wealth is found in burials of the period: some include
large quantities of pottery, jewelry, and other artifacts, whereas others
from the same sites lack any such materials.

1. Which of the following, if true, would most clearly undermine the


author's statement in the last sentence of the passage regarding the
distribution of wealth in Western Pueblo settlements?

A. Only community members of exceptional wealth are likely to have


been buried with their personal possessions.

B. Members of communities with extensive agricultural systems are


usually buried without personal possessions.

C. Most artifacts found in burial sites were manufactured locally rather


than imported from other communities.

D. Burial artifacts are often ritual objects associated with religious


practices rather than being the deceased's personal possessions.

E. The quality of burial artifacts varies depending on the site with which
they are associated.

解析: 题目问哪项能“削弱作者关于财富分布说法”。此题可
以只用解答逻辑题的思维和技巧来攻克。定位到最后一段的内容:关
于财富的分布,最后一段提到“存在精英阶级,发现了财富的分布”
。这样的推论过程属于“果因推理”,即“通过一个现象,得到一个
说法”。在此题中是通过“财富的分布(现象)(前提)推出有精英
阶层这一说法(结论)”。

“果因推理”的削弱角度只有两种:(1)给出他因;(2)推理
链中的前提推不出相应结论。出题机构往往会选择第一种出题方式,
因为如果选第二种出题方式,必须要把文章中的现象和说法重复一遍
,太容易让考生觉得这个选项沾边了。所以我们要优先找一个由“他
因”能推出“现象”的选项。

D项指出“财富是因为宗教因素,并不是因为存在精英阶层”,
属于“他因削弱”的反驳思路,即为答案。

2. According to the passage, which of the following is probably true of


the storage spaces mentioned in line 6?

A. They were used by the community elite for storage of their own food
supplies.

B. They served a ceremonial as well as a practical function.

C. Their size is an indication of the wealth of the particular community


to which they belonged.

D. Their existence proves that the community to which they belonged


imported large amounts of food.

E. They belonged to and were used by the community as a whole.

解析: 通过“storage spaces”找定位句,且读完整个定位句


,切不可只读半句。That formal management of food resources w
as needed is suggested by the large size of storage spaces l
ocated around some communal Great Kivas(underground ceremon
ial chambers). Though no direct evidence exists that such s
paces were used to store food, Western Pueblo communities la
cking sufficient arable land to support their populations co
uld have preserved the necessary extra food, including impor
ted foodstuffs, in such apparently communal spaces.

定位句大致描述两件事情:(1)在Kivas旁边;(2)用来储备
东西。

A项:elite是原定位句没有的内容,也是A项可能出错的地方。

B项:Kivas是“ceremonial chamber”,但在Kivas旁边的“sto
rage space”是否也有“ceremonial”的功能,我们并不知道,所以
“ceremonial”是B项可能出错的地方。

C项:“indication of the wealth”是新话题,排除此项。

D项:定位句提到了“进口食物”,但无法知道是否是“大量的
(large amounts)”,排除此项。

E项:认为“storage space”被使用,定位句也确实这样描述了
,所以E项和之前总结出的话题沾边。

得出答案为E项。

读一个选项的时候,不看认为对的地方,只挑这个
做题技巧:
选项是否有地方出错,如一个选项没有出错的空间,则这个选项大
概率为答案。

3. The primary purpose of the passage is to

A. outline the methods by which resources were managed within a


particular group of communities.

B. account for the distribution of wealth within a particular group of


communities.

C. provide support for a hypothesis concerning the social structure of a


particular society.
D. explain how political leadership changed in a particular historical
situation.

E. present new evidence that contradicts previous theories about a


particular historical situation.

解析: 文章第一段中告诉我们Peoblo部落有一个“阶级分层”
(hierarchical organizational structures),后文都在针对这点
进行细节展开。第二段告诉我们存在一个精英阶级(elite),也属
于是“阶级分层”的一个细节。所以全文就在告诉我们“Peoblo存在
一个阶级制度下的社会结构”。带着这个预测的答案方向去看选项,
只有C项合理。
第三部分 GRE填空

第十二章 填空解题思维

一、余氏GRE填空题解题思维

即使背诵了单词也不一定能在GRE填空中得到高分,因为填空题依
然在考查学生的逻辑思维。在14年的GRE教学生涯中,我总结了以下
GRE填空考查的三个角度:(1)长难句的理解;(2)学术性词汇的掌
握;(3)客观性思维。背诵词汇只是解填空题的基础,还需要用逻辑
思维来理解句意以及找到空格与空格外信息的联系。

以下九项原则是解答填空题的关键思维,考生要牢记。

原则1:读句子时有四大注意点——“词汇一致性”“不同分
类”“空格方向”“选项关系”

1. “词汇一致性”

解读: 读句子的时候,要看句子中是否有词汇一致性的情况。
词汇一致性是指不同的词汇描述的是同样的对象,或针对相同的对象
举例、列出子集。例如,前一个词是“人”,后一个词是“男人”,
就要把这两个词联系在一起看,因为它们是在描述相同的对象。
例1 Many of us will take on more risk if we perceive ____ factor;
people will drive faster when wearing a seat belt.

解析: 分号前后的两个句子描述的话题和方向一致。后文“dri
ve faster”就是针对之前“take on more risk”所举的例子,所以
“wearing a seat belt”对应了空格的内容。

例2 A bird's feathers would seem to be a(i)____ design for protecting


a bird from attack by microscopic organisms. They create a warm, moist
space next to the skin that could be an ideal incubator for spores. Wild birds
rarely(ii)____ skin diseases, however. The chemicals in the sebum include
an array of antibacterial and antifungal agents that allow the bird's skin
to(iii)____

解析: 在第二句话和第一句话之间没有找到表示逆接关系的词
(比如however、but 等),这说明两句话之间是顺接关系。第二句
话中的“spores”对应了前一句中的“microscopic organisms”。
第三句话中的“antibacterial and antifungal agents”对应了“m
icroscopic organisms”和“spores”。这几个不同的词描述的对象
是同一个,相互对应。词汇一致性的思维不仅适用于填空题,在阅读
题中也适用。

例3 Caricature can be revealing as well as amusing, and Ager's novel is


both: Ager's delineation of class, ethnic, and generational struggle is
exaggerated for comical effect, but it nonetheless ____.

解析: 冒号前有“revealing as well as amusing”,冒号之


后的“comical effect”对应“amusing”,所以空格要对应“revea
ling”。
如何判断“词汇一致性”?

答: 在读句子的过程中,若读到的信息与前文是同义词、近义
词、上义词或反义词,即满足“词汇一致性”。“反义词”也算具有意思上
的一致性,需要和前文局部信息结合起来看。

2. “不同分类”

解读: 在句中如果可以读出两个不同的分类,则其属性也不同
。这里的关键词是“不同”,不能把同一个类别下的例子理解为两个
分类。最常见的不同分类是“现在”与“过去”。

例 His new role gives the normally clownish actor a chance to impress
audiences with his(i)____. He is among the most uninhibited comic
performers around, but here he buttons his lip and stares straight ahead.
Perhaps without quite knowing it, the audience waits for a wink, a hint that
some of the(ii)____spirit that animated his previous movies might be
lurking inside the(iii)____ manner he presents in this film.

解析: 此题中的分类是“新”(new role)与“一般”(norma


lly)(不需要刻意去记固定的分类,考题中读到的不同类别即为不
同分类,其属性是相反的)。第二句话“He is among the most uni
nhibited comic performers”修饰“一般”情况,因此“新”角色
的特征就是“抑制,收敛(inhibited)”(意思相关即可)。故第
一空选C项。第二空所在句的“animated his pervious movies”是
“normally”的同义改写,都是描述过去一般情况下的表现,所以找
一个和“uninhibited”相关的词汇即可,选出D项。第三空之后的“
he presents in this film”又是在描述“新”的情况,与“inhabi
t”相关的只有“reserved”,得出答案为H项。

考生们要切记作答GRE填空题时不能感觉哪个词读得顺就填哪个。
要通过已有的信息线索来反推空格的内容,填空题考查的核心依然是逻
辑思维。

3. “空格方向”

解读: 遇到读不懂的句子,要先判断“空格方向”,再看选项
,进而缩小选择的范围。根据教学经验,我总结出以下两个关于“空
格方向”的思考角度:(1)好词/坏词;(2)空格与空格外的信息
之间的关系是“等于”还是“不等于”。别想其他的内容,只根据这
两大方向看选项。

例  Women in the mining towns of the American West were strictly


stereotyped into neat categories of public and private, good and bad, but the
100 intrepid female prospectors in Zanjani's book managed to(i)____
those categories. In addition to providing documentation that demolishes the
all-male version of prospecting, Zanjani uses the examples of her female
lonersto(ii)____. some of the(iii)____ generalizations about Euro-
American women as uniformly nurturant and sociable pioneers.

解析: 假设考生们读不懂此题,如何通过判断空格方向来解题

第一空所在的句子与之前句子是转折关系(but),所以带着“
不等于”的方向来看选项。“居住(inhabit)”“揭示(reveal)
”“使混乱(confound)”这三个词中,哪个词更倾向于“不等于”
?很明显得出答案为C项。

假设根本读不懂第二空所在句,根据空格前的loners和尾句soci
able pioneers来判断,考生们觉得空格处应该填“等于号”还是“
不等于号”?loners很显然与“孤独”相关,其与sociable肯定是取
反的关系,则第二空按“不等于”的方向看D、E、F三项。“发明(i
nvent)”和“持续下去(perpetuate)”与“不等于”不相关,“
击穿,刺穿(puncture)”相对而言和“不等于”更贴近,所以第二
空答案为D项。

第三空修饰一个说法(generalizations)。之前第一空和第二
空都是逆接的方向,第三空是填“好词”还是“坏词”?肯定是“不
好的方向”可能性更大。“轻率,肤浅(facile)”符合题意,得出
答案为H项。

考生们需要注意作答填空题时并不一定要直接得出答案,我们要做
的是先缩小选择范围,如果在这个范围中只有一个选项,则不需要再进
行对比或进一步思考了,直接得出答案即可。这样的思考过程也是决策
思维在考试中的实际应用。

4. “选项关系”

解读: 作答GRE填空题需要通过已有信息来推测空格的内容。已
有信息不仅包含句子本身,还包含选项。正确的选项必须和句子构成
合理的逻辑关系,且话题类似。当句子并没有提示解题信息或线索时
,可结合选项来找出正确答案。判断“选项关系”,可以看空格与空
格间是同义关系还是反义关系,或看哪几个选项说的是相同的话题。

例1 Parker's model of human affairs reflects(i)____ outlook, in stark


contrast to the generally(ii)____ premises that her colleagues in the
economics department adopt in their work. Accordingly, her
conclusions(iii)____ theirs.

解析: 第一空和第二空之间的“contrast to”表示两空为取反


的关系,所以在第一空和第二空的备选项中找出一对反义词,得出答
案为A、D项。其他选项并不能构成“反义词(取反)”的逻辑关系。

第三空修饰“Parker's model”的特征。考生要切记在整场GRE
考试中都要遵循的一个做题原则:和已有信息不冲突的选项都需要留
下待比较。不仅是填空题,也包括逻辑题和阅读题。此题G项和之前
谈论的话题没有关系,H和I项可以再比较。如果选择H项,“not so
sunny”和第一空的“sanguine”内容冲突,所以不可选。得出答案
为I项。

填空题中选项优先级可理解为:

冲突的选项<不冲突的选项<和原文话题沾边的选项<出错概率小
的选项

① 和已有信息冲突的选项不能成为答案,如果发现两个选项与已
有信息冲突,剩下一个选项不冲突,而且这个选项通过已有信息推测不
出来,只能选这个不冲突的选项。

② 如果发现与已有信息不冲突的选项不止一个,则要选一个和已
有信息及话题关联度最高、意思最接近的选项,所以就有了这样的比较
级关系:不冲突的选项<和原文话题沾边的选项。

③ 如几个选项都与原文话题沾边,则需要找一个出错概率小或描
述范围大、表达内容虚的选项。此步骤用到的答题思维和阅读题、逻辑
题相通。

例2 Historians of science argue that chemistry had____scientific


revolution in that several chemists failed to produce radical innovations
parallel to those in other sciences at the time, and chemistry consequently
matured later.

分析: 很多学生在复习GRE一段时间后会发现单词已经不是障碍
了,但还是分辨不出句子中的逻辑关系,只有看了解析之后,才发现
词汇对应的信息。一直以来,网络上流传的或机构教授的GRE学习方
法基本都是在已知答案的情况下进行倒推,这并不是考生在考场上见
到题目之后实际会有的反应。更多的情况是考生读不懂句子、看不出
逻辑关系提示词等,而此书在很大程度上就是帮助考生了解在读不懂
或找不到逻辑关系的情况下该如何解题。假设学生读完此题之后头脑
一片空白,不知切入点,只知道正确答案肯定要和已有的信息相关,
那么可以直接看选项,如果有选项和上下文信息沾边,即可放着待比
较。(逆向思考:不需要先找句子的提示点,只要选项和已有信息有
联系,则这个信息就是句子的提示点。)

发现此题只有C项中的“推迟(postponed)”和后文的“later
”在意思上相呼应,其他选项与空格外信息无关,故答案为C项。

原则2:句间如无转折词,即为顺接关系
解读: 前文我们讲过句子与句子之间的关系(句间关系)一共
有三种——顺接;逆接;新话题。填空题篇幅有限,一共由2~4个句
子组成,不可能在这几个句子中说到两个截然无关的话题,所以GRE
填空题中的句间关系不是顺接就是逆接。简单来说,如果在两个句子
之间并没有找到转折词(取反词),则默认这两个句子是顺接,即话
题和态度一致。

例 With the grand ambition of sending unbreakable coded messages,


some physicists are using exotic tools—to shut out(i)____. But a wire and
a few resistors may(ii)____a message as securely, according to a physicist
who claim to have devised a simple and uncracked scheme. The idea shows
that more(iii)____methods might compete with budding quantum
cryptography.

分析: 根据第一空之前的信息,第一空要填入一个“不好的东
西”。乍一看A、B、C三项都是不好的东西,则要找的选项不仅要与
前文方向一致,话题对应更要精准。前文描述“不可破解(unbreaka
ble)的信息”,故应填入C项,因为prying eyes(偷看的眼睛)与
“是否破解”的话题关系最近。

第二空要修饰“信息(message)”,“as securely(一样安全
地)”中的“as(一样地)”提示空格处要填入一个之前已经出现过
的修饰“信息”的同义词。前文是“sending messages”,故而第二
空填入D项“传输(convey)”最合适。

第三空之后的“密码学(cryptography)”可能有考生不认识,
但末句肯定与之前第二句话是顺接的逻辑关系,因为两句话之间没有
出现“取反词”。第三空要填入修饰“方法(methods)”的词。G项
“平凡的(mundane)”与前句中的“simple”相呼应,而H和I项并
没有可对应的信息,所以G项更合理。

原则3:话题沾边——空格与空格外信息属于一个话题

例 Barker's account of how mores have evolved over time is


illuminating. It reveals as(i)____ and(ii)____ some values that have
often regarded as(iii)____ , while uncovering other values that do indeed
seem to be universal.

分析: 完全读懂句意再推理出空格的信息属于事倍功半的备考
方法,更可能的情况是在考场上根本读不懂抽象语境的句子。既然正
确答案一定和空格外的信息是同一个话题,那么根据空格外的话题来
找选项即可。

即使学生没读懂此题,但至少会发现while从句之后在描述“价
值观是否宏大(value universal)”的话题,根据话题看第一空与
第二空的选项。前两空是顺接关系,所以要找一对方向相同且话题与
“范围大小”相关的选项,故得出答案为A、E(“短暂”和“相对”
均属于“范围小”这一话题)。如果第二空不选E项,选择D或F项,
那么第一空没有对应逻辑的词可以选择,故能确定前两空答案为“A
、E”。“通常认为(often regard)”与“确实(do indeed)”为
不同分类,则第三空选G项。

原则4:填空也考查客观性思维
解读: 填空题中依然会考查客观性思维,不可以靠感觉和主观
判断做题。

例 Hyana Kusiemko and her colleagues speculate that the(i)


____support among low-income works for increases in the minimum wage is
a form of last-place aversion: people who are in a marginally better position
than the worst off seek to(ii)____ to distinguish themselves from those in
last place.

分析: “增加工资”一定是受员工欢迎的好事吗?解题时切不
可只凭主观思维而不考虑句子的信息。

此题中的“last-place aversion”(不喜欢最差排名)提示我
们,比收入垫底的人好一点的人(people who are in a marginally
better position than the worst off)想将他们与最后一名区分开
,而增加工资会让他们之间的差距更小,这样就与比收入垫底的人好
一点的人们的初衷相反了,所以他们不希望增加工资,故第一空选C
项 “tepid(不热情)”。第二空选E项,同样对应了之前“last-pl
ace aversion”,因为他们不喜欢最差排名,所以要与最后一名区分
开(distinguish themselves from those in last place)。

原则5:“取同”关系优先

解读: GRE填空题考查的是空格与已有信息之间的逻辑关系是“
取同”还是“取反”。当一个空格既可以通过“取同”关系推理,又
可以通过“取反”关系推理时,优先使用“取同”逻辑来做题。譬如
,“你今天穿的衣服不是红色的”,则你今天穿的衣服可以是除了红
色之外的任何颜色;但如果“你今天穿的衣服接近蓝色”,则你今天
穿的衣服肯定和“蓝色”沾边。所以用“取同”的逻辑来找答案,会
大大缩小选择范围。

例 By deliberately dripping paint on their canvases, expressionists


affirmed that paint is not____entity, something merely to be moved where
the artist pleases, but rather a material possessing a fluid energy that the
painter attempts to control.

分析: 题干中与空格相关的线索有两个:一个是逗号之后的信
息“something merely to be moved where the artist pleases”
;另一个是“not... but...”形成的搭配。

空格与第二个逗号之后的信息是顺接关系,因为看到逗号就相当
于看到了约等于号。“not A but B”的结构中,A与B构成逆接(转
折)的逻辑关系。在既有顺接关系又有逆接关系的情况下,优先使用
顺接关系来解题。

带着“be moved where the artist pleases”的信息来看选项


,得出答案为D项。
原则6:找一个“虚的、描述范围大”的选项

解读: 与阅读题和逻辑题中找“虚”的解题思维一致,虚的、
描述范围大的选项表示此项错误概率比较小。

例1 Both inquiries were pushed forward by obsessive, heavy-handed


investigators with political agendas; both dragged on interminably, with
investigators ultimately chasing after details(i)____ the original alleged
offenses. And just as the first inquiry long ago grew too(ii)____for most
citizens to grasp, in the same way the second inquiry hinged on
immunological experiments so(iii)____—and produced so many
conflicting interpretations of the allegedly fabricated experimental records-
that impartial observers rarely knew whatto believe.

分析: 通过语境,并没有发现转折关系,则第一空答案要在A与
C中选择。

先看第三空。第三空肯定与之后的并列句“and... to believe
”联系更密切,后句讲“so many conflicting”,显然“arcane(
难懂的)”与这一话题更近,“明确(unambiguous)”与“many co
nflicting”信息冲突,“重要(momentous)”话题并不沾边,故第
三空答案为G。

第二空之前的“just as the first inquiry”表明两个问题属


性一致,根据第三空的“arcane(难懂的)”可推测出第二空选“co
nvoluted(困难,难理解)”。

最后看第一空,A与C项和空格外信息不冲突,哪个范围大?“fo
undation(基础)”本身可以是“background(背景)”的一个组成
部分,所以“background(背景)”的范围更大,得出答案为C项。

注意:与GRE阅读题和逻辑题的解题思路一致,若“范围小、程度
高、语气重”的选项与句中信息相呼应,优先选择这一选项。如无对
应,“范围小、程度高、语气重”与“范围大”的选项都可成为备选答案
时,才选择“范围大”的选项。

例2 Although Uruk in southern Mesopotamia has been(i)____as


being both the first city and the model for later ones, at least two sites in
northern Mesopotamia have yielded clear evidence of urbanization long
before the existing evidence from Uruk, and other discoveries indicate that
some of the(ii)____early urbanism were invented not in southern
Mesopotamia but in the north. These findings have led some archaeologists
to(iii)____a serious reconsideration about when and where the first cities
arose.

分析: 第一句话的主句表明“其他地方可能是第一个城市”,
所以让步关系从句中要填一个使“Uruk”就是第一个城市这一逻辑关
系成立的词汇,A和C项都可作为备选项。

第二空所在的句子和第二个and之前的句子是并列关系,所以答
案应与“其他城市才是第一个城市”方向一致,E和F项都和已知信息
冲突(“derivative”和前句中的“clear”冲突,“traditional t
heories”和前文反驳“the existing evidence”的信息冲突),故
第二空选D项。

最后一句话和前一句话之间并没有转折词,按照句间关系不是逆
接就是顺接的思路,依然要以“其他城市才是第一个城市”为话题方
向,故第三空应选I项。如果选择“ignore(忽视)”,则与前句意
思不一致,不对老观点再进行考虑相当于支持了老观点,错误。如选
“evaluate(评价)”,则结论既可以是支持老观点也可以是削弱老
观点,并不符合前句支持“其他地方才是第一个城市”的话题方向,
故G和H两项都不正确。

第二空和第三空顺利选出D和I项。第一空的“established(确
定)”与“presented(呈现)”如何进行对比?很明显“establish
ed(确定)”的语气更重,任何方向的顺接都可以说是“presented
(呈现)”,所以A项属于范围小、语气重的选项,C项属于“虚的、
范围大”的选项。后文有没有针对老观点的描述语气重的词汇呢?“
serious reconsideration”中的“serious”提示了加强语气,因此
第一空填入“established(确定)”更符合此题的语境。得出答案
为A项。

原则7:句内关系优于句间关系

解读: 一个空格可以与多处信息相呼应,我们要思考的是这个
空格和哪处信息逻辑关联最强,不可使用跳跃式思维。肯定优先考虑
和空格所在句的“句内关系”,若“句内关系”不能帮助我们解题,
才思考“句间关系”。

例 The essential difference between writing nonfiction and writing


fiction is that the artist can(i)____ a completed vision of the world, while
the journalist never can, the real world being always(ii)____. Art provides
freedom from the bewildering complexities of constant change. Indeed,it is
the very(iii)____of well-wrought fiction that can sometimes make it feel
more real than reality.
分析: 假设此题没有读懂,但artist是“writing fiction”的
同义改写,journalist是“writing nonfiction”的同义改写,这点
必须看出。

根据第一句话可直接判断出前两空的答案。第二空选“unfinish
ed”(“完不完整”的话题)可呼应之前的“completed”,第一空
选“realize”可呼应“never can”(“能不能”的话题)。前两空
答案为C和F项。

第三空处于新句子中,则优先思考新句子里面是否有与空格相关
的信息,“freedom from bewildering complexities”与“流行”
没有关系,与“稀少”也不沾边,只与“简洁性”有联系,故答案为
I项。

原则8:出现代词要找其指代对象

解读: 此条与做阅读题和逻辑题的要点一致,只有找到代词的
指代对象,句意才不容易混淆。并且代词是一个很好的解题切入点,
比如“such+空格”就表明空格的内容一定在空格外的信息里有提过
,可以直接找选项,看哪个选项词汇与已有信息之间存在联系。

例 Recent proposals for fixing the climate have taken the form of large-
scale geoengineering projects such as launching mirrors into space to reflect
solar radiation away from Earth, undertakings that are vastly more(i)____,
than anything a nineteenth-century rainmaker could have cooked up. What is
unclear, as one looks back at the history of weather modification research, is
whether this resourceful ambition will be(ii)____, or if, by contrast, it
serves to make the scientific community's(iii)____ that much more
devastating.
解析: 通过第一空之前的信息,很容易判断出第一空的方向是
“好方向”,但选项内容都不与空格外的信息冲突,所以都放着待比
较。

第二空之前有一个信息线索“this resourceful ambition”,


“this”的出现说明“ambition”在前文有被提及,所以第一空填B
项“enterprising”,其与“ambition”话题最近。

“by contrast”表明第二空与第三空之间是“取反”关系,要
从第二空与第三空的选项中找出一对反义词,并且第三空要找一个“
不好”方向的词汇,因为第三空最后有“that much more devastati
ng”这个负面信息。第二空与第三空答案分别为F、H项。

原则9:一句话中的多重否定可两两抵消

解读: 一句话中出现多个否定词会干扰学生理解句意,建议把
否定词两两抵消。

例1 Despite its best efforts to stimulate sales, the bookselling business


remains far from ____, for it has high fixed costs in wages and rent, and
falling prices make these ever harder to spur.

A. effective

B. healthy
C. innovative

D. robust

E. stingy

F. Parsimonious

解析: Despite的出现表明主句与空格处之间是“取反”关系,
主句中far from 本身就是取反词,于是两两抵消,空格填入与“sti
mulate sales”方向相同的选项即可。

符合“好”方向的选项为前四项,其中找一对同义词出来,即答
案为B、D项。

例2 Although it is not uncommon for journalists to portray political


inexperience on the part of public officials as an(i)____, it was
nevertheless surprising when members of the press treated the new senator's
obvious(ii)____ as an extraordinary virtue.

解析: 在含有转折关系的主句中如果出现表示否定意思的副词
,如此句中的“nevertheless”,那么主句和从句之间虽然有转折关
系,但否定词并不可两两抵消。主句中写不写nevertheless之类的副
词,都和从句是“让步关系”(转折关系)。

看到surprising、astonish、little、few、belie、far from
等典型的“取反”词时,就相当于看到了“不等于号”。所以此题是
“although”与“surprising”两个否定词抵消,两空之间的逻辑关
系“取同”。
从改写的位置来对应,第一空呼应“virtue”,第二空呼应“in
experience”,得出答案为A、F项。

二、“余翔六十六条”

以下是GRE填空最常见的出题要点总结,虽然不能概括所有GRE填
空的出题点,但只要熟练掌握了这六十六条出题要点,攻克GRE填空基
本就只剩下背单词了。由于篇幅有限,这里只能列出简约版的六十六条
要点,考生要做到看到相关要点就立刻想到其对应的解读内容。

要点1:otherwise

解读: 可理解为“本应该……事实上没有这样”。出现在句中
即表示存在转折关系,属于“取反”词。

“扫码看相关知识点讲解”

要点2:A or even B

解读: 如果“or”单独使用,其前后既可以是“顺接”,又可
以是“逆接”,并不一定表示“取反”关系。但如果是“A or even
B”,这个“even”就表明A和B是“取同”的顺接,但A和B不可以是
同义词,因为even的存在必须让B是A的递进。误导选项会出现一个A
的同义词,切记不可选。

要点3:too… to …

解读: “too… to…”并不是一律翻译成“太……以至于不能


……”。即并不是使用了“too…to…”就“自带否定”。只有“too
”之后的条件与“to”之后的内容呈因果关系时,才带有“否定意思
”。若无因果关联,相当于“so...that...”的用法,并无否定含义

要点4:not A but B

解读: 此短语类似“not so much A as B”,A与B都属于“取


反”的逻辑关系。

要点5:A or B

解读: 如同上文所分析,A or B 不一定是“取同”或是“取反


”,需要借助语境分析。“both A and B”的搭配中,A和B肯定是“
取同”的关系。

要点6:so... that...
解读: “so+条件 that +结果”,如果空格在此表达的一边,
则根据另外一边的信息来推测答案。

要点7:belie

解读: 看到belie相当于看到了“不等于号”,最经典的“取反
”词。

“扫码看相关知识点讲解”

要点8:far from

解读: far from 表示出现“取反”的逻辑关系,因为from表示


“远离”。此短语也是高频出现的典型取反词。

要点9:surprising

解读: surprising、astonishing、little、few、belie是5个
特别常见的取反词。
要点10:even

解读: 出现even不一定表示“取同”或“取反”。当even翻译
成“甚至”时,其为递进关系,属于“取同”;若even翻译成“即使
”,其为“逆接”关系,属于“取反”。

要点11:despite

解读: despite、although、though、even though都表让步,


让步关系和转折关系都与主句的意思相反。与despite接近的一个词
是albeit(尽管),它们都属于介词。

要点12:although & but

解读: 转折关系与让步关系有区别,也有共性。共性在于所引
导的从句都与主句的意思相反。区别在于转折关系是从句话题直接与
主句相反,让步关系是从句话题与主句态度上相反,且不能是一个话
题。比如“她很漂亮,但是眼睛一个大一个小”。眼睛大小不一属于
“不漂亮”,转折关系是直接把主句的话题“取非”。让步关系是“
她很漂亮,但成绩不太好”,态度上主句说好的方向,从句说坏的方
面,且话题也不一致。让步关系的从句不可直接把主句的话题“取非
”。

要点13:逗号、分号、破折号
解读: 看到逗号、分号和破折号相当于看到了“约等于号”。
特别是逗号,解题过程中常常以逗号之后的信息为切入点。

要点14:商业与研究

解读: 这是一个常见的出题要点:如果商业公司资助了研究机
构,研究机构的结果就会被怀疑不公正。研究机构包含大学。

“扫码看相关知识点讲解”

要点15:过去与现在

解读: 在句子中读到和时间有关的词汇,比如now、until等,
就要思考这个句子是否存在“过去”与“现在”的不同分类。

要点16:qualified(熟词僻义)

解读: “qualified, qualification”在GRE填空中更多的情况


下考查的是“限制”的含义。“qualified=limited”,“unqualifi
ed”可以和“absolute”等价。

“扫码看相关知识点讲解”

获取更多熟词僻义的总结,请添加余老师微信“yuxiangoyes”
或扫二维码查看。

要点17:because

解读: 因果句是“取同”方向,要么同义改写,要么上义词或
下义词改写,以下都表示因果关系:because, due to, in that, th
anks to , so… as to…(so A as to be B:因为 A,所以 B)。

要点18:常规与特殊

解读: 可以理解为不同分类。

要点19:until
解读: until表示“在什么之前”,是典型的可能会有“时间分
类”的提示词。类似词汇还有once、formerly、initial、pristine
、erstwhile、hereto、hitherto、now、future、begin、start、cr
eate、no longer等。分类不同会导致属性不同,具体来说是要找相
反的动作、目的、原因、状态、观点等。反之,如果分类相同,则默
认属性相同。

要点20:现在时与过去时

解读: 要注意“现在”与“过去”的分类会在动词的时态上体
现,并不是出现明显的时间标志词。

要点21:not only...but also...

解读: “not only A but also B”中的A和B是递进关系,属于


顺接的方向。

要点22:indeed

解读: indeed表示“的确,确实”,属于顺接的方向。

要点23:A rather than B

解读: 很明显A与B是“取反”的逻辑关系。
要点24:anecdote & science

解读: anecdote可以理解为“非科学”,与“科学”是不同的
分类。

要点25:paradoxically

解读: 出现此词表示句子里面要有一对反义词。

要点26:on the other hand

解读: 出现该搭配表示前后是取反关系。

要点27:or & not...or... & without...or

解读: 不要只凭技巧判断取同或取反关系,需要借助常识理解
句意,或缩小答案选择范围。

要点28:状语

解读: 句子开头如果是状语,则这个状语是一定是由之后的主
语发出的。句子开头的状语的形式是:“adj/doing/done/介词短语
,主谓宾”。
要点29:同位语

解读: 如果空格之后是同位语或定语从句,那么这些信息都是
与空格最近的逻辑线索,优先将其作为解题切入点。

要点30:other

解读: 出现“other”,则必须有与其对应的词汇出现。高手做
填空题和阅读题的感觉一样,是将前后文结合在一起读,要注意逻辑
呼应以及保持词汇一致性。

要点31:and

解读: “and”连接两个并列句,常见情况是空格在其中一个并
列句中,那么就要优先从另外一个并列句中找答案。

要点32:now

解读: 凡是在句中出现与时间相关的概念,就要注意是否存在
时间分类的情况。

要点33:空格+“不好”词汇

解读: 这是一个固定的考查思路。如果空格之后跟的是一个负
面的词汇,比如“污染”“交通拥挤”“障碍”,绝大多数的情况下
空格里要填入表示减轻、缓和、消除等意思的词汇。

要点34:as /such/ this /their

解读: 上述四个词是常见的线索提示词,也可理解为解题切入
点,之后接的信息一定在句中的其他地方已经说过了。

“扫码看相关知识点讲解”

要点35:定语从句

解读: 定语从句优先修饰离其最近的可修饰名词。可修饰名词
是指可被定语从句修饰的名词,例如“which are....”这样表示复
数的定语从句只能修饰从which往左数最近的一个复数名词,如果“w
hich”之前是单数名词,则此名词不可修饰。

要点36:subordinate A of B
解读: 类似“replace A with B”,两个结构中的A和B都要“
取反”。

要点37:so…(A)as to be…(B)

解读: 类似“so…(A)that…(B)”,A和B都属于“取同”
,即意思相同、程度类似。

要点38:whereas

解读: whereas与while、although类似,表示对照或让步,并
不是转折关系。

要点39:while

解读: while并不是百分之百表示“让步”的含义,也可能表示
“顺接”,具体要看语境。但while大概率表示“对照”或“让步”
,其所在的从句和主句是“取反”的逻辑关系。

要点40:at best

解读: at best提示“轻微的贬义”的态度。
要点41:if only because & only because

解读: “if only because”含有贬义。“only because”表示


作者认可的唯一原因。两者的语言色彩有明显差别。

要点42:enervated

解读: enervated看上去类似energy,但其意思和“有能量”的
含义截然相反,与debilitate是同义词,表示“使没有力量”。

要点43:discreet & discrete(形近词)

解读: discreet表示“小心,谨慎”,discrete表示“离散”
,这两个词长得很像(形近),考生可这么记忆:“ee”是两个眼睛
,discreet联想成“小心谨慎”。discreet最后两个字母互换之后,
discrete表示“离散”,名词discretion又变成“小心谨慎”的含义
了。已有的题库中还考过三次discretionary,其意思为“可自由支
配的”。

“扫码看形近词”
更多的形近词汇总可添加余老师微信“yuxiangoyes”或扫描右
侧二维码。

要点44:evaluate

解读: 按已有的题库来分析,evaluate很少成为答案。学生们
可这么理解:GRE填空中的评论家的观点要么是赞扬,要么是批评,
所以中性词evaluate极少成为正确答案。

要点45:in the sense that

解读: 意为“部分原因是……”,此短语可理解为“因果关系
”。

要点46:no longer

解读: no longer只对过去的动作状态提出否定,所以每当看到
no longer,就要先考虑过去的状态或动作。

要点47:not only...but also & but also

解读: 在短语“not only A but also B”中,A和B是“取同”


的方向。但如果之前没有not only,单独出现but also的时候,其左
右两边的话题要“取反”。
要点48:and yet

解读: 短语的前后表示“转折”,属于“取反”关系。

要点49:形容词前置的让步关系

解读: 形容词前置位于句首,表示一种让步关系,翻译成“尽
管如何”,与后文构成“取反”的逻辑关系。

要点50:after a few months

解读: 说明是时间对比型题目。总之,句中任何表示时间的状
语都要密切关注,看是否出现“时间分类”的情况。

要点51:little more than

解读: 如空格在这个短语之后,需要选择一个负面意思的词汇

要点52:turn…(A)to…(B)

解读: 意思上与此短语类似的表达还有:shift from A to B,


oscillate between A and B, reconcile A and B 等。A和B一律是
“取反”关系。
要点53:not so much...(A)as...(B)

解读: 此短语的难点在于很多学生不认识,不知其意思如何。
“not so much A as B”相当于“not A but B”,意为“不是……
而是……”,A与B构成“取反”逻辑。

要点54:身份与动作

解读: GRE填空中特定的身份常搭配固定的动作,比如批评家的
动作是赞扬与批评,很少出现评价。类似的其他身份与动作:学者的
动作是解释社会;医生的动作是让病人恢复健康,常选的动作是salv
age等相关意思的词汇;企业家考虑的问题是如何获利;下级对上级
的指令永远都是服从等。

要点55:all but

解读: all but后接不同的词性,意思有细微差异。“all but


+名词”理解为“除了……之外都……”。“all but+形容词”的搭
配中“all but”相当于“almost”(几乎)。

要点56:nothing more than

解读: 考题中的这些固定搭配往往会使很多考生产生困惑,望
考生们牢记下面四个在考题中常见的短语。
at best:最多

nothing more than/little more than:只不过是

anything more than:不仅仅是

nothing but:绝不是

要点57:less than

解读: less than表示否定,看到less than等于看到了not。

要点58:party

解读: 如果考试中考查的是“党派”的意思,那就没法考“同
义”或“反义”逻辑了。如果在考试中考“倾向,偏见”的意思,相
当于“biased”。

要点59:取反词

解读: 取反词不一定只体现在否定词中,其他表示对立感觉的
词都可归为取反词。比如句子中讨厌A,则对于A的敌人要填入一个“
喜欢”方向的答案。“敌人”在这里也是取反词。

要点60:句子1:句子2, but句子3
解读: 在已有的题库中,上述结构的题目中句子1与句子3的事
件意思重复的情况更多。

要点61:不冲突的选项都先放着

解读: 重申一条最基础且核心的做题原则,众多学生无法突破
瓶颈是此步骤错了——GRE填空找选项的第一步是将不冲突的选项先
放着,不是直接找“对”的选项,而是挑出没有“错”的放着待比较

要点62:搭配形成合理句意

解读: 选项要与空格前后形成合理的逻辑搭配。如果只有一个
选项符合要找的答案方向,则不需要思考句意。如果不止一个选项符
合答案方向,则要将选项代入原文,进而判断所形成的句子语意是否
合理。

要点63:all the more +adj.

解读: 此短语有部分学生不认识。“all the more +adj.”等


于 “very +adj.”,有强调作用。

要点64:带着预测读下去
解读: 此处与阅读一致,为了降低复杂句子与结构对句意理解
造成的困难,建议学生带着预测读句子。比如读了although引导的从
句,则带着其相反的意思去读主句。

要点65:nevertheless 出现即是取反

解读: nevertheless在句中出现标志着会有一对“取反”逻辑
的词汇。

要点66:yet to become

解读: yet to become的意思是“有待成为”,其隐含着一个与


现在的对比,“有待成为”即“现在尚未成为”。
第十三章 考前必看机经词汇

扫码做1-50单词词测

1 prevaricate [prɪ'værɪkeɪt]

意思: vi. 支吾其词,搪塞

助记: 【记】pre +vary →预先变化,预先想好变化之词

记法: prevaricate → pre( 向 前 )+varicate(=to straddle the legs


叉开双腿)→叉开双腿向前走→一瘸一拐地走路→舌头在嘴里一拐一拐
地→支吾;varicate→variate(n. 变形)→叉开双腿身体就变了形

英文释义: to avoid giving a direct answer to a question in order to


hide the truth

同反义词: fence with/tergiversate

电影台词例句: And you have a way of prevaricating with your


answers, Mr. Cubitt.您也很擅长顾左右而言他,丘比特先生。
“扫码看相关知识点讲解”

2 vertiginous [vɜː'tɪdʒɪnəs]

意思: adj. 令人眩晕的

助记: vertigo;vertical +gin(音“井”)→竖着乘电梯下井,或在井


里转

同反义词: rotary/rolling/revolutionary/ unstable

电影台词例句: —Vertiginous or light-headed?—Light-headed.——


是眩晕还是昏睡?——是昏睡。

3 provenance ['prɒvənəns]

意思: n. 出处,起源

助记1: pro-“前”+ven-“来”,went“走”,来的前面是出处

助记2: prove(v. 证实)→proven(prove 的过去式,被证实了


的)→provenance(n. 起源)
英文释义: the place that something originally came from

同反义词: origin/birth

电影台词例句: This is the actual provenance of the painting.这是这


幅作品的真实出处。

4 dullard ['dʌlɑːd]

意思: n. 笨蛋

助记: dull(迟钝的)+ard→愚人,笨蛋

同反义词: simple/turkey/put/idiot/stupid

电影台词例句: Now I can tell the board firsthand what a dullard he is.
现在我可以直接跟董事会说他有多傻了。

5 betoken [bɪ'təʊkən]

意思: v. 预示,是……的预兆

同反义词: indicate/forecast/bid/shadow/announce

电影台词例句: A light boyish step, and a gay tune in a high key,


betokened a visit from Grandfer Cantle.孩子似的轻盈脚步和声调高的快乐
曲子,说明是坎特尔大爷来了。

6 vengeful ['vendʒfl]
意思: adj.报复的,复仇心理的

同反义词: retaliatory/avenging

电影台词例句: I'm a very vengeful person.我是个报复心很重的


人。

7 veneer [və'nɪə(r)]

意思: vt. 镶盖,胶合,粉饰

英文释义: if you refer to the pleasant way that someone or something


appears as a veneer, you are critical of them because you believe that their
true, hidden nature is not good;

同反义词: garment/gild

电影台词例句: And this tenuous veneer around our planet is what


allows life to flourish. 而正是围绕我们行星的这个气体薄层让万物得以生
生不息。

8 vainglorious [ˌveɪn'ɡlɔːriəs]

意思: adj. 虚荣心强的,自负的

助记: vain glorious 徒劳的荣誉

电影台词例句: For the left, it's a vainglorious reminder of their


defeat.而对于左派来说,这是在提醒他们当初的失败。

9 throttle ['θrɒtl]
意思: vt. 掐脖子;扼杀

词根记忆: throt(=throat喉)+tle→掐脖子

英文释义: to attack or kill somebody by squeezing their throat in


order to stop them from breathing

同反义词: throttling thort/kill/silence

电影台词例句: full throttle. 全速前进。

10 tenor ['tenə(r)]

意思: n. 男高音;要点,要旨

词根记忆: ten(握住)+or→握住的东西→要点

同反义词: general idea/main points

电影台词例句: I think you'll like it, it has two out of the three tenors.
你们肯定会喜欢的,“三高”之二都有出场。

11 staid [steɪd]

意思: adj. 死板的,守旧的,陈腐

同反义词: earnest/serious

12 snarled [snɑːrld]
意思: adj. 纠缠不清的

同反义词: embroiled

13 smug [smʌɡ]

意思: adj. 自满的,自命不凡的

助记: s +mug(杯子)→杯子满了

英文释义: looking or feeling too pleased about something you have


done or achieved

同反义词: self-righteous

电影台词例句: Now he's just laughing at me. Look at him, that smug
Belushi bastard. I'll... 现在他在嘲笑我。你看他,那个自鸣得意的混蛋。
我将……

14 slapdash ['slæpdæʃ]

意思: adj. 草率的,马虎的

助记: slap 拍 +dash 冲撞

同反义词: brief/careless

电影台词例句: Sorry, it's sort of slapdash.不好意思,有点草率。

15 sidesplitting [‘saɪd9splɪtɪŋ]
意思: 令人捧腹大笑的

助记: side(旁边)+split(分开)+ting→(笑得)浑身散架

16 seditious [sɪ'dɪʃəs]

意思: adj. 煽动性的

助记: sedit(煽动)+ious→煽动性的;sedit→tides(pl. 潮汐)→


把群众的情绪煽动得像潮汐一样汹涌澎湃

同反义词: inflammatory/agitative

电影台词例句: The very suspicion of seditious utterances against the


government, suspected complicity in the Ku Klux Klan, or complaint by a
negro that a white man had been uppity to him were enough to land a citizen
in jail.只要有煽动反对政府的一点点嫌疑,有3K党同谋的嫌疑,或者有
黑人控告他态度傲慢,就足以让一个市民进监狱了。

17 schism ['skɪzəm; 'sɪzəm]

意思: n. 分歧,分裂,分立,分派(教会)

助记1: 形近于chasm“裂口”;school +ism“主义”,学校中各种主义


学说→分成很多 派;s +chisel“凿子”→凿裂;scissor“剪刀”,是分裂东
西用的

助记2: schism→sophism(n. 诡辩)

英文释义: strong disagreement within an organization, especially a


religious one, that makes its members divide into separate groups

同反义词: disrupture/disjuncture

电影台词例句: He wants to talk about my Orthodox Church schisms


article. 他想谈谈关于我那篇东正教分立文章的事。

18 saccharine ['sækərɪn; 'sækəriːn]

意思: adj. 糖质的,含糖的,奉承的,娇媚的

词根记忆: sacchar(糖)+ine→(说话)像糖一样甜腻的→(说话
声)娇滴滴的

同反义词: sugary; gluside/saxin

电影台词例句: Daddy, have a saccharine or nothing.爸爸,最多只能


吃块糖精。

19 risible ['rɪzəbl]

意思: adj. 可笑的,引人发笑的

助记: ris(笑)+ible→可笑的 同根词: derisive(嘲笑的)

英文释义: deserving to be laughed at rather than taken seriously

同反义词: ridiculous/laughing

电影台词例句: Its main weapon was, and remains, state control of


television, which repeats endlessly the risible claim that Venezuela is a victim
of an economic war. 在查韦斯时代,他们主要的宣传武器是电视,现在
仍是电视,即由国家控制电视,通过电视不断地发表关于委内瑞拉是经
济战的受害者的声明。

20 retrograde ['retrəɡreɪd]

意思: adj. 后退的,倒退的,退步的 vi. 倒退,退步,逆行

助记: retro(向后)+grade(步伐,走)→向后走→后退

同反义词: reversionary/degenerative; reverse/stem;


backward/sternforemost

电影台词例句: It's called retrograde memory loss. It's caused by the


swelling and frontal-lobe damage. 这叫作逆行性失忆,是由肿胀和额叶受
损引起的。

21 remonstrate ['remənstreɪt]

意思: v. 表示异议,抗议,责备,反对

助记1: re(反对)+monstrate(=to show 展示)→向大家展示反对


意见→抗议

助记2: demonstrate(v. 示威,抗议)

英文释义: to protest or complain about something or somebody

同反义词: blame for/object/protest against sth.

电影台词例句: Miss Temple seemed to remonstrate.坦普尔小姐似乎


在抗辩。

22 rash [ræʃ]
意思: adj.草率

英文释义: marked by defiant disregard for danger or consequence

同反义词: hasty/imprudent

23 rant [rænt]

意思: v.咆哮;口出狂言

助记1: (音)怨他,嚷他→大声责骂,咆哮

助记2: rant→grant(v. 同意,准许)

助记3: rant→rants→trans(前缀,表示“横穿,转移”)

英文释义: to speak or complain about something in a loud and/or


angry way

同反义词: gnar/ball out/objurgate/roar/snarl

电影台词例句: Robert will rant for a few days, but he won't do


anything. He adores the man.罗伯特这几天会怒不可遏,但也仅限于此。
他喜欢那个男人。
“扫码看相关知识点讲解”

24 progeny ['prɒdʒəni]

意思: n. 子孙,结果

助记: progeny→pro +geny(音“珍妮”)→唯一的后代叫珍妮

英文释义: a person's children; the young of animals and plants

同反义词: production/child/outcome/seed

电影台词例句: Parents very very rarely disown their progeny.父母很


少会跟子女断绝关系。

25 plucky ['plʌki]

意思: adj. 勇敢的

同反义词: bold/nervy

电影台词例句: All this mayhem? It isn't some plucky underground


resistance movement. It's an extinction burst. 这通乱战?不是顽强的地下
抵抗运动,而是回光返照。

26 pithy ['pɪθi]

意思: adj. 言简意赅的,简练的

助记: pith“精髓”

同反义词: medullated/marrowy

电影台词例句: Yeah. Pithy analysis.是啊。精辟的分析。

27 pilferage ['pɪlfərɪdʒ]

意思: n.行窃;偷盗

英文释义: to steal things of little value or in small quantities,


especially from the place where you work

28 phony ['fəʊni]

意思: adj. 假的,伪造的,欺骗的 n. 假货,赝品,骗子

同反义词: spurious/unreal; personator/stumour

电影台词例句: It's all fake news. It's phony stuff. It didn't happen. 这
些都是假新闻。这都是假消息。并没有发生这种事。

29 petulant ['petjulənt]
意思: adj. 暴躁的,易生气的,难以取悦的

助记: pet +“吐”音→宠物吐→给宠物吃好吃的,它吐出来不吃发脾


助记: petulant→petul(=pet 生气)+ant→容易生气 的→脾气坏的

英文释义: bad-tempered and unreasonable, especially because you


cannot do or have what you want

同反义词: fierce/cranky

电影台词例句: The direct insult followed by the petulant whine? 先


公然侮辱,再怨天尤人?

“扫码看相关知识点讲解”

30 parlous ['pɑːləs]

意思: adj. 危险的,靠不住的

助记: perilous(a. 危险的)变体


同反义词: dangerous/critical/risky;
considerably/remarkably/finely/tremendously

31 obsequious [əb'siːkwiəs]

意思: adj. 谄媚的,逢迎的

助记: ob“不好的”+sequence“随后,跟随”→不好的跟随行为→谄
媚的,溜须拍马的

英文释义: trying too hard to please somebody, especially somebody


who is important

同反义词: flattering/well-oiled

电影台词例句: I am the Master of Whisperers. My role is to be sly,


obsequious and without scruples.我是情报总管,就该诡计多端,阿谀逢
迎,不择手段。

“扫码看相关知识点讲解”

32 natter ['nætə(r)]
意思: n./v. 唠叨,闲聊

同反义词: rabbit on/chatter; complaint/small talk

电影台词例句: Here we are nattering on about our big celebrity. 我们


光顾着唠叨我们的大明星了。

33 muckrake [‘mʌkreɪk]

意思: vi. 揭发丑闻

34 maunder ['mɔːndə(r)]

意思: vi. 絮叨,闲逛,徘徊

同反义词: hang about/wander about

35 macabre [mə'kɑːbrə]

意思: adj. 恐怖的,令人毛骨悚然的,以死亡为主题的

助记: 来自法语,原指“骷髅舞蹈”

英文释义: unpleasant and strange because connected with death and


frightening things

同反义词: terrible/horrible/awesome/fearful/dire

电影台词例句: Are you sure Lady Margaery doesn't find all this a bit
macabre? 你确定玛格丽小姐不会感到可怕吗?
36 inundate ['ɪnʌndeɪt]

意思: 泛滥,淹没

助记: in(进入)+und(波浪)+ate→卷入波浪→淹没

英文释义: to give or send somebody so many things that they cannot


deal with them all

同反义词: flood/flow

电影台词例句: The team have been inundated with enquiries from


patients. 医疗小组收到病人太多的咨询。

37 imbue [ɪm'bjuː]

意思: v.灌输,使渗透

同反义词: indoctrinate/infuse

电影台词例句: In Shamanic belief, the land is imbued with magical


properties which aid communication with the spirit world. 在萨满教的信仰
中,大地充满神奇的灵性,可以帮助人们和神灵沟通。

38 hilarious [hɪ'leəriəs]

意思: adj. 欢闹的,愉快的

词根记忆: hilar(高兴)+ious→高兴的
英文释义: extremely funny

同反义词: frisky/riproarious

电影台词例句: See? They're missing out on hilarious jokes like that.


瞧?他们错过了像这样的令人捧腹的笑话。

39 hector ['hektə(r)]

意思: v.凌辱,威吓

助记: hect(许多)+or→装出有许多力量的人→虚张声势的人→
威吓

同反义词: make a fool of/make a mockery of/;


fourflusher/intimidator; ride roughshod over/bullyrag

电影台词例句: Normally, you'd be up in the patient room hectoring


her on what to do. 通常你会在病房里恐吓病人听从你的指挥。

40 hauteur [əʊ'tɜː(r)]

意思: n.傲慢

助记: 来自法语 haut(高)+eur→傲慢(参考haughty“傲慢的”)

英文释义: an unfriendly way of behaving towards other people


suggesting that you think that you are better than they are

同反义词: importance/big headed

电影台词例句: He did not take fits of chilling hauteur. 他没有动不动


露出冷冰冰的傲慢态 度来。
41 harangue [hə'ræŋ]

意思: n./v.长篇指责性演说

分割记忆: har看作hard, angue 看作argue, hard +argue→强硬的辩论


→长篇演说

英文释义: to speak loudly and angrily in a way that criticizes


somebody/something or tries to persuade people to do something

同反义词: preachment; oratorize/speechify

电影台词例句: And here she closed her harangue. 说到这里她结束


了长篇演说。

42 fustian ['fʌstiən]

意思: n.空洞的话,无意义的高调

助记: 发音记忆“发诗兴”→诗兴大发,讲无意义的空话

同反义词: flatulent/swollen

电影台词例句: The line is worth a hundred pages of fustian. 这一行


胜过一百页浮华的文章。

43 friable ['fraɪəbl]

意思: adj.易碎的
同反义词: delicate/tender/fragile/brittle

电影台词例句: Her atrium is extremely friable. 她的心房非常脆


弱。

44 flair [fleə(r)]

意思: n.本领,才华,天赋

助记: 和fair(公正的,美丽的)一起记

同反义词: discernibility/discriminating power

电影台词例句: I'm stolid, my interior, where this place has a certain


flair. 我的内心缺乏热情,但这个地方却蕴含某种热情。

45 ferret ['ferɪt]

意思: vt.搜出,查获;驱出

英文释义: to search for sth. that is lost or hidden among a lot of things

同反义词: rake out/seek out/seek/ beat for

46 elision [ɪ'lɪʒn]

意思: n.元音省略,音节省略

英文释义: the act of leaving out the sound of part of a word when you
are pronouncing it
同反义词: synaeresis

电影台词例句: This is called elision. 这叫作省节。

47 dissimulate [dɪ'sɪmjuleɪt]

意思: v. 掩饰,假装,装糊涂

助记: dis(不)+simul(相同)+ate→本来面目不同→掩饰

同反义词: act/mask/simulate

电影台词例句: They also spend plenty of time trying to dissimulate.


同时,还拿出大把的时间试图掩饰。

48 desultory ['desəltri]

意思: a.不连贯的,散漫的

助记: de +sult(跳)+ory→跳来跳去→散漫的

英文释义: going from one thing to another, without a definite plan


and without enthusiasm

同反义词: loose/intermittent

电影台词例句: His conduct during these first weeks had been


desultory beyond description. 在这两三个礼拜里,他的行动杂乱无章,简
直无法形容。

49 coterminous [kəʊ'tɜːmɪnəs]
意思: adj. 毗连的,有共同边界的

助记: 相当于conterminous终点站 con +term(边界,结束)


+ious→有共同边界的

同反义词: adjacent/conjoint

50 contumacious [ˌkɒntju'meɪʃəs]

意思: adj. 不服从的,违抗的

助记: con +tum(肿胀;骄傲)+acious→违抗的,不服从的

英文释义: lacking respect for authority

同反义词: resistant/stubborn

扫码做51-94单词词测

51 contrivance [kən'traɪvəns]
意思: n. 发明,发明的才能,想出的办法

英文释义: something that somebody has done or written that does not
seem natural; the fact of seeming artificial

同反义词: invention/stratagem

电影台词例句: Till the next morning, however, she was not aware of
all the felicity of her contrivance. 不过,她的神机妙算究竟带来了多大福
气,她一直到第二天早上才知道。

52 conscientious [ˌkɒnʃi'enʃəs]

意思: adj. 尽责的;小心谨慎的

助记1: 有了良心、道德心(conscience)自然会小心谨慎
(conscientious)

助记2: con(共同),sci(知道),entious(多)—知道的道理多
就会有责任心,本着良心做事就是尽责的、小心谨慎的

英文释义: taking care to do things carefully and correctly

同反义词: serious/earnest

电影台词例句: Bree's lucky to have someone as conscientious as you


working here.布里有你这样细心的人帮她真是幸运之极。

53 choleric ['kɒlərɪk]

意思: adj. 暴躁的,易怒的

助记: choler(胆汁)+ic→胆汁质的→易怒的。choler本身是一个
单词,意为“暴躁的”

英文释义: easily made angry

同反义词: fierce/testy

电影台词例句: Why are you so choleric about airports? 你为什么对


机场生气啊?

54 chasten ['tʃeɪsn]

意思: vt.(通过惩罚而使坏习惯等)改正、磨炼

助记: 来自chaste(纯洁的)+n→变纯洁→改正

同反义词: control/stay/restrain

电影台词例句: Whom Mammy loved, she chastened. 嬷嬷要是宠爱


谁,就会严加管教。

55 bumbling ['bʌmblɪŋ]

意思: adj. 装模作样,笨手笨脚的

同反义词: managing/ham-handed/droning

电影台词例句: Weeping can waste time; bumbling costs valuable


seconds. 哭泣会浪费时间;结结巴巴也会浪费宝贵的时间。

56 broach [brəʊtʃ]
意思: vt. 开瓶;提出(讨论)

助记: 开瓶时造成一缺口(make a breach while broaching)

英文释义: to begin talking about a subject that is difficult to discuss,


especially because it is embarrassing or because people disagree about it

同反义词: introduce/present/prefer/file/advance; cutting bit/drilling


head

电影台词例句: Oh, I like your broach. 我喜欢你的吊饰。

57 beget [bɪ'ɡet]

意思: v. 产生,引起,生(子女)

助记: be +get“得到”

同反义词: effect/attract/invite/produce/court

电影台词例句: Or did power and influence beget worries?是权势造


成烦恼?

58 barefaced ['beəfeɪst]

意思: adj. 公然的,无耻的,无面具的

助记: bare“空的,没有的”+face“脸”+d→不要脸的→无耻的

英文释义: showing that you do not care about offending somebody or


about behaving badly

同反义词: declared/flagrant
电影台词例句: Barefaced liars lie easily and often.厚颜无耻的撒谎
者会经常轻易地撒谎。

59 avocation [ˌævə'keɪʃn]

意思: n. 娱乐,消遣,副业,嗜好(不要把vocation“职业”和
vacation“度假”搞混)

助记: a(不)+vocation(职业)→非正规职业→副业

同反义词: weakness/habit/appetite/hobby

电影台词例句: We can say that one is lucky if his vocation coincides


with his avocation. 如果一个人的职业刚好与他的爱好一致,那么我们就
可以说他是非常幸运的。

60 waggish ['wæɡɪʃ]

意思: adj. 滑稽的,诙谐的

同反义词: funny/jocular

61 superciliousness [ˌsuːpə'sɪliəsnəs; ˌsjuːpə'sɪliəsnəs]

意思: n. 傲慢

英文释义: behaving towards other people as if you think you are


better than they are

同反义词: importance/hubris
“扫码看相关知识点讲解”

62 lacuna [lə'kjuːnə]

意思: n.空隙,空白

同反义词: clearance/gap/blank

电影台词例句: One such lacuna has been the absence of well-


preserved feathers from the Cretaceous—the period between 145m and 65m
years ago that ended with the mass extinction that wiped out the dinosaurs.
一个漏洞就是由于人们无法找到白垩纪时期保存完好的羽毛,因而该时
期羽毛化石的记录一直处于空白状态,白垩纪是距今1.45亿年到6500万
年的一段时期,末期发生了导致恐龙消失的物种大灭绝。

63 ruse [ruːz]

意思: n.诡计,骗术

联想记忆: 用玫瑰(rose)来骗取(ruse)姑娘的芳心→骗术,诡

英文释义: a way of doing something or of getting something by


cheating somebody

同反义词: strategy/device/scheme/fraud

电影台词例句: Can we drop that ruse soon? 能快点结束这种策略


吗?

64 blase [blɑː'zeɪ]

意思: adj.冷漠的,厌倦享乐的

助记: 对责骂(blame)已经厌倦(blase)

电影台词例句: One or two. Is that all? How blase? 一两根啊,仅此


而已吗?真无趣啊?

65 haggle ['hæɡl]

意思: vi. 争论,讨价还价 vt. 乱砍 n. 杀价,争论

同反义词: controversy/combat; cut and carve/debate/argue with

电影台词例句: Obviously you know how to haggle, so I'm not gonna


try and take you on. 你显然是杀价高手,所以我就不糊弄你了。

66 mordant ['mɔːdnt]

意思: adj.讥讽的,尖酸的

助记: 形近于rodent 咬的, 侵蚀性的;mordant 音“矛盾的”


助记2: Modern Times 是讽刺喜剧片

助记3: mordant→mord(=to bite 咬)+ant→像被咬了一口→腐蚀


英文释义: critical and unkind, but funny

同反义词: spicy/pungent

电影台词例句: Yet Ms Heti's mordant take on modernity encourages


introspection.然而,海蒂对现代生活的尖酸调侃却引人反思。

67 palaver [pə'lɑːvə(r)]

意思: v./n. 空谈,奉承

助记: palace(宫殿)+aver(承认,说话)→宫殿里的话→空谈

同反义词: negotiation/court/treaty; gas/rabbit on/flatter/lick one's


boots

电影台词例句: No tricks, just a little palaver. 没有骗局,只是一些


谈话。

68 perverse [pə'vɜːs]

意思: adj. 乖张的,倔强的,错误的,不正当的,误入歧途的,有


悖常理的

助记: per(始终)+verse(转)→始终和别人反着转→故意反对

同反义词: fallen/vicious
电影台词例句: It was a perverse grab for attention. 她是不顾一切想
吸引你的注意。

69 rapt [ræpt]

意思: adj. 全神贯注的

英文释义: so interested in one particular thing that you are not aware
of anything else

同反义词: concentrated/preoccupied

电影台词例句: He received no answer, all for the moment sitting rapt


in admiration of the performance described. 没有人回答他的问题,大家坐
在那儿,听了对演奏的这番形容,钦佩不已,一时都出了神。

70 cosset ['kɒsɪt]

意思: vt. 宠爱,溺爱 n. 宠物

助记: cosset→closet(n. 壁橱)→把自己宠爱的小东西都放在壁橱


英文释义: to treat somebody with a lot of care and give them a lot of
attention, sometimes too much

电影台词例句: In fact, big firms were largely a bloated, inefficient


and cosseted lot.事实上,大型企业在很大程度上集膨胀、效率低下及骄
纵于一身。

71 incumbent [ɪn'kʌmbənt]
意思: 现任者,在职者

助记1: in +cumb(躺),在里面躺着的→房客,办公室人员

助记2: incumbent→in(=upon)+cumb(=to lie)+ent→ lying


upon→正躺在位置上的人→现任者

英文释义: a person who has an official position

同反义词: dependent/anaclitic

电影台词例句: He won the majority votes in the second round run-off


against the incumbent. 他在第二轮决选中以多数票战胜了现任总统。

72 expostulate [ɪk'spɒstʃuleɪt]

意思: vi.(对人或行为的)抗议,告诫

助记1: ex(出,否定)+postulate(假定)→要求不要做某事

助记2: expostulate → ex(=out)+postulate(=to demand“要求”)


→要求某人从坏习惯或错误中抽身→劝诫

英文释义: to argue, disagree or protest about something

同反义词: preach to/warn sb. of sth.

电影台词例句: Tess did not expostulate.苔丝不再争辩了。

73 prolix ['prəʊlɪks]
意思: adj. 啰唆的,冗长的

助记: pro- +liquid→像水流一样多,长;pro +lex“词汇”→用词汇


多的

英文释义: using too many words and therefore boring

同反义词: interminable/verbose

电影台词例句: I will not be prolix, but it must be right that I mark


that I am the happiest and luckiest of men.我不会说太久,但我必须说我是
世界上最快乐、最幸运的男人。

74 fiasco [fi'æskəʊ]

意思: n. 惨败

助记: 和fresco(n.壁画)一起记

英文释义: something that does not succeed, often in a way that causes
embarrassment

同反义词: catastrophe/smashup

电影台词例句: Harry sighed, “Wish I'd followed him now, the match
was such a fiasco...”哈利叹了口气,“当时要跟踪他就好了,比赛输得这
么惨……。”

75 finicky ['fɪnɪki]

意思: adj. 苛求的,过分讲究的

助记: finical的变体,来自“fine(精细的)+ical”→精细的→挑剔
的,讲究的

同反义词: overnice

电影台词例句: Pawpaws, on the other hand, are inherently finicky. 另


一方面,万寿果天生就很难养。

76 regurgitate [rɪ'ɡɜːdʒɪteɪt]

意思: vt. 反刍,反胃,流回,涌回

英文释义: to bring food that has been swallowed back up into the
mouth again

同反义词: back flow/inverse flow

电影台词例句: Some animals mark their territory by regurgitating


their food. 有些动物通过反刍它们的食物来划清地盘。

77 lechery ['letʃəri]

意思: n. 好色

助记: 来自lecher(好色之徒),由licker(舔东西的人)而来

同反义词: sensuality/lasciviousness

电影台词例句: You're ordering tests to cover your lechery.


Interesting. 你借故检查来掩盖你好色的本质。有意思。

78 sleight [slaɪt]
意思: n. 熟练,巧妙,诡计,狡诈

助记: sl(看作sly狡猾)+eight→八面玲珑→巧计

同反义词: technique/skill/facility/mechanism/ scheme

电影台词例句: It's a sleight of hand fraud. 这是一种障眼法。

79 hiatus [haɪ'eɪtəs]

意思: 空隙,裂缝

助记: “he +at +us”,他在我们之间造成了裂缝,如第三者之类的

同反义词: crack/clearance/split

电影台词例句: You see, Grey? Good exposure to the esophageal


hiatus.看到了吗,Grey?很好地暴露了食管裂孔。

80 aphorism ['æfərɪzəm]

意思: n. 格言,警语

助记: a +phor(带来)+ism→带来智慧的话

英文释义: a short phrase that says something true or wise

同反义词: sententia/maxim

电影台词例句: His writing is elegant and urbane, full of paradoxes,


aphorisms and conceits. 他的作品语言优美雅致又充满悖论,各种金句和
巧妙的比喻接二连三。

81 erstwhile ['ɜːstwaɪl]

意思: adj.从前的,过去的

助记: ere(early)“以前的”+while“一段时间”

助记2: erstwhile→erst(在此以前)+while(n. 时间)→ 以前的

英文释义: former; that until recently was the type of person or thing
described but is not any more

同反义词: previous/former/ago; previously/ago/formerly/back

电影台词例句: Nothing remained of them or their erstwhile homes


but fragments of glass upon the floor.消失得无影无踪,只剩下地板上的玻
璃碎片。

82 gusto ['ɡʌstəʊ]

意思: n. 爱好;兴致勃勃

英文释义: enthusiasm and energy in doing something

电影台词例句: Let us sing with passion, gusto, fit to bust. 让我们在


欢乐中高歌、纵情欢闹。

83 lukewarm [ˌluːk'wɔːm]
意思: adj. 微温的,不热的

助记: luke(=tepid微温)+warm(温)→微温的

英文释义: slightly warm

同反义词: cold/nonchalant

电影台词例句: Oh, what beautiful lukewarm sentiment. 不冷不热的


情感可真是美好啊。

84 clout [klaʊt]

意思: n. 猛击,影响力 v. 猛击

助记: 和cloud一起记,像云遮住太阳→有影响力

英文释义: power and influence

电影台词例句: They lose their power, they lose their clout. 他们失去
了权力和影响力。

85 raillery ['reɪləri]

意思: n. 善意的嘲弄

助记1: 有栏杆(rail)挡着不让过,也是嘲弄

助记2: raillery→raill(嘲弄,取笑)+ery→嘲笑;raill→liar(n. 骗
子)→国王受到骗子的嘲弄

同反义词: gaggery
电影台词例句: A running fire of raillery and jests was proceeding
when Sam returned.萨姆返回时,相互正戏谑嘲弄,玩笑开得火热。

86 gambit ['ɡæmbɪt]

意思: n.开始

助记: game +bit→下棋中让一点,一个子来开局,国际象棋中的


开局弃子法

同反义词: threshold/conception/beginning/fresh/start

87 prophylactic [ˌprɒfə'læktɪk]

意思: adj. 预防疾病的 n. 预防药

助记: pro(预先)+phylact(保护)+ic

88 panache [pə'næʃ]

意思: n. 炫耀,假威风

助记: 近于pinnacle“小尖塔”;pan(平底锅)+音“车”:拿着锅吃
饭→炫耀

英文释义: the quality of being able to do things in a confident and


elegant way that other people find attractive

同反义词: effulgence/flashiness
电影台词例句: Okay, well, you know what? I would hate to muck up
your promposal with my flair and panache.跟你说,我聪明绝顶还一表人
才,可不想搞砸你的舞会邀请。

89 dainty ['deɪnti]

意思: adj.娇美的;挑剔的

词根记忆: dain(=dign“高贵”)+ty→高级食品

英文释义: small and delicate in a way that people find attractive

同反义词: delicious/elegant/particular; deliciousness

90 lugubrious [lə'ɡuːbriəs]

意思: adj. 悲哀的,忧郁的

助记1: 音“路孤悲”,考Gre道路上孤单悲凉

助记2: “卢沟 bring us”→ 卢沟桥带给我们一段悲哀的往事

英文释义: sad and serious

同反义词: sad/miserable/woeful

电影台词例句: “I am drinking,” replied the tippler, with a lugubrious


air.“我在喝酒,”酒鬼阴沉忧郁地回答道。

91 pillory ['pɪləri]
意思: v. 公开批评

助记1: pile(桩子)+lorry(卡车)→绑在卡车后面的桩子,戴着
枷锁游街示众

助记2: pillory 音“劈了锐”→劈了枷锁,锐气大增

英文释义: to criticize somebody strongly in public

电影台词例句: Philip Morris was pilloried in the press and they


issued an apology for this heartless calculation.菲利普·莫里斯公司遭到媒
体谴责,他们为这项冷血的计算公开道歉。

92 desiccate [‘desɪkeɪt]

意思: vt.(使)完全干涸,脱水

词根记忆: de +sicc(干)+ate→弄干,脱水;参考dehydrate(脱
水)

同反义词: dry out/exsiccate

电影台词例句: Grass clings to life even as desiccating winds remove


what little moisture remains in the soil. 即使风把土壤中仅存的水分带走,
草也能存活下来。

93 redolent ['redələnt]

意思: adj. 有……香味的,令人想起……的


助记1: 近于odor “气味”;red +oil→红油,正红花油是芳香的

助记2: red(红色)+olfactory(oil +factory 油厂,嗅觉的)→红色


又有味道的,是红花

助记3: redolent→red(=off“散发”)+ol(=to smell“发出气味”)


+ent→giving off an odour→香的

英文释义: making you think of the thing mentioned

同反义词: aromatic/odoriferous

电影台词例句: The air was redolent of steak and fried onions. All of
it the same as it ever was. 空气中弥漫着煎牛排和洋葱的香气。一切照
旧,从未改变。

94 insouciant [ɪn'suːsiənt]

意思: adj. 漫不经心的,漠不关心的

同反义词: indifferent/careless

“扫码看相关知识点讲解”
第十四章 考前必练20道经典机经题
1. The memoir comes as a surprise—a probing recollection of emotional
transformation by an author not naturally given to ____.

A. fulmination

B. equivocation

C. introspection

D. loquaciousness

E. affability

2. The idea of a “language instinct may seem ____to those who think of
language as the zenith of the human intellect and of instincts as brute
impulses.

A. jarring

B. plausible

C. gratifying

D. inevitable

E. conciliatory

3. Although the inhabitants of Sardinia had initially offered little or no


resistance to Roman rule of their island, the landed gentry of west central
Sardinia later openly ____Rome's enemy Carthage during the Second Punic
War.
A. derided

B. queried

C. supported

D. accosted

E. envied

4. The depletion of soil and trees in southern West Virginia has resulted
in frequent flash floods, since when the mountain slopes are ____, storms
cause torrents of water to pour down them unimpeded.

A. riven

B. fecund

C. denuded

D. withered

E. mutable

5. Notoriously, consumer behavior can be mercurial, and the future


predilections of moviegoers are especially ____.

A. volatile

B. derivative

C. manifest

D. unsophisticated

E. predictable

6. In the(i)____world of the pioneering Walker Art Center, artful


provocation is(ii)____blockbuster entertainment, the(iii)____is
privileged over the tried-and-true, and willful insouciance is cultivated about
the market forces that rein in the creativity of most big museum
establishments.

7. There can be little question that by the fourteenth century the capacity
to(i)___a doubtless restricted written Latin was available in virtually all
English communities. It would, of course, be(ii)___to suppose that all the
wealth of material available in Latin since 1100 was put to service in the
cultural formation of every writer of business Latin in provincial England,
but even schoolbooks(iii)___some elements of classical education. Latin
literacy was not, in these circumstances, exclusive.

8. Studies suggest that both adults and children often regard temperature
as(i)____ how hot or cold something feels. This misconception is
not(ii)____ . For example, a study showed that significant numbers of
engineering students believed that, to someone in bare feet, carpet feels
warmer than floor tile in the same room because its temperature is higher,
even though the temperatures of the surfaces are(iii)____, both having
come to thermal equilibrium with the same surrounding environment.

9. During the 1950s and 1960s, theory-minded neoclassical economists


came to(i)____the field of labor economics, pushing their more fact-
oriented colleagues to the margins. In more recent years, the theorists have
become interested in just the sort of(ii)____issues they once associated
with other economists and whose study they once(iii)____.

10. Fifteen years ago, the author of this scholarly monograph would
have(i)___found a university press willing to publish it. But today
specialized books of this sort are(ii)___ species. Their main purchasers—
university libraries—have far less money to spend on these items than they
once did. Computerized catalogs, subscription content, expensive scientific
journals, exploding storage costs: all these demands are(iii)____library
budgets.

11. The history of dance put forward in this volume(i)___ its own
methodological rules. For example, the editor repeatedly criticizes labels and
their “flawed finality,”but the term “modem”is employed(ii)___
throughout the book.

12. The researcher found that in assessing others, many people hold an
unconscious view that competence and warmth are(i)____: when they
perceive a person to be highly capable, they infer that he or she must have a
tendency to be(ii)____ .

13. The claim that large budget deficits significantly depress private
investment and thereby hurt future wages and living standards is(i)____ ;
in reality, the(ii)____ effects of budget deficits are tiny.

14. If you follow your intuition, you will more often than not err by
misclassifying a random event as(i). We are far too willing to(ii)the
belief that much of what we see in life is random.

15. Mowry believes that effective politics demands a synthesis of the


practical and the visionary. He has never had much trouble coming up
with(i)____. But these days, he also thinks he has become better at
relating that larger picture directly to(ii)____policies.
16. Whereas the original editor had sacrificed clarity for brevity and her
successor had been too ____, the new editor took an intermediate approach,
explicit as to the purport of each document and not too ambitious in detail.

A. discriminating

B. prolix

C. bombastic

D. verbose

E. cryptic

F. inscrutable

17. William Perkins, his ____speaking style notwithstanding, has long


been seen as the moderate face of his political party.

A. fiery

B. genteel

C. bumbling

D. unremarkable

E. affable

F. impassioned

18. Epidemiological studies have found that occasional tea drinkers, and
those who avoid tea altogether, do not reap the benefits of lowered risk of
heart attack that those who ____the brew do.

A. eschew

B. appreciate
C. sip

D. forswear

E. swill

F. guzzle

19. The intraparty debate that had been a fertile source of policy
proposals was now ____, leading voters to wonder if the party would be
bereft of new ideas in the upcoming election cycle.

A. passionate

B. multifaceted

C. petty

D. moribund

E. intense

F. fading

20. Although ants and humans have lifestyles, they fight their foes for
many of the same economic reasons, including access to dwelling spaces, to
territory, and to food.

A. complex

B. divergent

C. atypical

D. conflicting

E. disparate

F. complicated

You might also like